വിമർശനം:
സൂര്യനാണല്ലൊ ഭൂമിയിലെ ഋതുക്കൾക്ക് കാരണം. അപ്പോൾ നരകത്തിൻ്റെ ശ്വാസമൊ/വിടുതിയൊ ആണ് കാലാവസ്ഥകൾക്ക് കാരണം എന്ന ഹദീസ് ശാസ്ത്ര വിരുദ്ധമല്ലെ ? കൂടാതെ പനി നരകത്തിൻ്റെ ചൂടാണെന്നും, മലക്ക് മേഘങ്ങളെ തെളിച്ചു കൊണ്ടു പോകുമ്പോഴുള്ള ശബ്ദവും വെളിച്ചവുമാണ് ഇടിമിന്നൽ എന്നും ഹദീസുകളിൽ വിവരിക്കപ്പെട്ടതെല്ലാം യാഥാർത്ഥ്യത്തോടും യുക്തിയോടും നിരക്കാത്തതല്ലെ ?
വിമർശനവിധേയമായ ഹദീസ്:
اشْتَكَتِ النَّارُ إلى رَبِّهَا، فَقالَتْ: يا رَبِّ أَكَلَ بَعْضِي بَعْضًا، فأذِنَ لَهَا بنَفَسَيْنِ، نَفَسٍ في الشِّتَاءِ، وَنَفَسٍ في الصَّيْفِ، فَهْوَ أَشَدُّ ما تَجِدُونَ مِنَ الحَرِّ، وَأَشَدُّ ما تَجِدُونَ مِنَ الزَّمْهَرِيرِ.“നരകം തൻ്റെ രക്ഷിതാവിനോട് പരാതിപ്പെട്ടു: “എൻ്റെ ചില ഭാഗം ചിലതിനെ തിന്നു”. അപ്പോൾ രണ്ട് ആശ്വാസം/ശ്വാസം* സമയം നരകത്തിന് അനുവദിക്കപ്പെട്ടു. ഒരു ആശ്വാസം/ശ്വാസം ശൈത്യകാലത്തും മറ്റൊന്ന് ഉഷ്ണകാലത്തുമാണ്. അത് നിങ്ങൾ (ഈ കാലാവസ്ഥകളിൽ) അനുഭവിക്കുന്ന തീക്ഷ്ണമായ ചൂടും തണുപ്പുമാണ്.” (സ്വഹീഹുൽ ബുഖാരി: 3260, സ്വഹീഹു മുസ്ലിം: 617)
* ഹദീസിലെ (النَّفَسُ) എന്ന പദത്തിന് ആശ്വാസം/ശ്വാസം എന്നീ രണ്ട് അർത്ഥവും കൽപ്പിക്കാവുന്നതാണ്.
മറുപടി:
“പാവ”ലോകത്ത്, ഒരു “വിശ്വാസി- നിഷേധി” സംവാദം നടക്കുന്നു!
വിശ്വാസികളുടെ പക്ഷത്തു നിന്ന് വിഷയമവതരിപ്പിച്ച “പാവ” പണ്ഡിതൻ തൻ്റെ അവതരണത്തിന് ഇപ്രകാരം വിരാമം കുറിച്ചു:
“ചുരുക്കത്തിൽ, നാം പാവകൾ സൃഷ്ടിക്കപ്പെട്ടിരിക്കുന്നത് മനുഷ്യ കുഞ്ഞുങ്ങൾക്ക് കളിക്കാനാണ്.
രാവിലെ പെൺകുട്ടിയാണ് നമ്മളെയും കൊണ്ട് കളിക്കുന്നത്. അതുകൊണ്ടാണ് അധികം പരിക്കൊന്നും നമുക്ക് ഉണ്ടാവാത്തത്. വൈകുന്നേരം നമ്മളെയും കൊണ്ട് കളിക്കുന്നത് ആൺകുട്ടിയാണ്. അതുകൊണ്ടാണ് വൈകുന്നേരം നമ്മുടെ പരിക്കുകൾ കൂടുന്നതും.
എന്തു കൊണ്ടാണ് നാം പാട്ട് പാടുന്ന പാവകളായത് ? മനുഷ്യ കുട്ടികൾക്ക് പാട്ടു കേൾക്കാൻ ഇഷ്ടമായതു കൊണ്ടാണത്.
എന്തുകൊണ്ടാണ് -പാവകളായ- നമ്മുടെ ശരീരം മിനുസമുള്ളതായത് എന്ന് ചിന്തിച്ചു നോക്കിയിട്ടുണ്ടോ? കൂർത്ത ഭാഗങ്ങളുണ്ടായാൽ, കുഞ്ഞുങ്ങളുടെ മൃദുലമായ കൈകളിൽ മുറിവേൽക്കും എന്നതുകൊണ്ടാണത്.
നമ്മുടെ വലിപ്പം ഇത്രയായി തിട്ടപ്പെടുത്തപ്പെട്ടു എന്നതിനും ഒരു കാരണമുണ്ട്. നാം നന്നേ ചെറുതായാൽ കുട്ടികൾ നമ്മെ വിഴുങ്ങിക്കളഞ്ഞേക്കും, നാം വളരെ വലുതായാൽ കുട്ടികൾക്ക് നമ്മെ താങ്ങാൻ കഴിയില്ല എന്നതിനാലുമാണത്.
നമ്മുടെ പ്രകൃതത്തിനും സൃഷ്ടിപ്പിനും ഓരോ ചലനത്തിനു പോലും അർത്ഥവും ലക്ഷ്യവുമുണ്ട് എന്ന് ചുരുക്കം…”
എതിർ പക്ഷത്തു നിന്ന് എഴുന്നേറ്റ് നിന്ന, നിഷേധിയായ പാവ പരിഹാസപൂർവ്വം ചിരിച്ചു കൊണ്ട് പറഞ്ഞു:
“യാഥാർത്ഥ്യങ്ങൾക്ക് നിരക്കാത്ത, ശാസ്ത്ര വിരുദ്ധമായ എത്ര വലിയ വിഡ്ഢിത്തങ്ങളാണ് മറുപക്ഷം ഇവിടെ വിശദീകരിച്ചിരിക്കുന്നത്!
രാവിലെ പെൺകുട്ടിയും വൈകുന്നേരം ആൺകുട്ടിയുമാണ് നമ്മളെയും കൊണ്ട് കളിക്കുന്നത് എന്നതിനാലല്ല നമ്മുക്ക് പരിക്ക് കൂടുകയും കുറയുകയും ചെയ്യുന്നത്. മറിച്ച് രാവിലെ നമ്മുടെ ശരീരം സ്വഭാവികമായും ശാന്തമായതു കൊണ്ടാണ് നമ്മുക്ക് പരിക്കുകൾ കുറയുന്നത്. വൈകുന്നേരങ്ങളിലാകട്ടെ നമ്മുടെ ശരീരം പ്രകൃത്യാ ചലനാത്മകമാണ്. അതുകൊണ്ടാണ് പരിക്കുകൾ കൂടുന്നത്. ആൺകുട്ടിയും പെൺകുട്ടിയും മാറി മാറി കളിക്കുന്നതിനാലാണ് പരിക്കുകളിൽ ഏറ്റക്കുറച്ചിലുകൾ ഉണ്ടാവുന്നത് എന്ന അന്ധവിശ്വാസം (നമ്മുടെ പരിക്കുകളെ കുറിച്ചും പ്രതിവിധികളെ കുറിച്ചുമുള്ള) വൈദ്യ ശാസ്ത്രത്തിൻ്റെ പുരോഗമനത്തിന് തടസ്സമാണ്.
മനുഷ്യ കുട്ടികൾക്ക് പാട്ടു കേൾക്കാൻ ഇഷ്ടമായതു കൊണ്ടാണ് നാം പാട്ടുകാരായത് എന്ന വാദം വിചിത്രം തന്നെ! മെക്കാനിക്കൽ ഫോണോഗ്രാഫ്, ടേപ്പ് റെക്കോർഡറുകൾ, ഇലക്ട്രോണിക് സിന്തസൈസർ എന്നിവയൊക്കെയാണ് നാം പാട്ടുകാരാവാൻ കാരണം എന്ന് ശാസ്ത്രം ഇന്ന് തെളിയിച്ചു കഴിഞ്ഞിട്ടുണ്ട്.
കുട്ടികളുടെ കൈകളിൽ മുറിവാവരുത് എന്നതാണ് നമ്മുടെ ശരീരത്തിൻ്റെ മിനുസത്തിന് കാരണം എന്ന വിശ്വാസം അതിനേക്കാൾ വലിയ തമാശ! സിലിക്കണു കൊണ്ട് നിർമ്മിക്കപ്പെട്ടതിനാലാണ് നമ്മുടെ ശരീരം ഇത്ര മൃദുലം എന്ന വിവരം പോലും ഇവർക്കില്ലെ?!
പിന്നെ, നമ്മളെ വിഴുങ്ങാൻ മാത്രം വലുതാണ് ഈ കുട്ടികൾ എങ്കിൽ നമ്മളെ താങ്ങാനും അവർക്ക് കഴിയേണ്ടതല്ലെ?…
ചുരുക്കത്തിൽ, നമ്മുടെ സൃഷ്ടിപ്പും പ്രകൃതിയും ചലനങ്ങളാസകലവും ആകസ്മികതകൾ മാത്രമാണ്. ഒന്നിനും വസ്തുനിഷ്ഠമായ ഒരു അർത്ഥവും ലക്ഷ്യവും ഇല്ല…
സംവാദം കരഘോഷങ്ങളുടെ ആരവങ്ങളോടെ തുടർന്നുകൊണ്ടിരുന്നു…
ഭൗതിക ലോകത്തെയും പ്രകൃതി പ്രതിഭാസങ്ങളെയും വിശകലനം ചെയ്യുമ്പോൾ (ഇസ്ലാം) മതവും ശാസ്ത്രവും മിക്കവാറും രണ്ട് വ്യത്യസ്ത മേഖലകളെയും കാരണങ്ങളെയുമാണ് ചർച്ച ചെയ്യുന്നത് എന്ന് പാവ ലോകത്തെ ഈ വിശ്വാസി-നിഷേധി സംവാദത്തിലൂടെ വായനക്കാർക്ക് മനസ്സിലായിരിക്കുമല്ലൊ. ശാസ്ത്രം പ്രപഞ്ചത്തിലെ ഭൗതിക കാരണങ്ങളെ കുറിച്ച് സംസാരിക്കുമ്പോൾ മതം ആ പ്രപഞ്ചത്തിൻ്റെ ഭൗതിക കാരണങ്ങൾക്ക് പിന്നിലെ ദൈവിക ലക്ഷ്യത്തെയും ഉദ്ദേശ്യത്തേയുമാണ് കൈകാര്യം ചെയ്യുന്നത്. ഈ രണ്ട് വിശദീകരങ്ങളും തമ്മിൽ വൈരുദ്ധ്യമില്ലെന്ന് “പാവ സംവാദ” ത്തിലൂടെ മനസ്സിലാക്കാമല്ലൊ. പാവകൾ വിനോദ ഉപകരണങ്ങളാണെന്നും അവയുടെ സൃഷ്ടിപ്പിലെ ഓരോ ഭൗതിക കാരണങ്ങൾക്കും (ആകാരം, മൃതുലത, പാട്ട്, പരിക്കുകൾ) പിന്നിൽ അഭൗതിക (പാവേതര) കാരണങ്ങൾ ഉണ്ട് എന്നും അംഗീകരിക്കാനും, ഈ ഭൗതികവും അഭൗതികവുമായ കാരണങ്ങളെ പരസ്പരം സംയോജിപ്പിക്കാനും ഒരു നിഷേധിക്ക് കഴിയില്ല. അതിന് കാരണം നിഷേധി, പാവലോകത്തിനപ്പുറം ഉള്ളതെല്ലാം നിഷേധിക്കുന്നു എന്നത് മാത്രമല്ല. ഒരു പ്രതിഭാസത്തിന് ഒരേ സമയം പല കാരണങ്ങളും വ്യാഖ്യാനങ്ങളും ഉണ്ടാവൽ ബുദ്ധിപരമാണ്, എന്ന് തത്ത്വത്തിലെങ്കിലും അംഗീകരിക്കാനുള്ള യുക്തിയൊ വിനയമൊ ഇല്ലാത്തതു കൊണ്ടു കൂടിയാണത്. പാവയുടെ മെയ്യഴകിന് കാരണം സിലിക്കണെന്ന ഭൗതിക കാരണമാണ് എന്നതോടൊപ്പം തന്നെ, പാവ കൊണ്ട് കളിക്കുന്ന കുട്ടിയുടെ പിഞ്ചു കൈയ്യിൽ മുറിവേൽക്കാതിരിക്കുക എന്ന ഉദ്ദേശ്യ-ലക്ഷ്യപരമായ (Intentional- Purposefull) കാരണം കൂടി ഉണ്ട് എന്ന് ഒരാൾ അംഗീകരിച്ചാൽ അവ തമ്മിൽ എങ്ങനെ വൈരുദ്ധ്യമാവും ?! പാവ ലോകത്തെ ശാസ്ത്രത്തിൻ്റെ വളർച്ചക്കോ പുരോഗമനത്തിനൊ ഈ ഉദ്ദേശ്യ-ലക്ഷ്യപരമായ കാരണം എങ്ങനെ തടസ്സമാവും ?!
പാവലോകത്തിനപ്പുറം ഒരു ലോകമില്ലെന്നും, പാവപ്രകൃതിക്ക് ഒരു ഉദ്ദേശ്യ-ലക്ഷ്യവും ഇല്ലെന്നും ഒരു പാവക്ക് കാരണങ്ങളൊന്നും കൂടാതെ വാദിക്കാം. പക്ഷെ ഈ നിഷേധത്തിന് തെളിവായി, പാവ ലോകത്തെ ഭൗതിക കാരണങ്ങൾ ഉദ്ധരിക്കുന്നത് ന്യായ വൈകല്യം മാത്രമാണ്. സിലിക്കൺ പദാർത്ഥം കൊണ്ടാണ് ഉണ്ടാക്കപ്പെട്ടിരിക്കുന്നത് എന്ന ഭൗതിക കാരണം, അതിന് പിന്നിലെ, കുട്ടിയുടെ കൈയ്യിൽ മുറിവേൽക്കാതിരിക്കുക എന്ന ഉദ്ദേശ്യ-ലക്ഷ്യപരമായ (Intentional- Purposefull) കാരണത്തെ തെളിയിക്കുകയല്ലെ ചെയ്യുന്നത്? അല്ലാതെ ഈ ഉദ്ദേശ്യ-ലക്ഷ്യപരമായ കാരണത്തെ നിഷേധിക്കാൻ, സിലിക്കൺ പദാർത്ഥം കൊണ്ട് ഉണ്ടാക്കപ്പെട്ടിരിക്കുന്നു എന്ന ഭൗതിക കാരണം എങ്ങനെ ന്യായമാവും ?!
***********************ഇനി വിമർശന വിധേയമായ ഹദീസിലേക്ക് വരാം… ഭൗമിക കാലാവസ്ഥകളുടെ ഭൗതിക കാരണങ്ങൾ വിശകലനം ചെയ്യുകയല്ല ഹദീസ് ചെയ്യുന്നത് എന്ന് വിമർശകർ മനസ്സിലാക്കണം. പ്രകൃതിശാസ്ത്രങ്ങളുടെ ഭൗതിക വിശദീകരണങ്ങൾക്കപ്പുറം അദൃശ്യവും മനുഷ്യർക്ക് അപ്രാപ്യവുമായ ആത്മീയ അന്തസാരങ്ങൾ പങ്കു വെക്കുകയാണ് (ഇസ്ലാം) മതം ചെയ്യുന്നത്. ശാസ്ത്രം, പ്രകൃതി പ്രതിഭാസങ്ങളുടെ ഭൗതിക കാരണങ്ങൾ (material cause) ചർച്ച ചെയ്യുമ്പോൾ, മതം പ്രകൃതി പ്രതിഭാസങ്ങളുടെ അന്തിമ കാരണമാണ് (Final cause) അനാവരണം ചെയ്യുന്നത്; ഒരു പ്രകൃതി പ്രതിഭാസത്തിൻ്റെ ദൈവിക ഉദ്ദേശ്യം അല്ലെങ്കിൽ ദൈവികലക്ഷ്യം… അല്ലെങ്കിൽ ഒരു കാര്യം സ്വാഭാവികമായി വികസിച്ചെത്തുന്ന അതിൻ്റെ അവസാനം, ഇതൊക്കെയാണ് മതത്തിൻ്റെ ചർച്ചാവിഷയം. (കൂടുതൽ വിവരങ്ങൾക്ക്: https://www.snehasamvadam.org/തെറ്റിദ്ധരിക്കപ്പെട്ട-9/)
ഭൗതിക കാരണവും (material cause) അന്തിമ കാരണവും (Final cause) തമ്മിലുള്ള വ്യത്യാസം മനസ്സിലാവാത്തവർ ശാസ്ത്ര പുസ്തകങ്ങളിൽ മനപാഠമാക്കിയ വരികളിൽ നിന്നും “വ്യത്യസ്തമായ” എന്ത് വിശദീകരണങ്ങളും വർണനകളും കണ്ടാലും അതൊക്കെ ശാസ്ത്ര “വിരുദ്ധമായി” തെറ്റിദ്ധരിക്കുന്നു എന്ന് മാത്രം.
വിമർശനവിധേയമായ ഹദീസിനെ മുസ്ലിം സമൂഹവും ആദ്യകാല ഹദീസ് ശാസ്ത്രജ്ഞരും പല രീതിയിലും മനസ്സിലാക്കിയതായി പൗരാണിക ഹദീസ് ഗ്രന്ഥങ്ങളിൽ നിന്നും മനസ്സിലാവുന്നു. വിശദാംശങ്ങളിൽ ശാസ്ത്രത്തിൻ്റെ മേഖലക്ക് അപ്പുറമുള്ള വിവരങ്ങൾ ഉണ്ടാവാമെങ്കിലും ഈ വ്യാഖ്യാനങ്ങളിൽ ഏത് സ്വീകരിച്ചാലും ബുദ്ധിക്കൊ ശാസ്ത്രത്തിനൊ എതിരായി ഒന്നും ഹദീസിൽ കണ്ടെത്താൻ കഴിയില്ല എന്നതാണ് വസ്തുത.
വ്യാഖ്യാനം 1:
ഹദീസ് അനാവരണം ചെയ്യുന്നത് ഭൗമികമായ കാലാവസ്ഥയെ സംബന്ധിച്ച് അല്ലേയല്ല. പ്രത്യുത, നരകത്തിലെ കഠിനമായ ചൂടിൻ്റെയും തണുപ്പിൻ്റെയും ഭീകരത എത്രത്തോളമാണ് എന്ന് ബോധ്യപ്പെടുത്തുകയാണ് ഹദീസിൻ്റെ ഉദ്ദേശ്യം.
അതായത്, വർഷത്തിൽ രണ്ട് തവണ നരകത്തിന് ആശ്വാസം/ശ്വാസം അല്ലാഹു അനുവദിച്ചു. ആ ആശ്വാസ/ശ്വാസ വേളയിൽ നരകത്തിൻ്റെ ചൂടിൻ്റെയും തണുപ്പിൻ്റെയും കാഠിന്യം കുറയുമല്ലൊ. എന്നാൽ നരകത്തിലെ ഈ കാഠിന്യം കുറഞ്ഞ ചൂടും തണുപ്പും പോലും, ഭൗമികമായി ശൈത്യകാലത്തും ഉഷ്ണകാലത്തും നാം അനുഭവിക്കുന്ന ഏറ്റവും തീക്ഷ്ണമായ ചൂടും തണുപ്പിനോടും തുല്യമായിരിക്കും. അഥവാ, ഭൗമിക കാലാവസ്ഥകളിൽ നാം അനുഭവിക്കുന്ന ഏറ്റവും തീക്ഷ്ണമായ ചൂടിനോടും തണുപ്പിനോടും തുല്യമായിരിക്കും നരകത്തിലെ ഏറ്റവും കാഠിന്യം കുറഞ്ഞ ചൂടും തണുപ്പും. എങ്കിൽ നരകത്തിൻ്റെ സ്വാഭാവിക താപവും കുളിരും എത്ര അസഹനീയമായിരിക്കും !!!
فَهْوَ أَشَدُّ ما تَجِدُونَ مِنَ الحَرِّ، وَأَشَدُّ ما تَجِدُونَ مِنَ الزَّمْهَرِيرِ. “അത് നിങ്ങൾ (ഈ കാലാവസ്ഥകളിൽ) അനുഭവിക്കുന്ന തീക്ഷ്ണമായ ചൂടും തണുപ്പുമാണ്.” എന്ന ഹദീസിലെ ഭാഗം ഇതാണ് സൂചിപ്പിക്കുന്നത്.
ഇസ്ലാം വിരോധികളുടെ ഹദീസ് നിരൂപണങ്ങൾ ഉദയം കൊള്ളുന്നതിന് നൂറ്റാണ്ടുകൾക്ക് മുമ്പ് തന്നെ ഇത്തരമൊരു വ്യാഖ്യാനം, മുസ്ലിം ഹദീസ് പണ്ഡിതർ ഹദീസിന് നൽകിയിട്ടുണ്ട് എന്ന് നാം മനസ്സിലാക്കണം. ഉദാഹരണത്തിന്, ഹിജ്രാബ്ദം 476 ൽ (1083 CE) ഭൂജാതനായ കാദി ഇയാദ് എഴുതിയത് കാണുക: “…ഭൗമികമായ ഉഷ്ണകാലത്തെ നരകത്തോട് സദൃശ്യപ്പെടുത്തുകയും ഉപമിക്കുകയുമാണ് ഹദീസ്, അതിനാൽ നരകത്തിൻ്റെ കഠിനമായ ഉഷ്ണത്തെ ഭയപ്പെടുക, അത് വന്നു ഭവിക്കുന്ന അവസ്ഥയെ ഇല്ലാതാക്കുക.” (ശർഹു മുസ്ലിം: ഇമാം നവവി :5:120, ശർഹു സുനനു ഇബ്നു മാജ: സുയൂത്വി: 1:321, ഫത്ഹുൽ ബാരി: ഇബ്നു ഹജ്ർ: 10: 186)
രണ്ട് ശ്വാസങ്ങൾ/ആശ്വാസങ്ങളിലൂടെ, നരകത്തിലെ രണ്ട് ഘട്ടങ്ങളിലുണ്ടാവുന്ന ഏറ്റവും കുറഞ്ഞ ചൂടും തണുപ്പും, ഭൂമിയിലെ ഏറ്റവും കഠിനമായ ഉഷ്ണ കാലത്തെയും ശൈത്യകാലത്തെയും പോലെയായിരിക്കും എന്ന് ചുരുക്കം.
വ്യാഖ്യാനം 2:
“നരകം” ഒരു ഭാഗത്ത് ഉറച്ചു നിൽക്കുന്ന ഒരു അസ്തിത്വത്തെ മാത്രം വിളിക്കുന്ന സാങ്കേതിക പദമല്ല എന്നതാണ് ഇസ്ലാമിക വീക്ഷണം.
1. എല്ലാ ശിക്ഷയെയും ശിക്ഷാ ഹേതുവെയും “നരകം” എന്ന് വിളിക്കാം എന്നാണ് ഹദീസുകളിൽ നിന്ന് മനസ്സിലാവുന്നത്.
അന്യായമായ ഭൂമി അപഹരിച്ചാൽ “നരക”ത്തിൻ്റെ ഒരു കണ്ടമാണ് ഒരാൾ സ്വന്തമാക്കുന്നത് എന്ന് നബി (സ) പറഞ്ഞത് ഉദാഹരണം: فإنَّما أقْطَعُ له قِطْعَةً مِنَ النَّارِ. (സ്വഹീഹുൽ ബുഖാരി: 7169)2. പ്രയാസകരമായ ഭൗമീകാനുഭവങ്ങൾ ഒരു വിശ്വാസിയുടെ പാപങ്ങളെ മായ്ച്ചു കളയാനും ഇല്ലാതാക്കാനും ഉതകുന്നതാണ്. അതിലൂടെ നരകശിക്ഷയിൽ നിന്നും ഒരു വിശ്വാസി മോക്ഷം നേടുന്നു. ഇത്തരം പ്രയാസങ്ങളെയും ക്ലേശങ്ങളെയും (ഉദാഹരണം, കഠിനമായ വെയിൽ, പനി…) “നരകം” എന്നു തന്നെയാണ് ഹദീസുകൾ പരിചയപ്പെടുത്തിയിട്ടുള്ളത്… മനുഷ്യരുടെ പാപങ്ങളെ കരിച്ചു കളയുന്നതായ എല്ലാ ചൂടും പ്രയാസവും ക്ലേശങ്ങളും നരകമാണ്…
لسَّفَرُ قِطْعَةٌ مِنَ العَذَابِ“യാത്ര ശിക്ഷയിൽ നിന്നുള്ള ഒരു കഷ്ണമാണ്…” (സ്വഹീഹുൽ ബുഖാരി: 1804 )
فإنَّ شِدَّةَ الحَرِّ مِن فَيْحِ جَهَنَّمَ “കൊടും ചൂട് നരകത്തിൻ്റെ ചൂടിൽ നിന്നുമുള്ളതാണ്…” (സ്വഹീഹു മുസ്ലിം: 617)
الْحُمَّى مِنْ فَيْحِ جَهَنَّمَ فَابْرُدُوهَا بِالْمَاءِ “പനി നരകത്തിൻ്റെ ചൂടിൽ നിന്നും ഉള്ളതാണ്…” (സ്വഹീഹുൽ ബുഖാരി: 5725)
ഇബ്നുൽ ക്വയ്യിം (ജനനം ഹിജ്രാബ്ദം: 751 : CE 1292) എഴുതി:
“രണ്ടു രീതിയിലാണ് ഈ ഹദീസ് മനസ്സിലാക്കാവുന്നത്:
a) ഭൗമികമായ ക്ലേശങ്ങളും പനി പോലെയുള്ള രോഗങ്ങളുമെല്ലാം നരക ശിക്ഷക്ക് ചെറിയ സാമ്യതയോടെ അല്ലാഹു നിശ്ചയിച്ചത് മനുഷ്യർക്ക് നരകാനുഭവത്തിൻ്റെ ഒരു ലഘുവായ അനുഭൂതി മനസ്സിലാക്കാനും പാഠം ഉൾക്കൊള്ളാനുമാണ്. (Final cause – ലേ)
ഈ ഉദ്ദേശ്യത്തെ അടിസ്ഥാനപ്പെടുത്തി പിന്നീട് അതിന് അനുയോജ്യമായ ഭൗതിക കാരണങ്ങൾ (meterial cause -ലേ) അല്ലാഹു പനിക്കും രോഗത്തിനും നിർണയിച്ചു.
ആശ്വാസം, വിടുതി , സന്തോഷം, ആസ്വാദനങ്ങൾ എന്നിവ പരലോകത്തെ സ്വർഗത്തിൻ്റെ ഉള്ളടക്കങ്ങളാണ്. എന്നാൽ അവയെ കുറിച്ച് മനസ്സിലാക്കാനും ഉൾകൊള്ളാനും സങ്കൽപ്പിക്കാനും മനുഷ്യർക്ക് കഴിയാനായി അതിന് അനുയോജ്യമായ സുഖ-സന്തോഷാനുഭവങ്ങളും ഭൂമിയിൽ അല്ലാഹു നിശ്ചയിച്ചു.
b) പനിയുടെ രൂക്ഷതയെ നരകത്തിൻ്റെ കാഠിന്യവുമായി ഉപമിക്കുകയാണ് ഹദീസ് ചെയ്തത്. നരക ശിക്ഷയെ കുറിച്ച ബോധം നമ്മിൽ അങ്കുരിക്കപ്പെടുകയാണ് ഈ ഉപമയിലൂടെ നബി (സ) ഉദ്ദേശിച്ചത്. ( അല്ലാതെ പനിയുടെ ഭൗതിക കാരണം (meterial cause) വിശദീകരിക്കുകയല്ല.) (അത്ത്വിബ്ബുന്നബവിയ്: ഇബ്നുൽ ക്വയ്യിം: 21)
ഇബ്നു ഹജർ (ജനനം ഹിജ്രാബ്ദം: 773 : CE 1371) എഴുതി:
“പനിയെ നരകത്തോട് ചേർത്തി പറഞ്ഞതിൻ്റെ വ്യാഖ്യാനം പലതാണ്: പനി അക്ഷരാർത്ഥത്തിൽ നരകത്തിൽ നിന്നുള്ളതാണ് എന്നാണ് ഒരു വ്യാഖ്യാനം. പനിയിലൂടെ ശരീരത്തിൽ ഉളവാകുന്ന താപം, മനുഷ്യർ നരകത്തെ ലഘുവായി അനുഭവിക്കാനും ഉൽബുദ്ധരാവാനും വേണ്ടി അല്ലാഹു നിശ്ചയിച്ചതാണ്. ആ ഉദ്ദേശ്യത്തിൻ്റെ അടിസ്ഥാനത്തിൽ അല്ലാഹു ഭൗതികമായ കാരണങ്ങൾ പനിക്ക്, പിന്നീട് നിശ്ചയിച്ചു നൽകി… ബസ്സാർ തൻ്റെ മുസ്നദിൽ പ്രവാചക പത്നി ആഇശയിൽ (റ) നിന്ന് നിവേദനം ചെയ്തതായി വന്ന ഹദീസിലും, ഇമാം അഹ്മദ്, ത്വബ്റാനി എന്നിവർ പ്രവാചകാനുചരന്മാരായ ഇബ്നു മസ്ഊദ്, അബൂ ഉമാമ എന്നിവരിൽ നിന്നും ഉദ്ധരിച്ച ഹദീസിൽ ഇപ്രകാരം വ്യക്തമായി പ്രസ്ഥാവിക്കപ്പെട്ടിരിക്കുന്നു: الحمى حظ المؤمن من النار. “പനി, വിശ്വാസിക്ക് നരകത്തിൽ നിന്നും നൽകപ്പെടുന്ന ഒരു വിഹിതമാണ്.”…” (പാപങ്ങൾ പൊറുക്കപ്പെടാനുള്ള കാരണമായി പനിയെ പരിഗണിക്കുകയും നരക ശിക്ഷയുടെ വിഹിതം പനിയായി ലഘൂകരിച്ച് ഇഹലോകത്ത് തന്നെ അല്ലാഹു നൽകുകയും ചെയ്യുന്നു. അപ്പോൾ പനി അക്ഷരാർത്ഥത്തിൽ നരകത്തിൽ നിന്നും ഉള്ളതാണെന്ന് പറയാമല്ലൊ.” (ഫത്ഹുൽ ബാരി: ഇബ്നു ഹജ്ർ: 10: 186)
പനി എന്തുകൊണ്ട് ചൂടുള്ളതായി ? എന്ന ചോദ്യത്തിന് പല കാരണങ്ങളും പല വീക്ഷണകോണിൽ നിന്ന് കൊണ്ട് പറയാവുന്നതാണ്. പനിയുടെ ഭൗതിക കാരണം (meterial cause), ഹദീസിൽ പറയപ്പെട്ട ആത്മീയ കാരണത്തോട് ഒരിക്കലും വിരുദ്ധമാവുന്നില്ല. ഹദീസ് സംസാരിക്കുന്നത് പനിയുടെ അവസാന കാരണം അല്ലെങ്കിൽ അന്തിമ കാരണത്തെ (Final cause) സംബന്ധിച്ചാണ് എന്ന് എത്രയോ നൂറ്റാണ്ടുകൾക്ക് മുമ്പ് തന്നെ ഇമാം ഇബ്നുൽ ക്വയ്യിമും ഇബ്നു ഹജറും വ്യക്തമാക്കിയിരിക്കുന്നു !!
ഈ കാരണങ്ങൾ തമ്മിൽ വ്യത്യസ്തമാണ് എങ്കിലും പരസ്പര വിരുദ്ധമല്ല. പനിയുടെ ഭൗതിക കാരണങ്ങളെ സംബന്ധിച്ച തുടർച്ചയായ അന്വേഷണങ്ങളിലൂടെ ഇത് ഏതൊരാൾക്കും മനസ്സിലാക്കാവുന്നതാണ്:
എന്തു കൊണ്ട് നമ്മെ പനി ബാധിക്കുന്നു?
അണുബാധ മൂലം ശരീര താപനിലയിലെ താൽക്കാലിക വർദ്ധനവാണ് പനി. ശരീരത്തിൻ്റെ പ്രതിരോധ സംവിധാനത്തിൽ നിന്നുള്ള മൊത്തത്തിലുള്ള പ്രതികരണത്തിൻ്റെ ഒരു ഭാഗമാണിത്. ( https://www.mayoclinic.org/diseases-conditions/fever/symptoms-causes/syc-20352759 )
എങ്കിൽ, പനിക്ക് എന്തുകൊണ്ട് ചൂട് ?
സാധാരണയായി വൈറസ്, ബാക്റ്റീരിയ ബാധയെ പ്രതിരോധിക്കാനും മുക്തമാക്കാനും വേണ്ടിയാണ് ശരീരം താപനിലയെ താൽക്കാലിക വർദ്ധിപ്പിക്കുന്നത്.
( https://www.mayoclinic.org/diseases-conditions/fever/symptoms-causes/syc-20352759 )
എങ്കിൽ, വൈറസ്, ബാക്റ്റീരിയ ബാധയെ ചൂട് എന്തു കൊണ്ട് പ്രതിരോധിക്കുന്നു?
വൈറസ് ബാധ മൂലമാണ് വൈറൽ പനി ഉണ്ടാകുന്നത്. വൈറസുകൾ വളരെ ചെറിയ പകർച്ചവ്യാധി ഏജൻ്റുകളാണ്. അവ നിങ്ങളുടെ ശരീരത്തിലെ കോശങ്ങളെ ബാധിക്കുകയും പെരുകുകയും ചെയ്യുന്നു. വൈറസിനെതിരെ പോരാടാനുള്ള നിങ്ങളുടെ ശരീരത്തിൻ്റെ മാർഗമാണ് പനി. പല വൈറസുകളും താപനിലയിലെ മാറ്റങ്ങളോട് സംവേദനക്ഷമമാണ്, അതിനാൽ നിങ്ങളുടെ ശരീര താപനിലയിലെ പെട്ടെന്നുള്ള വർദ്ധനവ് നിങ്ങളെ വൈറസുകൾക്ക് കോശങ്ങളിലുള്ള ആതിഥ്യം തടസ്സപ്പെടുത്തുന്നു.
(https://www.healthline.com/health/viral-fever)
എങ്കിൽ, വൈറസുകൾ എന്തുകൊണ്ട് താപനിലയിലെ മാറ്റങ്ങളോട് സംവേദനക്ഷമമായി ?
വൈവിധ്യമാർന്ന വൈറസുകൾ വ്യത്യസ്തമായ താപനില സംവേദനം പ്രകടിപ്പിക്കുന്നു. ഓരോ വൈറസും ആതിഥേയ കോശത്തിൽ ഉൽപ്പാദിപ്പിക്കുന്ന മിക്ക പ്രോട്ടീനുകളും താപനില സംവേദനം പ്രകടിപ്പിക്കുന്നു.
എന്തു കൊണ്ട് ?
the mechanism of temperature sensitivity is not known…
താപനില സംവേദനക്ഷമതയുടെ സംവിധാനം ഇതു വരെ നമുക്ക് അറിയില്ല. പക്ഷേ വൈറൽ എൻസൈമുകളുടെ ഘടനയിലെ ചെറിയ മാറ്റങ്ങളുമായി ബന്ധപ്പെട്ടതായിരിക്കാം അത്…
(https://www.ncbi.nlm.nih.gov )
ശാസ്ത്രമെത്ര വികസിച്ചാലും, പ്രകൃതി പ്രതിഭാസങ്ങളുടെ വികാസത്തിലെ ഒരു ഘട്ടത്തിലെ ഭൗതിക കാരണം (meterial cause) മാത്രം വിശദീകരിക്കാനെ അതിന് കഴിയു. അതിൻ്റെ ആത്യന്തികമായ കാരണം (അഥവാ ദൈവിക ലക്ഷ്യം) വിശദീകരിക്കാൻ ശാസ്ത്രത്തിന് കഴിയില്ല. കാരണം ശാസ്ത്രത്തിൻ്റെ വിഷയത്തിനും പരിമിതിക്കും അപ്പുറമാണത്.
എന്തുകൊണ്ട് പനി ചൂടായി. തണുപ്പൊ മറ്റെന്തെങ്കിലും ആവാമായിരുന്നില്ലെ. വൈറസുകളിൽ എന്തുകൊണ്ട് ചൂടിനോട് തന്നെ സംവേദനമുണ്ടായി? എന്തുകൊണ്ട് എന്നതിനുള്ള ഉത്തരം മനുഷ്യ ശരീരത്തെയും വൈറസിനെയും സൃഷ്ടിച്ച ദൈവത്തിനല്ലെ പറയാനാകു? ദൈവത്തിൻ്റെ ഉദ്ദേശ്യം അല്ലെങ്കിൽ ലക്ഷ്യമാണ് ആത്യന്തികമായി ഒരു പ്രതിഭാസത്തിൻ്റെ ഏറ്റവും അടിസ്ഥാനപരമായ പ്രകൃതത്തെ നിർണയിക്കുക.
ആ ദൈവം പറയുന്നു… നരകത്തെ നിങ്ങൾ ഓർക്കാനും, പാപങ്ങൾ പൊറുത്തു നൽകാനും വേണ്ടി നരകച്ചൂടിൻ്റെ ഒരു പൊട്ട് ചൂട് നിങ്ങൾക്ക് നൽകാനാണ് പനി ചൂടായി നിശ്ചയിച്ചത്. ഈ ദൈവിക ഉദ്ദേശ്യത്തെ ഭൗതികമായി നടപ്പാക്കാൻ ഭൗതികമായ കാരണങ്ങൾ ദൈവം നിശ്ചയിക്കുകയും ചെയ്തു. ആ ഭൗതികാരണമാണ് ശാസ്ത്രത്തിലൂടെ മനുഷ്യർ കണ്ടെത്തുന്നത്. അതിനു പിന്നിലെ ദൈവിക ഉദ്ദേശ്യമാണ് മതത്തിലൂടെ മനുഷ്യർ കണ്ടെത്തുന്നത്. ഈ രണ്ട് കാരണങ്ങളും ആശയപരമായി പരസ്പരം എതിരല്ല. അവ രണ്ടും തമ്മിൽ വ്യത്യസ്തമായിരിക്കാം, പക്ഷെ തമ്മിൽ തെല്ലും വൈരുധ്യമില്ല.
സമാനമായ രീതിയിൽ തന്നെയാണ്, മറ്റു പ്രതിഭാസങ്ങളുടെ ആത്മീയ വ്യാഖ്യാനങ്ങളെ മനസ്സിലാക്കാൻ.
“മലക്ക് മേഘങ്ങളെ തെളിച്ചു കൊണ്ടുപോകുമ്പോളുള്ള ശബ്ദവും വെളിച്ചവുമാണ് ഇടിമിന്നൽ…” എന്ന് ചില നിവേദനങ്ങൾ കാണാം. ഹദീസ് സ്വഹീഹാണെന്ന് വന്നാൽ തന്നെ ഇടിമിന്നൽ എന്ന പ്രതിഭാസത്തിൻ്റെ ഭൗതിക കാരണം വിശദീകരിക്കുകയല്ല ഹദീസിലെ ഉദ്ദേശ്യം. മറിച്ച് പ്രകൃതി പ്രതിഭാസങ്ങളുടെ ഭൗതിക കാരണങ്ങൾക്കപ്പുറം അഭൗതികവും ആത്യന്തികവുമായ ദൈവിക ഉദ്ദേശ്യമാണ് ഹദീസിൽ വർണിക്കപ്പെടുന്നത്.
ഇടിയുടെയും മിന്നലിൻ്റെയും ഭൗതിക കാരണങ്ങളുടെ (Meterial Cause) അറ്റം തേടി പോയാൽ ഭൗതിക കാരണങ്ങൾക്കപ്പുറം മറ്റെന്തെങ്കിലുമൊരു കാരണം ആവശ്യമായി വരുന്ന ഘട്ടം എത്തുന്നുണ്ട്. ഇത് എല്ലാ പ്രകൃതി പ്രതിഭാസങ്ങളുടെ കാര്യത്തിലും കണ്ടെത്താനാവും. ഉദാഹരണമായി ഇടി-മിന്നൽ ചർച്ച ചെയ്യാം.
എന്താണ് (What) മിന്നൽ ?
മേഘങ്ങൾ, വായു, അല്ലെങ്കിൽ ഭൂമി എന്നിവയ്ക്കിടയിലുള്ള അന്തരീക്ഷത്തിൽ വൈദ്യുതിയുടെ ഒരു ഭീമാകാരമായ തീപ്പൊരിയാണ് മിന്നൽ.
(https://www.nssl.noaa.gov/education/svrwx101/lightning)
എങ്ങനെയാണ് (How) വൈദ്യുതി ഉടലെടുക്കുന്നത്?
ഈ പ്രകൃതി പ്രതിഭാസത്തിൻ്റെ വികസനത്തിൻ്റെ പ്രാരംഭ ഘട്ടത്തിൽ, മേഘത്തിലെ പോസിറ്റീവ്, നെഗറ്റീവ് ചാർജുകൾക്കിടയിലും മേഘത്തിനും ഭൂമിക്കും ഇടയിൽ വായു ഒരു ഇൻസുലേറ്ററായി പ്രവർത്തിക്കുന്നു; ചാർജുകളിലെ വ്യത്യാസം വളരെ വലുതാകുമ്പോൾ, വായുവിൻ്റെ ഈ ഇൻസുലേറ്റിംഗ് കപ്പാസിറ്റി തകരുകയും വൈദ്യുതിയുടെ ദ്രുതഗതിയിലുള്ള പുറം തള്ളൽ ഉണ്ടാകുകയും ചെയ്യുന്നു, ഇതിനെ മിന്നൽ എന്നറിയപ്പെടുന്നു. (https://www.weather.gov/safety/lightning-science-overview)
എങ്കിൽ എങ്ങനെയാണ് (How) ഈ പോസിറ്റീവ്, നെഗറ്റീവ് ചാർജുകൾ മേഘങ്ങളിൽ ഉണ്ടാവുന്നത്?
മേഘങ്ങൾക്കുള്ളിൽ, ജലബാഷ്പം ചുറ്റി സഞ്ചരിക്കുന്നത് തുടരുന്നു. ജലതന്മാത്രകൾ പരസ്പരം കൂട്ടിമുട്ടുകയും പരസ്പരം ഉരസുകയും ഇലക്ട്രോണുകൾ പുറം തള്ളുകയും ചെയ്യും, അതാണ് വൈദ്യുതി: ഇലക്ട്രോണുകൾ. ചാർജുകളുടെ ശേഖരണത്തോടെ മേഘവും ചാർജ്ജ് ഉള്ളതായി പരിണമിക്കുന്നു, മുകളിൽ പോസിറ്റീവ് ചാർജും അടിയിൽ നെഗറ്റീവ് ചാർജും. (https://scied.ucar.edu/learning-zone/storms/thunder-and-lightning)
മേഘങ്ങളുടെ അടിത്തട്ടിലുള്ള നെഗറ്റീവ് ചാർജുകൾ ഉപരിതലത്തിലെ പോസിറ്റീവ് ചാർജിലേക്ക് ആകർഷിക്കപ്പെടുന്നു. വിപരീത ചാർജ്ജുള്ള വസ്തുക്കൾ inverse-square law അനുസരിച്ച് ആകർഷിക്കപ്പെടുന്നുവെന്ന് കൂലോംബിൻ്റെ നിയമം (Coulomb’s law) പഠിപ്പിക്കുന്നു. (https://www.scu.edu/illuminate/thought-leaders/phil-kesten/when-lightning-strikes.)
എന്തുകൊണ്ട് (Why) പോസിറ്റീവ്, നെഗറ്റീവ് ചാർജുകൾ തമ്മിൽ ആകർഷിക്കപ്പെടുന്നു ?
ഒരു പോസിറ്റീവ് ചാർജിൽ അധിക പ്രോട്ടോണുകൾ അടങ്ങിയിരിക്കുന്നു, അതേസമയം നെഗറ്റീവ് ചാർജിൽ കുറഞ്ഞ പ്രോട്ടോണുകളാണ് അടങ്ങിയിട്ടുള്ളത്. പോസിറ്റീവ് ചാർജിനെ നെഗറ്റീവ് ചാർജിനോട് അടുപ്പിക്കുമ്പോൾ, പോസിറ്റീവ് ചാർജിൽ നിന്നുള്ള അധിക സംഖ്യ പ്രോട്ടോണുകൾ നെഗറ്റീവ് ചാർജിലേക്ക് മാറ്റപ്പെടും, അങ്ങനെ രണ്ട് ചാർജുകളിലെയും പ്രോട്ടോണുകളുടെ എണ്ണം സന്തുലിതമാകും. ഒരു പോസിറ്റീവ് ചാർജിൽ നിന്ന് നെഗറ്റീവ് ചാർജിലേക്കുള്ള പ്രോട്ടോണുകളുടെ ഈ വെർച്വൽ ട്രാൻസ്ഫർ അവയ്ക്കിടയിൽ ആകർഷകമായ ബലത്തിന് കാരണമാകുന്നു.
എങ്കിൽ, നെഗറ്റീവ് പോസറ്റീവ് ചാർജുകൾക്കിടയിലെ ആകർഷണീയതയെ വിശദീകരിക്കുക മാത്രമാണ് പ്രോട്ടോണുകളുടെ സംഖ്യാ സന്തുലിതവൽക്കരം വിവരിക്കുന്നതിലൂടെ ചെയ്യുന്നുള്ളു. എന്തുകൊണ്ടാണ് (Why) നെഗറ്റീവ് പോസറ്റീവ് ചാർജുകൾക്കിടയിലെ ആകർഷണീയത ? പോസിറ്റീവ് ചാർജിൽ നിന്നുള്ള അധിക സംഖ്യ പ്രോട്ടോണുകൾ നെഗറ്റീവ് ചാർജിലേക്ക് മാറ്റപ്പെടുന്നത് എന്തു കൊണ്ടാണ് (Why)?
ഭൗതികശാസ്ത്ര ചർച്ചകൾ കൊണ്ട് സജീവമായ ഒരു വിഖ്യാത വെബ്സൈറ്റിൽ ഈ, “എന്തു കൊണ്ട് (Why)?” എന്ന ചോദ്യത്തിന് നൽകപ്പെട്ട ചില മറുപടികൾ രസകരമാണ്:
മറുപടി 1:
“മൗലികശക്തികൾ പോലുള്ളവ എന്തിനാണ് നിലനിൽക്കുന്നതെന്ന് വിശദീകരിക്കാൻ ഭൗതികശാസ്ത്രം ശ്രമിക്കുന്നില്ല, അവ എങ്ങനെ പെരുമാറുന്നുവെന്ന് വിശദീകരിക്കാൻ മാത്രമാണ് ഭൗതികശാസ്ത്രം ശ്രമിക്കുന്നത്. ഈ ശക്തികൾ സ്പേയ്സിൻ്റെ വിവിധ symmetryകൾ മൂലമാണെന്ന് നമുക്ക് പറയാം. ഉദാഹരണത്തിന്, വൈദ്യുത ചാർജ് ഒരു U(1) U(1) gauge symmetry യുമായി ബന്ധപ്പെട്ടിരിക്കുന്നു എന്ന് വേണമെങ്കിൽ പറയാം. എന്നാൽ ആ പ്രത്യേക symmetry നിലനിൽക്കുന്നത് എന്തുകൊണ്ടാണെന്ന് അപ്പോഴും വിശദീകരിക്കപ്പെടുന്നില്ല.”
മറുപടി 1:
“എന്തുകൊണ്ട്” എന്ന ചോദ്യത്തിനുള്ള ഉത്തരം നൽകാൻ ഭൗതികശാസ്ത്രം ശ്രമിക്കുന്നില്ലെങ്കിലും, ഒരു പാർശ്വഫലമായി അത് പലപ്പോഴും “എന്തുകൊണ്ട്” എന്നത് വിശദീകരിക്കാറുണ്ട്. “എന്തുകൊണ്ട്” എന്നാൽ “എന്താണ് കാരണം”, ഭൗതികശാസ്ത്രം സംഭവങ്ങൾ തമ്മിലുള്ള ബന്ധത്തെ വിവരിക്കുന്ന ശാസ്ത്രമാണ്. അതിനാൽ, അത് കാരണങ്ങളെ (cause) വിവരിക്കുന്നത്, ദാർശനിക അർത്ഥത്തിലല്ല, പ്രായോഗിക അർത്ഥത്തിൽ മാത്രമാണ്…”
മറുപടി 3:
“സ്കൂളിൽ, നാം ലളിതമായിട്ടാണ് ശാസ്ത്രത്തെ കുറിച്ച് പഠിച്ചത്. തത്ത്വചിന്തകർ “ജീവതത്ത്വശാസ്ത്രമായ” (ontological) അർത്ഥം എന്ന് വിളിക്കുന്ന, പ്രപഞ്ചത്തെക്കുറിച്ചുള്ള സത്യം വിശദീകരിക്കുന്നതുപോലെയാണ് നാം ശാസ്ത്രത്തെക്കുറിച്ച് സ്കൂളിൽ സംസാരിച്ചിരുന്നത്. വാസ്തവത്തിൽ, ഭൗതികശാസ്ത്രം എന്നത് കാര്യങ്ങൾ വിശദീകരിക്കുകയും കാര്യങ്ങൾ പ്രവചിക്കുകയും ചെയ്യുന്ന മാതൃകകൾ ഉണ്ടാക്കുകയും ചെയ്യുന്ന ദൗത്യമാണ്…”
4. “It just so happens that…”
“എന്തൊ അങ്ങനെയാണ് രണ്ട് ചാർജുകളുടെയും പ്രകൃതി…”
(https://physics.stackexchange.com/questions/535448/why-do-positive-charges-attract-negative-charges )
മുകളിൽ വിവരിച്ച അന്വേഷണങ്ങളിലെ ചോദ്യങ്ങൾ ഒരു തവണ കൂടി ശ്രദ്ധിച്ചു നോക്കൂ…
എന്താണ് (What) മിന്നൽ ?
എങ്ങനെയാണ് (How) വൈദ്യുതി ഉടലെടുക്കുന്നത്?
എങ്കിൽ എങ്ങനെയാണ് (How) ഈ പോസിറ്റീവ്, നെഗറ്റീവ് ചാർജുകൾ മേഘങ്ങളിൽ ഉണ്ടാവുന്നത്?
എന്തുകൊണ്ട് (Why) പോസിറ്റീവ്, നെഗറ്റീവ് ചാർജുകൾ തമ്മിൽ ആകർഷിക്കപ്പെടുന്നു ?
എന്ത് (what)? എന്ന ചോദ്യത്തിലൂടെ തുടങ്ങി, എങ്ങനെ (How)? എന്ന ചോദ്യത്തിൽ ഭൗതിക ശാസ്ത്ര വിശദീകരണങ്ങൾ അവസാനിക്കുന്നു. എന്തുകൊണ്ട് (Why)? എന്ന ചോദ്യം എത്തിയപ്പോഴേക്കും തത്ത്വശാസ്ത്രത്തെയും (Philosophy) ജീവതത്ത്വശാസ്ത്രത്തെയും (ontology) കുറിച്ചൊക്കെ സംസാരിക്കേണ്ടതൊ വിമർശിക്കേണ്ടതൊ ആയി വരുന്നു. അല്ലെങ്കിൽ ഈ എങ്ങനെ (How)? തന്നെയാണ് എന്തുകൊണ്ട് (Why)? എന്ന് അങ്ങ് “വിശ്വസി”ച്ച് നാസ്തികരെ പോലെ സമാധാനമടയാൻ ശ്രമിക്കുക എന്ന ദുർഗതിയിലെത്തുന്നു.
ഇനി ഇടിവെട്ടിനെ കുറിച്ച് ഭൗതിക ശാസ്ത്ര ചർച്ചയിലേക്ക് വന്നാലും ഇടിവെട്ട് ഏറ്റവനെ പാമ്പ് കടിച്ച അവസ്ഥയായിരിക്കും നാസ്തികർക്ക് !
എന്താണ് (What) ഇടി ?
മിന്നൽ മൂലമുണ്ടാകുന്ന ശബ്ദമാണ് ഇടി.
മിന്നൽ എങ്ങനെയാണ് (How) ഈ ശബ്ദം ഉണ്ടാക്കുന്നത്?
മിന്നൽ വായുവിലൂടെ കടന്നുപോകുമ്പോഴാണ് ഇടിമുഴക്കം ഉണ്ടാകുന്നത്. മിന്നൽ, വായുവിനെ പൊടുന്നനെ ചൂടാക്കുകയും വായു വികസിക്കുകയും ചെയ്യുന്നു. മിന്നൽ ചാലിലെ വായുവിൻ്റെ താപനില 50,000 ഡിഗ്രി ഫാരൻഹീറ്റിലെത്തിയേക്കാം, സൂര്യൻ്റെ ഉപരിതലത്തേക്കാൾ 5 മടങ്ങ് ചൂടാണിത്. ഫ്ലാഷിനുശേഷം, വായു തണുക്കുകയും വേഗത്തിൽ ചുരുങ്ങുകയും ചെയ്യുന്നു. ഈ ദ്രുതഗതിയിലുള്ള വികാസവും സങ്കോചവും, നാം ഇടിമുഴക്കം പോലെ കേൾക്കുന്ന ശബ്ദ തരംഗത്തെ സൃഷ്ടിക്കുന്നു. (https://www.weather.gov/safety/lightning-science-thunder)
മിന്നൽ ചൂടാക്കുമ്പോൾ വായു എങ്ങനെയാണ് (How) വികസിക്കുന്നത് ?
കാരണം, വായുവിൻ്റെ കണികകൾ ചൂടാകുമ്പോൾ, പരസ്പരം അകന്നുപോകുകയും കൂടുതൽ ഇടത്തിലേക്ക് വ്യാപിക്കുകയും ചെയ്യുന്നു. ചൂടാകുമ്പോൾ വായു കൂടുതൽ ഇടം പിടിക്കുന്നുവെന്ന് നമുക്ക് പറയാം. കണികകൾ പരസ്പരം അകന്നു പോകുന്നതിനാൽ, വായുവിൻ്റെ സാന്ദ്രത കുറയുകയും ഭാരം കുറയുകയും ചെയ്യുന്നു. തൽഫലമായി, ചൂടുള്ള വായു ഉയരുന്നു. നേരെമറിച്ച്, തണുപ്പിക്കുമ്പോൾ വായുവിൻ്റെ കണികകൾ പരസ്പരം അടുക്കുന്നു. ഇക്കാരണത്താൽ, വായുവിൻ്റെ അളവ് കുറയുന്നു, വായു കൂടുതൽ സാന്ദ്രവും ഭാരമുള്ളതുമാകുന്നു. (https://www.toppr.com/guides/science/winds-storms-and-cyclones/air-expands-on-heating/)
ചൂടാക്കുമ്പോൾ തന്മാത്രകൾ വേഗത്തിൽ നീങ്ങുകയും ചലിക്കുകയും ചെയ്യും. അവ ചലിക്കുമ്പോൾ പരസ്പരം അകന്നുപോകുകയും ചെയ്യുന്നു. അതിനാൽ, മറ്റ് പദാർത്ഥങ്ങളെപ്പോലെ വായുവും ചൂടാക്കുമ്പോൾ വികസിക്കുകയും തണുപ്പിക്കുമ്പോൾ ചുരുങ്ങുകയും ചെയ്യുന്നു. തന്മാത്രകൾക്കിടയിൽ കൂടുതൽ ഇടം ഉള്ളതിനാൽ, വായുവിന് ചുറ്റുമുള്ള പദാർത്ഥത്തേക്കാൾ സാന്ദ്രത കുറവാണ്, ചൂടുള്ള വായു മുകളിലേക്ക് പൊങ്ങിക്കിടക്കുന്നു. (https://www.grc.nasa.gov)
എന്തുകൊണ്ടാണ് (Why) ചൂട് തന്മാത്രകളെ ചലിപ്പിക്കുന്നത്?
താപനില കൂടുന്നതിനനുസരിച്ച് കണികകൾ ഗതികോർജ്ജം (kinetic energy) നേടുകയും വേഗത്തിൽ നീങ്ങുകയും ചെയ്യുന്നു. (https://www.education.vic.gov)
താപനില കൂടുന്നതിനനുസരിച്ച് കണികകൾ ഗതികോർജ്ജം നേടുന്നു എന്ന വാചകവും, താപനില തന്മാത്രകളെ ചലിപ്പിക്കുന്നു എന്ന വാചകവും ഒന്നു തന്നെയാണ്. ആദ്യം പറഞ്ഞ കാര്യത്തെ രണ്ടാമത്തെ വാചകത്തിൽ സാങ്കേതിക പദം ഉപയോഗിച്ച് പറഞ്ഞു എന്ന് മാത്രം ! ചോദ്യം അപ്പോഴും അവശേഷിക്കുന്നു.
താപനില കൂടുന്നത് എന്തുകൊണ്ട് (Why) ചലനത്തെ ത്വരിതപ്പെടുത്തുന്നു?!
ഉത്തരം വിശ്വാസികളെ സംബന്ധിച്ചിടത്തോളം ലളിതമാണ്. പ്രകൃതി പ്രതിഭാസങ്ങളുടെ ഭൗതിക കാരണങ്ങൾ ശാസ്ത്രം വിശദീകരിക്കട്ടെ. എന്നാൽ പ്രകൃതി പ്രതിഭാസങ്ങളെല്ലാം ആശ്രിത വസ്തുക്കളാണ് (contingent). ഒന്ന് മറ്റൊന്നിനെ ആശ്രയിച്ചാണ് പ്രപഞ്ചം മുഴുവൻ നിലനിൽക്കുന്നത്. പല നിലക്കും ആവാമായിരുന്ന പ്രപഞ്ചത്തിലെ വസ്തുക്കൾ/പ്രതിഭാസങ്ങൾ എന്തുകൊണ്ട് നിലവിലെ സ്ഥിതിയിൽ തന്നെ ആയി ?! ഋതുക്കളും പനിയും ഇടിയും മിന്നലുമെല്ലാം എങ്ങനെയും ആകാമായിരുന്നു. തീർത്തും ഇല്ലാതിരിക്കുകയും ചെയ്യാമായിരുന്നു. എന്നിട്ടും അവ ഉണ്ടാവാനും നിലവിലെ പ്രകൃതിയിൽ ആകാനുമുള്ള കാരണമെന്താണ്?! അതിനുള്ള ഉത്തരം നിരാശ്രിതമായ ഒരു അസ്തിത്വം (ദൈവം) അവയെയെല്ലാം പ്രത്യേക ലക്ഷ്യത്തോടെ, താൻ ഉദ്ദേശിക്കുന്ന രൂപത്തിലാണ് ഉണ്ടാക്കിയത് എന്നതാണ്. ഈ പ്രകൃതി പ്രതിഭാസങ്ങളിലെ ദൈവിക ഉദ്ദേശ്യങ്ങളെ സംബന്ധിച്ചാണ് മതവും ഹദീസുകളും എല്ലാം സംസാരിക്കുന്നത്.
വ്യാഖ്യാനം 3:
സൂര്യൻ നരകത്തിൻ്റെ ഭാഗമായി പരിണമിക്കപ്പെടും എന്ന് മുഹമ്മദ് നബി (സ) വ്യക്തമാക്കിയിട്ടുണ്ട്. إن الشمس والقمر ثوران في النار يوم القيامة സൂര്യനും ചന്ദ്രനും നരകത്തിൻ്റെ ഭാഗമാക്കപ്പെടും എന്ന് വ്യക്തമായി ഹദീസിൽ വിശദീകരിക്കപ്പെട്ടിട്ടുണ്ട്. (ബസ്സാർ: 15: 243, സിൽസിലതു സ്വഹീഹ: 1: 242)
അപ്പോൾ, ഫലത്തിൽ സൂര്യനിൽ നിന്നുള്ള സ്വാധീനങ്ങൾ നരകത്തിൽ നിന്നുള്ള പ്രതിഫലനങ്ങളായി പറയാവുന്നതാണ്. ഈ ഭാഷാ പ്രയോഗത്തിനെ അറബി അലങ്കാരശാസ്ത്രത്തിൽ പരിചയപ്പെടുത്തപ്പെടുക اعتبار ما يكون എന്നാണ്. (വസ്തു എന്തായി മാറുമെന്നു പരിഗണിച്ച് അതിനെ പരിചയപ്പെടുത്തുക.) (കിതാബു ഇൽമുൽ ബയാൻ: അൽ മജാസുൽ മുർസൽ: അബ്ദുൽ അസീസ് അതീക്: പേജ്: 161)
സൂര്യനാണല്ലൊ ഭൗമിക താപത്തിൻ്റെ കേന്ദ്രം. സൂര്യനും നരകമാണെങ്കിൽ, ഋതുക്കളുടെ കാരണം നരകമാണെന്ന് സ്വഭാവികമായും പറയാവുന്നതാണല്ലൊ.
ഭൂമിയുടെ ഭ്രമണമാണല്ലൊ രാപ്പകലുകളുടെ കാരണം (Cause). രാത്രിയിൽ സൂര്യൻ ദൈവ സിംഹാസനത്തിനടിയിൽ സുജൂദ് (സാഷ്ടാംഗം) ചെയ്യാൻ പോവുകയാണ് എന്ന് ഹദീസിൽ വന്നിരിക്കുന്നത് അശാസ്ത്രീയത അല്ലെ?
……………………………വിമർശന വിധേയമായ ഹദീസ്:
അബൂ ദർറ് (റ) പറഞ്ഞു: സൂര്യൻ അസ്തമിച്ചപ്പോൾ പ്രവാചകൻ (സ) അബൂ ദർറിനോട് (റ) ചോദിച്ചു: എവിടേക്കാണ് സൂര്യൻ പോകുന്നത് എന്ന് താങ്കൾക്കറിയാമോ? ഞാൻ (അബൂ ദർറ് (റ) ) പറഞ്ഞു: അല്ലാഹുവും അവന്റെ ദൂതനുമാണ് ഏറ്റവും കൂടുതൽ അറിയുന്നത്. പ്രവാചകൻ (സ) പറഞ്ഞു: ദൈവത്തിൻ്റെ സിംഹാസനത്തിന് കീഴിൽ സാഷ്ടാംഗം ചെയ്യാനാണ് അതിൻ്റെ പ്രയാണം. എന്നിട്ട് അത് സമ്മതം ചോദിക്കും, അപ്പോൾ സമ്മതം നൽകപ്പെടും. (സമ്മതം നൽകപ്പെടുകയും “നീ വന്നിടത്തേക്ക് മടങ്ങുകയും ചെയ്യുക” എന്ന് പറയപ്പെടുകയും ചെയ്താൽ അത് മടങ്ങുകയും ഉദയസ്ഥാനത്തു തന്നെ ഉദിക്കുകയും ചെയ്യും.) അത് സാഷ്ടാംഗം ചെയ്യുകയും സമ്മതം നൽകപ്പെടാതിരിക്കുകയും ചെയ്യുന്ന ഒരു കാലം (ലോകാവസാനം) അടുത്തിരിക്കുന്നു. അന്ന് അതിനോട് പറയപ്പെടും: നീ വന്നിടത്തേക്ക് മടങ്ങുക. (പടിഞ്ഞാറ് നിന്ന് ഉദിക്കുകയും ചെയ്യുക.) അപ്പോൾ അത് പടിഞ്ഞാറു നിന്ന് ഉദിക്കും. (ലോകാവസാനം സംഭവിക്കും.) അതാണ് അല്ലാഹു ഈ വാക്യത്തിൽ പറഞ്ഞത്: ”സൂര്യന് അതിന് സ്ഥിരമായുള്ള ഒരു സ്ഥാനത്തേക്ക് സഞ്ചരിക്കുന്നു. പ്രതാപിയും സര്വ്വജ്ഞനുമായ അല്ലാഹു കണക്കാക്കിയതാണത്.” (ക്വുർആൻ: 36: 38) (സ്വഹീഹുൽ ബുഖാരി: 7424)
മറുപടി:
ഞാൻ ഒരു കസേരയിലിരുന്നാണ് ഈ ലേഖനം എഴുതുന്നത്. “കസേര”യുടെ കാരണം (Cause) എന്താണ് എന്ന് ഞാൻ അരിസ്റ്റോട്ടിലിനോട് ചോദിച്ചാൽ, വ്യത്യസ്തമായ നാല് കാര്യകാരണങ്ങൾ (Cause) കസേരയ്ക്കുണ്ട് എന്ന് അരിസ്റ്റോട്ടിൽ പറയും. മരവും കോടാലിയും ആണിയും ചുറ്റികയുമാണ് കസേരയുടെ കാരണം (Cause) എന്ന് പറയാവുന്നതാണ്. ഇതിന് material cause എന്നാണ് പറയുക.
കസേരക്ക് ആവശ്യമായ വസ്തുക്കളുടെ ക്രമീകരണം, ആകൃതി അല്ലെങ്കിൽ പ്രത്യേകമായ രൂപത്തിലേക്ക് പരിവർത്തനം എന്നിവയെല്ലാമാണ് കസേരയുടെ കാരണം (Cause) എന്നും പറയാവുന്നതാണ്. അഥവാ നിർമ്മിക്കപ്പെടുന്ന ചട്ടക്കൂട് നൽകുന്ന വസ്തുവിൻ്റെ പ്രത്യേക സത്ത അല്ലെങ്കിൽ സ്വഭാവമാണ് ആ കാര്യത്തിൻ്റെ കാരണം എന്നർത്ഥം. ഇതിന് Formal cause എന്നാണ് പറയുക.
കസേരയുടെ കാരണം (Cause), ആശാരി അല്ലെങ്കിൽ വാസ്തുവിദ്യ ആണ് എന്നും പറയാമല്ലൊ. അഥവാ പ്രാവർത്തികമായൊ ആശയപരമായൊ അതിനെ യാഥാർത്ഥ്യവൽക്കരിക്കുന്നത് എന്തോ/ആരോ അതാണ് അതിൻ്റെ കാരണം (Cause) എന്ന്. ഇതിനെ efficient cause അല്ലെങ്കിൽ moving cause എന്ന് പറയുന്നു.
സ്റ്റൈലിൽ കാലിൻമേൽ കാലും കയറ്റി വച്ച് ഇരിക്കാൻ ഒരു സംഗതി വേണം എന്ന ലക്ഷ്യമൊ ഉദ്ദേശ്യമൊ ആണ് കസേരയുടെ കാരണം (Cause) എന്ന് പറയുന്നതും ശരിയാണ്. ഇതിനെ final cause എന്നാണ് വിളിക്കപ്പെടുന്നത്. ഒരു വസ്തു/കാര്യം ഉണ്ടാവാനുള്ള ആത്യന്തികമായ കാരണമാണ് final cause.
(Western Philosophy I: page: 43, SGOU-SLM – MA PHILOSOPHY)
ഇങ്ങനെ പല രീതിയിലും ഒരു കാര്യത്തിൻ്റെ കാരണം വിശദീകരിക്കാവുന്നതാണ്. കാര്യത്തിൻ്റെ ഉള്ളടക്കത്തിലെ ഏത് ഘടകത്തെയും ഭാഗത്തെയും പരിഗണിച്ചാണ് നമ്മൾ കാരണം (Cause) പറയാനുദ്ദേശിക്കുന്നത്, അതിനനുസൃതമായി പല രീതിയിലും ഒരു കാര്യത്തിന് കാരണങ്ങൾ പറയാം എന്നർത്ഥം. അതിൽ ഒരു രീതിയിൽ കാരണം (Cause) പറയുന്നത് മറ്റൊരു രീതിയിൽ കാരണം (Cause) പറയുന്നതിനോട് എതിരാവുന്നില്ലല്ലൊ. ഉദാഹരണത്തിന് കസേരയുടെ കാരണം മരവും കോടാലിയും ആണിയും ചുറ്റികയുമടങ്ങുന്ന വസ്തുക്കളാണ് എന്ന് പറയുന്നത് കസേരയുടെ കാരണം ആശാരിയൊ അല്ലെങ്കിൽ വാസ്തുവിദ്യയൊ ആണ് എന്ന് പറയുന്നതിന് എതിരല്ലല്ലൊ. രണ്ടും ശരിയാണ്. ഈ സാമാന്യ ബുദ്ധി ഉപയോഗിച്ച് വേണം, ഒരു കാര്യത്തിന് മതം മുന്നോട്ടു വെക്കുന്ന, ആത്മീയമൊ അഭൗതികമൊ ആയ കാരണങ്ങളെ മനസ്സിലാക്കാൻ. “ദൈവം മഴ നൽകി” എന്ന് ഒരു മത വിശ്വാസി പറയുമ്പോൾ “ഏഴാനാകാശത്തിനപ്പുറം ദൈവം കുറച്ച് വെള്ളമുണ്ടാക്കി, നിലത്തേക്ക് എറിയുന്നതാണ് മഴ… അല്ലാതെ മേഘങ്ങൾ, സൂര്യൻ, താപം, ബാഷ്പീകരണം, സാന്ദ്രീകരണം എന്നിവയൊന്നും മഴയുടെ കാരണമല്ല!” എന്നാണ് വിശ്വാസി അല്ലെങ്കിൽ മതം പറയുന്നത് എന്ന് തെറ്റിദ്ധരിക്കാൻ മാത്രം ബുദ്ധിശൂന്യരാണോ നാസ്തികർ ?! അങ്ങനെയാണെങ്കിൽ അതേ മതം തന്നെ മഴയുടെ കാരണങ്ങളായി കാർമേഘത്തെയും സാന്ദ്രീകരണത്തെയും (ക്വുർആൻ: 7: 57) പലയിടത്തും അംഗീകരിക്കുന്നത് എന്ത് കൊണ്ടാണ് ?! കാര്യം സാമാന്യ ബുദ്ധിയുള്ളവർക്ക് (അഥവാ വിശ്വാസികൾക്ക്) വ്യക്തമാണ്. ഒരിടത്ത് മഴയുടെ material cause അല്ലെങ്കിൽ Formal cause പറയുമ്പോൾ മറ്റൊരിടത്ത് അതിൻ്റെ efficient cause അല്ലെങ്കിൽ final Cause ആണ് വിശദീകരിക്കപ്പെടുന്നത്. രോഗം ശമിപ്പിക്കുന്നത് ദൈവമാണ് എന്ന് ക്വുർആൻ പറയുന്നു. (ക്വുർആൻ: 26: 80). എല്ലാ രോഗങ്ങൾക്കും മരുന്നുണ്ട് എന്നും, രോഗം വന്നാൽ ചികിത്സിച്ച് ശമിപ്പിക്കണം എന്നും മുഹമ്മദ് നബി (സ) പറയുന്നു. (സ്വഹീഹുൽ ബുഖാരി: 5678). അപ്പോൾ രോഗശമനത്തിന് കാരണം ദൈവമാണൊ? അതൊ മരുന്നാണൊ ? എന്ന് ഒരു വിശ്വാസിയും ഇതുവരെ സംശയമുന്നയിച്ചിട്ടില്ല. രോഗശമനത്തിൻ്റെ കാരണം ദൈവമാണ് എന്ന് പറയുമ്പോൾ അതിൻ്റെ ആത്യന്തികമായ കാരണം (Cause) ദൈവമാണെന്നാണ് ഉദ്ദേശ്യം. രോഗശമനത്തിൻ്റെ കാരണം മരുന്നാണ് എന്ന് പറയുമ്പോൾ അതിൻ്റെ material cause മരുന്നാണ് എന്നുമാണ് ഉദ്ദേശ്യം എന്നത് സാമാന്യ ബോധമാണ്.
അപ്പോൾ ഇത്തരം കാരണവാദങ്ങൾ വ്യത്യസ്തമാണെങ്കിലും വൈരുദ്ധ്യമല്ല എന്ന് ആദ്യമെ മനസ്സിലാക്കി വേണം വിഷയത്തിലേക്ക് കടക്കാൻ.
സൂര്യനും -ഗ്രഹങ്ങൾ പോലെ തന്നെ – രണ്ട് തരം ചലനങ്ങളുണ്ട്. ഒന്ന് അതിൻ്റെ സ്വയം ഭ്രമണം. രണ്ട്, മിൽകിവേയ് ഗാലക്സിയുടെ കേന്ദ്രത്തിന് ചുറ്റുമുള്ള അതിൻ്റെ അതി ദീർഘമായ പ്രയാണം/സഞ്ചാരം.
സൂര്യൻ സ്വന്തം അച്ചുതണ്ടിൽ കറങ്ങുന്നത്, മധ്യരേഖയിൽ 25.6 ദിവസവും ധ്രുവങ്ങളിൽ 33.5 ദിവസവും എന്ന ഭ്രമണ കാലയളവോടെയാണ്.
സൂര്യൻ ക്ഷീരപഥ ഗാലക്സിയുടെ മധ്യഭാഗത്ത് ചുറ്റുന്നത് മണിക്കൂറിൽ ശരാശരി 828,000 കിലോമീറ്റർ വേഗതയിലാണ്. എന്നാൽ ഇത്രയും ഉയർന്ന നിരക്കിൽ പോലും, ക്ഷീരപഥത്തിന് ചുറ്റും ഒരു പൂർണ്ണ ഭ്രമണപഥം വെക്കാൻ സൂര്യന് ഏകദേശം 230 ദശലക്ഷം വർഷങ്ങൾ ആവശ്യമാണ്! (starchild.gsfc.nasa.gov)
സൂര്യൻ്റെ സ്വയംഭ്രമണവും സഞ്ചാരവും ദൈവത്തിനുള്ള ആരാധനയും സുജൂദും (സാഷ്ടാംഗം) ആണെന്ന് പറയുമ്പോൾ സൂര്യ ചലനത്തിൻ്റെയൊ രാപ്പകലുകളുടെയോ material cause അല്ല ഹദീസ് അവതരിപ്പിക്കുന്നത്. മറിച്ച് പ്രപഞ്ചത്തിൻ്റെ ഇത്തരമൊരു ഘടനക്കും, രൂപത്തിനും, സ്വഭാവത്തിനും ഒരു ദൈവികമായ ഉദ്ദേശ്യമുണ്ട്. ആ ദൈവിക ഇച്ഛയെ പ്രപഞ്ചത്തിലെ സകല സൃഷ്ടിപ്പിൻ്റെയും പ്രതിഭാസത്തിൻ്റെയും Final cause ആയി പറയാമല്ലൊ. പ്രത്യേക ഉദ്ദേശ്യത്തോടെയും ലക്ഷ്യത്തോടെയുമാണ് സർവ്വതും സൃഷ്ടിച്ചത്. അപ്പോൾ പ്രപഞ്ചത്തിലെ സകല സൃഷ്ടിപ്പിൻ്റെയും പ്രതിഭാസത്തിൻ്റെയും പരമവും ആത്യന്തികവുമായ Final cause ദൈവത്തിൻ്റെ ഉദ്ദേശ്യവും ലക്ഷ്യവുമാണ്.*
(* ഈ അർത്ഥത്തിൽ, ചർച്ചയുടെ ഒഴുക്കിനായി, ഭൗതിക പ്രതിഭാസങ്ങളുടെ അഭൗതികവും ദൈവിക ഉദ്ദേശ്യ പ്രകാരവുമുള്ള കാരണങ്ങളെ Final cause എന്നാണ് തുടർന്നുള്ള ഭാഗങ്ങളിൽ വിശേഷിപ്പിക്കുക.)
അങ്ങനെ വരുമ്പോൾ അത് ഭൗതികമായ കാര്യകാരണങ്ങളിൽ നിന്നും വ്യത്യസ്തമായിരിക്കാം, പക്ഷെ വിരുദ്ധമല്ല. ശാസ്ത്രത്തിൻ്റെ ചർച്ചയും വിഷയവും പ്രപഞ്ചത്തിൻ്റെ material cause കളിൽ കേന്ദ്രീകരിച്ചു കൊണ്ടാണ് എന്ന് ഓർക്കുക.
ഉദാഹരണത്തിന്, സൂര്യൻ്റെ സ്വയംഭ്രമണം അതിൻ്റെ പൂർവ്വസ്ഥിതിയിലെ കോണീയ ആക്കം നിലനിർത്തുന്നത് മൂലമാണ്. അഥവാ, സൂര്യൻ രൂപംകൊണ്ട വാതക മേഘത്തിന് കോണീയ ആക്കം ഉണ്ടായിരുന്നു, ആ കോണീയ ആക്കം അത് രൂപപ്പെടുമ്പോൾ സൂര്യനിലേക്ക് കൈമാറി, അത് -ഇന്ന് നാം നിരീക്ഷിക്കുന്ന ഭ്രമണം- സൂര്യന് നൽകി എന്ന് ഒരു ഭൗതികശാസ്ത്രജ്ഞൻ വിശദീകരിക്കും. ഈ ശാസ്ത്ര വിശദീകരണം സൂര്യഭ്രമണത്തിൻ്റെ material cause അല്ലെങ്കിൽ efficient cause മാത്രമാണ്. ഈ ശാസ്ത്ര വിശദീകരണം സൂര്യഭ്രമണത്തിൻ്റെ അവസാന കാരണം (final cause) എങ്ങനെയാണ് ആവുക? തീർച്ചയായും അത് അവസാന കാരണം ആവില്ല. കാരണം സൂര്യൻ്റെ ഭ്രമണം, വാതക മേഘത്തിൽ നിന്നുളള കോണീയ ആക്കം നിലനിർത്തുന്നത് മൂലമാണ് എന്നതാണ് അവസാന കാരണമെങ്കിൽ വാതക മേഘത്തിൻ്റെ കോണീയ ആക്കത്തിന് കാരണമെന്താണ്? അതിന് കാരണമായി ഒരു material cause ശാസ്ത്രീയമായി വിശദീകരിക്കപ്പെട്ടു എന്ന് കരുതുക. എങ്കിൽ അതിന് കാരണമെന്താണ് എന്ന് വീണ്ടും ചോദിക്കപ്പെടാം. അതിനു കാരണമായി ഒരു material cause ശാസ്ത്രീയമായി വിശദീകരിക്കപ്പെട്ടു എന്ന് കരുതുക. എങ്കിൽ അതിന് കാരണമെന്താണ് എന്ന് വീണ്ടും ചോദിക്കപ്പെടാം. ഇങ്ങനെ കാരണാന്വേഷണം തുടന്നു തുടർന്നു പോയാൽ, material cause ന് അപ്പുറം, ഒരു അവസാന കാരണം വിശദീകരിക്കപ്പെട്ടെ മതിയാവൂ. ഈ അവസാന കാരണമാണ് മതം “ദൈവേച്ഛ” യിലൂടെ അവതരിപ്പിക്കുന്നത്. അതെങ്ങനെ ശാസ്ത്രം വിശദീകരിക്കുന്ന material cause ന് എതിരാവും?! ഒരു വസ്തുവിൻ്റെ അവസാന കാരണം (final cause) ആ വസ്തു തന്നെ (material cause) ആവൽ ബുദ്ധിപരമായി അസാധ്യമാണ് എന്ന് മതവും തത്ത്വശാസ്ത്രവും നമ്മെ പഠിപ്പിച്ചു.
അപ്പോൾ സൂര്യൻ്റെ സ്വയംഭ്രമണം, വാതക മേഘത്തിൽ (Gas cloud) നിന്നുള്ള കോണീയ ആക്കം നിലനിർത്തുന്നത് മൂലമാണ് എന്ന ശാസ്ത്രീയ വീക്ഷണം സൂര്യഭ്രമണത്തിൻ്റെ material cause ആയി മതവും മതവിശ്വാസിയും അംഗീകരിക്കുന്നു. എന്നാൽ സൂര്യഭ്രമണത്തിന് ഒരു അവസാന കാരണമുണ്ട്. അത്, സൂര്യനെ സൃഷ്ടിച്ച ദൈവത്തിൻ്റെ കൽപ്പനകൾക്ക് വിധേയമായി അവനെ ആരാധിക്കലും അവന് സാഷ്ടാംഗം ചെയ്യലും അനുവാദം വാങ്ങലുമൊക്കെയാണ്. ഇതെങ്ങനെയാണ് ശാസ്ത്ര വിരുദ്ധമാവുക എന്ന് ബുദ്ധിപരമായൊ ഫിലോസഫിക്കലായൊ തെളിയിക്കാൻ ഏത് നാസ്തികനാണ് സാധിക്കുക?!
അപ്പോൾ സൂര്യഭ്രമണത്തിൻ്റെയും സൂര്യസഞ്ചാരത്തിൻ്റെ Final cause ദൈവത്തിന് സാഷ്ടാംഗം ചെയ്യലും അനുവാദം വാങ്ങലുമൊക്കെയാണ് എന്ന് ഹദീസിൽ വന്നതിൽ ശാസ്ത്ര “വിരുദ്ധമായി” ഒന്നും തന്നെ ഇല്ല. ശാസ്ത്രത്തിന് “അപ്രാപ്യമായ” വിഷയമാണത് എന്ന് വേണമെങ്കിൽ പറയാമെന്ന് മാത്രം.
*****************************ഇനി, ദൈവത്തിൻ്റെ സിംഹാസനത്തിന് അടിയിൽ സൂര്യൻ പോവുന്നു എന്ന് ഹദീസ് പറയുമ്പോൾ, സൂര്യൻ അതിൻ്റെ ഭ്രമണപഥത്തിൽ നിന്ന് നീങ്ങി സഞ്ചരിക്കുന്നു എന്ന് വരില്ലെ എന്നതാണ് അടുത്ത ചോദ്യം. ഇല്ല എന്നതാണ് ഉത്തരം. കാരണം, ദൈവ സിംഹാസനം “ആകാശഭൂമികളെ മുഴുവന് ഉള്കൊള്ളുന്നതാകുന്നു” എന്ന് ക്വുർആൻ (2: 255) വ്യക്തമാക്കിയിട്ടുണ്ട്. അപ്പോൾ ഏത് അച്ചുതണ്ടിൽ സഞ്ചരിച്ചും ദൈവത്തിന് സുജൂദും (സാഷ്ടാംഗവും) ആരാധനകളും നിർവ്വഹിക്കാം. സഞ്ചരിക്കുക പോയിട്ട് ഒരു ചലനം പോലും കൂടാതെ ദൈവത്തിന് സുജൂദ് (സാഷ്ടാംഗം) നിർവഹിക്കാം. കയ്യും കാലും അവയവങ്ങളുമില്ലാതെയും ദൈവത്തിന് സുജൂദ് (സാഷ്ടാംഗം) നിർവഹിക്കാം എന്നാണ് ഇസ്ലാം വിശദീകരിക്കുന്നത്. എല്ലാം ഓരോ സൃഷ്ടികളുടെയും രൂപവും പ്രകൃതിയും സൃഷ്ടിപ്പും അനുസരിച്ചാണ് അവ സുജൂദ് (സാഷ്ടാംഗം) നിർവ്വഹിക്കുക; മനുഷ്യർ നിർവ്വഹിക്കുന്നത് പോലെയല്ല. ക്വുർആൻ പറയുന്നു:
“ആകാശങ്ങളിലുള്ളവരും ഭൂമിയിലുള്ളവരും, സൂര്യനും ചന്ദ്രനും നക്ഷത്രങ്ങളും, പര്വ്വതങ്ങളും വൃക്ഷങ്ങളും ജന്തുക്കളും, മനുഷ്യരില് കുറെപേരും അല്ലാഹുവിന് സുജൂദ് (സാഷ്ടാംഗവും) ചെയ്തുകൊണ്ടിരിക്കുന്നു എന്ന് നീ കണ്ടില്ലേ?” (ക്വുർആൻ: 22: 18)
“ആകാശങ്ങളിലും ഭൂമിയിലുമുള്ളവരും, ചിറക് നിവര്ത്തിപ്പിടിച്ചു കൊണ്ട് പക്ഷികളും അല്ലാഹുവിന്റെ മഹത്വം പ്രകീര്ത്തിച്ചു കൊണ്ടിരിക്കുന്നു എന്ന് നീ കണ്ടില്ലേ? *ഓരോരുത്തര്ക്കും തന്റെ പ്രാര്ത്ഥനയും കീര്ത്തനവും എങ്ങനെയെന്ന് അറിവുണ്ട്.* അവര് പ്രവര്ത്തിക്കുന്നതിനെപ്പറ്റി അല്ലാഹു അറിയുന്നവനത്രെ.” (ക്വുർആൻ: 24: 41)
ചന്ദ്രനും നക്ഷത്രങ്ങളും, പര്വ്വതങ്ങളും വൃക്ഷങ്ങളും എന്തിനേറെ നിഴലുകൾ പോലും ദൈവത്തിന് സാഷ്ടാംഗം നമസ്കരിക്കുന്നു (13: 15) എന്നാണ് ക്വുർആൻ പറയുന്നത്! നിഴലുകൾക്ക് ശരീരമുണ്ടൊ? അവയവങ്ങളുണ്ടോ? അവയൊന്നും കൂടാതെ തന്നെ, ഓരോ സൃഷ്ടിക്കും അതിൻ്റെതായ സുജൂദ് (പ്രമാണം) അർപ്പിക്കാൻ സാധിക്കുന്നതാണ്.
وكل من ذل وخضع لما أمر به، فقد سجد“ആർ താൻ കൽപ്പിക്കപ്പെട്ട ഒരു കാര്യത്തിനും മുന്നിൽ പരമമായ വിധേയത്വവും കീഴൊതുക്കവും പ്രകടിപ്പിച്ചുവൊ അയാൾ ‘സുജൂദ്’ ചെയ്തു…” എന്ന് പൗരാണിക അറബി ഭാഷാ പണ്ഡിതരെല്ലാം, സുജൂദ് എന്ന അറബി പദത്തെ വിശദീകരിച്ചിട്ടുണ്ട്. (ലിസാനുൽ അറബ്: ഇബ്നു മൻളൂർ (ജനനം: 630 ഹിജ്രാബ്ദം.1232 CE) 3: 206 )
സുജൂദിന് തലയൊ കാലോ അവയവങ്ങളൊ നിശ്ചലാവസ്ഥയൊ ശരീരം വെക്കാനുള്ള പ്രതലമൊ ഒന്നും നിർബന്ധമില്ല എന്നർത്ഥം. ഓരോ സൃഷ്ടികൾക്കുമനുസരിച്ച് അവരുടെ സുജൂദും വ്യത്യസ്തമാണ്. മനുഷ്യർ അർപ്പിക്കുന്ന സുജൂദ് തന്നെ അവസ്ഥകൾക്കനുസരിച്ച് ആപേക്ഷിക രൂപമാറ്റങ്ങൾ ഉണ്ടാവാം. മനസ്സു കൊണ്ട് പോലും സുജൂദ് ചെയ്യുന്ന അവസ്ഥയുണ്ടാവാം. അവയങ്ങൾ കൊണ്ട് സാഷ്ടാംഗം ചെയ്യുമ്പോഴും ആശയപരമായി അത് സുജൂദ് അല്ലാതിരിക്കുന്ന അവസ്ഥകളും ഉണ്ട്. അത്തരം സന്ദർഭങ്ങളിൽ അത് ഭാഷാപരമായ സുജൂദ് മാത്രമാണ്; ആദമിന് മലക്കുകൾ അർപ്പിച്ച “സുജൂദ്” (ആദരപ്രകടം) പോലെ.
ഹദീസ് വായിക്കുമ്പോൾ വിമർശകരുടെ മനസ്സിൽ സൂര്യൻ ഒരു മൃഗമൊ, മനുഷ്യനൊ, കൈകാലുകളും അവയവങ്ങളുമുള്ള ഒരു ജീവിയൊ ഒക്കെ ആയി പ്രത്യക്ഷപ്പെടുന്നു. അത്തരമൊരു സൂര്യൻ “പോകുന്നതും” “വണങ്ങുന്നതും” “മടങ്ങുന്നതും” എല്ലാം അവർ സങ്കൽപ്പിക്കുന്നു. അതുകൊണ്ടാണ് സൂര്യനെ സംബന്ധിച്ച ഈ പരാമർശങ്ങളൊന്നും വിമർശകർക്ക് ഉൾക്കൊള്ളാൻ കഴിയാത്തത്. എന്നാൽ സൂര്യൻ അതിൻ്റെ രൂപത്തിലും പ്രകൃതിയിലും ഭ്രമണത്തിലും ഇവയെല്ലാം ചെയ്യുന്നുണ്ട് എന്നതാണ് ഇസ്ലാമിക വീക്ഷണം. പ്രപഞ്ചം മുഴുവൻ വിശാലമായ ദൈവത്തിൻ്റെ സിംഹാനത്തിന് ചുവട്ടിൽ സുജൂദ് ചെയ്യാൻ… ഭ്രമണപഥത്തിൽ നിന്ന് മാറി സഞ്ചരിക്കുകയൊ, ശരീരാവയങ്ങൾ ഉണ്ടാവുകയൊ ഒന്നും വേണ്ടതില്ല. അവയൊക്കെ വേണമെന്ന്, നാം സങ്കൽപ്പിക്കുന്നു എന്ന് മാത്രം. കാരണം മനുഷ്യർ സുജൂദ് ചെയ്യാറുള്ളത് അപ്രകാരമാണണല്ലൊ. അതുകൊണ്ട് സൂര്യൻ്റെ സുജൂദും അങ്ങനെ ആയെ തീരൂ എന്ന് വിമർശകരുടെ ദുർബുദ്ധി വാശി പിടിക്കുന്നു.
ശൈഖ് സ്വലിഹ് അൽമുനജ്ജിദ് എഴുതി:
“സൂര്യൻ അതിൻ്റെ ഭ്രമണപഥത്തിലൂടെ അനുസ്യൂതം സഞ്ചരിക്കുകയും കറങ്ങുകയും ചെയ്യുമ്പോൾ തന്നെ സുജൂദ് നിർവഹിക്കുന്നുണ്ട്. ഇതാണ് ഹദീസിൻ്റെ പദങ്ങൾ പ്രത്യക്ഷത്തിൽ സൂചിപ്പിക്കുന്നത്. تجري لا مستقرا لها സൂര്യൻ യാതൊരു ഭംഗവുമില്ലാതെ “അനുസ്യൂതം സഞ്ചരിക്കുന്നു” എന്ന പ്രവാചകശിഷ്യരായ ഇബ്നു അബ്ബാസ്, ഇബ്നു മസ്ഊദ് എന്നിവരുടെ (ജാമിഉ അഹ്കാമിൽ ക്വുർആൻ: 15/28-29) വ്യാഖ്യാനവും ഇക്കാര്യത്തെ ഉറപ്പിക്കുന്നു.
ഇത് തന്നെയാണ് ഇബ്നു ഹജറിൻ്റെ (ജനനം: 773 ഹിജ്രാബ്ദം.1371 CE) വ്യാഖ്യാനത്തിൽ നിന്നും മനസ്സിലാവുന്നത്. സൂര്യൻ ദൈവത്തിന് സുജൂദ് ചെയ്യുന്നു എന്നതുകൊണ്ട് അതിൻ്റെ സഞ്ചാരം നിർത്തി ഒരു സ്ഥലത്ത് തങ്ങുന്നു എന്ന് അതിനർത്ഥമില്ല. ദൈവ സിംഹാസനത്തിന് കീഴിൽ സുജൂദ് ചെയ്യുന്നു എന്നത് കൊണ്ട് ജനങ്ങളുടെ കാഴ്ച്ചയിൽ നിന്ന് അകന്ന് മറ്റെവിടെയെങ്കിലും പോവണം എന്നുമില്ല. കാരണം ദൈവിക സിംഹാസനം ആകാശ ഭൂമികൾക്കും സൂര്യനുമെല്ലാം മുകളിലും എല്ലാത്തിനെയും ഉള്കൊള്ളുന്നതുമാകുന്നു (ക്വുർആൻ 2: 255)
സൂര്യൻ ആകാശങ്ങൾക്കപ്പുറം ഉയർന്ന് പോയി സിംഹാസനത്തിന് താഴെ സുജൂദ് ചെയ്യുന്നു എന്ന് ഹദീസ് സൂചിപ്പിക്കുന്നില്ല, ഭ്രമണപഥത്തിലൂടെ സഞ്ചരിക്കുമ്പോൾ തന്നെ, സൂര്യൻ സുജൂദ് ചെയ്യുന്നു എന്നാണ് ഹദീസ് സൂചിപ്പിക്കുന്നത് എന്ന് ഇമാം ഇബ്നു കസീർ (ജനനം: 701 ഹിജ്രാബ്ദം.1373 CE) നൂറ്റാണ്ടുകൾക്കപ്പുറം വ്യക്തമാക്കിയിട്ടുണ്ട്. (അൽബിദായ വന്നിഹായ: 1:33)”
(https://www.google.com/amp/s/www.islamweb.net/amp/ar/fatwa/99520/#ip=1)
******************************സൂര്യൻ ദൈവത്തിന് മുന്നിൽ സാഷ്ടാംഗം ചെയ്യാൻ “പോകുന്നു”… ദൈവ സമ്മതം ആരായുന്നു. ദൈവം സമ്മതം നൽകുന്നു. തുടർന്നുള്ള സഞ്ചാരത്തിലേക്ക് മടങ്ങി പോകാൻ പറയുന്നു. സൂര്യൻ പ്രയാണം തുടരുന്നു… സൂര്യൻ്റെ “പോക്കും ” “മടക്കവും” തുടങ്ങിയ പരാമർശങ്ങളൊക്കെ അശാസ്ത്രീയമല്ലെ എന്നതാണ് മറ്റൊരു ചോദ്യം.
സൂര്യൻ ദൈവത്തിനടുത്തേക്ക് “പോകുന്നു”, തൻ്റെ തുടർ സഞ്ചാരത്തിനുള്ള അനുവാദം ലഭിക്കുമ്പോൾ “മടങ്ങുന്നു”. ഈ “പോക്കും”, “മടക്കവും” ആണ് സൂര്യസഞ്ചാരം അല്ലെങ്കിൽ കറക്കം. !
ഗാലക്സിയുടെ കേന്ദ്രത്തിന് ചുറ്റും ഏകദേശം വൃത്താകൃതിയിൽ എന്ത് കൊണ്ട് സൂര്യൻ സഞ്ചരിക്കുന്നു എന്ന് ചിന്തിച്ച് നോക്കൂ. സൂര്യനു ചുറ്റും ഗ്രഹങ്ങളും ഭൂമിക്ക് ചുറ്റും ചന്ദ്രനും എന്തിന് കറങ്ങി കൊണ്ടിരിക്കുന്നു എന്ന് ചിന്തിച്ച് നോക്കൂ. അവ നേർ രേഖയിലൊ, അലക്ഷ്യമായൊ സഞ്ചരിക്കാത്തതെന്ത്. ഇതിന് കാരണം ആകേന്ദ്രബലമാണ് (centripetal force). ആകേന്ദ്രബലം ഒരു വസ്തുവെ, അത് സഞ്ചരിക്കുമ്പോൾ വൃത്തത്തിൻ്റെ മധ്യഭാഗത്തേക്ക് വലിക്കുകയോ തള്ളുകയോ ചെയ്യുന്നു, ഇത് കോണീയമോ വൃത്താകൃതിയിലുള്ളതോ ആയ ചലനത്തിന് കാരണമായി മാറുന്നു. ഗാലക്സിയുടെ മധ്യത്തിലേക്കുള്ള നക്ഷത്രങ്ങളിലെ ആകേന്ദ്രബലം, ചന്ദ്രനെ ഭൂമിക്ക് ചുറ്റുമുള്ള ഭ്രമണപഥത്തിലും ഭൂമിയെ സൂര്യനുചുറ്റും ഭ്രമണപഥത്തിൽ നിലനിർത്തുന്ന അതേ അപകേന്ദ്രബലമാണ്: അത് ഗുരുത്വാകർഷണ ബലമാണ്. അഥവാ, നമ്മുടെ ഗാലക്സിയിൽ ധാരാളം പിണ്ഡം (Mass) അടങ്ങിയിരിക്കുന്നു. അതിൽ നക്ഷത്രങ്ങൾ, വാതകം, ഗ്രഹങ്ങൾ, ഇരുണ്ട ദ്രവ്യം (dark matter) എന്നിവ ഉൾപ്പെടുന്നു. നമ്മുടെ ഗാലക്സിയുടെ കേന്ദ്രത്തിൽ ഒരു തമോഗർത്തവും (black hole) ഉണ്ട്. ഗാലക്സിയുടെ ആകെ പിണ്ഡത്തിൻ്റെ ഒരു ദശലക്ഷത്തിൽ ഒന്ന് മാത്രമാണ് കേന്ദ്രത്തിലെ തമോഗർത്തം. പിണ്ഡം (mass) ഗുരുത്വാകർഷണത്തിനും (gravity) ഗുരുത്വാകർഷണം ഭ്രമണപഥങ്ങൾക്കും (Orbit) കാരണമാകുന്നതിനാൽ, ഗാലക്സിയിലെ സൂര്യനടക്കം എല്ലാ വസ്തുക്കളുടെയും ഭ്രമണപഥങ്ങൾ ഗാലക്സിയുടെ ആകെ പിണ്ഡം മൂലമാണ് ഉണ്ടാകുന്നത് എന്ന് പറയാം.
അപ്പോൾ സൂര്യൻ ഗാലക്സിയുടെ കേന്ദ്രത്തിലേക്ക് ഗുരുത്വാകർഷണം മൂലം അടുക്കുമ്പോൾ അതേസമയം അപകേന്ദ്രബലം (centrifugal force) സൃഷ്ടിക്കപ്പെടുന്നു, അത് സൂര്യനെ ഗാലക്സിയുടെ കേന്ദ്രത്തിൽ നിന്ന് അകറ്റുന്നു. ഒരു പിണ്ഡം (Mass) തിരിക്കുമ്പോൾ അതിന്മേലുള്ള പ്രത്യക്ഷമായ പുറത്തേക്കുള്ള ബലമാണ് അപകേന്ദ്രബലം.
മൊത്തം ആകേന്ദ്രബലവും (centripetal force) അപകേന്ദ്രബലവും (centrifugal force) തുല്യവും എതിർദിശയിലുള്ളതുമാണ്, അതിനാൽ അവ പരസ്പരം റദ്ദാക്കുകയും സൂര്യൻ അതിൻ്റെ ഭ്രമണപഥത്തിൽ (orbit) ഗാലക്സിയുടെ മധ്യത്തെ ചുറ്റുകയും ചെയ്യുന്നു. ഭൂമി അതിൻ്റെ ഭ്രമണപഥത്തിൽ (orbit) സൂര്യനെ ചുറ്റുന്നത് പോലെ.
അപ്പോൾ ഒരു സൂര്യസഞ്ചാരം അല്ലെങ്കിൽ സൂര്യൻ്റെ ഭ്രമണപഥം (orbit) എന്നാൽ ഒരു കേന്ദ്രത്തിൽ നിന്നും ഒരേ സമയം ആകർഷിക്കപ്പെടുകയും വികർഷിക്കപ്പെടുകയും (അടുക്കുകയും അകന്നുപോകുകയും) ചെയ്യലല്ലെ ?! സൂര്യഭ്രമണത്തിൻ്റെ ഭ്രമണപഥ രൂപീകരണത്തെ സംബന്ധിച്ച അത്ഭുതകരമായ ഈ ശാസ്ത്രീയ സൂചന നൽകി എന്നത് ഹദീസിൻ്റെ മൗലികതയെ ഊട്ടി ഉറപ്പിക്കുന്നു.
അപ്പോൾ സൂര്യൻ്റെ Rotation (കറക്കം, സഞ്ചാരം) ഒരു പോക്കും വരവും തന്നെയാണ്. ഗാലക്സിയുടെ കേന്ദ്രത്തിലേക്കുളള “മടക്കവും” അതിൽ നിന്ന് തെന്നിമാറിയുള്ള “പോക്കും” ആകുന്നു. ഈ പോക്കും മടക്കവും സൃഷ്ടിക്കുന്ന സൂര്യൻ്റെ ഭ്രമണപഥ കറക്കത്തിൻ്റെ material cause അല്ലെങ്കിൽ efficient cause ആകേന്ദ്രബലവും (centripetal force) അപകേന്ദ്രബലവും (centrifugal force) ആകുന്നു. പക്ഷെ അതിൻ്റെ അവസാന കാരണം ദൈവ സന്നിധിയിലേക്കുള്ള “പോക്കി”ൻ്റെയും “വരവി”ൻ്റെയും വിരാമമൊ ഭംഗമൊ ഇല്ലാത്ത പരമ്പരയാണ് എന്ന് ഹദീസ് വ്യാഖ്യാനിക്കുന്നു.
ആകേന്ദ്രബലവും അപകേന്ദ്രബലവും ഗുരുത്വാകർഷണ ബലവുമൊക്കെ എങ്ങനെ അവസാന കാരണമാവും?! ആ കാരണങ്ങളെല്ലാം അവസാനിക്കാത്ത ചോദ്യങ്ങൾക്ക് വിധേയമാവുന്നുണ്ട്. ഗുരുത്വാകർഷണ ബലത്തിന് കാരണമെന്താണ്? വസ്തുക്കളുടെ പിണ്ഡമാണെങ്കിൽ, പിണ്ഡമെന്തിന് ആകർഷണം ഉണ്ടാക്കുന്നു? പിണ്ഡം സ്പേസ്ടൈം എന്ന് വിളിക്കപ്പെടുന്ന, പ്രപഞ്ചത്തിൻ്റെ നിർമ്മാണത്തെ (fabric of the universe) വളക്കുന്നത് കാരണമാണ് എന്നാണ് ഉത്തരമെങ്കിൽ, അടുത്ത ചോദ്യം പ്രപഞ്ചത്തിൻ്റെ നിർമ്മാണത്തെ പിണ്ഡം എന്തിന് വളക്കുന്നു? എന്നാണ്. ഇതിന് ഒരു പ്രമുഖ ഭൗതികശാസ്ത്ര വെബ്സൈറ്റിൽ വന്ന ഒരു ഉത്തരം കാണുക:
“എന്തുകൊണ്ട്? (why) എന്ന ചോദ്യങ്ങൾക്ക് ഭൗതികശാസ്ത്രം അന്തിമ ഉത്തരം നൽകേണ്ടതുണ്ടോ?എൻ്റെ അഭിപ്രായത്തിൽ, അത് ഭൗതികശാസ്ത്രജ്ഞൻ്റെ ലക്ഷ്യമല്ല, മാത്രമല്ല അത് ഭൗതികശാസ്ത്രത്തിൻ്റെ പരിധിക്കപ്പുറവുമാണ്.
ചുറ്റുമുള്ള അനന്തമായ സങ്കീർണ്ണമായ ലോകത്തിൻ്റെ ചില വശങ്ങൾ മനസ്സിലാക്കുന്നതിനും പ്രവചിക്കുന്നതിനുമുള്ള നമ്മുടെ ഉപകരണങ്ങളായാണ് ഭൗതികശാസ്ത്രം കൂടുതലും സിദ്ധാന്തങ്ങളെ നിർമ്മിക്കുന്നത്. ചിലപ്പോഴൊക്കെ നമ്മൾ ഒരു പൊതു പ്രയോഗക്ഷമതയുള്ള ഒരു സിദ്ധാന്തത്തിലേക്ക് എത്തുന്നു, അത് ഒന്നിലധികം വിച്ഛേദിക്കപ്പെട്ട നിരീക്ഷണങ്ങൾക്ക് ഗംഭീരമായ സൈദ്ധാന്തിക ഏകീകരണം നൽകുന്നു…” (https://physics.stackexchange.com/questions/250709/why-does-matter-curve-space-time)
ഇതു തന്നെയല്ലെ മതവിശ്വാസികൾ കാലാകാലങ്ങളായി തൊണ്ട കീറി പറഞ്ഞു മനസ്സിലാക്കാൻ ശ്രമിച്ചു കൊണ്ടിരുന്നത്?! ശാസ്ത്രം ഉത്തരം അന്വേഷിക്കുന്നത് How (എങ്ങനെ?) ചോദ്യങ്ങൾക്കാണ്. മതത്തിൻ്റെയും തത്ത്വശാസ്ത്രത്തിൻ്റെയും മേഖലയാണ് എന്തുകൊണ്ട്? (why) എന്ന ചോദ്യങ്ങൾ.
സൂര്യപ്രയാണവുമായി ബന്ധപ്പെട്ട, പ്രത്യേകം ശ്രദ്ധേയമായ ഒരു വിവരം കൂടി ഈ ചർച്ചയിൽ കൂട്ടിവായിക്കാൻ അർഹതപ്പെടുന്നു: “സൂര്യൻ ക്ഷീരപഥത്തിന് ചുറ്റും കറങ്ങുമ്പോൾ, മറ്റ് താരാപഥങ്ങളെ അപേക്ഷിച്ച് സൂര്യൻ മുകളിലേക്കും താഴേക്കും നീങ്ങുന്നതായി കാണപ്പെടുന്നു”. (https://public.nrao.edu, www.forbes.com)
സഞ്ചാരത്തിനിടയിൽ സൂര്യൻ്റെ ഈ മുകളിലേക്കും താഴേക്കും നീങ്ങലുകൾ എന്തായിരിക്കും ?! മുകളിലേക്കും താഴേക്കും നീങ്ങലുകൾക്കും material/efficient cause കൾക്കപ്പുറം ആത്യന്തികമായ ഒരു ഉദ്ദേശ്യ സമ്പൂർണമായ കാരണം ഉണ്ടാവും എന്നതാണ് ഒരു വിശ്വാസി കരുതുന്നത്. കാരണം പ്രപഞ്ചത്തിൽ ഒന്നും അലക്ഷ്യവും ആകസ്മികവുമായിട്ടില്ല. ദൈവം എല്ലാം സൃഷ്ടിച്ചത് ലക്ഷ്യപൂർവ്വമാണ്. അവയിൽ ചിലത് നാം അറിയുന്നു, മിക്കവയും നാം അറിയുന്നില്ല. “അറിവില് നിന്ന് അല്പമല്ലാതെ നിങ്ങള്ക്ക് നല്കപ്പെട്ടിട്ടില്ല.” (ക്വുർആൻ: 17: 85)
******************************** ഇനി, സൂര്യ സഞ്ചാരവും രാപ്പകലുകളുടെ നിർമ്മിതിയുമായി ബന്ധപ്പെട്ട വിഷയത്തിലേക്ക് അടുത്തതായി വരാം.
“ഭൂമിയുടെ ഭ്രമണമാണ് രാപ്പകലുകളുടെ കാരണം (cause); അല്ലാതെ സൂര്യ സഞ്ചാരം അല്ല. അപ്പോൾ ഹദീസിലെ അത്തരമൊരു വിശദീകരണം ശാസ്ത്രീയമായി തെറ്റാണ്” എന്നതാണ് വിമർശനം. ഈ വിമർശനവും പല കാരണങ്ങളാലും ബാലിശമാണ്:
1. സൂര്യൻ നമ്മുടെ ഗാലക്സിയുടെ കേന്ദ്രത്തിന് ചുറ്റും ചലിച്ച് കൊണ്ടിരിക്കുന്നു എന്ന, ഒരുപാട് നാൾ ശാസ്ത്രജ്ഞർക്ക് അജ്ഞമായ ഒരു വിവരമാണ് ഹദീസ് സംസാരിക്കുന്നത് എന്ന അത്ഭുതകരമായ വസ്തുത വിമർശകർ “ശ്രദ്ധാപൂർവ്വം” അവഗണിക്കുന്നു.
രാവും പകലും ആവർത്തിക്കുന്നതിൽ ഭൂമിയുടെ സ്വയം ഭ്രമണം മാത്രം പങ്കു വഹിക്കുന്നുള്ളു എന്ന വാദം ഈ ലോകത്ത് ഒരു ശാസ്ത്ര വിജ്ഞാനിയും പറയില്ല. കാരണം, ഇവയിൽ സൂര്യൻ്റെ പ്രയാണത്തിനാണല്ലൊ പ്രാഥമികമായ പങ്ക്. ഗാലക്സിയുടെ ക്രേന്ദഭാഗത്തിന് ചുറ്റും കോണീയ ആക്കത്തോടെ സൂര്യൻ സഞ്ചരിച്ചു കൊണ്ടിരിക്കുന്നത് കൊണ്ടല്ലെ -അടിസ്ഥാനപരമായി – രാപ്പകലുകളുടെ ആവർത്തനം ഉണ്ടാവുന്നത്? അഥവാ സൂര്യൻ്റെ സഞ്ചാരം പൊടുന്നനെ നിന്നാൽ… അല്ലെങ്കിൽ സൂര്യൻ പൊടുന്നനെ എതിർ ദിശയിലേക്ക് സഞ്ചരിക്കുകയും ഗ്രഹങ്ങൾ സൂര്യൻ്റെ ഗുരുത്വാകർഷണ വലയത്തിനപ്പുറത്തേക്ക് മുന്നോട്ട് നീങ്ങിയാൽ പിന്നെ രാവും പകലും ഉണ്ടോ? ഒരിക്കലുമില്ല. സൂര്യൻ്റെ പെട്ടെന്നുള്ള സ്തംഭനം സംഭവിക്കുകയും ഭൂമിയും മറ്റു ഗ്രഹങ്ങളും മുന്നോട്ട് ഗമിക്കുകയും ചെയ്യുന്നതോടെ ഭൂമിയുടെ ഒരു ഭാഗത്ത് മാത്രം പകലും മറുവശത്ത് രാത്രി മാത്രം ദിവങ്ങളോളമൊ ആഴ്ച്ചകളോളം നിലനിൽക്കുകയും പിന്നീട് ഭൂമി എന്നെന്നേക്കുമായ ഇരുട്ടിലേക്കും (രാത്രിയിലേക്കും) അനന്തമായ ശൂന്യാകാശത്തേക്ക് ഒഴുകി പോവുകയും ചെയ്യും. (സൂര്യൻ പടിഞ്ഞാറ് നിന്ന് കിഴക്കോട്ട് സഞ്ചരിക്കുന്നതായി ഭൂമിയിൽ അനുഭവപ്പെടും.)
അപ്പോൾ ദൈവ സമ്മതം നിരന്തരം ലഭിച്ച് സൂര്യൻ അതിൻ്റെ പ്രയാണം അനുസ്യൂതം തുടരുന്നു എന്നതിനാലാണ് അടിസ്ഥാനപരമായി രാപ്പകലുകൾ ഉണ്ടാവുന്നത് എന്ന് വിശദീകരിക്കുന്നതിൽ ശാസ്ത്രീയമായി ഒരു അബദ്ധവുമില്ല.
സൂര്യൻ്റെ ഈ സഞ്ചാരം നിലച്ചാൽ എന്താണ് സംഭവിക്കുക:
“സൂര്യൻ അത്ഭുതകരമായി അപ്രത്യക്ഷമായാൽ, ഭൂമിയും (സൗരയൂഥത്തിലെ മറ്റെല്ലാ വസ്തുക്കളും) അവയുടെ -ഏതാണ്ട് വൃത്താകൃതിയിലുള്ള- ഭ്രമണപഥങ്ങൾ പിന്തുടരുന്നതിനുപകരം, ബഹിരാകാശത്തേക്ക് ഒരു നേർരേഖയിൽ മുന്നോട്ട് നീങ്ങുന്നത് തുടരും… ഭൂമി ബഹിരാകാശത്ത് ഒഴുകുന്നത് തുടരും. അങ്ങനെ സംഭവിച്ചാൽ പോലും, സൂര്യൻ ഭൂമിയിൽ നിന്ന് അപ്രത്യക്ഷമായി ആഴ്ചകൾക്കുള്ളിൽ നാം മരവിച്ച് മരണമടയും!
(https://www.sciencefocus.com/space/what-would-happen-to-earths-orbit-if-the-sun-vanished)
സൂര്യൻ്റെ ചലനം നിന്നാൽ എന്ത് സംഭവിക്കും? എന്ന ചോദ്യത്തിന് Brainly ലെ ഒരു ശാസ്ത്ര കുതുകി എഴുതിയ മറുപടിയിലെ വാചകങ്ങൾ ശ്രദ്ധിക്കൂ:
സൂര്യൻ്റെ ചലനം നിന്നാൽ ഒരു ദിശയിലേക്ക് മാത്രം സൂര്യോദയം നിലനിൽക്കും, അതിലൂടെ മഞ്ഞ് മലകൾ ഉരുകാൻ തുടങ്ങുകയും വെള്ളപ്പൊക്കത്താൽ നാം കഷ്ടപ്പെടുകയും ചെയ്യും… (https://brainly.in/question/43106937)
ഭൂമി എതിർ ദിശയിൽ കറങ്ങിയാൽ സൂര്യൻ പടിഞ്ഞാറ് നിന്ന് ഉദിക്കുകയും കിഴക്ക് അസ്തമിക്കുകയും ചെയ്യുന്നതായി ഭൗമികമായി അനുഭവപ്പെട്ടേക്കാം. എന്നാൽ അത് ഒരു ലോകാവസാനത്തിലേക്ക് എത്തി ചേരില്ല എന്നാണ് ശാസ്ത്രസൂചന.
അങ്ങനെ സംഭവിച്ചാൽ, “ഭൂമി വ്യത്യസ്തമായിരിക്കുമെങ്കിലും ജീവയോഗ്യം തന്നെയായിരിക്കും” എന്നാണ് പഠനങ്ങൾ പറയുന്നത്. ( https://www.sciencefocus.com/planet-earth/what-would-happen-if-earth-spun-the-other-way )
എന്ന് മാത്രമല്ല കൂടുതൽ മനോഹരമായ ഒരു ലോകമാണ് അതിലൂടെ സംജാതമാവുക എന്നു കൂടി പഠനങ്ങൾ നിഗമിക്കുന്നു:
“മൊത്തത്തിൽ, പിന്നിലേക്ക് കറങ്ങുന്ന ഭൂമി പച്ചനിറമുള്ള ഭൂമിയാണെന്ന് ഗവേഷകർ കണ്ടെത്തി. ആഗോള മരുഭൂമിയുടെ വിസ്തീർണം ഏകദേശം 16 ദശലക്ഷം ചതുരശ്ര മൈലിൽ (42 ദശലക്ഷം ചതുരശ്ര കിലോമീറ്റർ) നിന്ന് ഏകദേശം 12 ദശലക്ഷം ചതുരശ്ര മൈൽ (31 ദശലക്ഷം ചതുരശ്ര കിലോമീറ്റർ) ആയി ചുരുങ്ങും. മുൻ മരുഭൂപ്രദേശങ്ങളിൽ പകുതിയോളം പുല്ലുകൾ മുളച്ചു, മറുപകുതിയിൽ മരച്ചെടികൾ ഉയർന്നുവരും…” ( https://www.livescience.com/62405-what-if-earth-rotation-reversed.html )
അപ്പോൾ, സൂര്യൻ പടിഞ്ഞാറ് ഭാഗത്ത് നിന്ന് ഉദിച്ച് കിഴക്ക് അസ്തമിക്കുകയും അതോടൊപ്പം ലോകാവസാനവും ഭൂമിയുടെ സർവ്വനാശവും ഉണ്ടാവുക എന്ന ഒരു പ്രതിഭാസം സംഭവിക്കണമെങ്കിൽ സൂര്യൻ്റെ പ്രയാണത്തിൽ മാറ്റം സംഭവിച്ചെ തീരൂ. ഈ മാറ്റമാണ് ഹദീസിൽ സൂചിപ്പിക്കപ്പെട്ടത്, ഭൂമിയുടെ സഞ്ചാരഗതിയിലുള്ള മാറ്റമല്ല എന്നത് ഹദീസിൻ്റെ ശാസ്ത്രീയതക്ക് മാറ്റ് കൂട്ടുന്നു.
2. സൗരയൂഥത്തിൻ്റെ ഘടന, ഭൂമി, സൂര്യൻ ചന്ദ്രനക്ഷത്രാദികൾ, അവയുടെ രൂപം, അവയുടെ സഞ്ചാരം എന്നിങ്ങനെ ഖഗോളശാസ്ത്രവുമായി ബന്ധപ്പെട്ട മിക്കവാറും ശാസ്ത്രീയ വിവരങ്ങളും വസ്തുതകളും ക്വുർആനിലൂടെയും പ്രവാചകനിലൂടെയും മുസ്ലിംകൾക്ക് നൽകപ്പെട്ടിരിരുന്നു. ഉദാഹരണത്തിന് ക്വുർആനിലെ കേവലം മൂന്ന് വചനങ്ങളിലൂടെ മുസ്ലിംകൾക്ക് ലഭിച്ച ഖഗോളശാസ്ത്ര വിജ്ഞാനങ്ങൾ എന്തൊക്കെയായിരുന്നു എന്ന് നമ്മുക്കൊന്ന് പരിശോധിക്കാം:
“അവനത്രെ രാത്രി, പകല്, സൂര്യന്, ചന്ദ്രന് എന്നിവയെ സൃഷ്ടിച്ചത്. ഓരോന്നും ഓരോ ഭ്രമണപഥത്തിലൂടെ നീന്തിക്കൊണ്ടിരിക്കുന്നു.” (ക്വുർആൻ: 21:33)
“സൂര്യന് ചന്ദ്രനെ പ്രാപിക്കാനാവില്ല. രാവ് പകലിനെ മറികടക്കുന്നതുമല്ല. ഓരോന്നും ഓരോ (നിശ്ചിത) ഭ്രമണപഥത്തില് നീന്തിക്കൊണ്ടിരിക്കുന്നു.”(ക്വുർആൻ: 36:40)
“രാവിനെ അവന് പകലില് പ്രവേശിപ്പിക്കുന്നു. പകലിനെ രാവിലും പ്രവേശിപ്പിക്കുന്നു. സൂര്യനെയും ചന്ദ്രനെയും അവന് (തന്റെ നിയമത്തിന്) വിധേയമാക്കുകയും ചെയ്തിരിക്കുന്നു. അവയോരോന്നും നിശ്ചിതമായ ഒരു പരിധി വരെ സഞ്ചരിക്കുന്നു. അങ്ങനെയുള്ളവനാകുന്നു നിങ്ങളുടെ രക്ഷിതാവായ അല്ലാഹു.” (ക്വുർആൻ: 35: 13)
ഈ മൂന്ന് ക്വുർആൻ വചനങ്ങൾ താഴെ പറയുന്ന ഖഗോളശാസ്ത്ര വസ്തുതകളെ നൂറ്റാണ്ടുകൾക്ക് മുമ്പ് തന്നെ പ്രഖ്യാപിച്ചു:
* ഭൂമി, സൂര്യൻ, ചന്ദ്രൻ എന്നിങ്ങനെ എല്ലാ ഗോളങ്ങളും തുടർ പ്രയാണത്തിലും ചലനത്തിലുമാണ്.
* كُلࣱّ فِی فَلَكࣲ یَسۡبَحُونَ“…ഓരോന്നും ഓരോ ഭ്രമണപഥത്തിലൂടെ നീന്തിക്കൊണ്ടിരിക്കുന്നു.” (ക്വുർആൻ: 21: 33) എന്ന വാചകത്തിലെ ‘യസ്ബഹൂൻ'(يسبحون) എന്ന പദത്തിലൂടെ സൂര്യചന്ദ്രനക്ഷത്രാദികൾ ഭ്രമണപഥത്തിലൂടെ സഞ്ചരിക്കുക മാത്രമല്ല, സ്വയം തിരിയുക കൂടി ചെയ്യുന്നുവെന്ന് സൂചിപ്പിക്കുന്നു.
* എല്ലാ ഗോളങ്ങളും ഒരു ഭ്രമണപഥത്തിലൂടെ (orbit) യാണ് സഞ്ചരിക്കുന്നത്.
* ഭ്രമണപഥങ്ങൾ (orbit) ഏതാണ്ട് വൃത്താകൃതിയിലാണ്; ദീർഘവൃത്താകൃതി (elliptical).
ക്വുർആനിൽ ഭ്രമണപഥം എന്നതിന് പ്രയോഗിക്കപ്പെട്ട “ഫലക്” (الفَلَك) എന്ന പദം തന്നെ വൃത്താകൃതിയെ സൂചിപ്പിക്കുന്നുണ്ട്.
الفَلَك: مَدَار النجوم. والجمع: أفلاك. وفَلَكَ كل شيء: مُستداره ومٌعظَمه. وفَلَكَ البحر: مَوْجه المستدير المتردّدഫലക് (الفَلَك) എന്ന പദം വട്ടത്തെയും വളഞ്ഞ, കറങ്ങി വരുന്ന വഴിയെയുമെല്ലാമാണ് സൂചിപ്പിക്കുന്നത് എന്ന് പൗരാണിക അറബി ഭാഷാ പണ്ഡിതൻമാർ വ്യക്തമാക്കിയിട്ടുണ്ട്. (ഫിൽ മുഹ്കമുൽ മുഹീത്വ്: ഇബ്നു സീദ:)
ഫലക് (الفَلَك) എന്ന പദം വൃത്താകൃതിയെ സൂചിപ്പിക്കുന്നു എന്ന് പ്രവാചക ശിഷ്യനായ ഇബ്നു അബ്ബാസും വിശദീകരിച്ചിട്ടുണ്ട്. (അസ്വവാഇകു ശ്ശദീദ അലാ ഇത്തിബാഇൽ ഹൈഅതിൽ ജദീദ: ഹമൂദ് ബിൻ അബ്ദുല്ല അത്തുവൈജിരി, ഫൈദുൽ ക്വദീർ: മുനാവി:1:647)
* كُلࣱّ فِی فَلَكࣲ یَسۡبَحُونَ“…ഓരോന്നും ഓരോ ഭ്രമണപഥത്തിലൂടെ നീന്തിക്കൊണ്ടിരിക്കുന്നു.” (ക്വുർആൻ: 21: 33) എന്ന വാചകത്തിലെ “ഓരോന്നും ഓരോ ഭ്രമണപഥത്തിലൂടെ” (كُلࣱّ فِی فَلَكࣲ) എന്നത് ഒരു ഉച്ഛാരണപരമായ അനുലോമവിലോമ വാചകമാണ്. അനുലോമവിലോമപദം/വാചകം (palindrome) എന്നാൽ, madam അല്ലെങ്കിൽ racecar പോലെയുള്ള മുന്നോട്ടും പിന്നോട്ടും ഒരു പോലെ വായിക്കാവുന്ന വാക്കുകളും, സംഖ്യകളും, ശൈലികളും ഒക്കെ ഉൾക്കൊള്ളുന്ന ഒരു ശ്രേണിയാണ്.
كُلࣱّ فِی فَلَكࣲ ك ل ف ي ف ل كഈ അനുലോമവിലോമ വാചകളിലെ അക്ഷരങ്ങൾ തമ്മിൽ വരച്ചു ചേർത്താൽ ഗോളങ്ങളുടെ ഭ്രമണപഥത്തിൻ്റെ (orbit) രൂപം ദൃശ്യമാവും!
* ഭൂമിയും മറ്റു ഗ്രഹങ്ങളും ഉപഗ്രഹങ്ങളുമെല്ലാം ഉരുണ്ട ആകൃതിയിലാണ് എന്നും ഈ മൂന്ന് വാക്യങ്ങളിൽ നിന്നും തെളിയുന്നു. അതുകൊണ്ട് തന്നെ ഭൂമിയും ഗ്രഹങ്ങളും ഉപഗ്രഹങ്ങളുമെല്ലാം ഗോളാകൃതിയിലാണ് എന്ന കാര്യത്തിൽ പ്രവാചക കാലഘട്ടം മുതൽക്കെ, മുസ്ലിം പണ്ഡിതന്മാർക്കിടയിൽ ഇജ്മാഅ് (ഏകാഭിപ്രായം) ഉണ്ടായിരുന്നു. ഇക്കാര്യത്തിൽ മുസ്ലിം പണ്ഡിതന്മാർക്കിടയിൽ ഇജ്മാഅ് (ഏകാഭിപ്രായം) ഉണ്ടെന്നും, യാതൊരു അഭിപ്രായ വ്യത്യാസവും അവർക്കിടയിൽ നിലവിലുള്ളതായി അറിയില്ല എന്നും ഹിജ്റാബ്ദം രണ്ടാം ഒന്നാം നൂറ്റാണ്ടുകാരായ, ഇമാം അഹ്മദുബ്നു ഹമ്പലിൻ്റെ ശിഷ്യഗണങ്ങൾ, ഹിജ്റാബ്ദം 256 മരണപ്പെട്ട (869 CE) ഇബ്നു മുനാദി, ഹിജ്റാബ്ദം 384 മരണപ്പെട്ട (994 CE) ഇബ്നു തൈമിയ, ഹിജ്റാബ്ദം 661 മരണപ്പെട്ട (1263 CE) ഇബ്നു തൈമിയ, ഹിജ്റാബ്ദം 543 ൽ ഭൂജാതനായ ഇമാം റാസി (1149 CE) തുടങ്ങി ഒട്ടനവധി പൗരാണിക പണ്ഡിതന്മാർ വ്യക്തമാക്കിയിട്ടുണ്ട്. (മജ്മൂഉൽ ഫതാവാ: 25/195, 6/586, അൽഫസ്ൽ ഫിൽ മിലലി വൽഅഹ്വാഇ വന്നിഹൽ:2/78)
ക്വുർആനിലും (39: 5) ഹദീസിലും വന്ന (യുകവ്വിറു يكور പന്ത് പോലെ ഉരുട്ടുന്നു എന്ന) പ്രസ്താവനകളിൽ നിന്ന് അറിവിൽ നേതൃപദവി (ഇമാം) അലങ്കരിക്കുന്ന സർവ്വ പണ്ഡിതരും ഭൂമിയുടെ ഗോളാകൃതി അംഗീകരിക്കുന്നവരായിരുന്നു എന്നും ഇമാം ഇബ്നു ഹസം വിശദീകരിക്കുകയുണ്ടായി. (അൽഫസ്ൽ ഫിൽ മിലലി വൽഅഹ്വാഇ വന്നിഹൽ: 2/78)
ഖഗോളശാസ്ത്രവുമായി ബന്ധപ്പെട്ട മിക്കവാറും ശാസ്ത്രീയ വിവരങ്ങളും വസ്തുതകളും ക്വുർആനിലൂടെയും പ്രവാചകനിലൂടെയും മുസ്ലിംകൾക്ക് നൽകപ്പെട്ടിരിരുന്നു എന്നർത്ഥം. ഇത് ഈ കാലഘട്ടത്തിലെ മുസ്ലിംകൾ മുന്നോട്ട് വെക്കുന്ന ഒരു അവകാശവാദമല്ല എന്ന് പ്രവാചകശിഷ്യന്മാരുടെയും പൗരാണിക മുസ്ലിം പണ്ഡിതന്മാരുടെയും ക്വുർആൻ വ്യാഖ്യാനത്തിൽ നിന്നും തന്നെ ഇവിടെ തെളിയിച്ചു കഴിഞ്ഞു. സൗരയൂഥത്തിൻ്റെ ഘടന, ഭൂമി, സൂര്യൻ ചന്ദ്രനക്ഷത്രാദികൾ, അവയുടെ രൂപം, അവയുടെ സഞ്ചാരം എന്നിങ്ങനെ സർവ്വതും മുസ്ലിംകൾ പണ്ടു മുതലേ -ക്വുർആനിൽ നിന്നും ഹദീസുകളിൽ നിന്നും – മനസ്സിലാക്കിയിരുന്നു. എന്നിട്ടും ഹദീസിലെ സൂര്യൻ്റെ സാഷ്ടാംഗവും അവർക്ക് അവരുടെ ഖഗോളശാസ്ത്ര വിജ്ഞാനങ്ങൾക്ക് എതിരായി ഒരു കാലഘട്ടത്തിലും തോന്നിയില്ല. അതിന് കാരണം സൂര്യഭ്രമണത്തിൻ്റെ material cause നെ പറ്റി പഠിപ്പിക്കുകയല്ല മതത്തിൻ്റെ ആത്യന്തികമായ ലക്ഷ്യം എന്ന തിരിച്ചറിവാണ്. ഈ പ്രകൃതി പ്രതിഭാസങ്ങളുടെ ആത്മീയ മണ്ഡലവും പരമമായ കാരണങ്ങളും (ദൈവിക ലക്ഷ്യം) വിശദീകരിക്കുകയാണ് മതം ചെയ്യുന്നത് എന്ന കേവല ബുദ്ധിയും തത്ത്വജ്ഞാനവും അവർക്കെല്ലാം ഉണ്ടായിരുന്നു. അനുഭവം മാത്രമാണ് ജ്ഞാനത്തിനു കാരണമെന്നുള്ള എൻപിരിസസ (empiricism) ജ്വരവും ശാസ്ത്രത്തെ മതവൽകരിക്കുന്ന സയൻ്റിസമൊ (scientism) മുസ്ലിംകളിലെ തത്ത്വജ്ഞാന ബോധത്തെയും യുക്തിയെയും ഊറ്റി കളഞ്ഞിട്ടില്ല എന്നതാണ് ഈ തിരിച്ചറിവിന് കാരണം.
ചുരുക്കത്തിൽ, അശാസ്ത്രീയവും അയുക്തികരവുമായ ഒന്നും ഹദീസിൻ്റെ ഉള്ളടക്കത്തിൽ ഇല്ല. ശാസ്ത്രീയ പ്രവചനങ്ങളുടെയും സൂചനകളുടെയും ഒരു അത്ഭുത കലവറയാണ് യഥാർത്ഥത്തിൽ ഈ ഹദീസ്.
വിമർശനം:
സൂര്യനെ സൃഷ്ടിക്കപ്പെടുന്നതിന് മുമ്പ് ചെടികളും മരങ്ങളും സൃഷ്ടിക്കപ്പെട്ടു എന്ന ബൈബിളിലെ ഉൽപ്പത്തി പ്രതിബാധനത്തിലെ അബദ്ധം മുഹമ്മദ് നബിയും കോപ്പിയടിച്ചു. തന്നിമിത്തം ഹദീസിലേയും ഉൽപ്പത്തി പ്രതിബാധനത്തിൽ ധാരാളം അബദ്ധങ്ങൾ ഉള്ളടങ്ങിയിട്ടില്ലെ ?!
മറുപടി:
ക്വുർആനിൽ, പ്രവാചകൻ ഇബ്റാഹീമിന്റെ (അബ്രഹാം) കാലത്തെ ഈജിപ്തിലെ ഭരണാധികാരിയെ സംബന്ധിച്ച് “അല്ലാഹു അധികാരം നൽകിയ” വ്യക്തി (آتَاهُ اللَّهُ الْمُلْكَ) (ക്വുർആൻ: 2: 258) എന്നാണ് പറയപ്പെടുന്നത്… പ്രവാചകൻ യൂസുഫിന്റെ (അ) (ജോസഫ്) കാലത്തെ ഈജിപ്തിലെ ഭരണാധികാരിയെ സംബന്ധിച്ച് “രാജാവ്” (ملك King) എന്നാണ് പറയപ്പെടുന്നത്; മൂസാ നബിയുടെ(അ) (മോശെ) കാലത്തെ ഭരണാധികാരിയെ “ഫിർഔൻ” (Pharaoh فرعون ഫറവോൻ) എന്നുമാണ് പറയപ്പെടുന്നത്.
ജോസഫിന്റെ മരണത്തിന് ശേഷമാണ് ഈജിപ്തിലെ ഭരണാധികാരികളെ സൂചിപ്പിക്കാൻ “ഫറവോൻ” എന്ന പദവി ഉപയോഗിക്കാൻ തുടങ്ങിയത്.
എന്നാൽ ബൈബിളിൽ, ജോസഫിന്റെ കാലത്തെ ഭരണാധികാരിയെയും “ഫറവോൻ” എന്ന പേരിലാണ് പരിചയപ്പെടുത്തുന്നത്.
ജോസഫിനും മുമ്പ്, അബ്രഹാമിന്റെ കാലത്തെ ഭരണാധികാരിയേയും “ഫറവോൻ” എന്ന പേരിലാണ് (ആറു തവണ) പരിചയപ്പെടുത്തുന്നത്. (ഉല്പത്തി: 12: 10-20)
ജോസഫിന്റെ കാലഘത്തിലെ ഭരണാധികാരിയേയും “ഫറവോൻ” എന്ന പേരിൽ തന്നെയാണ് (തൊന്നൂറ് തവണ) പരിചയപ്പെടുത്തുന്നത്. (ഉല്പത്തി: 41: 14, 25, 46…)
അതിന് കാരണം, എല്ലാ കാലഘട്ടത്തിലും, ഈജിപ്റ്റിലെ ഭരണാധികാരികളെ എല്ലാം “ഫറവോൻ” എന്ന പേരിലാണ് വിളിക്കപ്പെട്ടിരുന്നത് എന്ന തെറ്റിദ്ധാരണയിൽ, മനുഷ്യരാൽ രചിക്കപ്പെട്ടതാണ് എന്നതിനാലാണ് ഈ ചരിത്രപരമായ് സ്കലിതം ബൈബിളിൽ വരാൻ കാരണം.
ക്വുർആൻ, മനുഷ്യ നിർമ്മിതമായിരുന്നെങ്കിൽ, ബൈബിൾ നോക്കി പകർത്തിയതായിരുന്നെങ്കിൽ ഇത്രയും സൂക്ഷ്മവും, സങ്കീർണവും, നൂറ്റാണ്ടുകൾക്ക് ശേഷം തിരിച്ചറിഞ്ഞതുമായ ഈ സ്കലിതം ക്വുർആനിലും സംഭവിക്കുമായിരുന്നില്ലെ? പക്ഷെ അങ്ങനെ സംഭവിച്ചില്ല ! ക്വുർആനിൽ പ്രവാചകൻ ഇബ്റാഹീമിന്റെ (അ) കാലഘട്ടത്തിലെ ഈജിപ്ത്യൻ ഭരണാധികാരിയെ “അല്ലാഹു അധികാരം നൽകിയ” വ്യക്തി (آتَاهُ اللَّهُ الْمُلْكَ) എന്നും (ക്വുർആൻ:2:258), പ്രവചകൻ യൂസുഫിന്റെ കാലഘട്ടത്തിലെ ഈജിപ്ത്യൻ ഭരണാധികാരിയെ “രാജാവ്” (ملك) എന്നും (ക്വുർആൻ: 12: 76) അഭിസംബോധനം ചെയ്ത അത്യത്ഭുതകരമായ സൂക്ഷ്മതയിൽ നിന്ന് രണ്ട് കാര്യങ്ങൾ സുതരാം വ്യക്തമാവുന്നു: ഒന്ന്, ക്വുർആൻ ബൈബിളിന്റെ കോപിയല്ല. ബൈബിളിൽ ദിവ്യബോധനത്തിൽ നിന്നുമുള്ള ഭാഗങ്ങൾ ഉള്ളടങ്ങിയിട്ടുണ്ട് എന്നതിനാൽ തന്നെ ബൈബിളും ക്വുർആനും വിഷയാതിഷ്ടിതമായ സാമ്യതകൾ ഉണ്ടാകാം എന്ന് മാത്രം.
രണ്ട്, ബൈബിളിൽ പരാമർശിക്കപ്പെട്ട സങ്കീർണവും അതിസൂക്ഷ്മവും മാനുഷികവുമായ തെറ്റ് ക്വുർആൻ ആവർത്തിക്കാതിരുന്നതും ക്വുർആന്റെ അത്യത്ഭുതകരമായ ചരിത്ര കൃത്യതയും തെളിയിക്കുന്നത് ക്വുർആന്റെ അമാനുഷികതയെയാണ്. ഇത് മനുഷ്യനിൽ നിന്നല്ല എന്നർത്ഥം.
സമാനമാണ് വിമർശന വിധേയമായ ഹദീസിന്റെയും അവസ്ഥ. ബൈബിളിൽ നിന്നുള്ള കോപ്പിയടിയാണ് ഹദീസും, അതിനാൽ അതിൽ ധാരാളം അബദ്ധങ്ങൾ ആവർത്തിക്കപ്പെട്ടു എന്നല്ല വിമർശന വിധേയമായ ഹദീസിൽ നിന്നും സൂക്ഷ്മ പരിശോധനയിൽ നിന്നും മനസ്സിലാവുക. പ്രത്യുത, ബൈബിളിൽ ഉൽപ്പത്തിയെ സംബന്ധിച്ച് പറയപ്പെട്ട വിശദീകരണത്തിലെ സൂക്ഷ്മമായ അബദ്ധങ്ങളും അശാസ്ത്രീയതകളും അത്യൽഭുതകരമായ വിധം ഹദീസിലെ ഉൽപ്പത്തി വിശദീകരണത്തിൽ ഒഴിവാക്കപ്പെട്ടിരിക്കുകയാണ് !
ഇത് മനസ്സിലാക്കാനായി ആദ്യം നമ്മുക്ക് ബൈബിളിലെ, ഉൽപ്പത്തി പുസ്തകം: അദ്ധ്യായം ഒന്നിലെ, 1 മുതൽ 31 വരെയുളള വാക്യങ്ങൾ വായിച്ചു നോക്കാം:
1 ആദിയിൽ ദൈവം ആകാശവും ഭൂമിയും സൃഷ്ടിച്ചു. 2 ഭൂമി പാഴായും ശൂന്യമായും ഇരുന്നു; ആഴത്തിന്മീതെ ഇരുൾ ഉണ്ടായിരുന്നു. ദൈവത്തിന്റെ ആത്മാവ് വെള്ളത്തിന്മീതെ പരിവർത്തിച്ചു കൊണ്ടിരുന്നു. 3 വെളിച്ചം ഉണ്ടാകട്ടെ എന്നു ദൈവം കല്പിച്ചു; വെളിച്ചം ഉണ്ടായി. 4 വെളിച്ചം നല്ലത് എന്നു ദൈവം കണ്ടു; ദൈവം വെളിച്ചവും ഇരുളും തമ്മിൽ വേർപിരിച്ചു. 5 ദൈവം വെളിച്ചത്തിനു പകൽ എന്നും ഇരുളിനു രാത്രി എന്നും പേരിട്ടു. സന്ധ്യയായി ഉഷസ്സുമായി, ഒന്നാം ദിവസം. 6 ദൈവം: വെള്ളങ്ങളുടെ മധ്യേ ഒരു വിതാനം ഉണ്ടാകട്ടെ; അതു വെള്ളത്തിനും വെള്ളത്തിനും തമ്മിൽ വേർപിരിവായിരിക്കട്ടെ എന്നു കല്പിച്ചു. 7 വിതാനം ഉണ്ടാക്കിയിട്ടു ദൈവം വിതാനത്തിൻകീഴുള്ള വെള്ളവും വിതാനത്തിന്മീതെയുള്ള വെള്ളവും തമ്മിൽ വേർപിരിച്ചു; അങ്ങനെ സംഭവിച്ചു. 8 ദൈവം വിതാനത്തിന് ആകാശം എന്നു പേരിട്ടു. സന്ധ്യയായി ഉഷസ്സുമായി, രണ്ടാം ദിവസം. 9 ദൈവം: ആകാശത്തിൻകീഴുള്ള വെള്ളം ഒരു സ്ഥലത്തു കൂടട്ടെ; ഉണങ്ങിയ നിലം കാണട്ടെ എന്നു കല്പിച്ചു; അങ്ങനെ സംഭവിച്ചു. 10 ഉണങ്ങിയ നിലത്തിനു ദൈവം ഭൂമി എന്നും വെള്ളത്തിന്റെ കൂട്ടത്തിനു സമുദ്രം എന്നും പേരിട്ടു; നല്ലത് എന്നു ദൈവം കണ്ടു. 11 ഭൂമിയിൽനിന്നു പുല്ലും വിത്തുള്ള സസ്യങ്ങളും ഭൂമിയിൽ അതതുതരം വിത്തുള്ള ഫലം കായ്ക്കുന്ന വൃക്ഷങ്ങളും മുളച്ചുവരട്ടെയെന്നു ദൈവം കല്പിച്ചു; അങ്ങനെ സംഭവിച്ചു. 12 ഭൂമിയിൽനിന്നു പുല്ലും അതതുതരം വിത്തുള്ള ഫലം കായ്ക്കുന്ന വൃക്ഷങ്ങളും മുളച്ചുവന്നു; നല്ലത് എന്നു ദൈവം കണ്ടു. 13 സന്ധ്യയായി ഉഷസ്സുമായി, മൂന്നാം ദിവസം. 14 പകലും രാവും തമ്മിൽ വേർപിരിവാൻ ആകാശവിതാനത്തിൽ വെളിച്ചങ്ങൾ ഉണ്ടാകട്ടെ; അവ അടയാളങ്ങളായും കാലം, ദിവസം, സംവത്സരം എന്നിവ തിരിച്ചറിവാനായും ഉതകട്ടെ; 15 ഭൂമിയെ പ്രകാശിപ്പിപ്പാൻ ആകാശവിതാനത്തിൽ അവ വെളിച്ചങ്ങളായിരിക്കട്ടെ എന്നു ദൈവം കല്പിച്ചു; അങ്ങനെ സംഭവിച്ചു. 16 പകൽ വാഴേണ്ടതിനു വലിപ്പമേറിയ വെളിച്ചവും രാത്രി വാഴേണ്ടതിനു വലിപ്പം കുറഞ്ഞ വെളിച്ചവും ആയി രണ്ടു വലിയ വെളിച്ചങ്ങളെ ദൈവം ഉണ്ടാക്കി; നക്ഷത്രങ്ങളെയും ഉണ്ടാക്കി. 17 ഭൂമിയെ പ്രകാശിപ്പിപ്പാനും പകലും രാത്രിയും വാഴുവാനും വെളിച്ചത്തെയും ഇരുളിനെയും തമ്മിൽ വേർപിരിപ്പാനുമായി 18 ദൈവം അവയെ ആകാശവിതാനത്തിൽ നിർത്തി; നല്ലത് എന്നു ദൈവം കണ്ടു. 19 സന്ധ്യയായി ഉഷസ്സുമായി, നാലാം ദിവസം. 20 വെള്ളത്തിൽ ജലജന്തുക്കൾ കൂട്ടമായി ജനിക്കട്ടെ; ഭൂമിയുടെമീതെ ആകാശവിതാനത്തിൽ പറവജാതി പറക്കട്ടെ എന്നു ദൈവം കല്പിച്ചു. 21 ദൈവം വലിയ തിമിംഗലങ്ങളെയും വെള്ളത്തിൽ കൂട്ടമായി ജനിച്ചു ചരിക്കുന്ന അതതുതരം ജീവജന്തുക്കളെയും അതതുതരം പറവജാതിയെയും സൃഷ്ടിച്ചു; നല്ലത് എന്നു ദൈവം കണ്ടു. 22 നിങ്ങൾ വർധിച്ചു പെരുകി സമുദ്രത്തിലെ വെള്ളത്തിൽ നിറവിൻ; പറവജാതി ഭൂമിയിൽ പെരുകട്ടെ എന്നു കല്പിച്ചു ദൈവം അവയെ അനുഗ്രഹിച്ചു. 23 സന്ധ്യയായി ഉഷസ്സുമായി, അഞ്ചാം ദിവസം. 24 അതതുതരം കന്നുകാലി, ഇഴജാതി, കാട്ടുമൃഗം ഇങ്ങനെ അതതുതരം ജീവജന്തുക്കൾ ഭൂമിയിൽനിന്ന് ഉളവാകട്ടെ എന്നു ദൈവം കല്പിച്ചു; അങ്ങനെ സംഭവിച്ചു. 25 ഇങ്ങനെ ദൈവം അതതുതരം കാട്ടുമൃഗങ്ങളെയും അതതുതരം കന്നുകാലികളെയും അതതുതരം ഭൂചരജന്തുക്കളെയും ഉണ്ടാക്കി; നല്ലത് എന്നു ദൈവം കണ്ടു. 26 അനന്തരം ദൈവം: നാം നമ്മുടെ സ്വരൂപത്തിൽ നമ്മുടെ സാദൃശ്യപ്രകാരം മനുഷ്യനെ ഉണ്ടാക്കുക; അവർ സമുദ്രത്തിലുള്ള മത്സ്യത്തിന്മേലും ആകാശത്തിലുള്ള പറവജാതിയിന്മേലും മൃഗങ്ങളിന്മേലും സർവഭൂമിയിന്മേലും ഭൂമിയിൽ ഇഴയുന്ന എല്ലാ ഇഴജാതിയിന്മേലും വാഴട്ടെ എന്നു കല്പിച്ചു. 27 ഇങ്ങനെ ദൈവം തന്റെ സ്വരൂപത്തിൽ മനുഷ്യനെ സൃഷ്ടിച്ചു, ദൈവത്തിന്റെ സ്വരൂപത്തിൽ അവനെ സൃഷ്ടിച്ചു, ആണും പെണ്ണുമായി അവരെ സൃഷ്ടിച്ചു. 28 ദൈവം അവരെ അനുഗ്രഹിച്ചു: നിങ്ങൾ സന്താനപുഷ്ടിയുള്ളവരായി പെരുകി ഭൂമിയിൽ നിറഞ്ഞ് അതിനെ അടക്കി സമുദ്രത്തിലെ മത്സ്യത്തിന്മേലും ആകാശത്തിലെ പറവജാതിയിന്മേലും സകല ഭൂചരജന്തുവിന്മേലും വാഴുവിൻ എന്ന് അവരോടു കല്പിച്ചു. 29 ഭൂമിയിൽ എങ്ങും വിത്തുള്ള സസ്യങ്ങളും വൃക്ഷത്തിന്റെ വിത്തുള്ള ഫലം കായ്ക്കുന്ന സകല വൃക്ഷങ്ങളും ഇതാ, ഞാൻ നിങ്ങൾക്കു തന്നിരിക്കുന്നു; അവ നിങ്ങൾക്ക് ആഹാരമായിരിക്കട്ടെ; 30 ഭൂമിയിലെ സകല മൃഗങ്ങൾക്കും ആകാശത്തിലെ എല്ലാ പറവകൾക്കും ഭൂമിയിൽ ചരിക്കുന്ന സകല ഭൂചരജന്തുക്കൾക്കും ആഹാരമായിട്ടു പച്ചസസ്യമൊക്കെയും ഞാൻ കൊടുത്തിരിക്കുന്നു എന്നു ദൈവം കല്പിച്ചു; അങ്ങനെ സംഭവിച്ചു. 31 താൻ ഉണ്ടാക്കിയതിനെയൊക്കെയും ദൈവം നോക്കി, അത് എത്രയും നല്ലത് എന്നു കണ്ടു. സന്ധ്യയായി ഉഷസ്സുമായി, ആറാം ദിവസം.”
ഏഴ് ദിവസങ്ങളിലെ ഓരോ ദിവസവും നടന്ന സൃഷ്ടിപ്പിനൊപ്പം “സന്ധ്യയായി ഉഷസ്സുമായി, ഒന്നാം ദിവസം…”, “സന്ധ്യയായി ഉഷസ്സുമായി, രണ്ടാം ദിവസം…” എന്ന് ആവർത്തിക്കപ്പെടുന്നത് ശ്രദ്ധിക്കുക. ആ ദിവസങ്ങളിലെ സൃഷ്ടിപ്പുകൾ കാലക്രമം അനുസരിച്ച് (chronological order) നടന്നു എന്നാണ് ബൈബിൾ വാദിക്കുന്നത്. എന്ന് വെച്ചാൽ… രണ്ടാം ദിവസം ഭൂമിയിൽ പുല്ലും വിത്തുള്ള സസ്യങ്ങളും വൃക്ഷങ്ങളും മുളപ്പിച്ചു… മൂന്നാം ദിവസം… പകലും രാവും തമ്മിൽ വേർപിരിവാനും കാലം, ദിവസം, സംവത്സരം എന്നിവ തിരിച്ചറിവാനായും… വെളിച്ചങ്ങൾ (സൂര്യ ചന്ദ്രൻമാരെ) സൃഷ്ടിച്ചു. നാലാം ദിവസം വെള്ളത്തിലെ ജലജന്തുക്കളെയും സൃഷ്ടിച്ചു… എന്നിങ്ങനെ കാലക്രമം അനുസരിച്ചാണ് സൃഷ്ടിപ്പ്. അപ്പോൾ സൂര്യനും ചന്ദ്രനും ഇല്ലാതെ ഒന്നാം ദിവസം ഉഷസ്സും സന്ധ്യയും എങ്ങനെ ആയി? സൂര്യനും ചന്ദ്രനും സൃഷ്ടിക്കപ്പെടുന്നതിന് മുമ്പ് രാവും പകലും എങ്ങനെ ഉണ്ടായി ?!സൂര്യപ്രകാശം സൃഷ്ടിക്കുന്നതിന് മുമ്പ് സസ്യങ്ങളും വൃക്ഷങ്ങളും എങ്ങനെ നിലനിൽക്കുന്നു ?! ഇങ്ങനെ ഒരുപാട് വൈരുധ്യങ്ങളും ശാസ്ത്രാബദ്ധങ്ങളും ബൈബിളിലെ ഉൽപ്പത്തി പ്രതിബാധനത്തിൽ വന്നു ഭവിക്കുന്നു.
എന്നാൽ ഹദീസിലെ ഉൽപ്പത്തി വിവരണത്തിൽ ഈ അബദ്ധങ്ങളും ശാസ്ത്ര വിരുദ്ധതയും ഒഴിവാക്കപ്പെട്ടിരിക്കുന്നു:
ദൈവം ശനിയാഴ്ച്ച മണ്ണ് സൃഷ്ടിച്ചു. ഞായറാഴ്ച്ച പർവ്വതങ്ങളെ ഉണ്ടാക്കി. തിങ്കളാഴ്ച്ച മരങ്ങളെ സൃഷ്ടിച്ചു. ചൊവ്വാഴ്ച്ച മക്റൂഹ് (വെറുക്കപ്പെട്ടത്, ദോഷകരമായ വസ്തുക്കൾ) സൃഷ്ടിച്ചു. ബുധനാഴ്ച്ച പ്രകാശം (ഗുണകരമായ വസ്തുക്കൾ) സൃഷ്ടിച്ചു. വ്യാഴാഴ്ച്ച ജന്തുക്കളെ ഭൂമിയിൽ പരത്തി. വെള്ളിയാഴ്ച്ചയുടെ അവസാന സന്ധിയിൽ, അസറിനും രാത്രിക്കും ഇടയിൽ ആദമിനെ (സ) സൃഷ്ടിച്ചു. (സ്വഹീഹു മുസ്ലിം: 2789) എന്നാണ് ഹദീസിലെ ഉള്ളടക്കം. ഇവിടെ ദിവസങ്ങളിലെ സൃഷ്ടിപ്പുകൾ കാലക്രമം അനുസരിച്ച് (chronological order) നടന്നതാണ് എന്ന ബൈബിളിലെ അബദ്ധം ഒഴിവാക്കപ്പെട്ടിരിക്കുന്നു ! ഒന്നാം ദിവസം, രണ്ടാം ദിവസം… തുടങ്ങിയ കാലക്രമമൊ, സന്ധ്യയായി ഉഷസ്സായി തുടങ്ങി തുടർച്ചയെയൊ പ്രസ്താവിച്ചില്ല എന്നത് അത്ഭുതകരമാം വിധം സൂക്ഷ്മത അടങ്ങുന്നതാണ്.
“എന്റെ മകൻ ജനിച്ചത് ഞായറാഴ്ച്ചയാണ്. അതുകഴിഞ്ഞ് രണ്ടാം ദിവസമാണ് ഞാൻ ജനിച്ചത്. അതു കഴിഞ്ഞ് മൂന്നാം ദിവസമാണ് എന്റെ അച്ഛൻ ജനിച്ചത്. അതുകഴിഞ്ഞ് നാലാം ദിവസം എന്റെ അമ്മ ജനിച്ചു…” എന്ന് പറയൽ ശുദ്ധ അസംബന്ധമാണ് എന്ന് പ്രത്യേകം തെളിയിക്കേണ്ടതില്ലല്ലൊ. ഈ ഉൽപ്പത്തി പ്രതിബാധനമനുസരിച്ച് എന്റെ മകൻ എന്നേക്കാൾ മൂത്തതും, ഞാൻ ജനിച്ച ശേഷമാണ് എന്റെ അച്ചനും അമ്മയും ജനിച്ചത് എന്നും വരുന്നു ! ഇത്തരമൊരു ഉൽപ്പത്തി പ്രതിബാധനമാണ് ബൈബിൾ അവതരിപ്പിക്കുന്നത്.
അതേ സമയം ഒരാൾ ഇപ്രകാരം പറഞ്ഞു എന്ന് കരുതുക:
“എന്റെ മകൻ ജനിച്ചത് ഞായറാഴ്ചയാണ്. ഞാൻ ജനിച്ചത് തിങ്കളാഴ്ചയാണ്. എന്റെ അച്ഛൻ ജനിച്ചത് ചൊവ്വാഴ്ചയാണ്. എന്റെ അമ്മ ജനിച്ചത് ബുധനാഴ്ചയാണ്.”
ഇവിടെ ഉൽപ്പത്തിയുടെ ക്രമമൊ കാല തുടർച്ചയൊ പറയപ്പെടുന്നില്ല; ദിവസങ്ങൾക്ക് മാത്രമെ കാലക്രമം സംജാതമാകുന്നുള്ളു. ഇതാണ് ഹദീസിലെ ഉൽപ്പത്തി പ്രതിബാധന രീതിയുടെ വ്യത്യാസം.
ഹദീസിലെ ഉൽപ്പത്തി പ്രതിബാധനത്തിലെ മറ്റൊരു വ്യത്യാസം കൂടി ശ്രദ്ധിക്കേണ്ടതുണ്ട്:
ഇബ്നുൽ അസീർ (റ) പറഞ്ഞു: “മരങ്ങളെ സൃഷ്ടിച്ചു. ചൊവ്വാഴ്ച്ച മക്റൂഹ് (വെറുക്കപ്പെട്ടത്) എന്നത് കൊണ്ട് ഇവിടെ ഉദ്ദേശ്യം ദോഷകരമായ കാര്യങ്ങളാണ്. കാരണം ഹദീസിൽ തുടർന്ന് പറയുന്നത് “ബുധനാഴ്ച്ച പ്രകാശം സൃഷ്ടിച്ചു” എന്നാണ്. പ്രകാശം എന്നാൽ ഉപകാര പ്രധമായ കാര്യങ്ങളാണ്. ദോഷകരമായ കാര്യങ്ങളെ മക്റൂഹ് (വെറുക്കപ്പെട്ടത്) എന്ന് വിളിക്കപ്പെടാൻ കാരണം, അവ ‘പ്രിയങ്കരമായ’ കാര്യങ്ങൾക്ക് വിപരീതമായത് കൊണ്ടാണ്.” (അന്നിഹായ: 4:169)
ചുരുക്കത്തിൽ, ബൈബിളിലെ ഉൽപ്പത്തി വിവരണത്തിൽ നിന്നുള്ള കോപ്പിയടിയായിരുന്നു ഹദീസും എങ്കിൽ അതിലും ബൈബിളിലെ അബദ്ധങ്ങൾ ആവർത്തിക്കപ്പെടുമായിരുന്നു. എന്നാൽ വിമർശന വിധേയമായ ഹദീസിന്റെ സൂക്ഷ്മ പരിശോധനയിൽ നിന്നും മനസ്സിലാവുക, ബൈബിളിൽ ഉൽപ്പത്തിയെ സംബന്ധിച്ച് പറയപ്പെട്ട വിശദീകരണത്തിലെ സൂക്ഷ്മമായ അബദ്ധങ്ങളും അശാസ്ത്രീയതകളും – വിശിഷ്യാ കാലക്രമം – അത്യൽഭുതകരമായ വിധം ഹദീസിലെ ഉൽപ്പത്തി വിശദീകരണത്തിൽ ഒഴിവാക്കപ്പെട്ടിരിക്കുന്നു എന്നാണ് ! ഇത് ഹദീസുകളുടെ ദിവ്യബോധന അടിത്തറയെയും സത്യതയെയുമാണ് തെളിയിക്കുന്നത്.
**********************************വിമർശനം: ആകാശഭൂമികളെ ആറ് ദിവസം കൊണ്ട് സൃഷ്ടിച്ചു എന്ന് ക്വുർആൻ പറയുന്നു. ആകാശത്തെ രണ്ട് ദിവസങ്ങൾ കൊണ്ടും ഭൂമിയെ രണ്ട് ദിവസങ്ങൾ കൊണ്ടും ഭൂമിയിലെ വിശദാംശങ്ങളെ രണ്ട് ദിവസങ്ങൾ കൊണ്ടും സൃഷ്ടിച്ചു. അപ്പോൾ പിന്നെ ഹദീസിൽ ഏഴ് ദിവസങ്ങൾ കൊണ്ട് നടന്ന സൃഷ്ടിപ്പ് ക്വുർആനിക പരാമർശത്തോട് എതിരല്ലെ ?
മറുപടി: ഒരിക്കലുമല്ല. കാരണം, ക്വുർആൻ സംസാരിക്കുന്നത് ഭൂമി സൃഷ്ടിക്കപ്പെടുന്നതിന് മുമ്പുള്ള, ഭൂമിയുടെ സൃഷ്ടിപ്പു കൂടി ഉൾപ്പെടുന്ന അഭൗമികമായ (ആറ്) ദിവസങ്ങളെ സംബന്ധിച്ചാണ്. ചർച്ച വിഷയകമായ ഹദീസിൽ പരാമർശിക്കുന്നത് ഭൂമി സൃഷ്ടിക്കപ്പെട്ടതിന് ശേഷം, ഭൂമിയിലെ വിശദാംശങ്ങളുടെ സൃഷ്ടിപ്പ് നടന്ന ഭൗമികമായ (ഏഴ്) ദിവസങ്ങളെ സംബന്ധിച്ചുമാണ്.
അറബി പദമായ يوم (യവ്മ്, ദിവസം) ഭൗമികമായ ദിവസത്തെ സൂചിപ്പിക്കാൻ മാത്രമല്ല ഉപയോഗിക്കപ്പെടുന്നത്. കാലത്തിന്റെ ഏതു തരം ഘട്ടങ്ങളെയും ഭാഗങ്ങളെയും യവ്മ് (ദിവസം) എന്ന പദം കൊണ്ട് വിവക്ഷിക്കാവുന്നതാണ്. ഇതിന് ഉദാഹരണങ്ങൾ ക്വുർആനിൽ തന്നെ കാണാവുന്നതാണ്:
“പിന്നീട് ഒരു ദിവസം കാര്യം അവങ്കലേക്ക് ഉയര്ന്ന് പോകുന്നു. നിങ്ങള് കണക്കാക്കുന്ന തരത്തിലുള്ള ആയിരം വര്ഷമാകുന്നു ആ ദിവസത്തിന്റെ അളവ്.” (ക്വുർആൻ: 32: 5)
“അമ്പതിനായിരം കൊല്ലത്തിന്റെ അളവുള്ളതായ ഒരു ദിവസത്തില് മലക്കുകളും ആത്മാവും അവങ്കലേക്ക് കയറിപ്പോകുന്നു.” (ക്വുർആൻ: 70: 4)
ആയിരം ഭൗമിക വർഷങ്ങളോടു തുല്യമായ ഒരു യവ്മ് (ദിവസം), ഭൗമികമായ അമ്പതിനായിരം കൊല്ലത്തിന്റെ അളവുള്ളതായ ഒരു യവ്മ് (ദിവസം) എന്നൊക്കെ ക്വുർആൻ പറഞ്ഞതിൽ നിന്നും കാലത്തിന്റെ ഏതു തരം ഘട്ടങ്ങളെയും ഭാഗങ്ങളെയും യവ്മ് (ദിവസം) എന്ന പദം കൊണ്ട് വിവക്ഷിക്കാവുന്നതാണ് എന്ന് വ്യക്തം.
ഈ വസ്തു മനസ്സിലാക്കിയതിന് ശേഷം ശൈഖ് സ്വാലിഹ് അൽ മുനജ്ജിദ് എഴുതുന്നത് വായിക്കുക:
“അല്ലാഹു ആകാശങ്ങളെ രണ്ട് അഭൗമികമായ ദിവസം കൊണ്ട് (അഥവാ രണ്ട് കാല- ഘട്ടം കൊണ്ട്) സൃഷ്ടിച്ചു. ശേഷം ഭൂമിയെ രണ്ട് അഭൗമികമായ ദിവസം കൊണ്ട് (അഥവാ രണ്ട് കാല- ഘട്ടം കൊണ്ട്) സൃഷ്ടിച്ചു. ശേഷം ഭൂമിയിലെ വിശദാംശങ്ങൾ രണ്ട് അഭൗമികമായ ദിവസം കൊണ്ട് (അഥവാ രണ്ട് കാല- ഘട്ടം കൊണ്ട്) സൃഷ്ടിച്ചു. അപ്പോൾ മൊത്തം ആറ് അഭൗമികമായ ദിവസങ്ങൾ കൊണ്ട് ആകാശ ഭൂമികളെ സൃഷ്ടിച്ചു.
ഭൂമിയിലെ വിശദാംശങ്ങൾ സൃഷ്ടിച്ച രണ്ട് അഭൗമികമായ ദിവസങ്ങൾ, ഏഴ് ഭൗമികമായ ദിവസങ്ങൾക്ക് തുല്യമാണ്. ഈ രണ്ട് അഭൗമികമായ ദിവസങ്ങളിൽ ( അഥവാ ഏഴ് ഭൗമികമായ ദിവസങ്ങളിൽ ) നടന്ന സൃഷ്ടിപ്പിനെ സംബന്ധിച്ചാണ് ഹദീസിൽ ചർച്ച ചെയ്യപ്പെടുന്നത്.
ആകാശ ഭൂമികളെ ആറ് ദിവസങ്ങൾ കൊണ്ട് സൃഷ്ടിച്ചു എന്ന് ക്വുർആൻ പറയുന്ന ദിവസങ്ങൾ സൂര്യ ചന്ദ്ര ഉദയാസ്തമങ്ങൾ കൊണ്ട് അളക്കപ്പെടുന്ന ഭൗമികമായ ദിവസങ്ങൾ അല്ല. ആ സമയത്ത് സൂര്യനും ഭൂമിയും സൃഷ്ടിക്കപ്പെട്ടിട്ടു തന്നെയില്ലല്ലൊ; പിന്നെ അതെങ്ങനെ ഭൗമികമായ ദിവസങ്ങൾ ആവും ?! യവ്മ് (ദിവസം) എന്നാൽ കാലത്തിന്റെ അപേക്ഷികമായ ഭാഗങ്ങൾ അല്ലെങ്കിൽ ഘട്ടങ്ങൾ മാത്രമാണ്.”
ശൈഖ് നാസ്വിറുദ്ധീൻ അൽബാനി പറഞ്ഞു:
“ഹദീസിൽ പറയപ്പെട്ടിട്ടുള്ള ഏഴ് ദിവസങ്ങൾ, (ആകാശഭൂമികൾ സൃഷ്ടിക്കപ്പെട്ട,) ക്വുർആനിൽ പ്രസ്ഥാപിക്കപ്പെട്ട ആറ് ദിവസങ്ങളല്ല. ഹദീസിൽ പ്രസ്ഥാവിക്കപ്പെട്ടിരിക്കുന്നത് ഭൂമിൽ അല്ലാഹു നടപ്പാക്കിയ സംവിധാനങ്ങളുടെ വിശതമായ വിവരണമാണ്. ക്വുർആനിൽ പ്രസ്ഥാപിക്കപ്പെട്ടതിന് പുറമെ – അധികമായി – നടന്ന സൃഷ്ടിപ്പുകളും സംവിധാനങ്ങളുമാണ് ഹദീസിലെ പ്രമേയം; അല്ലാതെ ക്വുർആനിൽ പ്രസ്ഥാപിക്കപ്പെട്ടതിന് എതിരായ വിവരങ്ങളല്ല… ” (മുഖ്തസ്വറുൽ ഉലുവ് ലിഅലിയ്യുൽ അളീം: 112)
ക്വുർആനിലെ ആറും ഹദീസിലെ ഏഴും, ഇവ രണ്ടും (ഭൗമികവും അഭൗമികവുമായ) രണ്ട് തരം ദിവസങ്ങളും, രണ്ട് വ്യത്യസ്ഥ വിഷയങ്ങളുമാണ് എന്ന് ഹദീസുകളിൽ തന്നെ വ്യക്തമാക്കപ്പെട്ടിട്ടുണ്ട്:
ഇമാം നസാഈയുടെ “സുനനുൽ കുബ്റാ” യിലെ (6/427/11392) ഒരു ഹദീസിൽ മുഹമ്മദ് നബി (സ), തന്റെ അനുചരനായ അബൂ ഹുറൈറയോട് (റ) ഇപ്രകാരം പറഞ്ഞതായി വന്നിരിക്കുന്നു: “അല്ലയൊ അബൂ ഹുറൈറ ! അല്ലാഹു അകാശഭൂമികളെയും അവക്കിടയിലുള്ളവയെയും ആറ് (അഭൗമിക) ദിവസങ്ങളിലായി (കാല ഘട്ടങ്ങളായി) സൃഷ്ടിച്ചു. ഏഴാമത്തെ ദിവസം അവൻ സിംഹാസനസ്ഥനായി. (ഭൂമിയിലെ വിശദാംശങ്ങളുടെ സൃഷ്ടിപ്പ് ഭൗമികമായ ദിവസങ്ങളിൽ പറഞ്ഞാൽ) മണ്ണ് ശനിയാഴ്ച്ച അവൻ പടച്ചു…(ശേഷം ഏഴ് ഭൗമികമായ ദിവസങ്ങളിലെ സൂക്ഷ്മായ സൃഷ്ടിപ്പുകൾ പ്രവാചകൻ (സ) എണ്ണി പറഞ്ഞു…”
ഈ ഹദീസിൽ നിന്ന് കാര്യങ്ങൾ വ്യക്തമായി മനസ്സിലാക്കാം. ആകാശ ഭൂമികളുടെ സൃഷ്ടിപ്പ് നടന്നത് അഭൗമികമായ ആറ് ദിവസങ്ങളിൽ (ഘട്ടങ്ങളിൽ) ആയാണ്. അതിൽ രണ്ട് അഭൗമിക ഘട്ടങ്ങൾ ഭൗമികമായ ഏഴ് ദിവസങ്ങൾക്ക് തുല്യമാണ്. ശേഷം ഭൗമികമായ ഓരോ ദിവസവും സൃഷ്ടിക്കപ്പെട്ട ഓരോ സൃഷ്ടിപ്പും പ്രവാചകൻ (സ) എണ്ണി പറഞ്ഞു. ശനിയാഴ്ച്ച മണ്ണ് സൃഷ്ടിച്ചു. ഞായറാഴ്ച്ച പർവ്വതങ്ങളെ ഉണ്ടാക്കി… അപ്പോൾ ക്വുർആനും ചർച്ചാ വിഷയകമായി ഹദീസും തമ്മിൽ വൈരുദ്ധ്യമില്ല എന്ന് മാത്രമല്ല. രണ്ട് തരം ദിവസങ്ങളെയും കൂട്ടിയിണക്കി തന്നെ ഒരു ഹദീസ് കാര്യങ്ങൾ വ്യക്തമാക്കുന്നുണ്ട്.
*******************************വിമർശനം:
ആകാശ ഭൂമികളെ ആറ് ദിവസങ്ങളിലായി സൃഷ്ടിച്ചു എന്ന് ക്വുർആനിൽ പലയിടത്തും പറയുന്നത്. എന്നാൽ ഇതിനോട് വിരുദ്ധമായി ആകാശ ഭൂമികളെ ഏട്ട് ദിവസങ്ങളിലായി സൃഷ്ടിച്ചു എന്ന് ക്വുർആനിൽ ഒരു അധ്യായത്തിൽ പരാമർശിക്കുന്നില്ലേ ?
മറുപടി:
“നീ പറയുക: രണ്ടുദിവസങ്ങളിലായി ഭൂമിയെ സൃഷ്ടിച്ചവനില് നിങ്ങള് അവിശ്വസിക്കുകയും അവന്ന് നിങ്ങള് സമന്മാരെ സ്ഥാപിക്കുകയും തന്നെയാണോ ചെയ്യുന്നത്? അവനാകുന്നു ലോകങ്ങളുടെ രക്ഷിതാവ്. അതില് (ഭൂമിയില്) – അതിന്റെ ഉപരിഭാഗത്ത് – ഉറച്ചുനില്ക്കുന്ന പര്വ്വതങ്ങള് അവന് സ്ഥാപിക്കുകയും അതില് അഭിവൃദ്ധിയുണ്ടാക്കുകയും, അതിലെ ആഹാരങ്ങള് അവിടെ വ്യവസ്ഥപ്പെടുത്തി വെക്കുകയും ചെയ്തിരിക്കുന്നു. നാലു ദിവസങ്ങളിലായിട്ടാണ് (അവനത് ചെയ്തത്.)ആവശ്യപ്പെടുന്നവര്ക്ക് വേണ്ടി ശരിയായ അനുപാതത്തില്. (ക്വുർആൻ: 41: 9, 10)
ഈ ആയത്തുകൾ പ്രകാരം രണ്ടുദിവസങ്ങളിലായി ഭൂമിയെ സൃഷ്ടിച്ചു. പര്വ്വതങ്ങള്, അഭിവൃദ്ധി, ആഹാരങ്ങള് തുടങ്ങിയ വിശദമായ സൃഷ്ടിപ്പുകൾ നാല് ദിവസങ്ങൾ കൊണ്ട് നടത്തി. അതിലേക്ക് ആകാശങ്ങളെ സൃഷ്ടിച്ച രണ്ട് ദിവസങ്ങൾ കൂടി ചേർത്താൽ… 2+4+2 എട്ട് ദിവസങ്ങൾ ആയില്ലെ ? ക്വുർആനിൽ മറ്റു പലയിടത്തും പറയുന്ന ആറ് ദിവസ കണക്കിന് എതിരല്ലെ ? എന്നതാണ് വിമർശനം.
ഈ വിമർശനം തെറ്റായ കണക്കുകൂട്ടലിനെ അടിസ്ഥപ്പെടുത്തിയാണ് വിമർശകർ ഉന്നയിക്കുന്നത്. ശരിയായ കണക്ക് ഇപ്രകാരമാണ്:
രണ്ടുദിവസങ്ങളിലായി ഭൂമിയെ സൃഷ്ടിച്ചു. തുടർന്ന് പര്വ്വതങ്ങള്, അഭിവൃദ്ധി, ആഹാരങ്ങള് തുടങ്ങിയ വിശദമായ സൃഷ്ടിപ്പുകൾ നടത്തി. فِي أَرْبَعَةِ أَيَّامٍ “നാലു ദിവസങ്ങളിലായിട്ടാണ് (അവനത് ചെയ്തത്.)” (ക്വുർആൻ: 41:10) അഥവാ നാലു ദിവസങ്ങളിലായിട്ടാണ് ഭൂമിയെയും തുടർന്ന് പര്വ്വതങ്ങള്, അഭിവൃദ്ധി, ആഹാരങ്ങള് തുടങ്ങിയ വിശദമായ കാര്യങ്ങളെയും സൃഷ്ടിച്ചത്. ഇതിലേക്ക് ആകാശങ്ങളെ സൃഷ്ടിച്ച രണ്ട് ദിവസങ്ങൾ കൂടി ചേർത്താൽ… 2+2+2 ആറ് ദിവസങ്ങൾ. 2+2(അഥവാ 4)+2 = 6.
ഇസ്ലാം വിമർശനങ്ങൾ ഉയർന്നു വരുന്നതിന് നൂറ്റാണ്ടുകൾ എത്രയൊ മുമ്പ് ഭാഷാ പണ്ഡിതരായ ക്വുർആൻ വ്യാഖ്യാതാക്കൾ ഈ കണക്ക് വ്യക്തമായി നമ്മെ കൂട്ടി പഠിപ്പിച്ചിട്ടുണ്ട്. അൽപം ചില ഉദാഹരണങ്ങൾ ഇവിടെ ചേർക്കാം:
ഇമാം ക്വുർതുബി (ജനനം: 1214 CE) പറഞ്ഞു: “ഭൂമിയും അതിലെ വിശദാംശങ്ങളും മൊത്തം നാല് ദിവസങ്ങൾ (ഘട്ടങ്ങൾ) കൊണ്ട് സൃഷ്ടിച്ചു എന്നാണ് ഇവിടെ ഉദ്ദേശിച്ചിരിക്കുന്നത്. ഉദാഹരണത്തിന് ഒരാൾ ഇപ്രകാരം പറഞ്ഞു എന്ന് കരുതുക: “ഞാൻ ബസ്വറയിൽ നിന്ന് ബഗ്ദാദിലേക്ക് പത്ത് ദിവസം കൊണ്ട് യാത്ര ചെയ്ത് എത്തി. കൂഫയിലേക്ക് പതിനഞ്ച് ദിവസങ്ങൾ കൊണ്ട് എത്തി. മൊത്തം പതിനഞ്ച് ദിവസം യാത്ര എടുത്തു.” (അൽ ജാമിഉ ലി അഹ്കാമിൽ ക്വുർആൻ: 15: 343)
കാരണം ബസ്വറയിൽ നിന്നും കൂഫയിലേക്കുള്ള യാത്രക്ക് ഇടയിലാണ് ബഗ്ദാദ്. ബഗ്ദാദ് യാത്ര, കൂഫയിലേക്കുള്ള യാത്രയുടെ ഒരു ഭാഗം മാത്രമാണ്. നമ്മുടെ നാട്ടിലെ ഒരു ഉദാഹരണം പറഞ്ഞാൽ ഇങ്ങനെയുണ്ടാവും: ഞാൻ കാസർകോട് നിന്നും മലപ്പുറം വരെ അഞ്ച് ദിവസം കൊണ്ട് യാത്ര ചെയ്തു. കാസർകോട് നിന്നും തിരുവനന്തപുരം വരെ പത്ത് ദിവസം കൊണ്ട് യാത്ര ചെയ്തു. മൊത്തം പത്ത് ദിവസങ്ങളാണ് യാത്ര; പതിനഞ്ചല്ല.
ഇമാം ബഗ്വി (ജനനം: 1044 CE) പറയുന്നു: ഭൂമിയും അതിലെ വിശദാംശങ്ങളും മൊത്തം നാല് ദിവസങ്ങൾ (ഘട്ടങ്ങൾ) കൊണ്ട് സൃഷ്ടിച്ചു എന്നാണ് ഇവിടെ ഉദ്ദേശിച്ചിരിക്കുന്നത്. ഭൂമി രണ്ട് ദിവസങ്ങൾ (ഘട്ടങ്ങൾ) കൊണ്ട് സൃഷ്ടിച്ചു. ഭൂമിയിലുള്ള സൂക്ഷ്മമായ സൃഷ്ടിപ്പുകൾ -ഭൂമി സൃഷ്ടിച്ച രണ്ട് ദിവസങ്ങൾ കൂടി ചേർത്ത് – നാല് ദിവസങ്ങൾ കൊണ്ട് സൃഷ്ടിച്ചു. അപ്പോൾ മൊത്തം നാല് ദിവസങ്ങൾ കൊണ്ട് അതിലെ വിശദാംശങ്ങളും സൃഷ്ടിച്ചു എന്നർത്ഥം; ആറ് ദിവസങ്ങൾ കൊണ്ടല്ല. (തഫ്സീറുൽ ബഗ്വി: 7:165)
അസ്സജാജ് പറഞ്ഞു: (ജനനം: 855 CE) وقال الزجاج : في تتمة أربعة أيام ، يريد بالتتمة اليومين . “രണ്ട് (അഭൗമിക) ദിവസങ്ങൾ കൊണ്ട് ഭൂമിയും രണ്ട് (അഭൗമിക) ദിവസങ്ങൾ കൊണ്ട് ഭൂമിയിലെ വിശദാംശങ്ങളും സൃഷ്ടിച്ചു. മൊത്തം നാല് (അഭൗമിക) ദിവസങ്ങൾ കൊണ്ട് ഭൂമിയും ഭൂമിയിലെ സൂക്ഷ്മമായ കാര്യങ്ങളും സൃഷ്ടിച്ചു എന്നർത്ഥം.”ഇത് അറബി ഭാഷാപണ്ഡിതനായ സമഖ്ശരി (ജനനം: 1074 CE) തന്റെ “കശ്ശാഫ്” എന്ന ഗ്രന്ഥത്തിൽ (3: 444) ഉദ്ധരിക്കുന്നു.
ഇന്ത്യൻ മുസ്ലിംകളുടെ ദേശക്കൂറും വിശ്വസ്തതയും ചോദ്യം ചെയ്തു കൊണ്ട് വർഗീയ ദ്രുവീകരണം സൃഷ്ടിച്ച് രാഷ്ട്രീയ നേട്ടങ്ങൾ കൊയ്യാൻ സംഘ് പരിവാറും കൂട്ടാളികളും അശ്രാന്ത പരിശ്രമത്തിലാണ്. ഇസ്ലാമിനെതിരെ നുണകളും അർദ്ധ സത്യങ്ങളും പ്രചരിപ്പിക്കുന്നതിനു പുറമെ ഇസ്ലാമിക പ്രമാണങ്ങളെ ദുർവ്യാഖ്യാനിച്ചും തെറ്റിദ്ധരിപ്പിച്ചും ജനമനസ്സുകളിൽ ഇസ്ലാം ഭീതി പടർത്താൻ നിരന്തരം പ്രയത്നിച്ചു കൊണ്ടിരിക്കുന്ന വെറുപ്പിന്റെ അപ്പോസ്തലന്മാർ ഏറ്റവുമൊടുവിൽ പുതിയ കുപ്പിയിലാക്കി ഇറക്കുമതി ചെയ്ത പഴയ വീഞ്ഞാണ് ‘ഗസ്വത്തുൽ ഹിന്ദ്’ (ഹിന്ദ് യുദ്ധം). ഇന്ത്യയോട് യുദ്ധം ചെയ്യാൻ പ്രവാചക കൽപ്പനയുണ്ടെന്നും ഏതു നിമിഷവും പൊട്ടാവുന്ന ബോംബാണ് മുസ്ലിംകളെന്നുമാണ് ദേശസ്നേഹികളും സമാധാന ചിത്തരുമായ ഇന്ത്യൻ മുസ്ലിംകളുടെ തലയിൽ വെച്ചുകെട്ടുന്ന പുതിയ ആരോപണം. ഈ അവസരത്തിൽ വാദപ്രതിവാദങ്ങളുടെ വൈകാരിക തലം ഒട്ടും സ്പർശിക്കാതെ, വിവാദ വിഷയകമായ ഹദീസിനെ സംബന്ധിച്ച ഒരു വൈചാരികമായ ചർച്ചയാണ് ഈ കുറിപ്പ് ലക്ഷ്യം വെക്കുന്നത്. ﻋﺼﺎﺑﺘﺎﻥ ﻣﻦ ﺃﻣﺘﻲ ﺃﺣﺮﺯﻫﻤﺎ اﻟﻠﻪ ﻣﻦ اﻟﻨﺎﺭ: ﻋﺼﺎﺑﺔ ﺗﻐﺰﻭ اﻟﻬﻨﺪ ﻭﻋﺼﺎﺑﺔ ﺗﻜﻮﻥ ﻣﻊ ﻋﻴﺴﻰ ﺑﻦ ﻣﺮﻳﻢ ﻋﻠﻴﻪ اﻟﺴﻼﻡ. “എന്റെ സമുദായത്തിലെ രണ്ട് സംഘത്തെ അല്ലാഹു നരകത്തിൽ നിന്നും സംരക്ഷിക്കും. ‘ഹിന്ദി’നോട് (ഇന്ത്യ) യുദ്ധം ചെയ്യുന്ന സംഘവും ഈസബ്നു മർയത്തോടൊപ്പം(അ) ഉണ്ടാകുന്ന സംഘവുമാണത്.” (നസാഈ: 2/64)
‘ഹിന്ദു’മായുള്ള യുദ്ധത്തെ സംബന്ധിച്ച് ചില ഹദീസ് ഗ്രന്ഥങ്ങളിൽ ഉദ്ധരിക്കപ്പെട്ടിട്ടുള്ള ഒരു നിവേദനമാണ് നാം മുകളിൽ ഉദ്ധരിച്ചത്. ഹദീസ് ‘സ്വഹീഹ്’ (സ്വീകാര്യതയുടെ ഹദീസ് നിദാന ശാസ്ത്ര മാനദണ്ഡങ്ങൾ പൂർത്തീകരിക്കപ്പെട്ടത്) ആണ് എന്ന് വന്നാൽ തന്നെ ഹദീസിന്റെ ഉള്ളടക്കത്തിൽ, ഇസ്ലാമോഫോബിയ പ്രചാരകർ ഊതി വീർപ്പിച്ച് ഉരുട്ടി കാണിക്കുന്നതു പോലെ ഇന്ത്യക്ക് ഭീഷണിയായ ഒന്നും തന്നെ അടങ്ങിയിട്ടില്ല എന്നതാണ് മറ്റൊരു സത്യം. ഹദീസിലെ ‘ഹിന്ദ്’ (اﻟﻬﻨﺪ) എന്ന പദത്തിനാണ് ‘ഇന്ത്യ’ എന്ന് പരിഭാഷ നൽകപ്പെടാറുള്ളത്.
In ancient times, India was much more extended to the North West and west (consisting of parts of modern Pakistan and Afghanistan). (https://www.culturalindia.net)
പൗരാണിക ഇന്ത്യയിൽ അധുനിക പാകിസ്ഥാനും അഫ്ഗാനിസ്ഥാന്റെ പല ഭാഗങ്ങളും ഉൾകൊണ്ടിരുന്നു എന്ന് നമുക്കേവർക്കും അറിയാമല്ലൊ. (ആധുനിക പാക്കിസ്ഥാനിൽ ഉൾപ്പെടുന്ന) സിന്ദു നദി തീരപ്രദേശങ്ങളെയാണ് പൗരാണിക കാലത്ത് ‘ഹിന്ദ്’ കൊണ്ട് ഉദ്ദേശിക്കപ്പെട്ടിരുന്നത്, നമ്മുടെ ഇന്നത്തെ ഇന്ത്യയെയല്ല എന്നതിനാൽ തന്നെ ‘ഹിന്ദി’നോടുള്ള യുദ്ധം എന്നതുകൊണ്ട് (ആധുനിക) ഇന്ത്യയോടുള്ള യുദ്ധമല്ല എന്ന് തിരിച്ചറിയാൻ വലിയ പ്രയാസമൊന്നുമില്ല.
കൂടാതെ, പ്രവാചക കാലഘട്ടത്തിലാവട്ടെ -പ്രവാചകനും പ്രവാചകാനുചരന്മാരും ഉൾപ്പെടെ- അറബികൾ ‘ഹിന്ദ്’ (ഇന്ത്യ) എന്ന് വിളിച്ചിരുന്നത് ആധുനിക ഇന്ത്യയെയല്ല. ഇന്ത്യയുടെ അന്നത്തെ ഭൂമിശാസ്ത്ര ഘടന പ്രകാരമായാലും ശരി പൗരാണിക അറേബ്യൻ മുസ്ലിംകളുടെ സാങ്കേതിക ഭാഷ പ്രകാരമായാലും ശരി, ഹദീസിൽ പറയപ്പെട്ടിട്ടുള്ള ‘ഹിന്ദ്’ അഥവാ ‘ഇന്ത്യ’ നമ്മുടെ രാജ്യമായ ഇന്ത്യ (ഭാരതം) അല്ലേയല്ല.
പൂർവ്വസൂരികളായ മുസ്ലിംകൾ ‘ഹിന്ദ്’ (ഇന്ത്യ) എന്ന് വിളിച്ചിരുന്നത് ‘ബസ്വറ’യെയാണ്. ഇന്നത്തെ ഇറാക്കിലെ പ്രസിദ്ധമായ ഒരു പട്ടണമാണ് ‘ബസ്വറ’. ‘ബസ്വറ’ ഉൾപ്പെടെയുള്ള പൗരാണിക ‘ഇറാക്’ -പ്രവാചക കാലഘട്ടത്തിൽ- പേർഷ്യൻ സാമ്രാജ്യത്തിന്റെ ഭാഗമായിരുന്നു. (https://mawdoo3.com)
അറേബ്യയിൽ നിന്നും ഇന്ത്യയിലേക്കുള്ള കരമാർഗ്ഗത്തിലാണ് ബസ്വറ നിലകൊള്ളുന്നത് എന്നത് കൊണ്ട്, ഇന്ത്യയുടെ ദിക്കിലുള്ള നാട് എന്ന നിലയിൽ ‘ബസ്വറ’യെ പ്രവാചക കാലഘട്ടം ‘ഹിന്ദ്’ (ഇന്ത്യ) എന്ന് വിളിച്ചു.
ﻭﻛﺎﻧﻮا ﻳﺴﻤﻮﻥ اﻟﺒﺼﺮﺓ ﻫِﻨﺪًا، ﻷﻧﻬﺎ ﻣﻦ ﺟﻬﺔ اﻟﻬﻨﺪ، ﻭﻣﻨﻬﺎ ﻳُﺴﻠﻚ ﺇِﻟَﻰ اﻟﻬﻨﺪ، ﻭﻟﻬﺬا ﻗﺎﻝ ﺧﺎﻟﺪ ﻟﻤﺎ ﻋﺰﻟﻪ ﻋﻤﺮ ﻋﻦ اﻟﺸﺎﻡ: ﺇﻥ ﻋﻤﺮ ﺃﻣﺮﻧﻲ ﺃﻥ [ ﺁﺗﻲ] اﻟﻬﻨﺪ. ﻗﺎﻝ اﻟﺮﻭاﻱ: ﻭﻛﺎﻧﺖ اﻟﻬﻨﺪ ﻋﻨﺪﻧﺎ اﻟﺒﺼﺮﺓ. ഇബ്നു റജബ് എഴുതി: പൂർവ്വകാല മുസ്ലിംകൾ ‘ബസ്വറ’ക്ക് ‘ഹിന്ദ്’ എന്നായിരുന്നു പേര് വെച്ചിരുന്നത്. ‘ബസ്വറ’, ഇന്ത്യയുടെ ദിക്കിലായതു കൊണ്ടും ബസ്വറയിലൂടെയാണ് ഇന്ത്യയിലേക്ക് എത്താനുള്ള കരമാർഗം എന്നതുകൊണ്ടുമായിരുന്നു അത്. ഉമർ, ഖാലിദിനെ ശാമിൽ നിന്നും നീക്കിയപ്പോൾ ഖാലിദ് ഇപ്രകാരം പറഞ്ഞത് അതുകൊണ്ടാണ്: എന്നോട് ഉമർ ‘ഹിന്ദി’ലേക്ക് (ഇന്ത്യ) ചെല്ലാൻ കൽപ്പിച്ചു. നിവേദകൻ പറയുന്നു: ‘ഹിന്ദ്’ (ഇന്ത്യ) എന്നാൽ ഞങ്ങളുടെ അടുക്കൽ ‘ബസ്വറ’യായിരുന്നു. (മജ്മൂഉ റസാഇലു ഇബ്നു റജബ്: 3:205)
‘ബസ്വറ’ക്കടുത്ത ‘ഉബുല്ല’ എന്ന സ്ഥലത്തെ ‘ഇന്ത്യൻ പുൽത്തകിടി’ (مرج الهند) എന്നാണ് വിളിക്കപ്പെട്ടിരിക്കുന്നത് എന്ന് ഇബ്നു ഖൽദൂൻ തന്റെ ‘താരീഖ്’ (2:507) ൽ പ്രസ്ഥാവിക്കുന്നുണ്ട്.
ഒട്ടനവധി ഹദീസ്-ചരിത്ര ഗ്രന്ഥങ്ങളിൽ ഉദ്ധരിക്കപ്പെട്ട സുപ്രധാനമായ ഒരു നിവേദനം ഇപ്രകാരമാണ്: “പ്രവാചക ശിഷ്യൻ ഖാലിദിബ്നു വലീദ്(റ) പറഞ്ഞു: ശാം അതിന്റെ സമൃതി ഇട്ടു തന്നതിന് ശേഷം വിശ്വാസികളുടെ നേതാവ്, ഉമർ ബിൻ ഖത്താബ് എനിക്ക് കത്തെഴുതി, ഞാൻ ഹിന്ദിലേക്ക് (ഇന്ത്യ) സഞ്ചരിക്കാൻ കൽപ്പന നൽകി – ഹിന്ദ് (ഇന്ത്യ) എന്നാൽ ഞങ്ങളുടെ മനസ്സിൽ ബസ്വറയാണ് – എനിക്കാകട്ടെ ഹിന്ദിലേക്ക് പോകാൻ വൈമനസ്യമുണ്ടായിരുന്നു…” (മുസ്നദു അഹ്മദ്: 4:90, ദലാഇലുന്നുബുവ്വ: 6:387, ജാമിഉൽ മസാനിദ്: 2: 389, അൽ ജിഹാദ്: ഇബ്നു അബീ ആസിം: 2: 666, മുഅ്ജമുൽ കബീർ: ത്വബ്റാനി: 4:137, മുഅ്ജമുൽ അവ്സത്വ് : 8:277, അൽ മഅ്’രിഫതു വത്താരീഖ് : ഫസ്വി: 3:115-116, അൽമുത്തഫകു വൽമുഫ്തറകു:3:1743-1744, താരീഖു ദിമശ്ക്: ഇബ്നു അസാകിർ: 40:310. ഒരു ഹദീസ് ആയിട്ടല്ലെങ്കിൽ, ചരിത്രപരമായ ഭൂമിശാസ്ത്ര (Historical geography) സാക്ഷ്യമായെങ്കിലും ഈ നിവേദനം പരിഗണിക്കപ്പെടേണ്ടതുണ്ട്.)
ﺃﻥ اﻟﺴﻠﻒ اﻟﺼﺎﻟﺢ «ﻛﺎﻧﻮا ﻳﺴﻤﻮﻥ (اﻟﺒﺼﺮﺓ) (ﻫﻨﺪاً)… ﺇﻥ اﻟﻬﻨﺪ ﻛﺎﻧﺖ ﻓﻲ ﻧﻔﻮﺳﻬﻢ اﻟﺒﺼﺮﺓ، ﻭﺑﻪ ﺗﻔﻬﻢ ﺳﺎﺋﺮ اﻷﺣﺎﺩﻳﺚ اﻟﻮاﺭﺩِ ﻓﻴﻬﺎ ﺫﻛﺮُ (اﻟﻬﻨﺪ) .
അബൂ ഉബൈദ മശ്ഹൂറിബ്നു ഹസൻ ഇബ്നു മഹ്മൂദ് ആലു സൽമാൻ വ്യക്തമാക്കുന്നു:
“പൂർവ്വസൂരികളായ സച്ഛരിതർ ‘ബസ്വറ’ക്ക് പേര് നൽകിയിരുന്നത് ‘ഇന്ത്യ’ (ഹിന്ദ്) എന്നായിരുന്നു… തീർച്ചയായും ഇന്ത്യയെന്നാൽ (ഹിന്ദ്) അവരുടെ മനസ്സിൽ ബസ്വറയാണ്. ഇന്ത്യയെ (ഹിന്ദ് ) സംബന്ധിച്ച് സ്മരിക്കുന്ന എല്ലാ ഹദീസുകളും ഇപ്രകാരം തന്നെയാണ് മനസ്സിലാക്കേണ്ടത്.” (അൽ ഇറാക്ക് ഫിൽ അഹാദീസി വ ആസാറുൽ ഫിതൻ: 1:360-364)
മസ്ഊദി (മരണം:346 ഹിജ്റ) പറഞ്ഞു: ബസ്വറയിൽ അതബതുബ്നു ഗസ്വാൻ ഹിജ്റ 16 അല്ലെങ്കിൽ 17 ന് ഖലീഫയുടെ കല്പന പ്രകാരം കടന്നു വന്നു. അന്ന് ബസ്വറയെ വിളിക്കപ്പെട്ടിരുന്നത് ‘ഇന്ത്യൻ ഭൂമി’ (അർദുൽ ഹിന്ദ് أرض الهند) എന്നായിരുന്നു. അത് വെളുത്ത പാറകളും ചരൽക്കല്ലുകളും നിറഞ്ഞ ഒരു ഭൂമിയായിരുന്നു. അതബതുബ്നു ഗസ്വാൻ ആണ് അതിനെ ഒരു പട്ടണമായി വാർത്തെടുക്കുന്നത്. ബസ്വറയും ചുറ്റുപാടുമുള്ള മറ്റു പട്ടണങ്ങളിൽ നിന്നും പേർഷ്യൻ സാമ്രാജ്യത്തേക്ക് ചെല്ലുന്ന യുദ്ധ സന്നാഹങ്ങളും സഹായങ്ങളും തടയുക കൂടി ബസ്വറക്ക് മേൽ ഉള്ള വിജയത്തിന് പിന്നിലെ ലക്ഷ്യത്തിൽ പെട്ടതായിരുന്നു.. (താരീഖുത്വബ്രി: 3:596, അത്തംബീഹ് വൽഇഷ്റാഫ് :1:310,)
At the time of the Muhammadan conquest, the county about Basra was called Arz-ul-Hind, the Land of India…
“മുഹമ്മദിയ അധിനിവേശ കാലഘട്ടത്തിൽ, ബസ്രയെ ‘അർസ്-ഉൾ-ഹിന്ദ്’, ഇന്ത്യൻ നാട് എന്നാണ് വിളിക്കപ്പെട്ടിരുന്നത്… ” എന്ന് പല ഇന്ത്യൻ ചരിത്രകാരന്മാരും വ്യക്തമാക്കിയിട്ടുണ്ട്. (The Indian Encyclopedia: Edited by Subodh Kapoor: Cosmo Publication: New Delhi: 2002, Vol: page: 4718)
ബ്രിട്ടീഷ് ആർമി ഓഫീസറും ഒറിയന്റലിസ്റ്റും പേർഷ്യൻ ഭാഷാ പണ്ഡിതനുമായ സർ എച്ച്. റൗളിൻസണും ഫ്രഞ്ച് ഒറിയന്റലിസ്റ്റും ചരിത്രകാരനുമായ ഹെൻറി കോർഡിയറും ഈ വസ്തുതയിലേക്ക് വിരൽ ചൂണ്ടുന്നുണ്ട്. (Cathay and the Way Thinker. Being a Collection of Medieval Notices of China)
പേർഷ്യൻ സാസാനിയ്യ ഭരണകൂടത്തിന്റെ കാലഘട്ടത്തിൽ (226 AD-651 AD) ഇറാഖിന്റെ ദക്ഷിണ ഭാഗത്തും ടൈഗ്രിസ് നദിയുടെ ഇടയിലുമായി ‘മീഷാൻ’ എന്ന പേരിൽ ഒരു ‘അമീർ ഭരണം’ (Emirate) നിലനിന്നിരുന്നു. മീഷാൻ രാജ്യത്തിന്റെ കേന്ദ്രസ്ഥാനം ബസ്വറയുടെ പേർഷ്യൻ ഗൾഫ് തീരത്തായിരുന്നു. ബസ്വറയുൾപ്പെടുന്ന ഇത്തരം എമിറേറ്റുകൾക്ക് ആ കാലഘട്ടത്തിൽ പേർഷ്യൻ സാമ്രാജ്യത്തിൽ തന്ത്രപ്രധാനവും അതി പ്രസക്തവുമായ സ്ഥാനമുണ്ടായിരുന്നു. ഇറാഖിൽ പേർഷ്യൻ സാമ്രാജ്യത്വ ശക്തികളോട് മുസ്ലിംകൾ ഏറ്റുമുട്ടുന്ന ആദ്യത്തെ യുദ്ധങ്ങളാണ് ഹഫീർ, ദാത്തു സലാസിൽ യുദ്ധങ്ങൾ. യുദ്ധത്തിനിടയിൽ തങ്ങളുടെ സൈന്യത്തിൽ നിന്നും ആരും ഓടി രക്ഷപ്പെടാതിരിക്കാൻ വലിയ ചങ്ങല വലയങ്ങളുമായാണ് പേർഷ്യക്കാർ യുദ്ധത്തിന് വന്നത് എന്നതിനാൽ ‘ദാത്തു സലാസിൽ’ അഥവാ ‘ചങ്ങല കെട്ടുകളുടെ യുദ്ധം’ എന്നാണ് ആ പോരാട്ടത്തെ മുസ്ലിംകൾ വിളിച്ചത്. (മുഖ്തസറു താരീഖിൽ ബസ്വറ :അലീ ളരീഫ് അൽ അഅ്സമി: 7-12)
ബസ്വറയെ ഇന്ത്യ എന്നായിരുന്നു പ്രവാചക കാലഘട്ടത്തിൽ വിളിക്കപ്പെട്ടിരുന്നത് എന്നത് ഒട്ടനവധി ചരിത്രജ്ഞർ തങ്ങളുടെ വിശ്വപ്രസിദ്ധ ചരിത്ര ഗ്രന്ഥങ്ങളിൽ വ്യക്തമാക്കിയിട്ടുണ്ട്:
* അബൂ യുസുഫ് (മരണം: 182: ഹിജ്റ): ‘അൽഖറാജ്’ (1:73).
* ഇബ്നു സഅ്ദ് (മരണം: 230: ഹിജ്റ): ‘ത്വബകാത്ത്’ (7:3).
* ഖലീഫ ബിൻ ഖയ്യാത്ത് (മരണം: 240: ഹിജ്റ): ‘താരീഖ് ‘ (1:117).
* ത്വബ്രി (മരണം: 310: ഹിജ്റ): ‘താരീഖ് ‘ (3:591).
* അദ്ദാരിമി അൽ ബുസ്തി (മരണം :354 ഹിജ്റ): ‘അസ്സീറത്തുന്നബവിയ്യ വ അഖ്ബാരിൽ ഖുലഫാ’ (2:476).
* മുത്വഹ്ഹിർ ഇബ്നു ത്യാഹിർ അൽമക്ദസി (മരണം: 355 ഹി): ‘അൽബദ്ഉ വത്താരീഖ്’ (5:175).
* ഇബ്നുൽ അസീർ (മരണം: 630 ഹി): ‘അൽ കാമിൽ ഫിത്താരിഖ് ‘ (2:316).
* ദഹബി (മരണം: 748 ഹി): ‘സിയറു അഅ്ലാമിന്നുബലാഅ്’ (2:393).
* ഇബ്നു കസീർ (മരണം: 774 ഹി): ‘അൽ ബിദായ വന്നിഹായ’ (7:57).
ഖലീഫ ഉമറിന്റെ കാലഘട്ടത്തിലാണ് ബസ്വറ മുസ്ലിംകൾ വിജയിച്ചടക്കുന്നതും പട്ടണമാക്കുന്നതും എന്നും ഇതിനെ പറ്റി പ്രവാചകൻ (സ) സുവിശേഷമറിയിച്ചിട്ടുണ്ട് എന്നും ഇബ്നുൽ വർദ്ദി (മരണം: 749) തന്റെ ‘താരീഖിൽ’ (1:137) രേഖപെടുത്തുന്നു. ഇന്ത്യയെ സംബന്ധിച്ച, നാം ചർച്ച ചെയ്യുന്ന ഹദീസിനെ സംബന്ധിച്ചാണ് ഇബ്നുൽ വർദ്ദി സൂചിപ്പിക്കുന്നത്.
വെള്ള കലർന്ന മിനുസമുള്ള പാറയെയും കല്ലുകളേയുമാണ് ‘ബസ്റ’ (اﻟﺒَﺼْﺮَﺓُ) എന്ന് അറബിയിൽ വിളിക്കപ്പെടുന്നത്. വളരെ കട്ടിയുള്ള ഭൂമിയെ ‘അൽ ബസ്റു’ എന്ന് പറയുമെന്ന് കസ്സാസ് തന്റെ ‘ജാമിഅ്’ ൽ ഭാഷാ പണ്ഡിതന്മാരിൽ നിന്നും ഉദ്ധരിക്കുന്നു. ഈ ഭൂമി മുസ്ലിംകൾ വിജയിച്ചടക്കിയപ്പോൾ നിറയെ കല്ലുകൾ ഉള്ളതിനാൽ അവർ ആ നാടിന് ‘ബസ്വറ’ എന്ന് പേര് മാറ്റി വിളിക്കാൻ തുടങ്ങി എന്ന് ഒട്ടനവധി ഭാഷാ പണ്ഡിതരും ചരിത്രകാരൻമാരും രേഖപ്പെടുത്തുന്നുണ്ട്. (താജുൽ ഉറൂസ്: 10:203, ഉംദത്തുൽ ക്വാരി: 6:57)
ശർഖി ഇബ്നുൽ ക്വുത്വാമി പറഞ്ഞു: മുസ്ലിംകൾ ബസ്വറയിലേക്ക് വന്നപ്പോൾ ദൂരെ നിന്ന് നിരീക്ഷിച്ച സന്ദർഭത്തിൽ കുറെ കല്ലുകളാണ് അവർ കണ്ടത്. അപ്പോൾ അവർ പറഞ്ഞു: ഇത് കല്ലുകൾ നിറഞ്ഞ ഭൂമി അഥവാ ബസ്വറ ആണ്. അങ്ങനെയാണ് മുസ്ലിംകൾക്കിടയിൽ ആ നാടിന് അപ്രകാരം പേര് വന്നത്. (താരീഖു മദീനത്തുൽ ബസ്വറ: അബ്ദുല്ലാഹിബ്നു ഈസബ്നു ഇസ്മാഈൽ അന്നജ്ദി :19)
ഹിന്ദ് അഥവാ ബസ്വറയോടുള്ള യുദ്ധം എന്തുകൊണ്ട് പ്രോത്സാഹിപ്പിക്കപ്പെട്ടു? ഹിന്ദ് അഥവാ ബസ്വറയോട് യുദ്ധം എന്തുകൊണ്ട് പ്രോത്സാഹിപ്പിക്കപ്പെട്ടു? എന്നതാണ് അവശേഷിക്കുന്ന ചോദ്യം.
മതേതരത്വവും ജനാധിപത്യവും രാഷ്ട്രങ്ങളുടെ ചട്ടക്കൂടും രാഷ്ട്രമീമാംസയുടെ അടിസ്ഥാന ശിലയുമായ, ആധുനിക ലോക വ്യവസ്ഥയിലല്ല ഹിന്ദ് അഥവാ ബസ്വറയോട് യുദ്ധം ചെയ്യുന്നത് പ്രവാചകൻ (സ) പ്രോത്സാഹിപ്പിച്ചത്. ഇസ്ലാമിന്റെ ആവിർഭാവകാലഘട്ടത്തിൽ പ്രവാചക ശിഷ്യന്മാർ നേതൃത്വം വഹിച്ച പടയോട്ടങ്ങളുടെ ലക്ഷ്യവും പശ്ചാത്തലവും ആദ്യമായി നാം മനസ്സിലാക്കണം. ആദർശ സഹവർത്തിത്വമോ ജനാധിപത്യ മൂല്യങ്ങളോ തൊട്ടു തീണ്ടിയിട്ടില്ലാത്ത തീവ്ര മത വികാരത്തിലും അന്ധമായ മൗലികവാദത്തിലും വേരുറച്ച മതാധിഷ്ഠിത ഏകാധിപത്യ ഭരണങ്ങളായിരുന്നു (Theocratic Autocracy) അന്നത്തെ രാഷ്ട്രങ്ങളിൽ നിലനിന്നിരുന്നത്. ചരിത്രത്തിന്റെ അപൂർവം ചില ദശകളിൽ ചില നാടുകളിൽ അപൂർവ്വം ഭരണാധികാരികൾ മാത്രമെ അപര വിശ്വാങ്ങൾക്കും ആദർശങ്ങൾക്കും പ്രത്യയ ശാസ്ത്രങ്ങൾക്കും സഹവർത്തിത്വത്തിന് അവസരം നൽകിയിരുന്നുള്ളു. മതസ്വാതന്ത്ര്യവും സഹസ്ഥിതിയും വിവിധ പ്രത്യയശാസ്ത്രങ്ങളോട് സമാധാനപരമായ സഹവർത്തിത്വവുമൊക്കെ തീർത്തും അസംഭവ്യങ്ങളായിരുന്ന പൗരാണിക ലോക വ്യവസ്ഥയുടെ പശ്ചാത്തലത്തിൽ നിന്നു കൊണ്ട് വേണം ഹദീസിലെ യുദ്ധാഹ്വാനത്തെ മനസ്സിലാക്കാൻ.
ഇസ്ലാം സ്വീകരിക്കുകയോ ആചരിക്കുകയോ അതിന്റെ ചിഹ്നങ്ങൾ പ്രകടിപ്പിക്കുകയോ, ഇസ്ലാം പ്രചരിപ്പിക്കുകയോ ചെയ്യുന്നതിനുള്ള സ്വാതന്ത്ര്യം നിഷേധിക്കപ്പെടുകയും അവയുടെ പേരിൽ മർദ്ദനങ്ങളും പീഢനങ്ങളും അനുഭവിക്കുകയും ചെയ്യേണ്ടിവരുന്ന അവസ്ഥ ഇസ്ലാമികേതര രാജ്യങ്ങളിൽ ഇല്ലാതാവുകയും ഇസ്ലാം മതം പൂർണമായും ഉൾക്കൊണ്ട് ജീവിക്കാൻ കഴിയുന്ന അവസ്ഥ സംജാതമാവുകയും ചെയ്യുന്നതിന് മാത്രമായിരുന്നു ഈ യുദ്ധങ്ങൾ.
“ഫിത്ന ഇല്ലാതാകുന്നതുവരെ യുദ്ധം നടത്തി കൊള്ളുക” ( وَقَاتِلُوهُمْ حَتَّىٰ لَا تَكُونَ فِتْنَةٌ ) എന്ന ഖുർആൻ വചനത്തിന് പ്രവാചക ശിഷ്യൻ ഇബ്നു ഉമർ (റ) നൽകിയ വ്യാഖ്യാനം കാണുക:
ﻓﻌﻠﻨﺎ ﻋﻠﻰ ﻋﻬﺪ ﺭﺳﻮﻝ اﻟﻠﻪ ﺻﻠﻰ اﻟﻠﻪ ﻋﻠﻴﻪ ﻭﺳﻠﻢ ﻭﻛﺎﻥ اﻹﺳﻼﻡ قليلا، ﻓﻜﺎﻥ اﻟﺮﺟﻞ ﻳﻔﺘﻦ ﻓﻲ ﺩﻳﻨﻪ: ﺇﻣﺎ ﻗﺘﻠﻮﻩ، ﻭﺇﻣﺎ ﻳﻌﺬﺑﻮﻧﻪ، ﺣﺘﻰ ﻛﺜﺮ اﻹﺳﻼﻡ ﻓﻠﻢ ﺗﻜﻦ ﻓﺘﻨﺔ
“അല്ലാഹുവിന്റെ തിരുദൂതരുടെ(സ) കാലത്ത് ഞങ്ങള് അങ്ങനെ (ഫിത്ന ഇല്ലാതാകുന്നതുവരെ യുദ്ധം നടത്തുക എന്ന പ്രവർത്തനം) ചെയ്തിട്ടുണ്ട്. അന്ന് ഇസ്ലാം (മുസ്ലിംകള്) അല്പമായിരുന്നു. അതിനാല്, ഒരു മുസ്ലിം തന്റെ മത കാര്യത്തില് ഫിത്നക്ക് (പരീക്ഷണത്തിന്/ കുഴപ്പത്തിന്) വിധേയനാകുമായിരുന്നു. ഒന്നുകില് അവിശ്വാസികൾ അവനെ വധിക്കുമായിരുന്നു, അല്ലെങ്കില് അവിശ്വാസികൾ അവനെ മര്ദ്ദനങ്ങൾക്കും പീഢനങ്ങൾക്ക് വിധേയമാക്കുകയും ചെയ്യുമായിരുന്നു. അങ്ങനെ, ഇസ്ലാം (മുസ്ലിംകള്) വര്ദ്ധിച്ചു, അപ്പോള് ഈ ഫിത്ന ഇല്ലാതായി….’ (സ്വഹീഹുൽ ബുഖാരി: 4514)
ഇസ്ലാം സ്വീകരിക്കുവാനും ജീവിതത്തിൽ പ്രാവർത്തികമാക്കാനും, വസ്ത്രധാരണ രീതി, ബാങ്ക്, ആരാധനാലയങ്ങൾ, മതപഠനം, ആഘോഷങ്ങൾ തുടങ്ങിയ ഇസ്ലാമിക ചിഹ്നങ്ങൾ ആചരിക്കുവാനും, ഇസ്ലാമിക പ്രബോധന-പ്രചാരണങ്ങളിൽ ഏർപ്പെടാനും ഒരു വിശ്വാസിക്ക് ദൈവത്തിന്റെ ഭൂമിയിൽ എവിടെയും സാധ്യമാകുന്ന സ്ഥിതി വിശേഷം നിലവിൽ വരണം. അഥവാ മുസ്ലിം സമൂഹം ഇസ്ലാം വിരുദ്ധ രാഷ്ട്രങ്ങളിൽ നിന്ദ്യരും മർദ്ദിതരുമായി കഴിയുന്ന അവസ്ഥ ഇല്ലാതാവുകയും അല്ലാഹുവിന്റെ മതം ഭൂമിയിലെ ഏത് സാമൂഹിക വ്യവസ്ഥിതിയിലും പ്രൗഢിയോടെ പുലർത്താൻ സാധിക്കണം. (അതിനർത്ഥം മറ്റു മതങ്ങളോ ആദർശങ്ങളോ ഇസ്ലാമിന് കീഴ്പ്പെടുന്ന അവസ്ഥ സൃഷ്ടിക്കണമെന്നോ ഭൂമിയിൽ മുഴുവൻ ഇസ്ലാമിക ഭരണം സ്ഥാപിക്കപ്പെടണം എന്നോ അല്ല. ഇസ്ലാമും മുസ്ലിംകളും മറ്റു മതങ്ങളാലും ആദർശങ്ങളാലും അടിച്ചമർത്തപ്പെടുകയും നിന്ദ്യരാക്കപ്പെടുകയും ചെയ്യുന്ന ദുരവസ്ഥ ഇല്ലാതാവുകയും ഇസ്ലാമിക പ്രബോധനം സാധ്യമാവുകയും ചെയ്യണം എന്നാണ്.) അതിനു തടസ്സമായി നിൽക്കുന്ന ഓട്ടോക്രസികളോടാണ് -ഇസ്ലാമിക രാഷ്ട്രത്തിന്റെ ഭരണാധികാരി എന്ന നിലയിൽ- ചില ‘സ്വതന്ത്ര്യ സമരങ്ങൾ/ യുദ്ധങ്ങൾ’ നടത്താൻ മുഹമ്മദ് നബി (സ) ആഹ്വാനം ചെയ്തത്. ഇതാണ് ഈ സ്വാതന്ത്ര്യ യുദ്ധങ്ങളുടെ ലക്ഷ്യം.
“മര്ദ്ദനം ഇല്ലാതാവുകയും, മതം അല്ലാഹുവിന് വേണ്ടിയാവുകയും ചെയ്യുന്നത് വരെ നിങ്ങളവരോട് യുദ്ധം നടത്തിക്കൊള്ളുക. എന്നാല് അവര് (യുദ്ധത്തില് നിന്ന്) വിരമിക്കുകയാണെങ്കില് (അവരിലെ) അക്രമികള്ക്കെതിരിലല്ലാതെ പിന്നീട് യാതൊരു കയ്യേറ്റവും പാടുള്ളതല്ല.” (ഖുർആൻ: 2:193)
“എന്നാല് അവര് (യുദ്ധത്തില് നിന്ന്) വിരമിക്കുകയാണെങ്കില് (അവരിലെ) അക്രമികള്ക്കെതിരിലല്ലാതെ പിന്നീട് യാതൊരു കയ്യേറ്റവും പാടുള്ളതല്ല.” എന്ന ഭാഗം ഈ യുദ്ധങ്ങളുടെ പശ്ചാത്തലത്തിലേക്ക് കൂടുതൽ വെളിച്ചം വീശുന്നുണ്ട്. ആധുനിക മതേതര ജനാധിപത്യ രാഷ്ട്രങ്ങളിൽ ഇഷ്ടമുള്ള മതം സ്വീകരിക്കാനും അനുഷ്ഠിക്കാനും പ്രചരിപ്പിക്കാനും സ്വാതന്ത്ര്യമനുവദിക്കുന്ന അവസ്ഥ നിലനിൽക്കുമ്പോൾ ഇസ്ലാമിലെ സ്വാതന്ത്ര്യ യുദ്ധങ്ങൾക്ക് പ്രസക്തിയില്ല.
മുമ്പ് സൂചിപ്പിച്ചത് പോലെ, സഹവർത്തിത്വവും ജനാധിപത്യവും ഒരു നിലക്കും ഉൾകൊള്ളാത്ത തീവ്ര മത വികാരത്തിലധിഷ്ഠിതമായ തിയോക്രസികളായിരുന്നു ഇസ്ലാം ഉദയം കൊണ്ട കാലഘട്ടത്തിൽ ലോകത്തുടനീളം നിലനിന്നിരുന്നത്; പ്രത്യേകിച്ച് അറബ് ഉപഭൂഖണ്ഡത്തിൽ അധികാരം വാണിരുന്നത്. അറബ് ഉപഭൂഖണ്ഡത്തിലെ ഗോത്ര വ്യവസ്ഥ മതസ്വാതന്ത്രത്തെ മുഴുവനായും നിഷേധിച്ചിരുന്നു. പ്രസ്തുത സമൂഹങ്ങളിലെയും ഗോത്രങ്ങളിലേയും രാജാക്കന്മാർക്ക് പോലും സമൂഹ മനസ്സിനെതിരായ ആദർശങ്ങളോ മതമോ സ്വീകരിക്കാനും വിശ്വസിക്കാനുമുള്ള അവകാശവും സ്വാതന്ത്ര്യവും ഇല്ലാതിരിക്കാൻ മാത്രം തീവ്രമായിരുന്നു അന്ന് നിലനിന്നിരുന്ന വർഗീയതയും അസഹിഷ്ണുതയും.
റോമൻ ചക്രവർത്തിക്ക് പ്രവാചകൻ (സ) ഒരു കത്തയക്കുകയുണ്ടായി. ഹാരിസിബ്നു ഉമൈർ അൽ അസ്ദിയായിരുന്നു കത്തുമായി പുറപ്പെട്ട ദൂതൻ. റോമൻ ചക്രവർത്തിയുടെ, ശാമിലെ ഗവർണറായ ശർഹബീലിബ്നു അംറ് അൽ ഗസ്സാനി, പ്രവാചകന്റെ ദൂതനെ ബന്ദിയാക്കുകയും കെട്ടിയിട്ട് കഴുത്തറുക്കുകയും ചെയ്തു. തങ്ങളുടെ ക്രിസ്ത്യൻ സാമ്രാജ്യത്തിന്റെ അതിർത്തിയിൽ കാലു കുത്താൻ പോലും ഒരു മുസ്ലിമിനും അനുവാദമില്ലെന്ന്, ദൂതന്മാരെ വധിക്കുക എന്ന, അക്കാലഘട്ടത്തിലെ അസഹിഷ്ണുതയുടെ പാരമ്യരൂപത്തിലൂടെ റോമൻ സാമ്രാജ്യം പ്രഖ്യാപിച്ചു. ഇതാണ് ഹിജ്റ എട്ടാം വർഷം ശാമിലെ ക്രിസ്ത്യാനികളും മുസ്ലിംകളും തമ്മിൽ നടന്ന യുദ്ധമായ മുഅ്ത യുദ്ധത്തിലേക്ക് നയിച്ചത് എന്ന് ചില ചരിത്ര ഗ്രന്ഥങ്ങൾ സൂചിപ്പിക്കുന്നു. (മഗാസി: വാക്വിദി: 2:755, ത്വബകാത്തു ഇബ്നു സഅ്ദ്: 4:256)
പ്രേർഷ്യൻ സാമ്രാജ്യത്വത്തിന്റെ ആദർശ അസഹിഷ്ണുത ചരിത്ര പ്രസിദ്ധമാണ്. സ്വാതന്ത്ര്യ നിഷേധത്തിൽ പരസ്പരം മത്സരിച്ചിരുന്ന ക്രിസ്ത്യൻ റോമും പേർഷ്യൻ ജൂതന്മാരും തമ്മിലുള്ള അടിച്ചമർത്തലുകളും വർഗീയ യുദ്ധങ്ങളും കലാപങ്ങളും അക്കാലഘട്ടത്തിന്റെ പ്രധാന ഇതിവൃത്തം ആയിരുന്നു.
ഹദീസിൽ ഉദ്ദേശിക്കപ്പെട്ട ഹിന്ദ് അഥവാ ബസ്വറ ആധുനിക ഇറാഖിന്റെ ഭാഗമാണെന്നും ക്രിസ്താബ്ദം 638 ൽ ഖലീഫ ഉമറിന്റെ കാലഘട്ടത്തിൽ മുസ്ലിംകൾ വിജയിച്ചടക്കുന്നതു വരെ ഇറാഖ് പേർഷ്യൻ സാമ്രാജ്യത്വത്തിന്റെ കീഴിലായിരുന്നു എന്നും സൂചിപ്പിച്ചല്ലൊ.
എൻസൈക്ലോപീഡിയ ഇറാനിക (Encyclopedia iranica) പറയുന്നു:
സാസാനിയൻ രാജാക്കൻമാരാണ് പേർഷ്യൻ സാമ്രാജ്യത്തെ നയിച്ച അവസാന രാജവംശം. പേർഷ്യൻ സാമ്രാജ്യത്തിനു മേൽ ഇസ്ലാമിന്റെ അധിജയത്തിന് തൊട്ട് മുമ്പ് (224 CE–650 CE) വരെ ഭരണത്തിന് നേതൃത്വം വഹിച്ച വംശപരമ്പരയാണ് സാസാനിയൻ രാജവംശം. (https://iranicaonline.org/articles/sasanian-dynasty )
അബ്ദുശ്ശാഫി മുഹമ്മദ് അബ്ദുല്ലത്തീഫ് എഴുതി:
“പേർഷ്യൻ രാജാവായ കിസ്റക്ക് -അദ്ദേഹത്തെ ഇസ്ലാമിലേക്ക് ക്ഷണിച്ചു കൊണ്ട്- മുഹമ്മദ് നബി (സ) അയച്ച കത്ത് അഹങ്കാരത്തോടെയും ധാർഷ്ട്യത്തോടെയും പിച്ചിച്ചീന്തുകയാണ് ആ ഏകാധിപതി ചെയ്തത്. മാത്രമല്ല യമനിൽ ഭരണം നടത്തിയിരുന്ന തന്റെ പ്രതിനിധിയായ ബാദാനോട് മുഹമ്മദ് നബിയെ പിടികൂടാനും കൊല്ലാനും ആവശ്യപ്പെടുകയും ചെയ്തു. രാജാധിരാജനായ തന്നെ മറ്റൊരു മതത്തിലേക്ക് ക്ഷണിക്കാനുള്ള തന്റേടത്തിനു ശിക്ഷയായി കൊണ്ടാണ് മുഹമ്മദ് നബിയെ വധിക്കാൻ അദ്ദേഹം ഉത്തരവിട്ടത് (താരീഖു ത്വബ്രി 2:654). തുടക്കം മുതലേ അങ്ങേയറ്റം ശത്രുതാ മനോഭാവത്തോടെയാണ് പേർഷ്യൻ സാമ്രാജ്യത്വം ഇസ്ലാമിനെ അഭിമുഖീകരിച്ചത്. കാലാന്തരത്തിൽ ഈ അനീതി നിറഞ്ഞ ശത്രുത ഇസ്ലാമിക രാഷ്ട്രത്തിന്റെ അതിർത്തികളിലുള്ള അറബ് ഗോത്രങ്ങളെ ഭരണകൂടത്തിനെതിരെ ഇളക്കി വിടുകയും സായുധ വിപ്ലവങ്ങൾക്ക് പ്രോത്സാഹിപ്പിച്ചു കൊണ്ടിരിക്കുകയും ചെയ്തു. ഇത് യുദ്ധത്തിലാണ് കലാശിച്ചത്. ആഭ്യന്തര ലഹളക്കാരുമായും സഹായികളായ പേർഷ്യക്കാരുമായും മുസ്ന ഇബ്നു ഹാരിസ അശൈബാനിയുടെ നേതൃത്വത്തിലുള്ള മുസ്ലിം സൈന്യം അറബ് ഗൾഫ് തീരത്ത് നിരന്തരം ഏറ്റുമുട്ടി. ഇറാൻ മുഴുവൻ വിജയിച്ചടക്കുകയും ഇസ്ലാമിക നേതൃത്വത്തിനു കീഴിൽ സമ്പൂർണ്ണമായി പ്രവേശിക്കുകയും ചെയ്യുന്നത് വരെ പേർഷ്യക്കാരിൽ നിന്നുമുള്ള വൈദേശിക കലാപങ്ങൾ കെട്ടടങ്ങിയില്ല. മുസ്ലിംകൾ അല്ല ശത്രുതയ്ക്ക് തുടക്കം കുറിച്ചത് എന്നതിനാൽ തന്നെ നീതിയുള്ള നിഷ്പക്ഷരായ ആർക്കും പേർഷ്യക്കാരോടുള്ള യുദ്ധത്തിൽ മുസ്ലിംകളെ പഴിക്കാൻ കഴിയില്ല… പേർഷ്യൻ രാജ്യങ്ങളിലുള്ള വിജയത്തിന്റെ സുവിശേഷങ്ങളും യുദ്ധാർജിത സ്വത്തുക്കളും മദീനയിൽ ഖലീഫാ ഉമറിലേക്ക് മുസ്ലിം സൈന്യത്തിന്റെ നേതാക്കളിൽ ഒരാളായ സഅദ്ബ്നു അബീ വഖാസ് അയക്കുകയും പേർഷ്യൻ രാജ്യങ്ങളിൽ യുദ്ധം തുടരാനുള്ള അനുവാദം ആവശ്യപ്പെടുകയും ചെയ്തപ്പോൾ ഉമർ (റ) അത് നിരസിച്ചു കൊണ്ട് പറഞ്ഞു: “അവർക്കും നമുക്കുമിടയിൽ ഒരു വലിയ പർവതമുണ്ടാവുകയും നാം അവരിൽ നിന്നും, അവർ നമ്മിൽ നിന്നും സുരക്ഷിതരായിരിക്കുകയും ചെയ്തിരുന്നെങ്കിൽ എന്നാണ് ഞാൻ ആശിച്ചു പോവുന്നത്… യുദ്ധാർജിത സ്വത്തിനേക്കാൾ മുസ്ലിംകളുടെ സുരക്ഷയെയാണ് ഞാൻ മുന്തിക്കുന്നത്” (താരീഖു ത്വബ്രി 4:28). ഖലീഫയുടെ ഈ നിലപാട് മുസ്ലിംകൾ യുദ്ധക്കൊതിയന്മാർ അല്ലായിരുന്നു എന്നതിന് ഏറ്റവും വലിയ തെളിവാണ്. ശക്തിയും വാളും ഉപയോഗിച്ച് ഇസ്ലാം പ്രചരിപ്പിക്കൽ ആയിരുന്നില്ല മുസ്ലിംകളുടെ ഉദ്ദേശ്യം. (അസ്സീറത്തുന്നബവിയ്യ വത്താരീഖുൽ ഇസ്ലാം: 1:259)
എൻസൈക്ലോപീഡിയ ഇറാനിക (Encyclopedia iranica) പറയുന്നു:
സാസാനിയൻ രാജവംശത്തിലെ ഏറ്റവും കൂടുതൽ കാലം (r. 309-79 CE) ഭരിച്ച ചക്രവർത്തി ‘ശാഹ്പോർ രണ്ടാമൻ’ പേർഷ്യൻ സ്വേച്ഛാധിപത്യത്തിന്റെയും അസഹിഷ്ണുതയുടേയും പ്രതീകമായിരുന്നു.
ഇസ്ലാമിന് മുമ്പത്തെ അറബ് ഗോത്രങ്ങളെ അടിച്ചമർത്താനും സാമ്രാജ്യത്തിന്റെ അതിർത്തികൾ സുരക്ഷിതമാക്കാനും അയാൾ ഒരു സൈനിക പ്രചാരണം ആരംഭിച്ചു. (താരീഖുത്വബ്രി, തജാറുബുൽ ഉമം: മസ്കവൈഹി).
ﻓﺄﻓﺸﻲ ﻓﻴﻬﻢ اﻟﻘﺘﻞ، ﻭﺳﻔﻚ ﻓﻴﻬﻢ ﻣﻦ اﻟﺪﻣﺎء ﺳﻔﻜﺎ ﺳﺎﻟﺖ ﻛﺴﻴﻞ اﻟﻤﻄﺮ، ﺣﺘﻰ ﻛﺎﻥ اﻟﻬﺎﺭﺏ ﻣﻨﻬﻢ ﻳﺮﻯ ﺃﻧﻪ ﻟﻦ ﻳﻨﺠﻴﻪ ﻣﻨﻪ ﻏﺎﺭ ﻓﻲ ﺟﺒﻞ، ﻭﻻ ﺟﺰﻳﺮﺓ ﻓﻲ ﺑﺤﺮ….ﻭﺇﻥ ﺳﺎﺑﻮﺭ ﺿﺮﻱ ﺑﻘﺘﻞ اﻟﻌﺮﺏ، ﻭﻧﺰﻉ ﺃﻛﺘﺎﻑ ﺭﺅﺳﺎﺋﻬﻢ ﺇﻟﻰ ﺃﻥ ﻫﻠﻚ. ﻭﻛﺎﻥ ﺫﻟﻚ ﺳﺒﺐ ﺗﺴﻤﻴﺘﻬﻢ ﺇﻳﺎﻩ ﺫا اﻷﻛﺘﺎﻑ
ത്വബ്രി എഴുതുന്നു: “അറബികളെ അയാൾ പരക്കെ കൊന്നു. മഴ കുത്തിച്ചൊരിയുന്നതു പോലെ അറേബ്യയിൽ രക്തം ചിന്നി ചിതറി. മരണത്തിൽ നിന്ന് ഓടി രക്ഷപ്പെടാൻ ശ്രമിച്ചവർ തന്നെ മനസ്സിൽ മന്ത്രിച്ചു; ഒരു പർവ്വതത്തിനു മൂലയിലുള്ള ഗുഹയും ഒരു സമുദ്ര പ്രാന്തത്തിലുള്ള ദ്വീപും തന്നെ ഒരിക്കലും രക്ഷിക്കില്ല എന്ന്. ‘ശാഹ്പോർ’ ചക്രവർത്തി അറബികളെ കൂട്ടക്കുരുതി നടത്തി. അറബികളുടെ നേതാക്കളുടെ തോളെല്ലുകൾ വലിച്ചൂരി അവരെ ക്രൂരമായ രീതിയിൽ കൊലപ്പെടുത്തി. അക്കാരണത്താൽ അയാളെ ‘ദുൽ അക്താഫ്’ (ﺫﻱ اﻻﻛﺘﺎﻑ) ‘തോളെല്ലുകളുടെ ഉടമ’ എന്ന് അറബികൾ വിളിച്ചു.” (താരീഖു ത്വബ്രി: 2:57)
‘ശാഹ് പോർ’ രണ്ടാമൻ ഇറാഖിലെ അയാദുകളെ ആദ്യം ആക്രമിച്ചു. പിന്നീട് പേർഷ്യൻ ഗൾഫ് മുറിച്ച് കടന്ന് ബഹ്റൈൻ, കത്വർ എന്നീ പ്രദേശങ്ങളും, തമീം, ബകർ, വാഇൽ, അബ്ദുൽ കൈസ് തുടങ്ങിയ ഗോത്രക്കാരുടെ വാസസ്ഥലമായ ഹജറും ആക്രമിച്ചു. (താരീഖു ത്വബ്രി) ഒട്ടുമിക്ക ഗോത്രവർഗ്ഗക്കാരേയും കശാപ്പു ചെയ്യുന്നതിന് പുറമെ അറബികൾക്ക് വെള്ളം കിട്ടാതിരിക്കാൻ ജലസ്രോതസ്സുകൾ മണ്ണിട്ടു മൂടി.
തുടർന്ന് കിഴക്കൻ അറേബ്യയിലും സിറിയയിലും ആക്രമണം അഴിച്ചു വിടുകയും യമാമ, ബക്കർ, തഗ്ലിബ് നഗരങ്ങൾക്കുമെതിരെ ആഞ്ഞടിച്ചു. (ഗററു അഖ്ബാരി മുലൂകുൽ ഫറസ്: സഗാലിബി, തജാറുബുൽ ഉമം: മസ്കവൈഹി)
ചില അറബ് ഗോത്രങ്ങളെ നിർബന്ധിതമായി മാറ്റിപ്പാർപ്പിക്കുകയും സാസാനിയൻ സാമ്രാജ്യത്തിലേക്ക് നാടുകടത്തുകയും ചെയ്യപ്പെട്ടു.
അറബികളിൽ നിന്ന് തുടർന്ന് ആക്രമണങ്ങൾ ഉണ്ടാകാതിരിക്കാൻ, ‘ശാഹ്പോർ’ II, “അറബി മതിൽ” എന്ന പേരിൽ ഒരു പ്രതിരോധ സംവിധാനം നിർമ്മിച്ചു. (T. Daryaee, “Memory and History: The Construction of the Past in Late Antique Persia,” Nāma-ye Irān-e Bāstan/The International Journal of Ancient Iranian Studies 1/2, 2001-02, pp. 1-14)
‘ശാഹ് പോർ’ II ന്റെ ഭരണകാലത്ത് സാസാനിയൻ സാമ്രാജ്യത്തിലെ ക്രിസ്ത്യാനികൾ നേരിടേണ്ടി വന്ന കൊടും പീഡനങ്ങൾ അർമേനിയൻ സ്രോതസ്സുകൾ പോലും രേഖപ്പെടുത്തുന്നുണ്ട്.
‘ശാഹ് പോർ’ രണ്ടാമന്റെ വേട്ടക്കിരയായ പ്രശസ്ത ക്രിസ്ത്യൻ രക്തസാക്ഷികളുടെ പേരുകളിൽ അയാളുടെ പ്രധാന കരകൗശലത്തൊഴിലാളിയായ പോസി (പുസൈ), അദ്ദേഹത്തിന്റെ മകൾ മാർത്ത, തെക്ല, ദാനക്, ബൗത എന്നിവരും ഉൾപെടുന്നു. സ്ത്രീകൾ ഉൾപ്പെടെ സാൻ, മാമ, മെസഖ്യ, അന്ന, അബ്യത്, സാത്തായ്, മെസഖ്യ എന്നിങ്ങനെ മറ്റ് ഒരുപാട് രക്തസാക്ഷികളും അവരുടെ കൂട്ടത്തിൽ ഉണ്ടായിരുന്നു. (Brock and S. A. Harvey, Holy Women of the Syrian Orient, Berkeley, 1998, pp. 68-77) (https://iranicaonline.org/articles/shapur-ii )
സൊസോമന്റെ Ecclesiastical History ൽ, ‘ശാഹ്പോർ’ രണ്ടാമന്റെ കീഴിൽ രക്തസാക്ഷികളായ പേർഷ്യൻ ക്രിസ്ത്യാനികളെക്കുറിച്ച് ഗണ്യമായ വിശദാംശങ്ങൾ അടങ്ങിയിരിക്കുന്നു: “ഈ കാലയളവിൽ രക്തസാക്ഷികളായ പുരുഷന്മാരുടെയും സ്ത്രീകളുടെയും പേരുകളിൽ -സ്ഥിരീകരിക്കപ്പെട്ടവരുടെ എണ്ണം- പതിനാറായിരത്തിലധികമായി കണക്കാക്കപ്പെടുന്നു. എന്നാൽ ഈ കാലയളവിൽ പേരറിയാത്ത, രക്തസാക്ഷികളായ സ്ത്രീകളുടെയും പുരുഷന്മാരുടെയും എണ്ണം പതിനാറായിരത്തിലും എത്രയോ അധികമായതിനാൽ പേർഷ്യക്കാരും, സിറിയക്കാരും, എഡെസ്സ നിവാസികളും, എത്ര ശ്രമിച്ചിട്ടും ആ കണക്ക് നിജപ്പെടുത്താൻ സാധിക്കാതെ പരാജയമടഞ്ഞു.” ( Ecclesiastical History, Book II, Chapter XIV )
ഇസ്ലാമിന്റെ ആവിർഭാവത്തിനു മുമ്പ് തന്നെ പേർഷ്യൻ റോമൻ സാമ്രാജ്യങ്ങൾ മത സ്വേച്ഛാധിപത്യത്തിൽ കേളികേട്ടവരായിരുന്നു. ഇരു സാമ്രാജ്യങ്ങളേയും ആദർശപരമായി നയിച്ചിരുന്ന സൊറോസ്ട്രിയർ, ക്രിസ്ത്യാനികൾ, ജൂതന്മാർ തുടങ്ങി വ്യത്യസ്ഥ ആദർശവിശ്വാസികൾ – അവർ ഓരോരുത്തർക്കും ലഭ്യമാകുന്ന രാഷ്ട്രീയ അനുകൂലാവസ്ഥക്ക് അനുസൃതമായി- നടപ്പാക്കിയിരുന്ന അന്യമത ധ്വംസനങ്ങളും ഭരണ വടം വലികളും അന്നത്തെ ലോക വ്യവസ്ഥയിലെ അസഹിഷ്ണുതയുടെ ആഴം വ്യക്തമാക്കിത്തരുന്നുണ്ട്.
ഇസ്ലാംഭരണത്തിനു മുമ്പുള്ള പേർഷ്യയിലെ അവസാനസാമ്രജ്യമായിരുന്ന സാസാനിയൻ സാമ്രാജ്യത്തിന്റെ കാലത്ത് സൊറോസ്ട്രിയൻ മതം സാമ്രാജ്യത്തിലെ ഔദ്യോഗികമതമായിരുന്നു. കൂടാതെ ഈ സമയത്ത് സൊറോസ്ട്രിയൻ മതം സംഘടനാരൂപം കൈവരിക്കുകയും ചെയ്തു.
സാസാനിയൻ കാലഘട്ടത്തിലെ സൊറാസ്ട്രിയനിസം (224-651) പേർഷ്യൻ ചക്രവർത്തിയായ ശഹൻ-ശായുടെ (രാജാധി രാജൻ) രക്ഷാകർതൃത്വം ആസ്വദിച്ചു പോന്നു.
ഹഖാമനീഷിയാൻ പേർഷ്യൻ രാജാക്കന്മാരുടെ മഹത്വത്തിന്റെ നേരിട്ടുള്ള അവകാശികളായി സ്വയം കണ്ട സാസാനിയൻ സാമ്രാജ്യത്വ ഭരണകൂടം മസ്ദ-ആരാധനയിൽ അധിഷ്ടിതമായ, സോറോസ്ട്രിയൻ വിശ്വാസത്തിന്റെ സംരക്ഷകരായി സ്വയം അവരോധിച്ചു. (Zoroastrians and Christians in Sasanian Iran: A.V Williams: Department of Religions and Theology, University of Manchester: Page: 2)
ഈ കാലഘട്ടത്തിൽ അഹ്രിമാനും ദേവുകളും വലിയ പ്രഹരവും ഉപദ്രവവും ഭരണകൂടത്തിൽ നിന്ന് നേരിട്ടു. അവരുടെ വിശ്വാസങ്ങൾ ഭൂമിയിൽ നിന്ന് പുറത്താക്കപ്പെടുകയും വിശ്വാസ്യത നഷ്ടപ്പെടുകയും ചെയ്തു. യഹൂദരും ബുദ്ധമതക്കാരും ബ്രാഹ്മണരും അരാമിക്, ഗ്രീക്ക് ഭാഷകൾ സംസാരിക്കുന്ന ക്രിസ്ത്യാനികളും സ്നാപനമേറ്റവരും മനിക്കേയന്മാരും പേർഷ്യൻ ഭൂമിയിൽ ആക്രമിക്കപ്പെട്ടു. ചിത്രങ്ങൾ തകർക്കപ്പെട്ടു, ആരാധനാലയങ്ങളും വാസസ്ഥലങ്ങളും നശിപ്പിക്കപ്പെട്ടു, അഗ്നിക്ഷേത്രങ്ങൾ ധാരാളമായി സ്ഥാപിക്കപ്പെട്ടു. (Mary Boyce, Textual sources for the study of Zoroastrianism (Manchester: Manchester University Press, 1984), 112, translating from M. Back, Die sassanidischen Staatsinschriften (Acta Iranica, 18, 1978), 384ff.)
ഏറ്റവും കൂടുതൽ കുപ്രസിദ്ധമായ പീഡന പരമ്പരയുടെ കണക്കുകൾ ശാഹ്പോർ രണ്ടാമന്റെ ഭരണകാലഘട്ടത്തിൽ (309-79) നിന്നുള്ള ഇരുപത്തിയൊമ്പത് രക്തസാക്ഷികളാണ്. മുഖ്യ ആത്മീയ നേതാവായ അദുർബാദിന്റെ നേതൃത്വത്തിൽ ക്രിസ്ത്യാനികളെ നാൽപത് വർഷത്തോളം സുസ്ഥിരവും നിഷ്കരുണവും ദംശിച്ചു കൊണ്ടിരുന്നു. കോൺസ്റ്റന്റൈൻ ക്രിസ്തുമതം സ്വീകരിച്ച് ക്രിസ്തുമതത്തെ റോമൻ സാമ്രാജ്യത്തിന്റെ മതമാക്കിയതിന് തൊട്ടുപിന്നാലെയാണ് പീഡനത്തിന്റെ തുടക്കം.
ക്രിസ്ത്യൻ പീഡനത്തിന്റെ രണ്ടാം കാലഘട്ടം യാസ്ദെജർദ് ഒന്നാമന്റെയും (399-421) പിന്നീടുള്ള വർഷങ്ങളിൽ അദ്ദേഹത്തിന്റെ പിൻഗാമിയായ ബഹ്റാം അഞ്ചാമന്റെയും ആദ്യ വർഷങ്ങളിലുമായിരുന്നു (c. 421-39).
മൂന്നാമതായി, യാസ്ദെജർദ് രണ്ടാമന്റെ (439-57) ഭരണകാലത്ത്, ക്രിസ്ത്യൻ വിരുദ്ധ മത വിദ്വേഷം പീഡനവും വർദ്ധിച്ചു. ക്രിസ്ത്യൻ അർമേനിയയെ സൊറോസ്ട്രിയനിസത്തിലേക്ക് നിർബന്ധ മതപരിവർത്തനം ചെയ്യാനുളള ശ്രമം ശക്തിയാർജിച്ചു.
പേർഷ്യൻ ക്രിസ്ത്യാനികളുടെ, സുറിയാനി ക്രിസ്ത്യൻ രക്തസാക്ഷികളുടെ പ്രവർത്തനങ്ങളെ (Acts of the Persian Martyrs) സംബന്ധിച്ച രേഖകളിൽ പീഡനവും താഢനവും രക്തസാക്ഷിത്വവും വികാരഭരിതമായ വാക്കുകളിൽ രേഖപ്പെടുത്തപ്പെട്ടിട്ടുണ്ട്. (Zoroastrians and christians in Sasanian Iran: A.V Williams: Department of Religions and Theology, University of Manchester: Page: 4)
സോറാസ്ട്രിയൻ പുരോഹിതന്മാർ സാധാരണക്കാരോടൊപ്പം ഭക്ഷണം കഴിക്കുന്നത് ഒഴിവാക്കണം എന്ന ശക്തമായ കൽപ്പന പുറപ്പെടുവിക്കപ്പെട്ടു. ഒരു അവിശ്വാസിക്ക് എന്തെങ്കിലും ദാനം നൽകുന്നത് നിരോധിച്ചുകൊണ്ടുള്ള സോറോസ്ട്രിയൻ മതശാസനകൾ ഇറക്കപ്പെട്ടു. മരിക്കുമെന്ന് നിങ്ങൾ കരുതുന്നില്ലെങ്കിൽ അല്ലാതെ, ഒരു അവിശ്വാസിയ്ക്ക് നിങ്ങൾ ഭക്ഷണമുൾപ്പെടെ ഒന്നും നൽകരുത് എന്നായിരുന്നു പുരോഹിത ശാസന. (Pahlavi Rivayat, ed. Williams, 1990, 14.7, voL 2, 27.)
ശക്തനായ ഒരു പ്രഭു, ഉദാഹരണത്തിന് ഗിർഡർ, രാജാവിന്റെ മേൽ സ്വാധീനം നേടുമ്പോളെല്ലാം, പല കാരണങ്ങളും ആരോപിച്ച് ക്രിസ്ത്യാനികളും മറ്റ് ന്യൂനപക്ഷങ്ങളും രാഷ്ട്രീയവും മതപരവുമായ ബലിയാടുകളാക്കപ്പെടുക എന്നത് ഒരു സ്ഥിരം പ്രതിഭാസമായിരുന്നു. (Zoroastrians and christians in Sasanian Iran: A.V Williams: Department of Religions and Theology, University of Manchester: Page: 8)
മത പരിത്യാഗം മഹാ കുറ്റകൃത്യമായി നിശ്ചയിക്കപ്പെട്ടു. ഭീഷണികളും പീഡനങ്ങളും ഉപയോഗിച്ച് ക്രിസ്തുമതത്തിൽ നിന്ന് സോറാസ്ട്രിയൻ മതത്തിലേക്ക് നിർബന്ധപൂർവ്വം വലിച്ചിഴക്കപ്പെടുന്ന വർണനകൾ ക്രിസ്ത്യൻ സാധുചരിത്രവർണ്ണന (hagiography) രേഖകൾ മുഴുവൻ നിറഞ്ഞു നിൽക്കുന്നു.
സൂര്യൻ, അഗ്നി, ജലം എന്നിവയെ ആരാധിക്കാൻ പല ക്രിസ്ത്യാനികളേയും സോറാസ്ട്രിയൻ പൗരോഹിത്യം വധഭീഷണിയിലൂടെയും മർദ്ദനങ്ങളിലൂടെയും നിർബന്ധിച്ചു കൊണ്ടിരുന്നു.
ഈസ്റ്റ് ചർച്ച് കത്തോലിക്കരുടെയും ബിഷപ്പുമാരുടെയും തലവനായ ‘മാർ സിമൺ ബാർ സബ്ബാ’യുടെ വധത്തിന് കാരണം, ക്രിസ്ത്യാനികൾക്ക് ഇരട്ട നികുതി ചുമത്താൻ വിസമ്മതിച്ചതാണ്. സിമണേയും അദ്ദേഹത്തിന്റെ എല്ലാ സഹോദരന്മാരെയും വാളാൽ ശിരസ്സു ചേദിച്ച് വധിക്കാൻ രാജ കൽപ്പന നൽകപ്പെട്ടു. ഈ കൂട്ടകൊലക്ക് തൊട്ടു മുമ്പ് സോറാസ്ട്രിയൻ മുഖ്യ പുരോഹിതൻ ഇരകളോട് ഇപ്രകാരം ആവശ്യപ്പെടുകയുണ്ടായി: “ദൈവമായ സൂര്യനെ ആരാധിക്കുക, രാജാക്കന്മാരുടെ രാജാവും എല്ലാ കർത്താക്കളുടെയും കർത്താവുമായ ‘ശാഹ്പോറി’ന്റെ ഹിതം അനുസരിച്ച് പ്രവർത്തിക്കുക, എങ്കിൽ നിങ്ങൾക്ക് ജീവിക്കാം.” അപ്പോൾ അവർ എല്ലാവരും ഉറക്കെ മറുപടി നൽകി: “തീർച്ചയായും ഞങ്ങൾ രാജാക്കന്മാരുടെ രാജാവും മുഴുവൻ ഭൂമിയുടെയും അധിപനും, നിത്യതയുടെ ഉടമയുമായ (സാക്ഷാൽ ദൈവത്തിന്റെ) ഹിതം പ്രവർത്തിക്കും.”
സിമണിന്റെ രക്തസാക്ഷിത്വത്തിന് സാക്ഷ്യം വഹിച്ച പുസൈ ക്രിസ്ത്യൻ യുവാവ്, മറ്റൊരു ക്രിസ്ത്യൻ വയോധികനെ വധിക്കുന്ന ചടങ്ങിൽ വെച്ച് പരസ്യമായി പ്രതികരിക്കുകയുണ്ടായി. ഇത് പേർഷ്യൻ രാജാധിരാജനോടുള്ള ധിക്കാരമായി പരിഗണിക്കപ്പെട്ടു. പുസൈയെ ചങ്ങലകളിൽ വലിച്ചിഴക്കുകയും ക്രിസ്തുമതം ഉപേക്ഷിക്കാൻ നിർബന്ധിക്കുകയും ചെയ്തു. വിസമ്മതിച്ചപ്പോൾ വധശിക്ഷക്ക് വിധേയമാക്കി. മാർ അബാ, മാർ ഗിവർഗീസ് എന്നിവരും ക്രിസ്തുമത രക്തസാക്ഷികളിൽ എണ്ണപ്പെടുന്നു. (Zoroastrians and christians in Sasanian Iran: A.V Williams: Department of Religions and Theology, University of Manchester: Page: 12)
പേർഷ്യൻ സാമ്രാജ്യത്വത്തിന്റെ ക്രൂര രാഷ്ട്രീയത്തിന്റെ മറ്റൊരു ഉദാഹരണമാണ് വലേറിയൻ ചക്രവർത്തിയുടെ വധം: റോമൻ ചക്രവർത്തിയായ (ക്രിസ്ത്യൻ വിരുദ്ധനായിരുന്നു) വലേറിയൻ പണം വാഗ്ദാനം ചെയ്ത് സാസാനിയൻമാരുമായുള്ള യുദ്ധം അവസാനിപ്പിക്കാൻ ആഗ്രഹിച്ചു. സാസാനിയൻ രാജാവായ ശാഹ്പോർ ഒന്നാമൻ -ചക്രവർത്തി നേരിട്ട് വന്ന് അദ്ദേഹത്തോട് അഭ്യർത്ഥിക്കാൻ ആഗ്രഹിക്കുന്നു എന്ന സന്ദേശവുമായി- അംബാസഡർമാരെ വലേറിയനിലേക്ക് തിരിച്ചയച്ചു. വലേറിയൻ ഒരു ചെറിയ സംഘത്തോടൊപ്പം ശാഹ്പോറിനടുത്തേക്ക് ചെന്നപ്പോൾ, അദ്ദേഹത്തെ സാസാനിയക്കാർ പിടികൂടി. ബൈസന്റൈൻ ചരിത്രകാരനായ സോസിമസിന്റെ അഭിപ്രായത്തിൽ, പേർഷ്യക്കാർക്കിടയിൽ ഒരു അടിമയെന്ന നിലയിൽ വലേറിയൻ തന്റെ ജീവിതത്തോട് വിട പറഞ്ഞു. പുരാതന ക്രിസ്ത്യൻ എഴുത്തുകാരനും കോൺസ്റ്റന്റൈൻ ഒന്നാമന്റെ ഉപദേശകനുമായ ലാക്റ്റൻഷ്യസ് വാദിക്കുന്നത്, ശാഹ്പോർ ഒന്നാമൻ കുതിരപ്പുറത്ത് കയറാൻ ആഗ്രഹിക്കുമ്പോഴെല്ലാം വലേറിയൻ ചക്രവർത്തിയെ മനുഷ്യ പാദപീഠമായി ഉപയോഗിച്ചിരുന്നു എന്നാണ്.” അദ്ദേഹത്തെ കൊന്നുകളയുകയും, അദ്ദേഹത്തിന്റെ തൊലി മാംസത്തിൽ നിന്ന് ഉരിച്ച് വേർപെടുത്തി, സിന്ദൂരം കൊണ്ട് ചായം പൂശി, ബാർബേറിയൻമാരുടെ ദൈവങ്ങളുടെ ക്ഷേത്രത്തിൽ പ്രതിഷ്ടിക്കുകയും ചെയ്തു” എന്നും ലാക്റ്റൻഷ്യസ് വാദിക്കുന്നു. (https://www.ancient-origins.net/history/what-really-happened-valerian-was-roman-emperor-humiliated-and-skinned-hands-enemy-008598)
വലേറിയന്റെ തൊണ്ടയിലേക്ക് ഉരുക്കിയ സ്വർണ്ണ ദ്രാവകം ഒഴിച്ചാണ് പേർഷ്യക്കാർ വധിച്ചതെന്നും പറയപ്പെടുന്നു.
പേർഷ്യൻ സാമ്രാജ്യം ജൂതവിരുദ്ധമായിരുന്ന കാലഘട്ടത്തിൽ ആദർശ അസഹിഷ്ണുതയുടെ പേരിൽ ജൂതന്മാർ മത ദ്വംസനങ്ങൾക്കും അക്രമങ്ങൾക്കും പാത്രമായി. പഴയനിയമത്തിന്റെ ഭാഗമായ ഗ്രന്ഥങ്ങളിൽ ഒന്നായ ‘എസ്തേറിന്റെ പുസ്തക’ത്തിൽ ‘പൂരിം’ എന്ന പേരിൽ അറിയപ്പെടുന്ന ജൂതാഘോഷത്തിനു പിന്നിലുള്ള കഥ പ്രതിപാദിക്കുന്നുണ്ട്. പൂരിം തിരുനാളിലെ സായാഹ്നത്തിലും അടുത്ത പ്രഭാതത്തിലും ഈ പുസ്തകത്തിന്റെ സമ്പൂർണ്ണപാഠം ജൂതർ ഉറക്കെ വായിക്കുന്നു. പേർഷ്യൻ രാജാവിന്റെ കൊട്ടാരത്തിൽ, ഹാമാന്റെ നേതൃത്വത്തിൽ ജൂതർക്കെതിരെ നടന്ന ഒരു ഉപജാപത്തിന്റേയും, തൽഫലമായി ഉണ്ടാകാനിരുന്ന വംശഹത്യയിൽ നിന്ന്, ജൂതന്മാർ രക്ഷപെടുന്നതിന്റേയും കഥയാണിത്. (The Religious Policy of Xerxes and the “Book of Esther” Robert J. Littman)
ഒരു ജൂത ഉദ്യോഗസ്ഥനായ മൊർദ്ദെകായ്, രാജാവിന്റെ ഉന്നത സഹായിയായ ഹാമാന്റെ മുമ്പിൽ തലകുനിച്ച് വണങ്ങാൻ വിസമ്മതിച്ചു എന്ന കാരണത്താലാണ് രാജ്യത്തെ എല്ലാ ജൂതന്മാരെയും ഉന്മൂലനം ചെയ്യാൻ രാജാവ് പദ്ധതിയിട്ടത്. യഹൂദ മതം ഒരു ജൂതനും, ദൈവമല്ലാതെ മറ്റേതെങ്കിലും വ്യക്തിയെയോ വിഗ്രഹത്തെയോ തലകുനിച്ച് വണങ്ങാൻ അനുവദിക്കുന്നില്ല.
ലബ്നാനിലെ പ്രസിദ്ധ (ക്രിസ്ത്യൻ) സാഹിത്യകാരനും ചരിത്രകാരനുമായ ജോർജി സൈദാൻ എഴുതി: പേർഷ്യയും റോമും തമ്മിലുളള ശത്രുത പൗരാണികമാണ്. ഒരു പക്ഷെ ബി.സി അഞ്ചാം നൂറ്റാണ്ടിനുമപ്പുറം അതിന്റെ വേരുകൾ എത്തി നിൽക്കുന്നുണ്ടാകാം. ലോകത്തെ അടക്കി ഭരിക്കാനുള്ള ഇരു സാമ്രാജ്യങ്ങളുടെയും അത്യാഗ്രഹമായിരുന്നു ഈ ശത്രുതയുടെ അടിത്തറ. നൂറ്റാണ്ടുകളോളം നീണ്ടു നിന്ന ഈ അധികാര വടം വലി ഇസ്ലാമിന്റെ ആവിർഭാവ കാലഘട്ടത്തിലും തുടർക്കഥയായിരുന്നു. പേർഷ്യൻ സാമ്രാജ്യത്തിന്റെ ആധിപത്യം ഇസ്ര അനൂഷർവാൻ ചക്രവർത്തിയിൽ എത്തിച്ചേർന്നപ്പോൾ റോമൻ സാമ്രാജ്യത്തെ അൽപാൽപ്പമായി പിടിച്ചടക്കാൻ അദ്ദേഹം സൈന്യ വ്യൂഹത്തെ വിന്യസിച്ചു. സിറിയ പിടിച്ചടക്കുകയും അന്താഖിയ ചുട്ടു നശിപ്പിക്കുകയും ഏഷ്യാ മൈനർ കൊള്ളയടിക്കുകയും ചെയ്തു. അന്നത്തെ റോമൻ ചക്രവർത്തിയായിരുന്ന ജസ്റ്റീനിയൻ ചക്രവർത്തിയും വിട്ടുവീഴ്ചയ്ക്ക് തയ്യാറല്ലായിരുന്നു. ക്രിസ്താബ്ദം 541 മുതൽ 561 വരെ ഇരുപതു വർഷം ഇരു രാഷ്ട്രങ്ങളും യുദ്ധത്തിൽ മുഴുകി.
പർവേസ് ചക്രവർത്തിയുടെ കാലഘട്ടത്തിൽ തന്റെ സുഹൃത്ത് മോറിസിന്റെ കൊലപാതകത്തിന് പ്രതികാരമെന്ന പേരിൽ റോമൻ സാമ്രാജ്യത്തെ പർവേസ് ചക്രവർത്തി ആക്രമിക്കുകയുണ്ടായി. ക്രിസ്താബ്ദം 614 ൽ സിറിയ പിടിച്ചടക്കി…
(പേർഷ്യയിൽ ജൂതന്മാർക്ക് സ്വാധീനമുണ്ടായിരുന്ന കാലത്ത്) ജൂതന്മാരുടെ അകമഴിഞ്ഞ സഹായത്താൽ ബൈസാന്റിയൻ പടയെ പർവേസ് ചക്രവർത്തി ഒന്നൊന്നായി കീഴടക്കി. ഈജിപ്ത്, അന്താഖിയ, ദമാസ്ക്കസ്, ബൈത്തുൽ മുഖദസ് തുടങ്ങിയവ പിടിച്ചടക്കി. ജറുസലേമിലെ ബൈത്തുൽ മുഖദസ് കൊള്ളയടിക്കാനും ക്രിസ്ത്യൻ പള്ളികളും പുണ്യപുരുഷന്മാരുടെ കല്ലറകൾ തീയിടാനും അവിടെയുള്ള വിലമതിക്കാനാകാത്ത സ്വത്തുക്കൾ പിടിച്ചു പറിക്കാനും തന്റെ സൈന്യത്തിന് പർവേസ് ചക്രവർത്തി അനുവാദം നൽകി. സിറിയ വരെ ഈ കൊലയും കൊള്ളയും തുടർന്നു. 90000 ക്രിസ്ത്യാനികളെ സൈന്യം കൊന്നൊടുക്കി… ഇതെല്ലാം കണ്ടിട്ടും കാണാത്ത മട്ടിൽ ഭീരുവായി (അന്നത്തെ) ഹെറാക്ലിയസ് ചക്രവർത്തി കൊട്ടാരത്തിൽ തന്നെ ഇരുന്നു; ക്രിസ്താബ്ദം 632 ൽ ഏഷ്യാ മൈനറിൽ വെച്ച് കൊല്ലപ്പെടുന്നത് വരെ… (താരീഖുത്തമദ്ദുനുൽ ഇസ്ലാമി:ജോർജി സൈദാൻ: 1: 43-48)
ഹെറാക്ലിയസിന്റെ കാലഘട്ടത്തിൽ ജൂതന്മാർ തങ്ങളുടെ സഹായികളായ പേർഷ്യക്കാരിൽ നിന്നും 80000 ക്രിസ്ത്യൻ ബന്ദികളെ വിലയ്ക്ക് വാങ്ങി അറുത്തു കൊന്നു… (താരീഖുത്തമദ്ദുനുൽ ഇസ്ലാമി: ജോർജി സൈദാൻ: 1: 43-48)
ഈ രാഷ്ട്രീയ വ്യവസ്ഥയിലെ, പേർഷ്യൻ അധിനിവേശത്തിനു കീഴിലായിരുന്ന ‘ബസ്വറ’യോടാണ് (ഹിന്ദ്) മുഹമ്മദ് നബി (സ) യുദ്ധത്തിന് ആഹ്വാനം ചെയ്തത്. ഈ യുദ്ധ ആഹ്വാനത്തിന് പശ്ചാത്തലവുമായി ഗാഢമായ ബന്ധമുണ്ട് എന്ന് വ്യക്തമാണല്ലൊ. ഏതെങ്കിലും ഒരു രാഷ്ട്ര നേതാവ് ഒരു രാജ്യത്തോട് ഒരു പ്രത്യേക പശ്ചാത്തലത്തിലും സാഹചര്യത്തിലും യുദ്ധത്തിന് പ്രോത്സാഹനം നൽകിയ ഒരു വാചകം എടുത്തു വെച്ച്, അവ എക്കാലത്തേക്കുമുള്ള കൽപ്പനയായി അവതരിപ്പിക്കുന്നതിലെ ദുരുദ്ദേശം തിരിച്ചറിയപ്പെടണം. ബ്രിട്ടീഷ് സാമ്രാജ്യത്വ അധിനിവേശത്തോട് സമരവും യുദ്ധവും പ്രഖ്യാപിച്ച് കൊണ്ടുള്ള ഇന്ത്യൻ സ്വാതന്ത്ര്യ സമര നേതാക്കളുടെ വാചകങ്ങൾ എടുത്തുദ്ദരിച്ച് ഇന്നത്തെ, അല്ലെങ്കിൽ എക്കാലത്തേയും ബ്രിട്ടനോട് ശത്രുത വെച്ചുപുലർത്താനാണ് അവയുടെ ഉൾസാരമെന്ന് ദുർവ്യാഖ്യാനിക്കുന്നതു പോലെ തികച്ചും ബാലിശമാണ് ഈ ഹദീസ് ദുർവ്യാഖ്യാനം.
ഹദീസിൽ ഉദ്ദേശിക്കപ്പെട്ട ഹിന്ദ് അഥവാ ബസ്വറ നൂറ്റാണ്ടുകളായി മുസ്ലിംകളുടേതായി കഴിഞ്ഞു. അപ്പോൾ ബസ്വറയോടുള്ള യുദ്ധ കൽപ്പന എക്കാലത്തേയും ബാധിക്കുന്ന, പൊതു കൽപനയാണെന്ന് ദുർവ്യഖ്യാനിച്ചാൽ ‘മുസ്ലിം ഇറാഖി’നോട് തന്നെ യുദ്ധം ചെയ്യാനാണ് മുഹമ്മദ് നബി (സ) കൽപ്പിച്ചത് എന്ന് ഹദീസിന് വിരുദ്ധാത്ഥം കൽപ്പിക്കേണ്ടി വരില്ലേ ?! ഇസ്ലാം മുഹമ്മദ് നബിയിലൂടെ കടന്നുവന്ന കാലഘട്ടത്തിൽ ലോകത്ത് നിലനിന്നിരുന്ന രാഷ്ട്രീയ വ്യവസ്ഥയെ സംബന്ധിച്ച് ചരിത്രത്തിന്റെ താളുകളിലൂടെ അൽപ്പമെങ്കിലും കണ്ണോടിച്ചാൽ ജനാധിപത്യ മൂല്യങ്ങളും മത സഹവർത്തിത്വത്തിലും ഊന്നി, ഇന്ന് ലോകത്ത് നിലനിൽക്കുന്ന രാഷ്ട്ര ഘടനയായിരുന്നില്ല അന്നത്തേത് എന്ന് മനസ്സിലാക്കാവുന്നതെയുള്ളു.
അന്യ മതങ്ങളേയോ ആദർശങ്ങളേയോ സംസ്കാരങ്ങളേയോ വെച്ചുപൊറുപ്പിക്കാത്ത സ്വേച്ചാധിപത്യ ഭരണകൂടങ്ങളായിരുന്നു ലോകത്ത് ഭൂരിഭാഗവും രാജ്യങ്ങളിലും നിലനിന്നിരുന്നത്. ജനാധിപത്യമോ മത സഹിഷ്ണുതയോ തൊട്ടു തീണ്ടിയിട്ടില്ലാത്ത കിരാത വാഴ്ച്ച. ഇതാണ് ഇസ്ലാമിന്റെ ആവിർഭാവ ഭൂമിക. ഈ സാമൂഹിക രാഷ്ട്രീയ പശ്ചാത്തലത്തിൽ ഊന്നി നിന്നു കൊണ്ടേ ഇസ്ലാമിലെ യുദ്ധങ്ങളെയും യുദ്ധ ആഹ്വാനങ്ങളേയും വായിച്ചറിയാവൂ. പ്രതിരോധത്തിനും മത സ്വാതന്ത്ര്യത്തിനും വേണ്ടിയായിരുന്നു ഇസ്ലാമിക യുദ്ധങ്ങൾ മുഴുവനും. അതല്ലാതെ ഇന്ന് ലോകത്ത് നിലനിൽക്കുന്ന മതേതര സ്വഭാവമോ ആദർശ സഹവർത്തിത്വമോ രാഷ്ട്രീയ പ്രകൃതിയായി സ്വീകരിച്ച സാമൂഹിക പശ്ചാത്തലത്തോടായിരുന്നില്ല പ്രസ്തുത സമരങ്ങൾ. ഈ സമൂഹത്തോട് അവകാശങ്ങൾക്കും സ്വാതന്ത്ര്യത്തിനും വേണ്ടി സായുധ സമരത്തിൽ ഏർപ്പെടുകയല്ലാതെ മറ്റെന്താണ് മുസ്ലിംകളുടെ – ആവിർഭാവ ഘട്ടത്തിൽ – മുമ്പിലുണ്ടായിരുന്ന വഴി !!
പ്രതിരോധ യുദ്ധങ്ങളല്ലാത്ത ഇസ്ലാമിക യുദ്ധങ്ങൾ തന്നെയും നിർബന്ധിത മതപരിവർത്തനത്തിനോ അന്യ മത ധ്വംസനത്തിനോ വേണ്ടിയല്ല നടന്നത് എന്നർത്ഥം. മറിച്ച്, മത സ്വാതന്ത്ര്യത്തിനും അവകാശങ്ങൾക്കും വേണ്ടിയാണ്. ഇസ്ലാമിക ഭരണത്തിനു കീഴിൽ കൊണ്ടുവന്ന് അന്യ മത സമൂഹങ്ങളെ അടക്കിഭരിക്കുകയല്ല ലക്ഷ്യം. മറിച്ച്, അന്നത്തെ തിയോക്രാറ്റിക്ക് ഓട്ടോക്രസികളിൽ (തീവ്രമതവികാരത്തിൽ അധിഷ്ഠിതമായ സ്വേച്ഛാധിപത്യ ഭരണകൂടങ്ങൾ) ഇസ്ലാമിൽ ആകൃഷ്ടരാകുന്നവർക്ക് ഇസ്ലാം സ്വീകരിക്കാനുള്ള വഴിയൊരുക്കാനും ഇസ്ലാം അനുസരിച്ചുള്ള ജീവിതം എളുപ്പമാവാനും ഇസ്ലാമിക പ്രബോധനം സാധ്യമാകാനും അവരുടെ മേൽ ഭരണം നേടിയെടുക്കുകയല്ലാതെ മറ്റൊരു വഴിയും അന്നത്തെ രാഷ്ട്രീയ ഘടനയിൽ നില നിന്നിരുന്നില്ല. ഇസ്ലാമിന്റെ കടന്നുവരവിനെ ആ കാലഘട്ടത്തിലെ ഗോത്ര സമൂഹങ്ങളും നാട്ടുരാജ്യങ്ങളും മത സമുദായങ്ങളും എത്രത്തോളം അസഹിഷ്ണുതയോടെയാണ് സ്വീകരിച്ചത് എന്ന ചരിത്രസാക്ഷ്യങ്ങൾ മുമ്പ് പല ലേഖനങ്ങളിൽ നാം വിശദീകരിച്ചിട്ടുണ്ട്. (വിശദാശംങ്ങൾക്കായി: www.snehasamvadam.org: ‘ഇസ്ലാമിന്റെ യുദ്ധഭൂമിക’ എന്ന ലേഖനം വായിക്കുക: https://bit.ly/3BHqSuX)
ഇത്തരമൊരു ലോക വ്യവസ്ഥയിൽ ഇസ്ലാമിന്റെ അതിജീവനവും പ്രചാരണവും മറ്റെന്തു മാർഗത്തിലൂടെയാണ് സാധ്യമാകുമായിരുന്നത്? ഇത്തരമൊരു ഭൂമികയിൽ സമ്പൂർണ അഹിംസ രാഷ്ട്രീയ നയമായി ഇസ്ലാം സ്വീകരിച്ചിരുന്നുവെങ്കിൽ ഇതര സമൂഹങ്ങൾ ഇസ്ലാമിനെ വിഴുങ്ങുമായിരുന്നു. ഒരു നൂറ്റാണ്ടിനപ്പുറം വെളിച്ചം കാണാതെ അജ്ഞതയുടെ അഗണ്യകോടിയിൽ അലിഞ്ഞില്ലാതെയാവുമായിരുന്നു (;ദൈവത്തിൽ നിന്നുള്ള മതമായിരുന്നില്ല ഇസ്ലാം എങ്കിൽ.)
പക്ഷെ ഇസ്ലാം ദൈവത്തിൽ നിന്നുള്ള ആദർശമായത് കൊണ്ടും പ്രാവർത്തിക മതമായത് കൊണ്ടും അനിവാര്യ ഘട്ടങ്ങളിലുള്ള സായുധ സമരങ്ങൾക്ക് അനുവാദം നൽകപ്പെട്ടു. ഇസ്ലാം ദൈവത്തിൽ നിന്നുള്ള സന്മാർഗ സന്ദേശമാണ്. അത് ലോകം മുഴുവനും – കുടിലും കൊട്ടാരവും വ്യത്യാസമില്ലാതെ – എത്തിക്കുക എന്നത് ദൈവത്തിന്റെ ബാധ്യതയാണല്ലോ. അവൻ സ്വയം ഏറ്റെടുത്ത ബാധ്യത. അതിന് സാമൂഹിക അസഹിഷ്ണുതയും രാഷ്ട്രീയ തടസ്സങ്ങളും വിഘ്നങ്ങളായി കൂടാ. സത്യത്തിന്റെ ശത്രുക്കൾ രാഷ്ട്രങ്ങളുടേയും നാട്ടുരാജ്യങ്ങളുടേയും അതിർത്തികളിൽ വാളേന്തി നിൽക്കുകയും വാതിലുകൾ കൊട്ടിയടക്കുകയും ചെയ്തിരുന്ന കാലഘട്ടത്തിൽ വാളുകൊണ്ടല്ലാതെ ആ തടസ്സത്തെ നീക്കാൻ കഴിയില്ലായിരുന്നു. ഇന്നത്തെ ലോക-രാഷ്ട്ര വ്യവസ്ഥ അന്നത്തേതല്ല. രാഷ്ട്രങ്ങളുടെ വാതിലുകൾ ഏത് ആദർശങ്ങൾക്കും മതങ്ങൾക്കും മുമ്പിൽ – താത്ത്വികമായെങ്കിലും – മലർക്കേ തുറന്നിടപ്പെട്ടിരിക്കുകയാണ്, ഇന്ന്. ഇവിടെ, വാതിലുകളുടെ കാവൽക്കാർ വാളേന്തി നിൽക്കുന്നില്ല. പിന്നെയെന്തിന് മുസ്ലിംകൾ വാളെടുക്കണം ?!. വാതിലുകൾ കൊട്ടിയടക്കപ്പെട്ടാലല്ലേ ചവിട്ടി തുറക്കേണ്ടതുള്ളു ?!. തുറന്നു കിടക്കുന്ന വാതിൽ ആരെങ്കിലും ചവിട്ടി തുറക്കാൻ ശ്രമിക്കുമോ?!!
ഇസ്ലാമിക ഭരണത്തിനു കീഴിൽ പൂർണ്ണ മത സ്വാതന്ത്ര്യങ്ങളോടെയും അവകാശങ്ങളോടെയും ജീവിക്കാൻ ഒരു അമുസ്ലിമിന് വഴിയൊരുക്കണമെന്നാണ് മുഹമ്മദ് നബി (സ) അദ്ദേഹത്തിന്റെ അനുചരന്മാരെ പഠിപ്പിച്ചത് എന്ന് സാന്ദർഭികമായി സൂചിപ്പിക്കട്ടെ. ഇസ്ലാം മുന്നോട്ടുവെക്കുന്ന ഭരണ വ്യവസ്ഥ തീവ്രമതവികാരത്തിൽ അധിഷ്ഠിതമായ സ്വേച്ഛാധിപത്യ ഭരണകൂടമല്ല; ജനാധിപത്യ മൂല്യങ്ങളിൽ അധിഷ്ഠിതമായ ആത്മീയഭരണമാണ്. അവിടെയും ഇസ്ലാമിക ഭരണത്തിനു കീഴിൽ പ്രസ്തുത സമൂഹങ്ങളെ അടക്കിഭരിക്കുകയല്ല ലക്ഷ്യം; മുമ്പ് സൂചിപ്പിച്ചതു പോലെ അന്നത്തെ തിയോക്രാറ്റിക്ക് ഓട്ടോക്രസികളിൽ ജീവിക്കുന്ന മനുഷ്യർക്കിടയിൽ ഇസ്ലാം പ്രബോധനം ചെയ്യാനും, ഇസ്ലാം സ്വീകരിക്കാനും ഉള്ള വഴി വെട്ടുക മാത്രമാണ്.
‘ഹിന്ദി’നോടുള്ള യുദ്ധത്തെ സംബന്ധിച്ച പ്രവാചക പ്രവചനം കഴിഞ്ഞ കാലഘട്ടത്തിൽ പുലർന്നിട്ടില്ല എന്നും അത് ഭാവിയിൽ നടക്കാനിരിക്കുന്നതാണ് എന്നും ചില മുസ്ലിം പണ്ഡിതന്മാർ വ്യാഖ്യാനിക്കാറുണ്ട്. ഈ പണ്ഡിതാഭിപ്രായം അംഗീകരിച്ചാലും, സ്വതന്ത്രാനന്തര അതിർത്തിരേഖ ഉൾകൊള്ളുന്ന ആധുനിക മതേതര ജനാധിപത്യ ഇന്ത്യയോടുള്ള യുദ്ധമല്ല ഇതു കൊണ്ടും ഉദ്ദേശിക്കപ്പെടുന്നത്. ഈ പണ്ഡിത വ്യഖ്യാനം സ്വീകരിച്ചാലും ഇന്ത്യക്കെതിരെയുള്ള ഭീകരാക്രമണങ്ങൾ ഒരു സാഹചര്യത്തിലും ഈ ഹദീഥിൽ പ്രദിപാദിക്കുന്നില്ല. അന്ത്യനാളിനോടനുബന്ധിച്ച് തീർത്തും വ്യത്യസ്ഥമായ ഒരു ലോക ഘടനയിൽ, ലോകം മുഴുവൻ ബലപ്രയോഗത്തിലൂടെ അധിനിവേശം നടത്തുന്ന, മസീഹുദ്ദജ്ജാൽ എന്ന ഒരു ഏകശസനാധിപന്റെ കാലത്താണ് സംഭവം നടക്കുക. ഈ ഏകശാസകനിൽ നിന്നും ഹിന്ദിനെ മോചിപ്പിച്ച് സ്വാതന്ത്ര്യത്തിലേക്കും, നീതിയിലേക്കും തിരിച്ച് കൊണ്ടുപോകാനാണ് ഹിന്ദ് യുദ്ധം.
യുദ്ധം നടക്കുന്ന ലോകം നമ്മൾ ഇന്ന് ജീവിക്കുന്ന ലോകമല്ല. യുദ്ധം ചെയ്യപ്പെടുന്ന ‘ഹിന്ദ്’ ഇന്നത്തെ മതനിരപേക്ഷ ജനാധിപത്യ ഇന്ത്യയുമല്ല. ദജ്ജാലെന്ന ഏകാധിപതിയുടെ കീഴിൽ ഞെരിഞ്ഞമരുന്ന ഒരു ഹിന്ദിനോട് അന്ത്യദിനത്തോട് അടുത്തായി നടക്കുന്ന ഒരു യുദ്ധത്തിനെ (ഒരു സ്വാതന്ത്ര്യ സമരത്തിനെ) സ്വതന്ത്രാനന്തര അതിർത്തിരേഖ ഉൾകൊള്ളുന്ന ആധുനിക മതനിരപേക്ഷ ഇന്ത്യയോടുള്ള യുദ്ധമായി ദുർവ്യാഖ്യാനിക്കുകയാണ് ഇസ്ലാമോഫോബിക്കുകൾ ചെയ്തു കൊണ്ടിരിക്കുന്നത്. എന്നാൽ സമാധാന ചിത്തരായി ജീവിക്കുന്ന സാധാരണ അമുസ്ലിംകളോട് എന്ത് നിലപാടാണ് സ്വീകരിക്കേണ്ടത് എന്നതുമായി ബന്ധപ്പെട്ട ഇസ്ലാമിക നിയമവും പ്രവാചക പാഠവും മറ്റു ഹദീസുകളിൽ വ്യക്തമായി വായിക്കാൻ സാധിക്കും, അതിനോട് എതിരായി അന്യായമായി ഒരു നാടിനേയും ആക്രമിക്കാൻ പ്രവാചക ശ്രേഷ്ഠൻ (സ) ഒരിക്കലും കൽപ്പിക്കുകയില്ല.
അല്ലാഹു പറഞ്ഞു: “മതകാര്യത്തില് നിങ്ങളോട് യുദ്ധം ചെയ്യാതിരിക്കുകയും, നിങ്ങളുടെ വീടുകളില് നിന്ന് നിങ്ങളെ പുറത്താക്കാതിരിക്കുകയും ചെയ്യുന്നവരെ സംബന്ധിച്ചിടത്തോളം നിങ്ങളവര്ക്ക് നന്മ ചെയ്യുന്നതും നിങ്ങളവരോട് നീതി കാണിക്കുന്നതും അല്ലാഹു നിങ്ങളോട് നിരോധിക്കുന്നില്ല. തീര്ച്ചയായും അല്ലാഹു നീതി പാലിക്കുന്നവരെ ഇഷ്ടപ്പെടുന്നു. മതകാര്യത്തില് നിങ്ങളോട് യുദ്ധം ചെയ്യുകയും നിങ്ങളുടെ വീടുകളില് നിന്ന് നിങ്ങളെ പുറത്താക്കുകയും നിങ്ങളെ പുറത്താക്കുന്നതില് പരസ്പരം സഹകരിക്കുകയും ചെയ്തവരെ സംബന്ധിച്ചു മാത്രമാണ് -അവരോട് മൈത്രികാണിക്കുന്നത്- അല്ലാഹു നിരോധിക്കുന്നത്. വല്ലവരും അവരോട് മൈത്രീ ബന്ധം പുലര്ത്തുന്ന പക്ഷം അവര് തന്നെയാകുന്നു അക്രമകാരികള്.” (കുർആൻ: 60:8,9)
പ്രവാചകൻ മുഹമ്മദ് (സ) പറഞ്ഞു: مَنْ قَتَلَ مُعَاهَدًا لَمْ يَرَحْ رَائِحَةَ الْجَنَّةِ، وَإِنَّ رِيحَهَا لَيُوجَد مِنْ مَسِيرَةِ أَرْبَعِينَ عَامًا. “സമാധാന സന്ധിയിലുള്ള ഒരു അമുസ്ലിമിനെ ആരെങ്കിലും കൊന്നാൽ അവന് സ്വർഗത്തിന്റെ സുഗന്ധം പോലും ലഭിക്കില്ല.” (സ്വഹീഹുൽ ബുഖാരി: ഹദീസ് നമ്പർ: 3166)
عَنْ صَفْوَانَ بْنَ سُلَيْمٍ عَنْ رَسُولِ اللَّهِ صلى الله عليه وسلم قَالَ “ أَلاَ مَنْ ظَلَمَ مُعَاهِدًا أَوِ انْتَقَصَهُ أَوْ كَلَّفَهُ فَوْقَ طَاقَتِهِ أَوْ أَخَذَ مِنْهُ شَيْئًا بِغَيْرِ طِيبِ نَفْسٍ فَأَنَا حَجِيجُهُ يَوْمَ الْقِيَامَةِ
സ്വഫ്വാനു ബ്നു സുലൈമില്(റ) നിന്ന് നിവേദനം: പ്രവാചകൻ (സ) പറഞ്ഞു: ‘അറിയണം, ആരെങ്കിലും സമാധാന സന്ധിയിലുള്ള അമുസ്ലിമിനെ ഉപദ്രവിക്കുകയോ, അവന് കിട്ടേണ്ട അവകാശങ്ങളില് കുറവ് വരുത്തുകയോ, സാധ്യമാകുന്നതിലുപരി വഹിക്കാന് അവനെ നിര്ബന്ധിക്കുകയോ, മനപ്പൊരുത്തമില്ലാതെ അവനില് നിന്നും വല്ലതും കവര്ന്നെടുക്കുകയോ ചെയ്താൽ ഉയിര്ത്തെഴുന്നേല്പ്പ് നാളില് ഞാന് അവനുമായി (ആ അമുസ്ലിമിന്റെ അവകാശങ്ങളുടെ കാര്യത്തിൽ) തര്ക്കത്തിലേര്പെടും. (അബൂദാവൂദ്: 3052).
ഇതാണ് സമാധാന ചിത്തരായി ജീവിക്കുന്ന സാധാരണ അമുസ്ലിംകളോട് സ്വീകരിക്കേണ്ട നിലപാടായി ഇസ്ലാം പഠിപ്പിച്ച നിയമവും ശാസനയും. എത്ര തവണ വ്യക്തമാക്കിയാലും ഈ ആയത്തുകളും ഹദീസുകളും വിമർശകർ കാണാത്ത മട്ടാണ് !!
ദജ്ജാലിനോടും ദജ്ജാലിന്റെ അധിനിവേശത്തോടുമുള്ള യുദ്ധം അന്ത്യദിനത്തോടനുബന്ധിച്ചാണ്, ഈസാ നബിയുടെ(അ) (യേശു) നേതൃത്വത്തിലാണ് നടക്കുക. (ഫത്ഹുൽ ബാരി: 6: 610, അത്തൗദീഹു ലി ശർഹിൽ ജാമിഅ്: 17:663)
അന്ത്യനാളിനോടടുത്ത് പ്രത്യക്ഷനാകുമെന്ന് പല മത ഗ്രന്ഥങ്ങളും പ്രവചിച്ച, അന്തിക്രിസ്തു (antichrist) എന്ന പേരിൽ അറിയപ്പെടുന്ന ഒരു ഭാവി വ്യക്തിത്വമാണ് ‘മസീഹു ദ്ദജ്ജാൽ’. ലോകത്ത് ജീവിച്ചിരുന്നതും ജീവിച്ചിരിക്കുന്നവരുമായ ഏകാധിപതികളേക്കാൾ ഏറ്റവും കിരാതനും ക്രൂരനുമായ ഏകാധിപതിയായിരിക്കും (dictator) മസീഹു ദ്ദജ്ജാൽ എന്നാണ് ഹദീസുകൾ പഠിപ്പിക്കുന്നത്. മനുഷ്യരെ ഭൗതീകവും ആത്മീയവുമായ പാരതന്ത്ര്യത്തിലേക്ക് വലിച്ചിഴക്കുന്ന ഈ ദജ്ജാലിന്റെ ഉപദ്രവങ്ങളിൽ നിന്ന് രക്ഷ തേടാൻ പ്രവാചകൻ (സ) തന്റെ അനുചരന്മാരോട് പ്രത്യേകം ഉപദേശിക്കുക കൂടി ചെയ്തതായി കാണാം. (സ്വഹീഹു മുസ്ലിം: 924)
ദജ്ജാലിനാൽ ഭാവിയിൽ, ലോകത്ത് വിതക്കപ്പെടാനിരിക്കുന്ന കുഴപ്പങ്ങളും ഛിദ്രതകളും സമാനതകൾ ഇല്ലാത്തതായിരിക്കും.
مَا بَيْنَ خَلْقِ آدَمَ إِلَى أَنْ تَقُومَ السَّاعَةُ فِتْنَةٌ أَكْبَرُ مِنْ فِتْنَةِ الدَّجَّالِ . പ്രവാചകൻ (സ) പറഞ്ഞു: “(ആദ്യ മനുഷ്യൻ) ആദമിനെ സൃഷ്ടിച്ചതു മുതൽ അന്ത്യദിനം സംഭവിക്കുന്നതു വരെ ദജ്ജാലിന്റെ കുഴപ്പത്തേക്കാൾ വലിയ ഒരു ആപത്തും ഇല്ലതന്നെ.” (മുസ്നദു അഹ്മദ്: 15831)
ലോക രാജ്യങ്ങൾ മുഴുവൻ വെട്ടിപ്പിടിച്ച് തന്റെ അധികാരത്തിനും ആജ്ഞാപനത്തിനും കീഴിലാക്കുന്ന ലോകം കണ്ട ഏറ്റവും വലിയ ഏകാധിപതിയായിരിക്കും ‘മസീഹു ദ്ദജ്ജാൽ’.
ليس من بلد إلا سيطؤه الدجال إلا مكة والمدينة ليس له من نقابها نقب إلا عليه الملائكة صافين يحرسونها
ദജ്ജാൽ കാലു കുത്താത്ത ഒരു നാടും അവശേഷിക്കില്ല; മക്കയും മദീനയും ഒഴികെ. ആ രണ്ട് നാടുകളെയും സംരക്ഷിച്ചു കൊണ്ട് മലക്കുകൾ വലയം ചെയ്യുന്നുണ്ടാകും. (സ്വഹീഹുൽ ബുഖാരി: 1881, സ്വഹീഹു മുസ്ലിം: 2943)
നാടുകളിൽ കാലുകുത്തുക എന്നതുകൊണ്ടുദ്ദേശം പടയോട്ടത്തിലൂടെ അധികാരത്തിന് കീഴിലാക്കുക എന്നാണ്. فلا يبقى له موضع إلا ويأخذه غير مكة والمدينة “മക്കയും മദീനയും തുടങ്ങിയ സ്ഥലങ്ങളല്ലാതെ ഒരു സ്ഥലവും അവൻ പിടിച്ചടക്കാത്തതായി അവശേഷിക്കില്ല” എന്ന് ചില നിവേദനങ്ങളിൽ കാണാം. (ഉംദത്തുൽ കാരി: 10:244)
“എല്ലാ ജല തടത്തിലും അവന്റെ അധികാരമെത്തും” (يبلغ سلطانه كل منهل) എന്ന് മറ്റു ചില നിവേദനങ്ങളിലും വന്നിരിക്കുന്നു. (മുസ്നദു അഹ്മദ്: 23139)
(സ്വാഭാവികമായും ദജ്ജാലിന്റെ ഈ അധിനിവേശം ഇന്ത്യയിലുമെത്തുമല്ലൊ. ആദർശ സ്വാതന്ത്ര്യവും ജനാധിപത്യ മൂല്യങ്ങളും ദജ്ജാലിനാൽ തിരോധാനം ചെയ്യപ്പെടുന്ന കാലത്ത് അവ തിരിച്ചു പിടിക്കാൻ രക്ഷകരായി കൊണ്ടാണ് ഈ പണ്ഡിതാഭിപ്രായപ്രകാരം മുസ്ലിംകൾ ഇന്ത്യയിലേക്ക് കടന്നുവരിക എന്നർത്ഥം)
ദജ്ജാലിന്റെ സാമ്രാജ്യത്വ അധിനിവേശത്തിനു കീഴിൽ മനുഷ്യ ജീവിതങ്ങൾ ഞെരിഞ്ഞമരും. അങ്ങനെ ലോകം മുഴുവൻ അക്രമവും അനീതിയും അടിച്ചമർത്തലുകളും നിറയുന്ന ഘട്ടത്തിൽ മഹ്ദി എന്ന സ്ഥാനപേരിലുള്ള ഒരു ഭരണാധികാരിയുടേയും പ്രവാചകനായ ഈസാ(അ)യുടേയും നേതൃത്വത്തിൽ അന്ത്യദിനത്തോടടുത്ത് നടക്കുന്ന സ്വാതന്ത്ര്യ സമരങ്ങൾ ഒരുപാട് ഹദീസുകളുടെ ഇതിവൃത്തമായിട്ടുണ്ട്. (ഇക്കൂട്ടത്തിൽ തന്നെയാണ് ‘ഗസ്വത്തുൽ ഹിന്ദും’ എന്നാണ് ഈ വിഭാഗം പണിതന്മാരുടെ അഭിപ്രായം)
أبشركم بالمهدي يبعث على اختلاف من الناس وزلازل فيملأ الأرض قسطاً وعدلاً كما ملئت جوراً وظلماً “ലോകം മുഴുവൻ അക്രമങ്ങളും സ്വേച്ഛാധിപത്യവും കൊണ്ട് നിറഞ്ഞ സന്ദർഭത്തിൽ ലോകത്തെ നീതിയും ന്യായവും കൊണ്ട് മഹ്ദി നിറക്കുമെന്ന്” ഹദീസിൽ പ്രസ്ഥാവിക്കുന്നുണ്ട്. (മുസ്നദു അഹ്മദ്: 11344, അൽ അഹ്കാമുശറഇയ്യ അൽ കുബ്റാ: അബ്ദുൽ ഹക്ക് അൽ ഇശ്ബീലി: 4/532, മജ്മഉസ്സവാഇദ്: 7/316)
അത്ഭുത സിദ്ധികൾ പലതും പ്രദർശിപ്പിച്ച് ജനങ്ങളെ വശീകരിക്കുന്നതിന് പുറമെ, ദജ്ജാലെന്ന ഏകശസനാധികാരി ആളുകളെ ചതിയിലൂടെ അവരെ അഗ്നിയിലേക്ക് നയിക്കുമെന്ന സൂചനകൾ ഹദീസുകളിൽ കാണാം.
‘അയാളുടെ കൂടെ രണ്ട് ജല തടങ്ങളുണ്ടാകും. ഒന്ന് പ്രത്യക്ഷ ദൃഷ്ട്യാ വെള്ളമായിരിക്കും. മറ്റൊന്ന് ദൃഷ്ട്യാൽ അഗ്നിയായിരിക്കും. എന്നാൽ വെള്ളമെന്ന് തോന്നിപ്പിക്കപ്പെടുന്ന തടങ്ങൾ യഥാർത്ഥത്തിൽ അഗ്നിയായിരിക്കും എന്നും’ ഹദീസുകൾ സൂചിപ്പിക്കുന്നുണ്ട്. (സ്വഹീഹു മുസ്ലിം: 5223)
അത്ഭുത പ്രവർത്തനങ്ങൾ പ്രദർശിപ്പിച്ച് ദിവ്യത്വം വാദിക്കുക കൂടി ചെയ്യും. അധികാരത്തിലൂടെ തന്റെ ദിവ്യത്വം ജനങ്ങളിൽ അടിച്ചേൽപ്പിക്കുക എന്നത് ദജ്ജാലിന്റെ പദ്ധതികളിൽ ഒന്നായിരിക്കും.
“ദജ്ജാൽ ജനങ്ങളോട് പറയും: ഈ വ്യക്തിയെ (ഒരു മുസ്ലിം) ഞാൻ കൊല്ലുകയും പിന്നീട് ജീവിപ്പിക്കുകയും ചെയ്താൽ (ഞാൻ നിങ്ങളുടെ ദൈവമാണെന്നതിൽ) നിങ്ങൾ സംശയിക്കുമോ? ജനങ്ങൾ പറയും: ഇല്ല. അപ്പോൾ ദജ്ജാൽ അയാളെ കൊല്ലുകയും ജീവിപ്പിക്കുകയും ചെയ്യും. അപ്പോൾ ജീവൻ തിരിച്ചു കിട്ടിയ വ്യക്തി പറയും: “അല്ലാഹുവാണേ, മുമ്പൊരിക്കലും നിന്നെക്കുറിച്ച് എനിക്കില്ലാതിരുന്ന വ്യക്തമായ തിരിച്ചറിവ് ഇന്നെനിക്ക് ലഭിച്ചു.” (അഥവാ നബി പറഞ്ഞ ദജ്ജാൽ നീ തന്നെയാണു എന്ന് എനിക്കിപ്പോൾ ഉറപ്പായി എന്നയാൾ ആണയിടും. കാരണം ഇപ്രകാരം ഒരു സംഭവമുണ്ടാകുമെന്ന പ്രവാചകന്റെ ഹദീസിന് ജീവിക്കുന്ന തെളിവായി അയാൾ മാറുമല്ലൊ) അപ്പോൾ ദജ്ജാൽ അയാളെ വീണ്ടും കൊല്ലാൻ ശ്രമിക്കും. പക്ഷെ അതിന് ദജ്ജാലിനെ അല്ലാഹു അനുവദിക്കില്ല.” (സ്വഹീഹുൽ ബുഖാരി: 6599)
തന്റെ ദിവ്യത്വത്തിൽ ജനങ്ങളെ നിർബന്ധിച്ച് വിശ്വസിപ്പിക്കാനായി അധികാരവും മായാജാലവുമൊക്കെ ദജ്ജാൽ ഉപയോഗിക്കും. എന്നിട്ടും വിശ്വസിക്കാത്തവർക്ക് വധശിക്ഷയാണ് ദജ്ജാലിന്റെ ഭരണകൂടം വിധിക്കുക. വിശ്വാസ സ്വാതന്ത്ര്യം നിശ്ശേഷം നിഷേധിക്കപ്പെടുന്ന ഈ സാമ്രാജ്യത്വ സ്വേച്ഛാധിപത്യത്തോടാണ് മുസ്ലിംകൾ പടക്കിറങ്ങുന്നത് എന്നർത്ഥം.
ഈ സ്വേച്ഛാധിപതിയുടെ രാജാധികാരത്തോടും അയാളുടെ കിങ്കരന്മാരോടുമാണ് അന്ത്യ നാളിനോടടുത്ത് സ്വാതന്ത്ര്യ സമരങ്ങൾ നടക്കുക. അല്ലാതെ സമാധാനത്തോടെ ജീവിക്കുന്ന അമുസ്ലീംകളോടോ അവരുടെ നാടുകളോടോ അല്ല. ചുരുക്കത്തിൽ, ഗസ്വത്തുൽ ഹിന്ദ് പരാമർശിക്കുന്ന ഹദീഥുകളെ തെറ്റിദ്ധരിപ്പിച്ചു കൊണ്ട് ഇന്ത്യൻ മുസ്ലിംകൾ ഇന്ത്യയോട് യുദ്ധം ചെയ്യാൻ തക്കം പാർത്തിരിക്കുന്നവരാണെന്ന പ്രചാരണം അബദ്ധജടിലവും അടിസ്ഥാനരഹിതവുമാണ്. അകാരണമായും അന്യായമായും ഒരു നാടിനോടും യുദ്ധം ചെയ്യാനും അക്രമിക്കുവാനും ഈ ഹദീസിൽ എന്നല്ല ഇസ്ലാമിക പ്രമാണങ്ങളിൽ എവിടെയും പഠിപ്പിക്കുന്നില്ല.
വിമർശനം:
“സൂര്യൻ ചൂടുള്ള ജലാശയത്തിൽ അസ്തമിക്കുന്നു” എന്ന് മുഹമ്മദ് നബി തന്റെ അനുചരനായ അബൂദർറിനോട് പറഞ്ഞതായി ഹദീസ് പ്രസ്ഥാവിക്കുന്നു.
മറുപടി:
സൂര്യൻ ചൂടുള്ള ജലാശയത്തിൽ അസ്തമിക്കുന്നുവെന്ന് പ്രവാചകൻ (സ) പറഞ്ഞതായി സ്വഹീഹായ ഒരു ഹദീസിലൂടെയും സ്ഥാപിതമായിട്ടില്ല. വിമർശകർ പ്രശ്നവൽകരിക്കുന്ന ഹദീസാകട്ടെ ദുർബലമാണ് (ദഈഫ് ضعيف).
ഹദീസിന്റെ നിവേദക പരമ്പര ഇപ്രകാരമാണ്:
ﺣﺪﺛﻨﺎ ﻋﺜﻤﺎﻥ ﺑﻦ ﺃﺑﻲ ﺷﻴﺒﺔ، ﻭﻋﺒﻴﺪ اﻟﻠﻪ ﺑﻦ ﻋﻤﺮ ﺑﻦ ﻣﻴﺴﺮﺓ اﻟﻤﻌﻨﻰ، ﻗﺎﻻ: ﺣﺪﺛﻨﺎ ﻳﺰﻳﺪ ﺑﻦ ﻫﺎﺭﻭﻥ، ﻋﻦ ﺳﻓﻲاﻥ ﺑﻦ ﺣﺴﻴﻦ، ﻋﻦ اﻟﺤﻜﻢ ﺑﻦ ﻋﺘﻴﺒﺔ، ﻋﻦ ﺇﺑﺮاﻫﻴﻢ اﻟﺘﻴﻤﻲ، ﻋﻦ ﺃﺑﻴﻪ، ﻋﻦ ﺃﺑﻲ ﺫﺭ، ﻗﺎﻝ: (സുനനു അബൂദാവൂദ്: 4002) വിമർശനവിധേയമായ ഹദീസ് ദുർബലമാണ് (ദഈഫ് ضعيف). (അതിനാൽ തന്നെ പ്രവാചകന്റെ (സ) പ്രസ്ഥാവനയായി അത് സ്ഥാപിതമാവുന്നില്ല.) കാരണങ്ങൾ: 1. ഹദീസിന്റെ നിവേദക പരമ്പരയിൽ സുഫ്യാനിബ്നു ഹുസൈൻ എന്ന റാവി (നിവേദകൻ) ഉണ്ട്. അദ്ദേഹം ദുർബലനാണ് (ദഈഫ്). യഅ്കൂബിബ്നു ശൈബ പറഞ്ഞു: അദ്ദേഹത്തിന്റെ ഹദീസിൽ ദുർബലമായവയുണ്ട്. ഉസ്മാനുബ്നു അബീ ശൈബ പറഞ്ഞു: അദ്ദേഹം വിശ്വസ്ഥനാണെങ്കിലും ഹദീസ് ഉദ്ധരിക്കുന്നതിൽ (ഓർമ്മക്കുറവ് കാരണം) വൈരുദ്ധ്യങ്ങൾ സംഭവിക്കാറുണ്ട്. ഇബ്നു സഅ്ദ് പറഞ്ഞു: അദ്ദേഹം വിശ്വസ്ഥനാണെങ്കിലും ഹദീസ് ഉദ്ധരിക്കുന്നതിൽ ധാരാളം തെറ്റുകൾ സംഭവിക്കാറുണ്ട്. (വിമർശനവിധേയമായ ഹദീസ് ഉദ്ധരിച്ച) അബൂദാവൂദ് തന്നെ, ഇബ്നു മഈനിൽ നിന്ന് ഉദ്ധരിക്കുന്നത് ഇപ്രകാരമാണ്: ‘സുഫ്യാനിബ്നു ഹുസൈൻ’ ഹദീസ് മനപാഠശേഷിയിൽ ഉന്നതശ്രേണി വഹിക്കുന്ന പണ്ഡിതന്മാരിൽപ്പെട്ട വ്യക്തിയായിരുന്നില്ല. (തഹ്ദീബു ത്തഹ്ദീബ്: 4:190) 2. മാത്രമല്ല, സൂര്യന്റെ അസ്തമയത്തെ സംബന്ധിച്ച്, ബുഖാരിയും മുസ്ലിമും -വിശ്വസ്ഥതയിലും ഓർമ്മശക്തിയിലും ഉന്നതശ്രേണിയലങ്കരിക്കുന്ന നിവേദകന്മാരിൽ നിന്ന്- ഉദ്ധരിച്ച ഹദീസിൽ നിന്നും വ്യത്യസ്ഥമായ ഒറ്റപ്പെട്ട വാചകങ്ങളോടെയാണ് ‘സുഫ്യാനുബ്നു ഹുസൈൻ’ ഉദ്ധരിച്ചിരിക്കുന്നത് എന്ന് ഇമാം ബസ്സാർ തന്റെ മുസ്നദിലും സൂചിപ്പിക്കുന്നുണ്ട്. (മുസ്നദുൽ ബസ്സാർ: 4010) “സൂര്യൻ ചൂടുള്ള ജലാശയത്തിൽ അസ്തമിക്കുന്നുവെന്ന” പ്രസ്ഥാവന (ദുർബല റാവിയായ) ‘സുഫ്യാനുബ്നു ഹുസൈന്റെ’ നിവേദനത്തിൽ മാത്രമെ കണ്ടെത്താനാവു. സൂര്യാസ്തമയത്തെ സംബന്ധിച്ച്, വിശ്വസ്ഥതയിലും ഓർമ്മശക്തിയിൽ ഉന്നതശ്രേണിയലങ്കരിക്കുന്ന നിവേദകന്മാർ ഉദ്ധരിച്ച ഹദീസിന് വിപരീതമായ, ഓർമ്മക്കുറവുള്ള ഒരു ദുർബലനായ റാവി ഉദ്ധരിച്ച നിവേദനമായതിനാൽ പ്രവാചകൻ (സ) പറഞ്ഞതായി ഈ ഹദീസ് സ്ഥാപിതമാകുന്നില്ല. (അന്നദ്റുൽ മകാസ്വിദി വദവാബിതുഹു: 47)മനുഷ്യർക്ക് ഉപദ്രവമുണ്ടാക്കുന്ന ചില ജീവികളെ ആവശ്യ സന്ദർഭത്തിൽ കൊല്ലാനുള്ള അനുവാദം പ്രവാചകൻ (സ) നൽകിയിട്ടുണ്ട്. ഇത്തരം ഉപദ്രവകാരികളായ ജീവികളെ ‘ഫവാസിക്‘ (فَوَاسِقُ) എന്നാണ് ഹദീസുകളിൽ വിളിച്ചിട്ടുള്ളത്. ഇത്തരം ജീവികളെ സാധാരണയായി Animal rights activist കളല്ലാത്ത എല്ലാവരും – മതത്തിന്റെയൊ ആദർശത്തിന്റെയൊ വ്യത്യാസമില്ലാതെ കൊല്ലാറുമുണ്ട്. (പക്ഷെ ഇത്തരം ഉപദ്രവകാരികളായ ജീവികളെ കൊല്ലാൻ, മുഹമ്മദ് നബി (സ) അനുവാദം നൽകി എന്നതുകൊണ്ട് ‘താൽകാലിക’ അഹിംസ വാദികളായി ഇസ്ലാം വിമർശകർ നാട്യം കളിക്കാറുണ്ടെന്ന് മാത്രം.) ഫവാസിക് (فَوَاسِقُ) ഉപദ്രവകാരികളായ ജീവികൾ എന്ന് പ്രവാചകൻ (സ) എണ്ണിയവ ഹദീസുകളിൽ നിന്ന് വായിക്കാം: خَمْسٌ مِنَ الدَّوَابِّ كُلُّهَا فَوَاسِقُ تُقْتَلُ فِي الْحَرَمِ: الْغُرَابُ، وَالْحِدَأَةُ، وَالْكَلْبُ الْعَقُورُ، وَالْعَقْرَبُ، وَالْفَارَةُ.
“മൃഗങ്ങളിൽ നിന്നുള്ള അഞ്ചെണ്ണം ‘ഫവാസികുകൾ’ (ഉപദ്രവകാരികൾ) ആകുന്നു. അവയെ ഹറമിൽ വെച്ചായാൽ (പോലും) കൊല്ലൽ അനുവദനീയമാണ്. അവ: കാക്ക, ഗരുഡൻ, കടിക്കുന്ന നായ്, തേൾ, എലി എന്നിവയാണ്.”
ചില നിവേദനത്തിൽ തേളിന് പകരം സർപ്പത്തെ പറയപ്പെട്ടിരിക്കുന്നു. മറ്റൊരു നിവേദനത്തിൽ കാക്കയെ ”അൽ ഗുറാബുൽ അബ്കഅ്” (وَالْغُرَابُ الأَبْقَعُ) എന്ന് പ്രത്യേകമായി വിശേഷിപ്പിക്കുകയും ചെയ്തിരിക്കുന്നു.
“അവയെ കൊല്ലുന്നതിൽ പാപമില്ല” എന്ന് ഒരു നിവേദനത്തിൽ കാണാം. (ബുഖാരി: 1829, മുസ്ലിം: 1198, തുർമുദി: 837, നസാഈ: 2888, ഇബ്നുമാജ: 3087)
മറ്റു ചില നിവേദനങ്ങളിൽ ഈ ‘ഫവാസിക്കു’കളുടെ കൂട്ടത്തിൽ ചെന്നായ, പുലി എന്നിവയെയൊക്കെ പ്രസ്ഥാവിച്ചതായും വന്നിരിക്കുന്നു(ഫത്ഹുൽ ബാരി: 4: 30) എന്നതിൽ നിന്നെല്ലാം ഇവയെ കൊല്ലാൻ അനുവാദം നൽകിയതിന്റെ കാരണം വ്യക്തമാണ്. അവ സാധാരണ വളർത്തു മൃഗങ്ങളിൽ നിന്നും വ്യത്യസ്ഥമായി ഉപദ്രവകാരികളാണ് എന്നതാണത്.
ഇമാം മാലിക് പറഞ്ഞു: മനുഷ്യനെ കടിക്കുകയും ആക്രമിക്കുകയും ഭയപ്പെടുത്തുകയും ചെയ്യുന്ന പുലി, സിംഹം, ചെന്നായ എന്നിവയും കടിക്കുന്ന നായയുടെ സ്ഥാനത്ത് തന്നെയാണ്. (അൽ മുൻതകാ ശർഹുൽ മുവത്വഅ് 2: 262)
ഇമാം ഐനി പറഞ്ഞു: “ഫവാസിക്കുകളിൽ കാക്കയെ എണ്ണിയപ്പോൾ, “വെള്ളയും കറുപ്പും നിറം കലർന്ന കാക്ക” (ഗുറാബുൽ അബ്കഅ് وَالْغُرَابُ الأَبْقَعُ) എന്ന് ഒരു ഹദീസിൽ പ്രത്യേകമായി വിശേഷിപ്പിച്ചിരിക്കുന്നു. കാക്കയെ കൊല്ലാൻ അനുവാദം നൽകിയത് കാക്ക ഇങ്ങോട്ട് ഉപദ്രവിക്കുന്നു എന്നതിനാലാണ്. “വെള്ളയും കറുപ്പും നിറം കലർന്ന കാക്ക”കളാണ് ഇങ്ങോട്ട് ഉപദ്രവിക്കുക. അതല്ലാത്ത, ഉപദ്രവകാരികളല്ലാത്ത കാക്കകളെ കൊല്ലാനും പാടില്ല എന്ന് ഹദീസിൽ നിന്ന് മനസ്സിലാക്കാം.” (ഉംദത്തുൽ കാരി 10:180)
ഇമാം ഇബ്നു ഹജർ പറഞ്ഞു: “ഹദീസിന്റെ ആശയത്തിൽ നിന്ന് കൊല്ലാൻ അനുവാദം നൽകിയതിന് കാരണം മനുഷ്യരെ ഉപദ്രവിക്കുക എന്നതാണ് എന്ന് വരുന്നു. അപ്പോൾ മനുഷ്യരെ ഉപദ്രവിക്കുന്ന ഏത് ജീവിയേയും ആവശ്യഘട്ടത്തിൽ ഫവാസിക്ക് എന്നതിൽ ഉൾപ്പെടുത്താം.” (ഫത്ഹുൽ ബാരി: 4: 30)
ഈ ഒരു അടിസ്ഥാനത്തിൽ നിന്നുകൊണ്ട്, പല്ലി ശല്യം അധികരിച്ചാൽ അവയുടെ ഉപദ്രവകാരണത്താൽ അവയെ കൊല്ലാനും പ്രവാചകൻ (സ) അനുവാദം നൽകി. പല്ലിയുൾപ്പെടെയുള്ള ‘ഫവാസിക്കു’കളെ തേടിപ്പിടിച്ച് കൊല്ലാനല്ല പ്രവാചക കൽകപ്പനയുടെ ഉദ്ദേശമെന്ന് “അവയെ കൊല്ലുന്നതിൽ കുറ്റമില്ല.” (لا حرج على من قتلهن) എന്ന ഹദീസിലെ (ബുഖാരി: 1828) വാചകത്തിൽ നിന്ന് മനസ്സിലാക്കാം. വേട്ടയാടേണ്ട ഒന്നല്ല പല്ലി എന്ന കാര്യത്തിൽ മുസ്ലിം പണ്ഡിതന്മാരെല്ലാം ഏകോപിച്ചിരിക്കുന്നു എന്ന് അബൂ അംറ് അൽ കുർതുബി (മരണം: 463 ഹി) പറയുന്നു. (അത്തംബീഹ് ലിമാ ഫിൽ മുവത്വഅ് മിനൽ മആനി വൽ അസാനീദ് 15:187)
കൂടാതെ, “പല്ലികളെ കൊല്ലുവാൻ പ്രവാചകൻ (സ) കൽപ്പിച്ചതായി ‘ഞാൻ’ കേട്ടിട്ടില്ല” എന്ന് പ്രവാചക പത്നി ആഇശ (റ) പറഞ്ഞതിന്റെ (സ്വഹീഹുൽ ബുഖാരി: 1831) അർത്ഥമെന്താണ്? പ്രിയ പത്നി ആഇശയോട് പല്ലികളെ കൊല്ലുന്നതിനെ സംബന്ധിച്ച് പ്രവാചകൻ (സ) സംസാരിച്ചിട്ടേയില്ല എന്നാണ് ! അഥവാ പല്ലിയെ കൊല്ലാൻ പറഞ്ഞത് പല്ലി ശല്യവും ഉപദ്രവവും ഉള്ളവരോട് മാത്രമാണ്. പ്രവാചകൻ (സ) സ്വയം പല്ലിയെ കൊന്നതായും ഒരു ഹദീസിലും ഇല്ല !!! (പല്ലിയെ കൊല്ലൽ മതത്തിൽ ഒരു പുണ്യകർമമായിരുന്നെങ്കിൽ പ്രവാചകൻ (സ) അത് നിരന്തരമായി അനുഷ്ടിക്കാതിരിക്കില്ലല്ലൊ.) ഇതും സൂചിപ്പിക്കുന്നത് പല്ലിയെ കൊല്ലൽ പല്ലി ശല്യവും ഉപദ്രവവും ഉള്ളവർക്കുള്ള ഒരു സ്വഭാവിക അനുമതി മാത്രമാണ്. അല്ലാതെ പല്ലിയെ കൊല്ലൽ ഒരു പ്രമേയമായോ കാമ്പയിനായോ അദ്ദേഹം അവതരിപ്പിച്ചിട്ടില്ല എന്നർത്ഥം.
പല്ലിയെ കൊല്ലാൻ അനുവാദം നൽകിയ ഹദീസുകളിൽ അതിനുള്ള കാരണവും വ്യക്തമാക്കിയിട്ടുണ്ട്. അവ ‘ഫുവൈസിക്ക്’ അഥവാ ‘കുറിയ ഉപദ്രവകാരികളാണ്’ എന്നതാണത്.
أَنَّ رَسُولَ اللَّهِ صَلَّى اللهُ عَلَيْهِ وَسَلَّمَ، قَالَ لِلْوَزَغِ: فُوَيْسِقٌ…
“അല്ലാഹുവിന്റെ തിരുദൂതൻ (സ) പല്ലികളെ സംബന്ധിച്ച് ‘ഫുവൈസിക്ക്’ (കുറിയ ഉപദ്രവകാരി) എന്ന് പറഞ്ഞു…” (സ്വഹീഹുൽ ബുഖാരി: 1831)
أَنَّ النَّبِيَّ صَلَّى اللهُ عَلَيْهِ وَسَلَّمَ” أَمَرَ بِقَتْلِ الْوَزَغِ ، وَسَمَّاهُ فُوَيْسِقًا”
“പല്ലികളെ കൊല്ലാൻ (അനുവാദം നൽകിക്കൊണ്ട്) പ്രവാചകൻ (സ) കൽപ്പന പുറപ്പെടുവിച്ചു. അതിനെ ‘ഫുവൈസിക്ക്’ (കുറിയ ഉപദ്രവകാരി) എന്ന് അദ്ദേഹം വിളിച്ചു.” (സ്വഹീഹു മുസ്ലിം: 2238)
ഒരു അന്ധവിശ്വാസത്തിന്റെയൊ മിഥ്യാ ധാരണയുടെയൊ അടിസ്ഥാനത്തിലല്ല പല്ലികളെ കൊല്ലാൻ പ്രവാചകൻ (സ) അനുവാദം നൽകിയത്. അവ മനുഷ്യർക്ക് ശല്യവും ഉപദ്രവവുമായി മാറുന്നതിന്റെ അടിസ്ഥാനത്തിലാണ് എന്ന് ഹദീസുകളിൽ വ്യക്തമായി പ്രഖ്യാപിച്ചിട്ടുണ്ട് എന്ന് ചുരുക്കം.
ഇമാം ദിംയരി പറഞ്ഞു: “പല്ലിയെ സംബന്ധിച്ച് ‘ഫുവൈസിക്ക്’ (കുറിയ ഉപദ്രവകാരി) എന്ന് വിശേഷിപ്പിക്കാൻ കാരണം, ഹറമിലും അല്ലാത്തിടത്തും കൊല്ലാൻ അനുവാദം നൽകപ്പെട്ട ഉപദ്രവകാരികളായ (ഫവാസിക്ക്) ജീവികളിൽ പല്ലി പെടുന്നു എന്നതിനാലാണ്. ‘ഫിസ്ക്’ (الفسق) എന്ന പദത്തിന്റെ അടിത്തറ ‘ഖുറൂജ് ‘ (الخروج പുറത്തുപോവുക) എന്നതാണ്. ഇപ്പറഞ്ഞ (അഞ്ച് ഉപദ്രവകാരികളായ ജീവികളും) മനുഷ്യനെ കടിച്ചും ഉപദ്രവിച്ചും ശാരീരിക അപായങ്ങൾ വരുത്തിയും ഭൂരിഭാഗം ജീവികളുടെയും സ്വഭാവത്തിൽ നിന്നും പ്രകൃതത്തിൽ നിന്നും ‘പുറത്തുപ്പോവുന്നു’ എന്നതിനാലാണ് അവക്ക് ഫവാസിക്ക് എന്ന പേര് നൽകപ്പെട്ടത്. (ഹയാത്തുൽ ഹയവാനുൽ കുബ്റാ: 2:546)
ശൈഖ് മുനജ്ജിദ് പറഞ്ഞു: فعلة قتله :الأذى والضرر. “അപ്പോൾ അവയെ കൊല്ലാനുള്ള കാരണം മനുഷ്യന് ഉപദ്രവങ്ങളും ശാരീരിക അപായങ്ങളും വരുത്തുന്നവയാണ് അവ എന്നതാണ്.” (ഇസ്ലാം: സുആൽ വൽജവാബ്: 289055)
“പല്ലിയെ ‘ഫുവൈസിക്ക’ എന്നാണ് വിളിക്കപ്പെട്ടത്. ത്വയ്യിബി പറഞ്ഞു: പല്ലിയെ ഇപ്രകാരം വിളിക്കാൻ കാരണം (ഫവാസിക്) ഉപദ്രവകാരികളായ അഞ്ച് ജീവികളെ പോലെ പല്ലിയിൽ നിന്നും ഉപദ്രവമുണ്ടാകാം എന്നതിനാലാണ്.” (ശർഹു സുനനു ഇബ്നുമാജ: 1: 232)
പല്ലികളെ കൊല്ലാൻ അനുവദിച്ചതിലെ കാരണം ചർച്ച ചെയ്യവെ ഇമാം നവവി പറഞ്ഞു: പല്ലികൾ പല ഉപദ്രവങ്ങളുമുണ്ടാക്കുന്ന ജീവിയാണെന്നതിൽ ഏകാഭിപ്രായമുണ്ട്… അവയെ കൊല്ലുന്നത് പ്രവാചകൻ (സ) പ്രോത്സാഹിപ്പിക്കാൻ കാരണം അവയിലെ ഉപദ്രവങ്ങളാണ്.” (ശർഹു മുസ്ലിം: 14:236)
അവ വെള്ള പാത്രങ്ങളിൽ മനുഷ്യന് ഉപദ്രവകരമായ പലതും നിക്ഷേപിച്ചു കൊണ്ടും ഗുരുതരമായ രോഗങ്ങളും വിഷങ്ങളും പടർത്തിക്കൊണ്ടും ഉപദ്രവങ്ങൾ ഏൽപ്പിക്കുന്നുവെന്ന് ഇമാം ഐനി വ്യക്തമാക്കുന്നു. (ഉംദത്തുൽ കാരി: 15: 250)
അബൂബക്കർ ഇബ്നുൽ അറബി പറഞ്ഞു: മൃഗങ്ങൾ രണ്ടു വിതമുണ്ട്. ഉപദ്രവിക്കുന്നവയും ഉപദ്രവിക്കാത്തവയും. ഉപദ്രവിക്കുന്നവയെ കൊല്ലാം. ഉപദ്രവിക്കാത്തവയെ കൊല്ലരുത്. പല്ലിയെ കൊല്ലാൻ അനുവദിച്ചത് അവ ഉപദ്രവകാരിയാണ് എന്ന അടിസ്ഥാനത്തിലാണ്. (ആരിദത്തുൽ അഹ്വദി: 6:276)
പല്ലികളെ കൊല്ലാനുള്ള ഭൗതീകമായ, മുഖ്യ കാരണം അവയിലെ ഉപദ്രവമാണ് എന്ന് ചുരുക്കം. ഇതാകട്ടെ ശാസ്ത്രത്തിന്റെ വെളിച്ചത്തിൽ ഒരു മണ്ടത്തരമൊ മിഥ്യയൊ ആണെന്ന വ്യാഖ്യാനത്തിനെതിരായാണ് വസ്തുതകൾ സംസാരിക്കുന്നത്.
പല്ലികളുൾപ്പെടെ ഗൗളിവർഗ ജീവികളിൽപ്പെട്ട (Lizard) അയ്യായിരത്തിലധികം വർഗങ്ങൾ ലോകത്തുണ്ട്. ഹദീസിലെ ‘വസഗ് ‘ (الْوَزَغِ ،الوَزَغَة) എന്ന പദം പല്ലി വർഗത്തിൽപ്പെട്ട (Lizard) ആയിരത്തോളം വരുന്ന വിഭാഗങ്ങളെ വിശേഷിപ്പിക്കാവുന്ന പേരാണ്; വീട്ടു പല്ലികളെ സംബന്ധിച്ച് മാത്രമല്ല. (https://mawdoo3.com/%D9%85%D8%A7_%D9%87%D9%88_%D8%AD%D9%8A%D9%88%D8%A7%D9%86_ %D8%A7%D9%84%D9%88%D8%B2%D8%BA) ഗെക്കോ (പല്ലി), പല്ലി വർഗത്തിൽപ്പെട്ട (Lizard) ആയിരത്തിലധികം ഇനം പല്ലികളിൽ ഏതെങ്കിലുമാണെന്ന് എൻസൈക്ലോപീഡിയ ബ്രിട്ടാനിക്ക വ്യക്തമാക്കുന്നു. (https://www.britannica.com/animal/gecko)
സൗദി അറേബ്യയിൽ കുറഞ്ഞത് 100 പല്ലികളും ഗൗളിവർഗ (Lizard) ഇനങ്ങളുണ്ട്, അവയിൽ പലതും പാശ്ചാത്യർക്ക് അജ്ഞാതമാണ്. സൗദി അറേബ്യ, ഉരകങ്ങളുടെ ഒരു അപൂർവ്വ കലവറയാണ്. മറ്റൊരു വാചകത്തിൽ പറഞ്ഞാൽ, പൗരാണിക കാലഘട്ടത്തിൽ, പൂർണമായും നാഗരികമായിട്ടില്ലാത്ത മരുഭൂ പ്രദേശങ്ങളിൽ ജീവിക്കുന്നവരെ സംബന്ധിച്ചിടത്തോളം നൂറു കണക്കിന് പല്ലി വർഗങ്ങൾ ശല്യം കൊണ്ടും ഉപദ്രവങ്ങൾ കൊണ്ടും അവരുടെ ദുസ്വപ്നമായി (Nightmare) അവ മാറിയിട്ടുണ്ടാവണം.
“വടക്കുകിഴക്കൻ ആഫ്രിക്ക മുതൽ തെക്കുപടിഞ്ഞാറൻ ഏഷ്യ വരെയുള്ള അറേബ്യൻ പ്രദേശങ്ങളിൽ ഗെക്കോകളുടേയും മറ്റു പല്ലി വർഗങ്ങളുടേയും സമൃദ്ധി നിലനിന്നിരുന്നു. പ്രോജക്റ്റ് സൈറ്റുകൾ ഇൻഡസ്ട്രിയൽ കോംപ്ലക്സുകൾ എന്നിവയുടെ ഭാഗമായ ശുദ്ധീകരണവും നഗരവികസനവും മൂലമുണ്ടായ പാരിസ്ഥിതിക മാറ്റങ്ങൾ, പല്ലി വർഗങ്ങളുടെ പ്രകൃതിദത്ത ആവാസവ്യവസ്ഥയുടെ ദ്രുതഗതിയിലുള്ള ഇടിവിന് കാരണമായി. എത്രത്തോളമെന്നാൽ തുരൈഫ് പ്രദേശത്തെ പല്ലി വൈവിധ്യത്തിന്റെ -പഠന കാലയളവിലെ- 16 ഇനം പല്ലികളെ രേഖപ്പെടുത്തപ്പെട്ടിരുന്നത്, ഇപ്പോഴത്തെ സർവേയിൽ -ഏറ്റവും സമൃദ്ധമായ കുടുംബം- 9 ഇനങ്ങളുള്ള ലാസെർട്ടിഡേ ആയി ചുരുങ്ങി.” (https://www.ncbi.nlm.nih.gov/pmc/articles/PMC4992096/)
വിവിധ തരം മരുഭൂ പല്ലികൾ, ഓന്തുകൾ, അരണകൾ തുടങ്ങിയ പല്ലിവർഗങ്ങളുടെ (Lizards) ആധിക്യത്തിൽ കേവലം പതിറ്റാണ്ടുകൾ കൊണ്ട് സംഭവിച്ച ഇടിവാണ് ഈ പഠനം സൂചിപ്പിക്കുന്നത്. എങ്കിൽ 14 നൂറ്റാണ്ടുകൾക്ക് മുമ്പ്, നാഗരിക വികസനങ്ങൾ സംഭവിക്കുന്നതിനപ്പുറം പൗരാണിക അറേബ്യയിലെ ജനങ്ങൾ ജീവിച്ചിരുന്ന മരുഭൂവന്യതയിൽ നിന്നു കൊണ്ടാകണം ഉപദ്രവകാരികളായ പല്ലികളെ കൊല്ലാൻ പ്രവാചകൻ (സ) അനുവാദം നൽകിയതിനെ സംബന്ധിച്ച് ചിന്തിക്കാൻ.
“പല്ലിവർഗങ്ങളുമായും പാമ്പുകളുമായുള്ള മനുഷ്യന്റെ ഇടപെടലിന്റെ വർദ്ധനവ് മനുഷ്യ സാൽമൊണെല്ലോസിസ് രോഗത്തിന്റെ വ്യാപനത്തിൽ പ്രാധാന പങ്കു വഹിക്കുന്നതായി പഠനങ്ങൾ സൂചിപ്പിക്കുന്നു, പ്രത്യേകിച്ചും ചെറിയ കുട്ടികളിൽ കണ്ടുവരുന്ന കൂടുതൽ ആക്രമണാത്മക അണുബാധകൾക്ക് കാരണമിതാണ്.” (https://www.ncbi.nlm.nih.gov/pmc/articles/PMC5617995/)
ഊഷര ഭൂമിയിൽ, ഒട്ടകപ്പുറത്ത് ദിവസങ്ങളോളവും മാസങ്ങളോളവും യാത്ര ചെയ്തും, മരച്ചുവട്ടിലും ഓലപ്പുരയിലും കിടന്നുറങ്ങിയുമെല്ലാം ജീവിച്ചിരുന്ന പൗരാണിക അറബിയെ വലച്ചിരുന്ന പ്രശ്നത്തെയാണ് പ്രവാചകൻ (സ) അഭിസംബോധന ചെയ്യുന്നത്. അല്ലാതെ ഇന്ന്, മിനുമിനുത്ത, റബ്ബറൈസ്ഡ് എക്സ്പ്രസ് ഹൈവേകളും, ശീതീകരിച്ച ആഡംഭര കാറുകളും, അംബരചുമ്പികളായ കെട്ടിടങ്ങളുമെല്ലാം ജീവിത ചിത്രങ്ങളായി പരിണമിച്ച പരിഷ്കൃത നഗകവാസികളോട്, കെട്ടിടത്തിന്റെ ഏതോ നിലയിലെ, ഏതോ മുറിയിൽ… ഏതോ മൂലയിൽ ആരുമറിയാതെ പതുങ്ങിയിരിക്കുന്ന പല്ലിയെ തേടിപ്പിടിച്ച് ‘ശിക്ഷിക്കാനു’ള്ള ആഹ്വാനമല്ല അത്. അങ്ങനെ ആ ഹദീസിനെ വ്യഖ്യാനിക്കുമ്പോൾ മാത്രമാണ് തെറ്റിദ്ധാരണകൾ ഉടലെടുക്കുന്നത്.
അറേബ്യൻ ഉപദ്വീപിലെ മരുഭൂമികളിൽ ഏറ്റവും കൂടുതൽ കണ്ടുവരുന്ന ഗൗളിവർഗ ജീവികളിൽ പല്ലികളോട് അടുത്ത ഒന്നാണ്, ഇരുണ്ട പുള്ളികളുള്ള, മണൽ നിറമുള്ള ‘ഡെസേർട്ട് മോണിറ്റർ’. അവ മരുഭൂമിയിലെ അന്തരീക്ഷവുമായി മികച്ച രീതിയിൽ ഇണങ്ങിച്ചേരുന്നു. മരുഭൂമിയിലെ ഏറ്റവും ആക്രമണാത്മക ഉരഗങ്ങളിൽ ഒന്നാണിത്. ഭീഷണി നേരിടുന്നതായി അനുഭവപ്പെട്ടാൽ അവ ‘ശരീരം വായു കൊണ്ട് വീർപ്പിക്കുകയും ഉച്ചത്തിൽ ചീറ്റുകയുകയും’ ചെയ്യും. പ്രതിരോധത്തിനായി വാൽ ഉപയോഗിച്ച് ചാട്ടവാറടി പോലെ വീശിയടിക്കുമെന്നതിന് പുറമെ വേദനാജനകമായ അവയുടെ കടി പലപ്പോഴും രോഗബാധയായി പരിണമിക്കാറുമുണ്ട്. ‘ഡെസേർട്ട് മോണിറ്റർ’ ഒരു സജീവ വേട്ടക്കാരനാണ്, വേട്ടയാടി പിടിക്കാനും കീഴടക്കാനും കഴിയുന്ന എന്തും അവ ഭക്ഷിക്കും; ഇതിൽ പെരുച്ചാഴി, എലി, അണ്ണാൻ, എട്ടുകാലി തുടങ്ങി മറ്റ് ഉരഗങ്ങൾ, ചെറിയ സസ്തനികൾ, പക്ഷികൾ, ചീഞ്ഞുനാറുന്ന ശവം വരെ ഉൾപ്പെടുന്നു. അറേബ്യൻ ഉപദ്വീപിലുടനീളം അതിന്റെ വിഹാര പരിധി വ്യാപിച്ച് കിടക്കുന്നു. (https://www.ddcr.org/FloraFauna/Detail.aspx?Class=Reptiles&Order=Reptiles&Referrer=Monitors& Subclass=Lizard%20Family&Name=Desert%20Monitor&Id=169)
മറ്റു ചില പല്ലികളും അവയുടെ ഉപദ്രവങ്ങളും പഠനങ്ങളിൽ നിന്ന് നമുക്ക് വായിക്കാം:
“ഹവായ് ദ്വീപിലുടനീളം നടത്തപ്പെട്ട ചില പഠനഫലങ്ങൾ സാൽമൊണെല്ല രോഗം ബാധിച്ച പല്ലികൾ ദ്വീപുകളിൽ വ്യാപകമായി വസിച്ചു വരുന്നുവെന്ന് കാണിക്കുന്നു. ഹവായ് ദ്വീപുകളിലെ പല്ലികൾ, പഴയ കെട്ടിടങ്ങളിൽ, ഇരുമ്പ് മേൽക്കൂരയുടെ ആവരണത്തിനും ചുമരിനും ഇടയിലും മതിലുകളിലെ വിള്ളലുകളിലും പ്രധാനമായും താമസിക്കുന്നു. മാത്രമല്ല, ലൈറ്റ് ഫർണിച്ചറുകളേയും ജനലുകളേയും ചുറ്റിപ്പറ്റി അവ ജീവിക്കുന്നു. സർവേയിൽ ഉൾപ്പെടുത്തിയ 13 സൈറ്റുകളിൽ, 76.9 ശതമാനം സാൽമൊണെല്ല ബാധിച്ച 10 പല്ലികളെ കണ്ടെടുക്കപ്പെട്ടു. ഇവിടെയുള്ള 9 വീടുകളിൽ 23ൽ 7 പല്ലികളുടെ കാഷ്ടത്തിൽ (30.4 ശതമാനം) സാൽമൊണെല്ല പോസിറ്റീവ് ആയി കണ്ടെത്തി. ഈ വീടുകളിൽ 63 പല്ലികളിൽ 27 എണ്ണം സാൽമൊണെല്ലക്ക് (42.9 ശതമാനം) പോസിറ്റീവ് ആണ് എന്ന് 1981 ൽ ഹെൽമ് കണ്ടെത്തി. ദ്വീപുകളിൽ പല്ലികളിൽ നിന്ന് മനുഷ്യരിലേക്ക് സാൽമൊണെല്ല പകരുന്നത് ഭക്ഷണവും വെള്ളവും അവയുടെ കാഷ്ടത്തിലൂടെ മലിനീകരിക്കപ്പെടുന്നതിലൂടെയാണ്.”
(Salmonella in Two Gecko Species on the Island of Hawaii: John G. Chan, Charlene Shero, Laura Young, Barney Bareng, Biology Discipline: University of Hawaii at Hilo: Hilo, Hawaii 96720)
മലേഷ്യയിൽ നടത്തിയ ഒരു പഠനത്തിൽ 83.3% വീട്ടിൽ വളർത്താനായി പിടിക്കപ്പെടുന്ന പല്ലികളും (Iguanidae, Agamidae, Scincidae, Gekkonidae, Varanidae) 25% കാട്ടു പല്ലികളും (Agamidae, Scincidae, Gekkonidae) സാൽമൊണെല്ല അണുബാധ വാഹകരാണെന്ന് തെളിയിക്കപ്പെട്ടു.
ജപ്പാനിൽ ഒരു വളർത്തുമൃഗ സ്റ്റോറിൽ നിന്ന് മാത്രം 66% (47/71) പല്ലിവർഗവും (Lizards) 100% (23/23) പാമ്പുകളും സാൽമൊണെല്ലയ്ക്ക് കാരണകാരികളായി കണ്ടെത്തപ്പെട്ടു.
ക്രൊയേഷ്യയിൽ, ഒരു സ്വകാര്യ ഉടമയുടെ അടുക്കലുണ്ടായിരുന്ന 48.4% വീട്ടിൽ വളർത്താനായി പിടിക്കപ്പെട്ട പല്ലിവർഗങ്ങളും, 8.9% പിടിക്കപ്പെട്ട പാമ്പുകളും സാൽമൊണെല്ലയ്ക്ക് പോസിറ്റീവായതായി കണ്ടെത്തി.
പോളണ്ടിലെ മൃഗശാലകളിലും സ്വകാര്യ സൂക്ഷിപ്പുകാരുടെ അടുക്കലുമുണ്ടായിരുന്ന മുപ്പത്തൊമ്പത് ശതമാനം (58/149) പല്ലിവർഗങ്ങളും, 29% (31/106) പാമ്പുകളും സാൽമണെല്ലയ്ക്ക് പോസിറ്റീവ് ആണെന്ന് കണ്ടെത്തി. കാനഡയിൽ, പോസ്റ്റ്മോർട്ടത്തിനായി സമർപ്പിച്ച 51% വളർത്തുമൃഗ പാമ്പുകളും 48% വളർത്തുമൃഗ പല്ലിവർഗങ്ങളും സാൽമൊണെല്ലയ്ക്ക് പോസിറ്റീവ് ആണെന്ന് കണ്ടെത്തി, സാൽമൊണെല്ല പോസിറ്റീവ് ആയ മൃഗങ്ങളിൽ മൂന്നിലൊന്നിന്റെയും മരണത്തിന് കാരണമായത് ‘സാൽമൊനെല്ലോസി’സാണ്. (https://www.ncbi.nlm.nih.gov/pmc/articles/PMC5617995/)
അമേരിക്കയിലെ ഒരു പഠനത്തിൽ കാട്ടിൽ നിന്ന് പിടികൂടി യു.എസ്.എയിലേക്ക് ഇറക്കുമതി ചെയ്ത 80% (88/110) ഇന്തോനേഷ്യൻ ‘ടോക്കെയ് ഗെക്കോസ്’ പല്ലികൾ (ഗെക്കോ ഗെക്കോ) ‘സാൽമൊണെല്ലയ്ക്ക് പോസിറ്റീവാണെന്ന് കണ്ടെത്തി. ഇതിൽ 14 വ്യത്യസ്ത സെറോഗ്രൂപ്പുകളും, 17 പ്രത്യേക സെറോടൈപ്പുകളും ഉൾപ്പെടുന്നു, അവയിൽ പലതും ആൻറിബയോട്ടിക്കുകളെ ചെറുക്കാൻ ശേഷിയുള്ളവയാണ്. (https://pubmed.ncbi.nlm.nih.gov/22607081/)
വിയറ്റ്നാമിലെ മെകോംഗ് ഡെൽറ്റയിൽ നിന്ന് 201 കാട്ടു പല്ലികളെ ശേഖരിച്ചു, അവയുടെ കാഷ്ടത്തിൽ സാൽമൊണെല്ലയുടെ അതിജീവന കാല പരിധിയെ നിർണ്ണയിക്കാൻ നടത്തിയ പഠനത്തിൽ, പരിശോധിച്ച 101 സാമ്പിളുകളിൽ 24 എണ്ണം (23.8%) സാൽമൊണെല്ല പോസിറ്റീവ് ആയിരുന്നു.
വിയറ്റ്നാമിലെ ഊഷ്മാവിൽ, പല്ലി കാഷ്ടത്തിലെ സാൽമൊണെല്ലയ്ക്ക് 6 ആഴ്ച അതിജീവിക്കാൻ കഴിയും. തെക്ക് കിഴക്കൻ ഏഷ്യൻ രാജ്യങ്ങളിലെ മനുഷ്യരിൽ സാൽമൊണെല്ലയുടെ സംഭരണത്തിലും സാൽമൊണെല്ല അണുബാധയുടെ ഉറവിടമായി വർത്തിക്കുന്നതിലും കാട്ടു പല്ലികൾ ഒരു പ്രധാന പങ്ക് വഹിക്കുന്നുവെന്ന് ഫലങ്ങൾ തെളിയിക്കുന്നു. (https://www.jstage.jst.go.jp/article/jvms/80/8/80_18-0233/_article)
‘സാൽമൊണെല്ല ബാക്ടീരിയ പ്രതിവർഷം 19,000 പേരെ ആശുപത്രികളിലേക്കും 380 മരണങ്ങളിലേക്കും നയിച്ചുവെന്ന് അമേരിക്കയിലെ Centers for Disease Control and Prevention റിപ്പോർട്ട് ചെയ്യുന്നു.
പല്ലികളെ വളർത്തുമൃഗങ്ങളായി ഉപയോഗിക്കപ്പെടുന്നത് വ്യാപകമായതോടെ, യുഎസിലെ 16 സംസ്ഥാനങ്ങളിൽ അപകടകരമായ സാൽമൊണെല്ല പൊട്ടിപ്പുറപ്പെടലുകൾ ഉണ്ടായി എന്ന് പുതിയ വാർത്താ റിപ്പോർട്ടുകൾ പുറത്തു വരുന്നു.’ (abcnews.go.com)
പ്രവാചക കാലഘട്ടത്തിൽ മരുഭൂവാസികളായ പൗരാണിക അറബികൾക്ക്, ഈ പല്ലികളിലും ഗൗളിവർഗ ജീവികളിലും (Lizard) ഉപദ്രവകാരികളായ വിഭാഗങ്ങളിൽ നിന്നും നിരന്തരം ഉപദ്രവമേൽക്കുന്നവരായിരുന്നു എന്നത് മുന്നിൽ വെച്ചാണ് ഹദീസിനെ സമീപിക്കേണ്ടത്. ഇത്തരം പല്ലികളെയാണ് കൊല്ലാൻ അനുവാദം നൽകപ്പെട്ടത് എന്നതാണ് പല ഗവേഷകരുടേയും പണ്ഡിതരുടെയും വീക്ഷണം. അറേബ്യ നാഗരീകമായി വികസിച്ചിട്ടില്ലാത്ത അക്കാലഘട്ടത്തിൽ ഇവയുടെ വിഹാര പരിധിയും തോതും ഇന്നത്തേക്കാൾ എത്രയൊ ഇരട്ടി കൂടുതലായിരിക്കും എന്ന് ചിന്തിക്കാവുന്നതെയുള്ളു.
ഈ ഗൗളിവർഗ ജീവികളെ (Lizards) സംബന്ധിച്ചാണ് ഹദീസ് സംസാരിക്കുന്നത് എങ്കിലും -ഉപദ്രവ ഹേതുവാണെങ്കിൽ – വീട്ടു പല്ലി ഉൾപ്പെടെ ഏത് പല്ലിയെയും കൊല്ലലും ഹദീസിലെ അനുവദിക്കപ്പെട്ട വിഭാഗമായി പരിഗണിക്കപ്പെടും എന്ന മറ്റൊരു വീക്ഷണവുമുണ്ട്. രണ്ടിലും തെറ്റൊന്നും കാണുന്നില്ല.
നമ്മെ ചുറ്റിപറ്റി ജീവിക്കുന്ന പല്ലികളിൽ വളരെ സാധാരണയായി കാണപ്പെടുന്ന ഒരു വിഭാഗമാണ് House geckos എന്നറിയപ്പെടുന്ന, വീട്ടു പല്ലികൾ അല്ലെങ്കിൽ ചുമർ പല്ലികൾ. ഹദീസിൽ കൊല്ലാൻ അനുവാദം നൽകപ്പെട്ടത് ഈ പല്ലി വർഗത്തെ സംബന്ധിച്ചു കൂടിയാണ് എന്ന് മനസ്സിലാക്കിയാൽ തന്നെ, മനുഷ്യർക്ക് അറപ്പുളവാക്കുകയും നിരന്തരം ശല്യപ്പെടുത്തുകയും ചെയ്യുന്നവയാണ് അവ എന്നതിലുപരി മനുഷ്യരിൽ ഗുരുതരമായ രോഗങ്ങൾക്ക് കാരണമായ സാൽമൊണെല്ല (Salmonella) എന്ന അണുക്കളുടെ വാഹകർ കൂടിയാണ് അവ എന്നും നാം തിരിച്ചറിയേണ്ടതുണ്ട്. ഉരഗങ്ങളുമായുള്ള സമ്പർക്കം പുലർത്തുന്നതിലൂടെയും സാൽമൊണെല്ല അണുബാധ ഉണ്ടാകാം, അവ സമ്പർക്കം പുലർത്തിയ പാത്രങ്ങൾ, ഭക്ഷണം, വെള്ളം ഉൾപ്പെടെ എന്തിൽ നിന്നും അണുബാധ ഉണ്ടാകാം. (https://www.cdc.gov/healthypets/diseases/salmonella.html) ഉഭയജീവികളുമായോ (ഉദാ. തവളകൾ), ഉരഗങ്ങളുമായോ (ഉദാ. പല്ലികൾ) അല്ലെങ്കിൽ അവയുടെ വിസർജ്ജത്തിൽ നിന്നോ കാഷ്ടത്തിൽ നിന്നോ നേരിട്ടോ അല്ലാതെയോ സമ്പർക്കം പുലർത്തുന്നതിലൂടെ സാൽമൊണെല്ല പടരാം. സാൽമൊണെല്ല ബാക്റ്റീരിയ സാധാരണയായി കുടലിനെയും, ചിലപ്പോഴെല്ലാം രക്തപ്രവാഹത്തെയും ബാധിക്കുന്നു. പല്ലികൾ ഈ ബാക്ടീരിയകളെ കുടൽ, വായ, കാഷ്ടം എന്നിവയിൽ വഹിക്കാറുണ്ട്.
ഈ ബാക്ടീരിയകൾ വയറിളക്കരോഗത്തിനുള്ള ഏറ്റവും സാധാരണമായ കാരണങ്ങളിലൊന്നാണ്. ന്യൂയോർക്ക് സ്റ്റേറ്റിൽ ഓരോ വർഷവും ഇക്കാരണത്താലുണ്ടാകുന്ന ആയിരക്കണക്കിന് കേസുകൾ റിപ്പോർട്ട് ചെയ്യപ്പെടുന്നു. മിക്ക കേസുകളും വേനൽക്കാലത്ത് സംഭവിക്കുന്നു. ചിലരിൽ ജീവനു ഭീഷണിയാവുന്ന അപകടങ്ങളും സൃഷ്ടിച്ചേക്കും. (https://www.health.ny.gov/diseases/communicable/zoonoses/salmonella/amphibian_reptilian_questions_and_answers.htm)
സാൽമൊണെല്ലയെ ഒരു ഭക്ഷ്യ രോഗകാരണമായ അണുവായാണ് കണക്കാക്കപ്പെടുന്നത്. അവ മൂലം മലിനമായ ഭക്ഷണത്തിലൂടെ -ലോകത്ത്- പ്രതിവർഷം 80 ദശലക്ഷം സാൽമൊനെലോസിസ് കേസുകൾ റിപ്പോർട്ട് ചെയ്യപ്പെടുന്നു. യു.എസ്.എയിൽ 6% സ്പോറാഡിക് സാൽമൊനെലോസിസ് കേസുകളും, 21 വയസ്സിന് താഴെയുള്ളവരിൽ 11% കേസുകളും ഉരഗങ്ങളും ഉഭയജീവികളുമായുള്ള സമ്പർക്കം മൂലമാണെന്ന് കണക്കാക്കപ്പെടുന്നു.
നൈജീരിയയിലെ സുക്കയിൽ, വീട്ടു പല്ലികളിലുള്ള സാൽമൊണെല്ല അണുബാധയെ സംബന്ധിച്ച അന്വേഷണ പഠനത്തിൽ തൊന്നൂറിൽ ഇരുപത് പല്ലികളിൽ സാൽമൊണെല്ല സാന്നിദ്ധ്യം കണ്ടെത്തി; 30 ശതമാനം വാഹക നിരക്കിൽ. (https://pubmed.ncbi.nlm.nih.gov/3833829 /)
150 ചുമർ പല്ലികളുടെ (Hemidactylus brookei) കുടലിലെ എയറോബിക് ബാക്ടീരിയ വ്യൂഹത്തെ കുറിച്ചു പഠനം നടത്തപ്പെട്ടപ്പോൾ സാൽമൊണെല്ലയുടെ 35 ഇൻസുലേറ്റുകളും എന്ററോബാക്ടീരിയേസിയിലെ (Enterobacteriaceae) മറ്റ് പല ഇനങ്ങളും ഉൾപ്പെടെ വിവിധതരം ബാക്ടീരിയകൾ കണ്ടെടുത്തു. ഷിഗെല്ല സോനെയി – 2, എഡ്വേർഡീസെല്ല ടാർഡ – 4, എന്റർടോബാക്റ്റർ എസ്പിപി – 8, സിട്രോബാക്റ്റർ ഫ്രോയിഡി – 3, സെറാട്ടിയ മാർസെസെൻസ് – 3, പ്രോട്ടിയസ് എസ്പിപി – 35, ക്ലെബ്സില്ല ന്യൂമോണിയ – 13, എസ്ഷെറിച്ച കോളി – 17, ഇൻസുലേറ്റുകൾ. എട്ട് സാൽമൊണല്ല സെറോടൈപ്പുകൾ എന്നിവ തിരിച്ചറിഞ്ഞു, അവയിൽ പ്രധാനം എസ്. വിറ്റിംഗ്ഫോസ് (S. hvittingfoss), എസ്. ടൈഫിമുറിയം (S.typhimurium) എന്നിവയാണ്. മനുഷ്യ ശരീരത്തിലെ എന്ററോപാഥോജനുകളുടെ വ്യാപനവുമായി ഈ കണ്ടെത്തലുകൾക്കുള്ള ബന്ധം വളരെ പ്രസക്തമാണ്. (https://pubmed.ncbi.nlm.nih.gov/3729372/)
“എല്ലാ ഉരഗങ്ങളിലും ബാക്ടീരിയ, വൈറസ്, പരാന്നഭോജികൾ, പുഴുക്കൾ എന്നിവയുൾപ്പെടെ നിരവധി അണുക്കൾ ഉണ്ട്. ഇവയിൽ പലതും ഉരഗ ഉടമകളുടെ കുടുംബത്തിലേക്ക് പകരാം. ഇവയിൽ ഏറ്റവും പ്രധാനപ്പെട്ടവ ഇനി പറയുന്നവയാണ്:
സാൽമൊണെല്ല: സാൽമൊണെല്ല സാധാരണയായി എല്ലാത്തരം ഉരഗങ്ങളിലും കാണപ്പെടുന്നു. ഉരഗങ്ങളുടെ കാഷ്ടവുമായി സമ്പർക്കത്തിൽ വന്ന എന്തെങ്കിലും വായിൽ വെക്കുമ്പോൾ ഉരഗങ്ങളിൽ നിന്ന് മനുഷ്യരിലേക്ക് ഇത് വ്യാപിച്ചേക്കാം. ഉദാഹരണത്തിന് ഉരഗങ്ങൾ/ ഉരഗജീവികളുമായുള്ള സമ്പർക്കം വഴി മലിനമായ ഫോർമുല കുപ്പികൾ കുടിക്കുന്നതിലൂടെ ശിശുക്കൾക്ക് സാൽമൊണെല്ല ബാധിക്കാം. സാൽമൊണല്ല അണുബാധ വയറിളക്കം, തലവേദന, പനി, വയറു വേദന എന്നിവയ്ക്ക് കാരണമാവുകയും സെപ്റ്റിസീമിയ (രക്തത്തിലെ വിഷബാധ) ഉണ്ടാവുകയും ചെയ്യാം. കഠിനമായ നിർജ്ജലീകരണവും സംഭവിക്കാം. 2008 ൽ 449 സാൽമൊനെലോസിസ് കേസുകൾ ഉണ്ടായിരുന്നു, ഇതിൽ പതിനഞ്ച് കേസുകൾ ഉരഗങ്ങളുമായി അടുത്തിടെ സമ്പർക്കമുണ്ടായ ആളുകളായിരുന്നു. ഈ പതിനഞ്ച് കേസുകളിൽ ഒമ്പത് പേർ ഒരു വയസ്സിന് താഴെയുള്ള കുഞ്ഞുങ്ങൾ ആയിരുന്നു.
ബോട്ടുലിസം: പക്ഷാഘാതത്തിനും മരണത്തിനും കാരണമാകുന്ന ക്ലോസ്ട്രിഡിയം ബാക്ടീരിയം പുറത്തുവിടുന്ന വിഷവസ്തു മൂലമുണ്ടാകുന്ന ഗുരുതരവും ജീവന് ഭീഷണിയുമായ രോഗമാണ് ബോട്ടുലിസം.
ക്യാംപിലോബാക്ടീരിയോസിസ് (മലവിസർജ്ജനം), ലെപ്റ്റോസ്പിറോസിസ് (കരൾ രോഗം), ട്രിച്ചിനെല്ലോസിസ് (നാഡീവ്യവസ്ഥ, ഹൃദയം, ശ്വാസകോശം, പേശികൾ എന്നിവയെ ബാധിക്കുന്ന രോഗം) എന്നിവ ഉരഗങ്ങളെ വളർത്തുന്നതുമായി ബന്ധപ്പെട്ടിരിക്കുന്നു. മിക്കതും ചികിത്സിക്കാവുന്നവയാണെങ്കിലും ചിലത് വളരെ ഗുരുതരമാണ്.” (https://www.hpsc.ie/a-z/zoonotic/reptilesandrisksofinfectiousdiseases/)
57 വീടുകളിൽ നിന്ന്, നൂറ് ‘ഏഷ്യൻ ഹൗസ് ഗെക്കോ’ പല്ലികളെ ശേഖരിച്ച് നടത്തിയ പഠനങ്ങളിൽ സാൽമൊണല്ലയുടെ മൂന്ന് സെറോടൈപ്പുകൾ കണ്ടെത്തി. അതിൽ ഒന്നായ ‘സാൽമൊണെല്ല വിർചോവ്’ (ഫേജ് തരം 8) ആക്രമണാത്മക രോഗവുമായി ബന്ധപ്പെട്ടിരിക്കുന്നതിനാൽ സാൽമൊനെലോസിസ് എന്ന പകർച്ചവ്യാധിയിൽ ഏഷ്യൻ ഹൗസ് ഗെക്കോയ്ക്ക് ഒരു പ്രധാന പങ്കുണ്ടെന്ന് കണ്ടെത്തി. (https://pubmed.ncbi.nlm.nih.gov/20973656/)
നമ്മെ ചുറ്റിപറ്റി ജീവിക്കുന്ന മറ്റൊരു പല്ലി വർഗമായ, iguanas പല്ലികൾ വീടിനകത്ത് എത്തുകയൊ, മുറ്റത്ത് കറങ്ങി നടക്കുകയൊ ചെയ്യാറുണ്ട്. അവയുടെ വേദനയേറിയ ദംശനം, മാംസം കീറുകയും, അവയുടെ പല്ലുകൾ ത്വക്കിൽ തറച്ചിരിക്കുകയും ചെയ്യും.
ഇവയിലൂടെ മനുഷ്യരിലേക്ക് എത്തുന്ന സാൽമൊനെലോസിസ്, അസുഖകരമായ ഇൻഫ്ലുവൻസക്ക് പുറമെ, ജീവനു ഭീഷണിയാവുന്ന അപകടങ്ങളും സൃഷ്ടിച്ചേക്കും. (https://www.crittercontrol.com/wildlife/lizard/lizards-in-the-house)
iguanas പല്ലികൾ പാമ്പുകളേയും അപകടകരമായ വേട്ട ജീവികളേയും വിഷജന്തുക്കളേയും വീട്ടിലേക്കും മുറ്റത്തേക്കും ആകർഷിച്ചേക്കാം. (https://www.crittercontrol.com/wildlife/lizard/lizards-in-the-house)
വേലി പല്ലികൾ, ഗെക്കോകൾ, ഏങ്കിൾസുകൾ എന്നിവ വൃത്തികെട്ട കാഷ്ടങ്ങൾ കൊണ്ട് പരിസരങ്ങളും വസ്ത്രങ്ങളും ഭക്ഷണപദാർത്ഥങ്ങളും മലിനമാക്കുകയും, പൊടുന്നനെയുള്ള ചലനങ്ങളിലൂടെ ആളുകളെ സംഭ്രമത്തിലാക്കുകയും ഭീതിപ്പെടുത്തുകയും ചെയ്യുന്നു.
വലിയ ഇഗുവാന പല്ലികൾക്ക് കടിക്കാനും മാന്താനും വാലുകൾ കൊണ്ട് അടിക്കാനും കഴിയും. അവ പുൽത്തകിടികൾ മാന്തികുഴിക്കുകയും നടപ്പാതകൾ തകർക്കുകയും ഫലങ്ങൾ കഴിക്കുകയും, പൂന്തോട്ടങ്ങളിൽ നാശങ്ങളുണ്ടാക്കുകയും ചെയ്യുന്നു.
പാശ്ചാത്യ വേലി പല്ലികൾ, ഗെക്കോകൾ, തവിട്ട്/പച്ച അനോളുകൾ തുടങ്ങിയ പല്ലി വർഗങ്ങൾ ജനലുകൾ, വാതിലുകൾ, തറകൾ എന്നിവയ്ക്ക് ചുറ്റുമുള്ള വിള്ളലുകളിലൂടെ സഞ്ചരിക്കുന്നതിൽ വിദഗ്ധരാണ്. (https://www.crittercontrol.com/wildlife/lizard/lizards-in-the-house)
ഇവക്കു പുറമെ ദോഷകാരികളും അപകടകാരികളുമായ അനേകം ഇനം പല്ലികൾ വേറെയുമുണ്ട്. ഉദാഹരണത്തിന്, ടോക്കെ ഗെക്കോ (Tokay gecko) പല്ലി വർഗം കടിക്കുന്നവയാണ്. ടോക്കെ പല്ലികൾ വളരെ ആക്രമണാത്മക സ്വഭാവമുള്ള പല്ലിയാണിത്. ഭീഷണിപ്പെടുത്തുമ്പോഴോ, ഭയപ്പെടുമ്പോഴോ മാത്രമല്ല അവ കോപിക്കുമ്പോഴും സമ്മർദ്ദം അനുഭവിക്കുമ്പോഴും കടിക്കും. പൂർണ്ണമായി വളർന്ന ടോക്കെയ് പല്ലിക്ക് ശക്തമായ താടിയെല്ലുണ്ട്, ഇത് ചർമ്മത്തിൽ മുറുകെ പിടിക്കുന്നു. അവയെ വെള്ളത്തിൽ മുക്കിയാലല്ലാതെ അവ കടി വിടുകയില്ല. ചർമ്മത്തിൽ നിന്ന് അവയെ വലിച്ചെടുക്കാൻ ശ്രമിക്കുന്നതോടെ അവ കടി മുറുക്കുകയെ ഉള്ളു. ടോക്കെ പല്ലികൾ രോഗകാരികളായ സൂക്ഷ്മ ജീവികളുടേയും വൈവിധ്യമാർന്ന അണുക്കളുടേയും വാഹകരാവാം. ഇവയിൽ ഭൂരിഭാഗവും മനുഷ്യർക്ക് അപകടകരമല്ല എങ്കിലും അവയിൽ ചിലത് ദോഷകരമായ ബാക്ടീരിയകളാണ്. കൂടാതെ ടോക്കെയുടെ തുളച്ചുകയറുന്ന ഒരു കടിയിലൂടെ ദോഷകരമായ പ്രോട്ടോസോവകളും കടന്നുപോയേക്കാം. ഇത്തരം അണുബാധകൾ കുട്ടികളെ എളുപ്പത്തിൽ അപകടത്തിലേക്ക് എത്തിച്ചേക്കും. (https://tokaygeckoguide.com/why-you-dont-want-to-get-bitten-by-a-tokay-gecko/1603/)
ഇബ്റാഹിം നബിയെ ശത്രുക്കൾ അഗ്നിയിലേക്ക് എറിഞ്ഞപ്പോൾ പല്ലികൾ തീ ഊതി ആളി കത്തിക്കാൻ ശ്രമിച്ചതിനാലാണ് തലമുറകൾക്കിപ്പുറവും പല്ലികളെ മുസ്ലിംകൾ കൊന്നു കൊണ്ടിരിക്കുന്നത് എന്ന വിമർശനത്തിന് യാതൊരു യാഥാർത്ഥ്യവുമില്ല. ഒരാൾ ചെയ്ത തെറ്റിന് അയാളുടെ സന്ധതികളിൽ കുറ്റമാരോപിക്കുന്ന മൗഢ്യതയെ നിശിതമായി വിമർശിച്ച മതമാണ് ഇസ്ലാം.
മുഹമ്മദ് നബി (സ) പറഞ്ഞു:
لا تَجْني نفسٌ على الأخرى
“ഒരാളുടെ കുറ്റം മറ്റൊരാളുടെ മേൽ ചുമത്തപ്പെടില്ല.” (സുനനു നസാഈ: 4833, ത്വബ്റാനി: 1384, മഅ്രിഫത്തു സ്വഹാബ: അബൂ നുഐം: 1391)
لا تَجْني أمٌّ على ولَدٍ
“മാതാവിന്റെ കുറ്റം സന്താനത്തിനു മേൽ ചുമത്തപ്പെടില്ല.” (സുനനു നസാഈ: 2/ 251, സുനനു ഇബ്നുമാജ: 2/ 147, സുനനു ഇബ്നു ഹിബ്ബാൻ: 1683)
ഇബ്റാഹിം നബിയുമായി ബന്ധപ്പെടുത്തി കൊണ്ട് ബുഖാരിയിൽ വന്ന ഹദീസ് ഇപ്രകാരമാണ്: “പല്ലികളെ കൊല്ലാൻ (അനുവാദം നൽകി കൊണ്ട്) പ്രവാചകൻ (സ) കൽപ്പന പുറപ്പെടുവിച്ചു. അദ്ദേഹം (സ) പറഞ്ഞു: അത് ഇബ്റാഹിം നബിയുടെ (അ) മേൽ തീ ഊതാൻ ശ്രമിച്ചിരുന്നു.” (സ്വഹീഹുൽ ബുഖാരി: 2628)
ഈ ഹദീസുമായി ബന്ധപ്പെട്ട വളരെ പ്രധാനപ്പെട്ട ചില കാര്യങ്ങൾ മനസ്സിലാക്കുന്നതോടെ ഹദീസ് സംബന്ധമായ തെറ്റിദ്ധാരണകൾ ഇല്ലാതാവുന്നതാണ്:
* പല്ലികളെ കൊല്ലാൻ അനുവദിച്ചതിലെ മുഖ്യ കാരണം അവയിലെ ഉപദ്രവങ്ങൾ തന്നെയാണ്. ആ മുഖ്യ കാരണം ധാരാളം ഹദീസുകളിലൂടെ പ്രവാചകൻ (സ) പഠിപ്പിച്ചു കഴിഞ്ഞു. അതിനു പുറമെ ഒരു അധിക കാരണം കൂടി പങ്കു വെക്കുകയാണ് ഈ ഹദീസിലൂടെ അദ്ദേഹം ചെയ്യുന്നത്.
* വീട്ടിലെ പല്ലികളെയല്ല ഹദീസിൽ തീയിലേക്ക് ഊതാൻ ശ്രമിച്ചു എന്ന് പറഞ്ഞത്. കാരണം, ഇബ്റാഹിം നബിയെ(അ) തീക്കുണ്ടാരമുണ്ടായി എറിഞ്ഞത് വീട്ടിനുള്ളിൽ അല്ലല്ലൊ. മരുഭൂമിലെ വിശാല മൈതാനത്താണ്.
* പല്ലികൾക്കും മറ്റു ഉരഗങ്ങൾക്കും – മനുഷ്യരെ പോലെ തന്നെ – ശ്വാസകോശമുണ്ട്. മനുഷ്യന്റെ ശ്വാസകോശം പോലെ തന്നെ അവയുടെ ശ്വാസകോശം വാരിയെല്ലുകൾക്കും വയറിലെ പേശികൾക്കുമിടയിലായാണ് സ്ഥിതി ചെയ്യുന്നത്. അവയും ഓക്സിജൻ ഉള്ളിലേക്ക് എടുക്കുകയും കാർബൺ ഡൈ ഓക്സൈഡ് പുറത്തുവിടുകയുമാണ് ചെയ്യുന്നത്. അതുകൊണ്ട് തന്നെ ഊതുക എന്നത് അവക്ക് ശാരീരികമായി അസാധ്യമായ ഒരു കാര്യമൊന്നുമല്ല.
‘ആക്രമണാത്മക ഉരഗങ്ങളിൽ ഒന്നായ ‘ഡെസേർട്ട് മോണിറ്റർ’ ഭീഷണി നേരിടുമ്പോൾ ‘ശരീരം, വായു കൊണ്ട് വീർപ്പിക്കുകയും ഉച്ചത്തിൽ ഊതുകയുകയും’ ചെയ്യും. മെഡിറ്ററേനിയൻ വീട്ടു പല്ലികൾ വഴക്കിനിടയിലും ഇണകളെ ആകർഷിക്കാനും ‘ചിലക്കുക’ പതിവാണ്. മധ്യ, തെക്കേ അമേരിക്കയിലെ ടേണിപ്പ്-ടെയിൽഡ് പല്ലികൾ തങ്ങളുടെ വിഹാര പരിധി അടയാളപ്പെടുത്തുന്നതിനായി പ്രാണികളെ അനുകരിക്കുന്ന ശബ്ദത്തിൽ ശബ്ദം പുറപ്പെടുവിക്കുന്നു. 14 ഇഞ്ച് (36 സെന്റീമീറ്റർ) ഉയരമുള്ള ഗെക്കോയായ ന്യൂ കാലിഡോണിയൻ പല്ലി ഉച്ചത്തിൽ അലറുന്നത് കാരണം, അത് “മരങ്ങളിലെ രാക്ഷസൻ” എന്ന പ്രാദേശിക വിളിപ്പേര് നേടി. ഏഷ്യയിൽ കാണപ്പെടുന്ന പുരുഷ ടോക്കെയ് പല്ലികൾ, ഇണചേരാൻ സൂചിപ്പിച്ചു കൊണ്ട് “ടോകേ-ടോക്കേ!” എന്ന് അത്യുച്ചത്തിൽ ശബ്ദിക്കുന്നു. ‘വായു ശ്വാസകോശങ്ങളിൽ നിന്ന് ഗ്ലോട്ടിസിലൂടെ പുറന്തള്ളുന്നതിലൂടെ’യാണ് പല്ലികൾ ഈ ശബ്ദങ്ങൾ പുറപ്പെടുവിക്കുന്നത്.’ (https://www.nationalgeographic.com/animals/article/151024-animal-behavior-lizards-reptiles-geckos-science-anatomy )
ഗൗളിവർഗത്തിന് വായു ഊതാനും വായു പുറം തള്ളി ചീറ്റാനും ചീറാനും അലറാനുമൊക്കെ കഴിയുമെന്ന് ചുരുക്കം.
* പല്ലികൾ തീ ഊതുന്ന ഡ്രാഗണാണോ എന്ന് പരിഹസിക്കുന്നവരുണ്ട്. പല്ലികൾ തീ ഊതി എന്ന് ഹദീസിൽ വന്നിട്ടില്ല എന്നതാണ് അവർക്കുള്ള മറുപടി. പല്ലി തീയിലേക്ക് ഊതിയാൽ എന്ത് സംഭവിക്കാനാണ്? അത് എങ്ങനെ ആളിക്കത്താനാണ്? എന്ന് ചോദിക്കുന്നവരോട് പറയാനുള്ളത് രണ്ട് കാര്യങ്ങളാണ്. ഒന്ന്, ശക്തമായി ഊതാൻ കഴിയുന്ന പല്ലികളുടെ ചില സവിശേഷതകളെ സംബന്ധിച്ച് നാം വിവരിച്ചു കഴിഞ്ഞു. രണ്ട്, പല്ലിയുടെ ഊത്തിന് വല്ല സ്വാധീനവും ആ തീയിൽ വരുത്താൻ കഴിഞ്ഞുവെന്ന് ഹദീസിൽ ഇല്ല. മറിച്ച്, ഹദീസ് സംസാരിക്കുന്നത് തീ ഊതി ആളിക്കത്തിക്കാൻ ആ ജീവികൾ ആശിക്കുകയും ശ്രമിക്കുകയും ചെയ്തുവെന്ന മനസ്ഥിതിയുടെ ജീർണതയെ മാത്രമാണ്.
ശാഹ് വലിയുല്ലാഹ് ദഹ്ലവി പറഞ്ഞു: “‘അവയുടെ ഊത്ത് തീയിൽ യാതൊരു സ്വാധീനവും സൃഷ്ടിക്കില്ലാ എന്നിരുന്നിട്ടും’ പിശാചിന്റെ പ്രേരണയോട് പ്രകൃത്യാ ഉള്ളതായ അവയുടെ അടുപ്പം കാരണം അവ ഇബ്റാഹിമിന്റെ തീ ഊതാൻ ശ്രമിച്ചു. പല്ലികളുടെ ഈ പ്രകൃതത്തിലെ പൈശാചികത പ്രവാചകൻ (സ) ദിവ്യബോധത്തിലൂടെ അറിഞ്ഞു. അതിനെ പറ്റി ഉണർത്തുകയും ചെയ്തു.” (ഹുജ്ജത്തുല്ലാഹിൽ ബാലിഗ: 2:282)
* പല്ലികളെ കൊല്ലാൻ അനുവദിച്ചതിലെ ‘ഭൗതീകമായ’ മുഖ്യ കാരണം പങ്കു വെച്ചതിന് ശേഷം ‘അഭൗതീകമായ’ ഒരു അറിവു കൂടി അനുബന്ധമായി പഠിപ്പിക്കുക മാത്രമാണ് ഈ ഹദീസിലൂടെ അദ്ദേഹം ചെയ്യുന്നത്. മറ്റു ജീവജാലങ്ങളിൽ നിന്ന് വ്യത്യസ്ഥമായി പല്ലികളുടെ പ്രകൃതത്തിലുള്ള മാനസികവും സ്വഭാവപരവുമായ വ്യതിയാനവും നീചതയുമാണ് ആ ‘അഭൗതീക’ജ്ഞാനം. ആ ജ്ഞാനം പല്ലികളെ സൃഷ്ടിച്ച, പല്ലികളുടെ ജൈവ പ്രകൃതിയും മനോ വിഹാരങ്ങളും രഹസ്യങ്ങളുമെല്ലാം അറിയുന്ന സ്രഷ്ടാവിന് മാത്രം ലഭ്യമാകുന്ന അറിവാണ്.
“ആകാശങ്ങളിലും ഭൂമിയിലുമുള്ളവരും, ചിറക് നിവര്ത്തിപ്പിടിച്ചു കൊണ്ട് പക്ഷികളും അല്ലാഹുവിന്റെ മഹത്വം പ്രകീര്ത്തിച്ചു കൊണ്ടിരിക്കുന്നു എന്ന് നീ കണ്ടില്ലേ? ഓരോരുത്തര്ക്കും തന്റെ പ്രാര്ത്ഥനയും കീര്ത്തനവും എങ്ങനെയെന്ന് അറിവുണ്ട്. അവര് പ്രവര്ത്തിക്കുന്നതിനെപ്പറ്റി അല്ലാഹു അറിയുന്നവനത്രെ.” (കുർആൻ: 24:41)
“കണ്ണുകളുടെ കള്ളനോട്ടവും, ഹൃദയങ്ങള് മറച്ച് വെക്കുന്നതും അവന് (അല്ലാഹു) അറിയുന്നു.” (കുർആൻ: 40:19)
“അവന്റെ പക്കലാകുന്നു അദൃശ്യകാര്യത്തിന്റെ ഖജനാവുകള്. അവനല്ലാതെ അവ അറിയുകയില്ല. കരയിലും കടലിലുമുള്ളത് അവന് അറിയുന്നു. അവനറിയാതെ ഒരു ഇല പോലും വീഴുന്നില്ല. ഭൂമിയിലെ ഇരുട്ടുകള്ക്കുള്ളിലിരിക്കുന്ന ഒരു ധാന്യമണിയാകട്ടെ, പച്ചയോ, ഉണങ്ങിയതോ ആയ ഏതൊരു വസ്തുവാകട്ടെ, വ്യക്തമായ ഒരു രേഖയില് എഴുതപ്പെട്ടതായിട്ടല്ലാതെ ഉണ്ടാവില്ല.” (കുർആൻ: 6:59)
പല്ലികളുടെ ജൈവ പ്രകൃതിയെയും മനോ വിഹാരങ്ങളെയും സംബന്ധിച്ച അവയുടെ സ്രഷ്ടാവിന്റെ ഈ ‘അഭൗതീക’ജ്ഞാനം യാതാർഥ്യമല്ലെന്ന് വിമർശകർക്ക് എന്തിന്റെ അടിസ്ഥാനത്തിലാണ് വാദിക്കാനാവുക?! ആ ആദൃശ്യവും അഭൗതീകവുമായ ഒരു വിവരം തെറ്റാണെന്ന് ഭൗതീകമായ ഏത് ശാസ്ത്രം കൊണ്ടാണ് വിമർശകർ തെളിയിക്കുക ?!!
* ഏതൊ ഒരു പല്ലി, ഇബ്റാഹിം നബിയെ(അ) ശത്രുക്കൾ തീക്കുണ്ടാരത്തിൽ എറിഞ്ഞപ്പോൾ തീ ആളിക്കത്തിക്കാൻ ശ്രമിച്ചു അതു കാരണം എല്ലാ തലമുറയിലുള്ള പല്ലികളെയും കൊല്ലണം എന്നല്ല ഹദീസിൽ ഉള്ളത്. ഒരു പല്ലിയെ പറ്റിയുള്ള നിരൂപണമല്ല ഈ ഹദീസ്. പല്ലി വർഗത്തെ സംബന്ധിച്ചാണ് ഹദീസ്. ഒരാളെ പോലും കൊന്നിട്ടില്ലാത്ത ഒരു നിരപരാധിയായ മനുഷ്യനെ ആദർശത്തിന്റെ പേരിൽ മാത്രം തീക്കുണ്ടാരത്തിലേക്ക് എറിഞ്ഞപ്പോൾ ജീവജാലങ്ങളിൽ പലതും ആ തീ കെടാൻ വേണ്ടി ആശിച്ചു. എന്നാൽ പല്ലി വർഗം (ഒരു പല്ലിയല്ല) അത് ആളിക്കത്തിക്കാൻ ആശിക്കുകയും അതിന് വേണ്ടി പരിശ്രമിക്കുകയും ചെയ്തു എന്ന ഒരു സംഭവത്തെ ആ ജീവവർഗത്തിന്റെ പ്രകൃതിപരവും മാനസികവുമായ നീചതക്ക് തെളിവായി അവയെ പടച്ച സ്രഷ്ടാവ് പഠിപ്പിച്ചു. ഇബ്റാഹിം നബിയുമായി ബന്ധപ്പെട്ട ഈ സംഭവം, പല്ലികളുടെ പ്രകൃതവുമായി ബന്ധപ്പെട്ട് അവയുടെ സ്രഷ്ടാവായ അല്ലാഹു അറിയുന്ന ഒരു വസ്തുതക്കുള്ള ഒരു ഉദാഹരണം മാത്രമാണ്. അല്ലാതെ മൂല കാരണമല്ല.
അപ്പോൾ പിന്നെ ഈ നീച വർഗത്തെ എന്തിന് സൃഷ്ടിച്ചു എന്നതാണ് മറ്റൊരു ചോദ്യം. പിശാചിനെ എന്തിന് സൃഷ്ടിച്ചു ? എന്ന് ചോദിക്കും പോലെ നിരർത്ഥകമാണ് ഈ ചോദ്യം. അല്ലാഹുവിന്റെ സൃഷ്ടികളിൽ നീച സൃഷ്ടികൾക്കും സ്ഥാനവും പ്രസക്തിയുമുണ്ട്. കൃത്യമായ യുക്തിയും തേട്ടവുമുണ്ട്. ഉദാഹരണത്തിന്, പല്ലിയെ കൊണ്ടുള്ള ഭൗതീകമായ ചില ഉപകാരങ്ങളും മാറ്റി വെച്ചാൽ തന്നെ, പല്ലിയുടെ ഈ പ്രകൃതത്തെ സംബന്ധിച്ച ഹദീസ് ആരെല്ലാം വിശ്വസിച്ച് സത്യവിശ്വാസിയാവും ആരെല്ലാം പരിഹസിച്ച് തള്ളി സത്യനിഷേധിയാവും എന്ന പരീക്ഷണം തന്നെ പല്ലിയുടെ സൃഷ്ടിപ്പിനു പിന്നിലെ യുക്തികളിൽ ഒന്നാണ്.
“തീര്ച്ചയായും അതിനെ നാം അക്രമകാരികള്ക്ക് ഒരു പരീക്ഷണമാക്കിയിരിക്കുന്നു.” (കുർആൻ: 37:63)
മുല്ലാ അലിയുൽകാരി പറഞ്ഞു: “പല്ലി ഉപദ്രവങ്ങൾ ചെയ്യുന്ന ഒരു ചെറു ജീവിയാണ്… പല്ലി ഇബ്റാഹിമിന്റെ മേൽ തീ ഊതാൻ ശ്രമിച്ചിരുന്നു എന്ന് പറഞ്ഞതിന്റെ ഉദ്ദേശം ഇബ്റാഹീമിന്റെ ശരീരത്തിന് താഴെ ശത്രുക്കൾ കത്തിച്ച തീയിൽ അത് ഊതി എന്നാണ്.
കാദി പറഞ്ഞു: ഇത് പ്രവാചകൻ (സ) പറയാൻ കാരണം പല്ലി വർഗത്തിന്റെ (സ്വഭാവപരമായ) നികൃഷ്ടത കൂടി വ്യക്തമാക്കാനാണ്. ഇബ്റാഹിം നബിയെ (അ) ശത്രുക്കൾ തീക്കുണ്ടാരത്തിൽ എറിഞ്ഞ സമയത്ത് പിശാച് പല്ലികളുടെ (പ്രകൃതത്തിലെ) നീചത കാരണം, ആ തീ ആളിക്കത്തിക്കാൻ (പലതിനേയും ഉപയോഗപ്പെടുത്തുന്ന കൂട്ടത്തിൽ) അവയെയും ഉപയോഗപ്പെടുത്താൻ ശ്രമിച്ചു. (ഈ മാനസികമായ നീച പ്രകൃതിക്ക് പുറമെ) അവ ശാരീരികമായും ഉപദ്രവകാരികളാണ്.
ഇബ്നുൽ മലക്ക് പറഞ്ഞു: അവയുടെ ഉപദ്രവത്തിൽ പെട്ടതാണ് അവ ഭക്ഷണങ്ങൾ കേടു വരുത്തുകയും പലയിടത്തും കാഷ്ടിച്ചിട്ട് വൃത്തികേടാക്കുകയും ചെയ്യുക എന്നത്. പ്രകൃത്യാ അവ ഉപദ്രവകാരികളാണ് എന്നർത്ഥം.” (മിർക്കാത്തുൽ മഫാത്തീഫ്: 7:2671)
ശൈഖ് മുനജ്ജിദ് പറഞ്ഞു: “ഇബ്റാഹിം നബിയെ(അ) ശത്രുക്കൾ തീക്കുണ്ടാരത്തിൽ എറിഞ്ഞ സമയത്ത് പിശാച് പല്ലി തീയിൽ ഊതാൻ ശ്രമിച്ചു എന്ന് പ്രവാചകൻ (സ) പറയാൻ കാരണം പല്ലി വർഗത്തിന്റെ (ആത്മീയമായവും മാനസികവുമായ) നീചതയെയും നികൃഷ്ടതയെയും അറിയിക്കാൻ വേണ്ടിയാണ്. എന്നാൽ അവയെ കൊല്ലാൻ അനുവാദം നൽകിയത് അക്കാരണത്താൽ മാത്രമല്ല. (അവയിലെ ഉപദ്രവങ്ങൾ കാരണമാണ്.)” (ഇസ്ലാം: സുആൽ വൽജവാബ്: 289055)
ഇനി, പല്ലികളെ കൊല്ലുന്നതിന്റെ മികവിനനുസരിച്ച് ഇനാം പ്രഖ്യാപിക്കുക വഴി ആ ജീവിയോട് ക്രൂരതയല്ലെ ചെയ്യുന്നത് എന്നാണ് മറ്റൊരു വിമർശനം. പല്ലിയെ അടിക്കുന്നതിന്റെ പ്രതിഫലത്തെ സംബന്ധിച്ച ഹദീസ് ഇപ്രകാരമാണ്:
“പല്ലിയെ ആരെങ്കിലും ഒരു അടിക്ക് കൊന്നാൽ അവന് നൂറ് നന്മ രേഖപ്പെടുത്തപ്പെടും. രണ്ടാമത്തെ അടിയിൽ കൊല്ലുന്നവന് (ആദ്യത്തെ അടിയിൽ തന്നെ കൊല്ലുന്നവനേക്കാൾ) താഴെ പ്രതിഫലമാണ് ലഭിക്കുക. മൂന്നാമത്തെ അടിയിൽ കൊല്ലുന്നവന് (രണ്ടാമത്തെ അടിയിൽ തന്നെ കൊല്ലുന്നവനേക്കാൾ) താഴെ പ്രതിഫലമാണ് ലഭിക്കുക.” (സ്വഹീഹു മുസ്ലിം: 3359)
അടി മത്സരത്തിനുള്ള ആഹ്വാനമല്ല. യഥാർത്ഥത്തിൽ, ജീവജാലങ്ങളോടുള്ള പ്രവാചകന്റെ(സ) കാരുണ്യത്തിനുള്ള ഏറ്റവും നല്ല ഉദാഹരണമാണ് ഈ ഹദീസ്. ക്രൂരതയായിരുന്നു ഈ വാചകത്തിന്റെ ഉൾപ്രേരണയെങ്കിൽ ആ ജീവിയെ ഇഞ്ചിഞ്ചായി കൊല്ലാനാണ് അഹ്വാനം നൽകപ്പെടുമായിരുന്നത്. അതിന് പകരം അവയെ കൊല്ലുകയാണെങ്കിൽ ഒറ്റ അടിക്ക് കൊന്ന് വേദനയിൽ നിന്ന് പൊടുന്നനെ ആശ്വാസം നൽകാനും അതിനാണ് കൂടുതൽ പ്രതിഫലമെന്നുമാണ് ഹദീസ്. രണ്ടാമതൊരടി ആവശ്യമുണ്ടെങ്കിൽ അതിൽ കൊന്നിരിക്കണം എന്നതിനാലും വീണ്ടും ആ ജീവിയെ വേദനയിൽ തളച്ചിടരുത് എന്നതിനാലും മൂന്നാമത്തെ അടിയേക്കാൾ പ്രതിഫലം രണ്ടാമത്തെ അടിക്ക് നിശ്ചയിച്ചു. അടിയുടെ എണ്ണം കൂടും തോറും പ്രതിഫലം കുറയുമ്പോൾ ഏറ്റവും കുറഞ്ഞ എണ്ണത്തിൽ അതിനെ കൊല്ലാൻ ആളുകൾ ശ്രദ്ധിക്കുകയാണ് സംഭവിക്കുക.
പല്ലികൾ ഉപദ്രവകാരികളാണ് എന്ന മുഖ്യ കാരണത്തിന് പുറമെ അവ പ്രകൃത്യാ നീച ചിന്തയുള്ളവയാണ് എന്ന അധിക കാരണവും ഉണ്ടായിട്ടും അവയെ കൊല്ലേണ്ടി വന്നാൽ, ഇഞ്ചിഞ്ചായി ക്രൂരമായി കൊല്ലരുത് എന്ന് നിഷ്കർഷിച്ചത് കാരുണ്യമല്ലെ.
പ്രവാചകൻ (സ) പറഞ്ഞു: “അല്ലാഹു സുകൃതവാനാണ്. നന്മയെ അവൻ ഇഷ്ടപ്പെടുന്നു. അതിനാൽ നിങ്ങൾ വിധിക്കുകയാണെങ്കിൽ നീതിയോടെ വിധിക്കുക. നിങ്ങൾ കൊല്ലുകയാണെങ്കിൽ (പോലും) അതിലും (കാരുണ്യമായ) നന്മ കാണിക്കണം. (മുഅ്ജമുൽ അവ്സത്ത്: ത്വബ്റാനി: 5735)
“അല്ലാഹു സുകൃതവാനാണ്. നന്മയെ അവൻ ഇഷ്ടപ്പെടുന്നു. അതിനാൽ നിങ്ങൾ വിധിക്കുകയാണെങ്കിൽ നീതിയോടെ വിധിക്കുക. നിങ്ങൾ കൊല്ലുകയാണെങ്കിൽ (പോലും) ഉരുവിനോട് നന്മ (കരുണ) പ്രവർത്തിക്കുക. (മൃഗത്തെ) അറുക്കുകയാണെങ്കിൽ നല്ല രീതിയിൽ അറുക്കുക. ആയുധത്തിന് മൂർച്ച കൂട്ടി ഉരുവിന് (വേദനയിൽ നിന്ന്) പെട്ടെന്ന് ആശ്വാസം നൽകുക.” (ത്വബ്റാനി: 7121, സ്വഹീഹുൽ ജാമിഅ്: 1824)
വല്ല ജീവികളും മനുഷ്യരെ ഉപദ്രവിക്കുകയും അപായപ്പെടുത്തുകയും അവയെ കൊല്ലൽ ആവശ്യമായി വരികയും ചെയ്താൽ ആ കൊലയിൽ പോലും കരുണയുണ്ടാകണം എന്നാണ് പ്രവാചകൻ (സ) പഠിപ്പിച്ചത്.
മനുഷ്യരെ ഉപദ്രവിക്കാത്ത മൃഗങ്ങളെയും ജീവികളേയും തിരിച്ച് ഉപദ്രവിക്കാനൊ കൊല്ലാനൊ പ്രവാചകൻ (സ) അനുവാദം നൽകിയിട്ടില്ല. എന്നു മാത്രമല്ല ഒരു ജീവി ഉപദ്രവിച്ചു എന്നതിന്റെ പേരിൽ ആ വർഗത്തിൽപ്പെട്ട ഉപദ്രവിക്കാത്ത മറ്റു അംഗങ്ങളെ കൊല്ലുന്നതു പോലും അല്ലാഹു വിലക്കി. ഫവാസിക്കുകളിൽ എല്ലാ നായകളെയും പ്രവാചകൻ (സ) ഉൾപ്പെടുത്തിയില്ല, “കടിക്കുന്ന നായ” യെയാണ് കൊല്ലാൻ അനുവധിച്ചത് എന്ന് ശ്രദ്ധിക്കുക.
ഒരിക്കൽ ഒരു പ്രവാചകനെ ഉറുമ്പ് കടിച്ചു. അപ്പോൾ അദ്ദേഹം ഉറുമ്പും കൂട്ടിലെ മുഴുവൻ ഉറുമ്പുകളേയും കൊന്നു. അതിന്റെ പേരിൽ അല്ലാഹു ആ പ്രവാചകനെ ചോദ്യം ചെയ്യുകയുണ്ടായി.
أنْ قَرَصَتْكَ نَمْلَةٌ أحْرَقْتَ أُمَّةً مِنَ الأُمَمِ تُسَبِّحُ!
“ഒരു ഉറുമ്പ് കടിച്ചു എന്നതിന്റെ പേരിൽ അല്ലാഹുവെ സ്തുതിക്കുന്ന ഒരു സമൂഹത്തെ തന്നെ നീ ചുട്ട് ചാമ്പലാക്കിയൊ !” (സ്വഹീഹുൽ ബുഖാരി: 3019)
മൃഗങ്ങളോടുള്ള കാരുണ്യം പ്രവാചകനോളം ഊന്നിപ്പറഞ്ഞ മറ്റൊരു മത വ്യക്തിത്വങ്ങൾ വിരളമാണ്. ചില ഉദാഹരണങ്ങൾ കാണുക:
അബ്ദുല്ലാഹിബ്നു അബ്ബാസ് (റ) നിവേദനം: അല്ലാഹുവിന്റെ ദൂതൽ ഒരു വ്യക്തിയുടെ അടുത്തു കൂടെ കടന്നുപോയി; അയാൾ തന്റെ കാൽ ഒരു ആടിന്റെ പുറത്തു വെച്ച് കത്തി മൂർച്ച കൂട്ടുകയാണ്. അടാകട്ടെ അയാളിലേക്ക് തുറിച്ച് നോക്കി കൊണ്ടിരിക്കുകയുമാണ്. പ്രവാചകൻ (സ) പറഞ്ഞു: ഇതിന് മുമ്പ് (കത്തിക്ക് മൂർച്ച കൂട്ടുക എന്ന പണി) ചെയ്യാമായിരുന്നില്ലേ ? (ഉരുവിന്റെ മുമ്പിൽ വെച്ചു തന്നെ അത് ചെയ്യണമായിരുന്നോ ?) അതിന് രണ്ട് വട്ടം കൊല്ലാനാണോ നീ ഉദ്ദേശിക്കുന്നത് ?! (മുസ്തദ്റക് ഹാകിം: 7570)
ഭക്ഷിക്കുവാനായി അറുക്കുക എന്ന ന്യായമായ കാരണത്താലാണെങ്കിലും അവയെ അതിയായി ഭയപ്പെടുത്തുന്നത് ക്രൂരതയാണെന്ന് പ്രവാചകൻ (സ) പ്രഖ്യാപിച്ചു.
മൃഗങ്ങൾക്കും അവയുടെ പ്രകൃതത്തിന് യോജിച്ച അവകാശങ്ങളുണ്ടെന്ന് അദ്ദേഹം നൂറ്റാണ്ടുകൾക്ക് മുമ്പ് പഠിപ്പിച്ചു :
അബ്ദുല്ലാഹിബ്നു ജഅ്ഫർ (റ) പറഞ്ഞു: അൻസ്വാരികളിൽ പെട്ട ഒരാളുടെ തോട്ടത്തിൽ പ്രവാചകൻ (സ) പ്രവേശിച്ചു. അപ്പോൾ അവിടെയതാ ഒരു ഒട്ടകം; അല്ലാഹുവിന്റെ ദൂതനെ (സ) കണ്ടതും അത് തേങ്ങി, അതിന്റെ കണ്ണിൽ നിന്നും കണ്ണുനീർ ഒഴുകി. അപ്പോൾ പ്രവാചകൻ (സ) അതിനടുത്ത് ചെന്ന് അതിന്റെ പൂഞ്ഞയും തലയുടെ പിൻഭാഗം തലോടി. അപ്പോൾ അത് ശാന്തമായി. അദ്ദേഹം ചോദിച്ചു: ഈ ഒട്ടകത്തിന്റെ ഉടമ ആരാണ് ? ആരുടേതാണ് ഈ ഒട്ടകം? അൻസ്വാരികളിൽ പെട്ട ഒരു യുവാവ് വന്ന് അദ്ദേഹത്തോട് പറഞ്ഞു: അല്ലാഹുവിന്റെ ദൂതരേ, ഈ ഒട്ടകം എന്റേതാണ്. അപ്പോൾ പ്രവാചകൻ (സ) പറഞ്ഞു: “അല്ലാഹു താങ്കൾക്ക് ഉടമപ്പെടുത്തി തന്ന ഈ മൃഗത്തിന്റെ കാര്യത്തിൽ താങ്കൾ അല്ലാഹുവെ സൂക്ഷിക്കുന്നില്ലേ ? താങ്കൾ അതിനെ പട്ടിണിക്കിടുന്നതായും (പ്രയാസകരമായ ജോലികൾ നൽകി) ക്ഷീണിപ്പിക്കുന്നതായും അത് എന്നോട് പരാതിപ്പെടുന്നു.” (സുനനു അബൂദാവൂദ്: 2549, മുസ്നദു അഹ്മദ്: 1745 )
ഏതു മൃഗത്തോടും കരുണ കാണിക്കൽ പുണ്യമാണ് എന്നതാണ് ഇസ്ലാമിലെ അടിസ്ഥാന തത്ത്വം.
അബൂഹുറൈറയിൽ നിന്ന്: (പ്രവാചകാനുചരന്മാർ) ചോദിച്ചു: അല്ലാഹുവിന്റെ ദൂതരേ, മൃഗങ്ങളോട് നന്മ ചെയ്യുന്നതിൽ ഞങ്ങൾക്ക് പ്രതിഫലമുണ്ടോ ? അദ്ദേഹം പറഞ്ഞു: പച്ച കരളുള്ള എന്തിനോടും നന്മ ചെയ്യുന്നതിന് പ്രതിഫലമുണ്ട്. (സ്വഹീഹുൽ ബുഖാരി: 2363)
സ്വാഭാവികമായും പല്ലിയും ഇതിൽ ഉൾപ്പെടും. അതുകൊണ്ട് തന്നെ ആവശ്യമില്ലാതെ, തിരഞ്ഞു നടന്ന് പല്ലികളെ കൊല്ലാനൊന്നും ഹദീസുകളിലില്ല. ഇനി കൊല്ലേണ്ടി വന്നാൽ തന്നെ പെട്ടെന്ന് കൊല്ലുവാനും അദ്ദേഹം കൽപ്പിച്ചു.
കൊല്ലൽ അനുവദനീയമായ ഫവാസിക്കുകളിൽ കടിക്കുന്ന നായയെ എണ്ണിയ അതേ പ്രവാചകൻ (സ) കടിക്കാൻ വരാത്ത നായ്ക്കളോട് പുണ്യം ചെയ്യാൻ പഠിപ്പിച്ചു:
അബൂഹുറൈറ (റ) നിവേദനം: പ്രവാചകൻ (സ) പറഞ്ഞു: ഒരു നായ ഒരു കിണറിന് ചുറ്റും ചുറ്റിനടക്കുകയായിരുന്നു; ദാഹം കൊണ്ട് അത് ചാകാറായിട്ടുണ്ടായിരുന്നു. അപ്പോൾ ഇസ്റാഈല്യരിലെ ഒരു അഭിസാരിക അതിനെ കണ്ടു. അവർ അവരുടെ ചെരുപ്പിന്റെ മേലാവരണമൂരി (അതിൽ കിണറ്റിലെ വെള്ളം നിറച്ച്,) നായയെ കുടിപ്പിച്ചു. അത് മൂലം അവരുടെ പാപങ്ങൾ അവർക്ക് പൊറുത്തു കൊടുക്കപ്പെട്ടു. (സ്വഹീഹുൽ ബുഖാരി: 3308)
അബ്ദുർ റഹ്മാനിബ്നു അബ്ദുല്ല തന്റെ പിതാവിൽ നിന്നും ഉദ്ധരിക്കുന്നു. അദ്ദേഹം പറഞ്ഞു:
ഞങ്ങൾ അല്ലാഹുവിന്റെ ദൂതനോടൊപ്പം(സ) ഒരു യാത്രയിലായിരിക്കേ അദ്ദേഹം അൽപ്പ നേരം വിശ്രമിക്കാൻ പോയി. ഈ സമയം ഞങ്ങൾ ഒരു പക്ഷിയേയും അതിന്റെ രണ്ട് കുഞ്ഞുങ്ങളേയും കണ്ടു. ഞങ്ങൾ അതിന്റെ കുഞ്ഞുങ്ങളെ എടുത്തു കൊണ്ടുപോയി. തള്ള പക്ഷി വന്ന് ചിറകുവിരിച്ച് വട്ടമിട്ട് പറക്കാൻ തുടങ്ങി. അപ്പോൾ അല്ലാഹുവിന്റെ ദൂതൻ (സ) പറഞ്ഞു: ആരാണ് കുഞ്ഞുങ്ങളെ എടുത്ത് അതിനെ സങ്കടത്തിലാക്കിയത്. അതിന്റെ കുഞ്ഞുങ്ങളെ അതിന് തിരിച്ച് നൽകുക. (സുനനു അബൂദാവൂദ് : 2675)
ജീവജാലങ്ങളെ അനാവശ്യമായും വിനോധത്തിനായും കൊല്ലുന്നത് പോയി അവയെ ശകാരിക്കുന്നതും ശപിക്കുന്നതും വരെ പ്രവാചകൻ (സ) വിലക്കി:
“നിങ്ങൾ കോഴിയെ ശകാരിക്കരുത്; തീർച്ചയായും അത് നമ്മെ നമസ്ക്കാരത്തിന് (പ്രഭാതവേളയിൽ) ഉണർത്തുന്നു.” (സുനനു അബൂദാവൂദ്: 5101, സുനനുൽ കുബ്റാ: നസാഈ: 10781)
ചുരുക്കത്തിൽ, പല്ലിയെ കൊല്ലാൻ അനുവാദം നൽകി കൊണ്ടുള്ള ഹദീസ് വിവാദവൽക്കരിക്കുന്നത്, മൃഗശാലയിലെ Do not feed Monkeys’ ഫലകം കുരങ്ങുകളെ പട്ടിണിക്കിട്ട് കൊല്ലാൻ ആഹ്വാനം ചെയ്യുന്ന ക്രൂരതയാണെന്ന് മുദ്രാവാക്യം വിളിക്കുന്നതു പോലെ ബാലിശമാണ്.
ഹദീസ് മേഖലയിൽ ഇമാം ബുഖാരി അർപ്പിച്ച സേവനങ്ങളെ പരിഹസിച്ചു കൊണ്ട് ഒരു ഇസ്ലാം വിമർശകന്റെ നിരൂപണം ഇപ്രകാരമാണ്:
അദ്ദേഹം ജീവിച്ചത് 60 വർഷം. 20 വയസിൽ ഹദീസ് ശേഖരണം തുടങ്ങി. 36 വയസിൽ ഹദീസ് ശേഖരണം പൂർത്തിയാക്കി. ഈ 16 വർഷത്തിൽ 16×365 = 5840 ദിവസം 5840×24 = 140160 മണിക്കൂർ 140160 ×60 =84,09,600 മിനിറ്റ്
8409600 മിനിറ്റു കൊണ്ട് 300000 ഹദീസ് ശേഖരിച്ചു. എങ്കിൽ ഒരു ഹദീസിന് 28 മിനിറ്റു ചെലവഴിക്കേണ്ടി വന്നിട്ടുണ്ട് (8,409,600÷300,000) കണക്ക് ശരിയല്ലേ. റിപ്പോർട്ടർമാരെ അന്വേഷിച്ചു കണ്ടെത്തണം അതിനൊരു സാക്ഷ്യം വേണം വുളു എടുക്കണം 2 റക്അത്ത് നമസ്കരിക്കണം. കണക്കങ്ങോട്ട് എത്തുന്നില്ലല്ലോ. ന്നാ ശരി മ്മക്ക് LP സ്കൂളിലെ കണക്കെടുക്കാം. 300000 ഹദീസ് 5840 ദിവസം കൊണ്ട് ശേഖരിക്കുമ്പോൾ ഒരു ദിവസം ശരാശരി 51 ഹദീസ്. റിപ്പോർട്ടറെയും സാക്ഷിയെയും അന്വേഷിച്ചു കണ്ടെത്താൻ ഒരു 20 മിനിറ്റ് വുളു എടുക്കാനും, നമസ്കരിക്കാനും, കേൾക്കാനും, എഴുതാനും ഒരു 15 മിനിറ്റ് അപ്പൊ 35 മിനിറ്റ്. അതായത് 51×35 = 1785 മിനിറ്റ് പക്ഷെ ഒരുദിവസം 1440 മിനിറ്റല്ലേ ഉള്ളൂ
…………. മറുപടി :ഇസ്ലാമിക പണ്ഡിതന്മാർക്കിടയിൽ ഒരു ചൊല്ലുണ്ട്:
من تكلم في غير فنه أتى بالعجائب
തന്റേതല്ലാത്ത ശാസ്ത്രത്തിൽ ഒരാൾ സംസാരിക്കാൻ ശ്രമിച്ചാൽ അമ്പരപ്പിക്കുന്ന വിഡ്ഢിത്തങ്ങൾ അയാൾ കൊണ്ടുവരും. (ഫത്ഹുൽ ബാരി: 3/466)
വിവരമില്ലാത്ത വിഷയത്തിൽ ആളാവാൻ നോക്കിയാൽ ഇത്തരത്തിലുള്ള അമ്പരപ്പിക്കുന്ന വിഡ്ഢിത്തങ്ങൾ വിളമ്പുമെന്ന് പണ്ട് മുതലേ ഹദീസ് പണ്ഡിതന്മാർ പറയാറുണ്ടെന്നർത്ഥം.
ഇമാം ബുഖാരി ഇരുപതാം വയസ്സിൽ ആരംഭിച്ചതും 16 വർഷം കൊണ്ട് പൂർത്തീകരിച്ചതും സ്വഹീഹുൽ ബുഖാരി എന്ന് ഇന്ന് അറിയപ്പെടുന്ന അൽജാമിഅ് അസ്സ്വഹീഹ് എന്ന ഗ്രന്ഥത്തിന്റെ രചനയാണ്. അല്ലാതെ ഇരുപത് വയസ്സിന് ശേഷം ഹദീസുകൾ പഠിക്കാൻ ആരംഭിക്കുകയും 16 വർഷം കൊണ്ട് പൂർത്തീകരിക്കുകയും ചെയ്തുവെന്നല്ല. ഹദീസ് നിവേദകന്മാരെ സംബന്ധിച്ച് 18 വസ്സുള്ളപ്പോൾ അത്താരീഖുൽ കബീർ എന്ന ഒരു ഗ്രന്ഥം ഇമാം ബുഖാരി രചിക്കുകയുണ്ടായി. (സിയറു അഅ്ലാമിന്നുബലാഅ്: 12:400)
എന്നു പറഞ്ഞാൽ ചെറുപ്പം മുതൽക്കു തന്നെ അദ്ദേഹം ഹദീസ് പഠനം ആരംഭിച്ചിരുന്നു. പത്താം വയസ്സിൽ ഇബ്നുൽ മുബാറക്, വകീഅ് എന്നിവരുടെ സർവ ഹദീസ് ഗ്രന്ഥങ്ങളും അദ്ദേഹം മനപാഠമാക്കി. (സിയറു അഅ്ലാമിന്നുബലാഅ്: 12:407)
പതിനൊന്ന് വയസ്സായപ്പോഴേക്കും പഠിപ്പിക്കുന്ന ഗുരുവിന് ഒരു ഹദീസിന്റെ പരമ്പരയിൽ വീഴ്ച്ച സംഭവിച്ചാൽ ഇമാം ബുഖാരി അങ്ങോട്ട് തിരുത്തി കൊടുക്കാൻ തുടങ്ങി. (സിയറു അഅ്ലാമിന്നുബലാഅ്: 12:392)
ഇങ്ങനെ ചെറുപ്പം മുതൽ 36 വയസ്സു വരെ വായിക്കുകയും പഠിക്കുകയും ചെയ്ത ലക്ഷക്കണക്കിന് ഹദീസുകളിൽ നിന്ന് ഏറ്റവും പ്രബലമായ നിവേദക പരമ്പരയുള്ളവ മാത്രമെടുത്താണ് സ്വഹീഹുൽ ബുഖാരി എന്ന് ഇന്ന് അറിയപ്പെടുന്ന അൽജാമിഅ് അസ്സ്വഹീഹ് എന്ന ഗ്രന്ഥം രചിക്കുന്നത്.
അഥവാ, 3 ലക്ഷം ഹദീസ് 16 വർഷം കൊണ്ട് റിപ്പോർട്ടർമാരെ അന്വേഷിച്ചും അവരിൽ നിന്നും നേരിട്ട് കേട്ടും സ്വഹീഹാണെന്ന് ഉറപ്പു വരുത്തിയും പഠിച്ചു എന്ന് ഇമാം ബുഖാരി സ്വയമോ അദ്ദേഹത്തെ സംബന്ധിച്ച് മറ്റു ഹദീസ് പണ്ഡിതരോ പറഞ്ഞിട്ടില്ല. താൻ ജീവിതത്തിൽ വായിക്കുകയും പഠിക്കുകയും ചെയ്ത ലക്ഷക്കണക്കിന് ഹദീസുകളിൽ നിന്ന് തെരെഞ്ഞെടുത്ത (അത് എത്ര ലക്ഷമായാലും ശരി) ഏഴായിരത്തോളം ഹദീസുകൾ കൊണ്ട് സ്വഹീഹുൽ ബുഖാരി എന്ന ഗ്രന്ഥം രചിച്ചു എന്നാണ് പറഞ്ഞത്. റിപ്പോർട്ടർമാരെ അന്വേഷിച്ചതും അവരിൽ നിന്നും നേരിട്ട് കേട്ടതും സ്വഹീഹാണെന്ന് ഉറപ്പു വരുത്തിയതുമൊക്കെ ഈ ഏഴായിരത്തോളം വരുന്ന ഹദീസുകളുടെ കാര്യത്തിൽ മാത്രമാണ് എന്നർത്ഥം. ഈ ഉദ്യമത്തിനാണ് 16 വർഷം എടുത്തത്.
ഇനി, 3 ലക്ഷം ഹദീസ് പഠിച്ചു എന്ന് പറഞ്ഞാൽ അതിന്റെ പരമ്പരകളുടെ എണ്ണമാണ് സൂചിപ്പിക്കുന്നത്. ഒരു ഹദീസിന് തന്നെ നൂറുക്കണക്കിന് നിവേദക പരമ്പരകളുണ്ടാകാം. അപ്പോൾ ആ നൂറ് പരമ്പര പഠിച്ചാൽ നൂറ് ഹദീസ് പഠിച്ചു എന്നാണ് ഹദീസ് പണ്ഡിതന്മാരുടെ സാങ്കേതിക ഭാഷയിൽ പറയുക. ഇത് ഇമാം നവവിയും, ഇമാം ഇബ്നു സ്വലാഹും, ഹാഫിദ് അൽ ഇറാകിയുമെല്ലാം വ്യക്തമാക്കിയിട്ടുണ്ട്. (തദ്രീബുറാവി: 1:99)
സ്വഹീഹുൽ ബുഖാരിയിൽ തന്നെ ‘കർമ്മങ്ങളെല്ലാം ഉദ്ദേശങ്ങൾക്ക് അനുസൃതമായി മാത്രമാണ് സ്വീകരിക്കപ്പെടുക’ (إنما الأعمال بالنيات) എന്ന ഒരു ഹദീസ് 7 തവണ ആവർത്തിച്ചിട്ടുണ്ട്. ഈ ഒരു ഹദീസ് തന്നെ (ഹുമൈദി, ഇബ്നു മസ്ലമ, മുഹമ്മദിബ്നുകസീർ, മുസദ്ദിദ്, യഹ്യബ്നു കസഅ, കുതൈബത്തിബ്നു സഈദ്, അബുന്നുഅ്മാൻ എന്നീ) വ്യത്യസ്ഥരായ ഏഴു നിവേദകന്മാരിൽ നിന്നാണ് ഉദ്ധരിച്ചിരിക്കുന്നത്. അതുകൊണ്ട് തന്നെ സ്വഹീഹുൽ ബുഖാരിയിലെ ഏഴായിത്തോളം വരുന്ന ഹദീസുകളുടെ എണ്ണത്തിൽ ‘കർമ്മങ്ങളെല്ലാം ഉദ്ദേശങ്ങൾക്ക് അനുസൃതമായി മാത്രമാണ് സ്വീകരിക്കപ്പെടുക’ എന്ന ഒരൊറ്റ ഹദീസ് ഏഴെണ്ണമായാണ് പരിഗണിക്കപ്പെട്ടിരിക്കുന്നത്. ഈ കണക്കിന് ഒരു നാട്ടിലെ ഒരു റിപ്പോർട്ടറിൽ നിന്ന് തന്നെ ആയിരക്കണക്കിന് ഹദീസുകൾ പഠിക്കാം.
“എങ്കിൽ ഒരു ഹദീസിന് 28 മിനിറ്റു ചെലവഴിക്കേണ്ടി വന്നിട്ടുണ്ട് (8,409,600÷300,000) കണക്ക് ശരിയല്ലേ.” (വിമർശകൻ)
ഒരു ഹദീസ് പഠിക്കാൻ 28 മിനുറ്റ് എന്ന കണക്കു കൊണ്ട് തെളിയിച്ച ആ അപാരതക്ക് മുമ്പിൽ നമിക്കുന്നു !
من طريق الدراوردي عن عمرو بن يحيى المازني عن أبيه عن أبي سعيد عن النبي صلى الله عليه وسلم قال: لا ضرر ولا ضرار
അറബി പഠിച്ച, മലയാളിയായ ഞാൻ മുകളിൽ കൊടുത്തിരിക്കുന്ന ഈ ഹദീസ് സനദു സഹിതം പഠിച്ചത് 38 സെക്കന്റും 37 മൈക്രോ സെക്കന്റിലുമാണ്. സ്റ്റോപ് വാച്ച് വെച്ച് നോക്കിയിട്ടുണ്ട്. അപ്പോൾ ഒരാളുടെ അടുത്ത് നിന്ന് ഇതുപോലെയുള്ള നൂറ് ഹദീസ് പഠിക്കാൻ എത്ര സമയമെടുക്കും മാഷേ…? 3837 സെക്കന്റ്. അഥവാ 63 മിനുറ്റ്. അഥവാ 1 മണിക്കൂർ 3 മിനുറ്റ്. അപ്പോൾ ഒരു ദിവസം ഏകദേശം പത്തര മണിക്കൂർ പഠിച്ചാൽ ഏകദേശം 1000 ഹദീസുകൾ പഠിക്കാം. അപ്പോൾ ഒരു വർഷത്തിലെ 300 ദിവസം ഏകദേശം പത്തര മണിക്കൂർ പഠിച്ചാൽ തന്നെ 3 ലക്ഷം ഹദീസ് പഠിക്കാം. ഇനി ദീർഘമായ ഹദീസുകൾ ഉൾപ്പെടുത്തി പഠിക്കാനെടുക്കുന്ന സമയ ദൈർഘ്യം നീളുമെന്ന് സമ്മതിച്ചാൽ തന്നെ കുറച്ച് വർഷങ്ങളുടെ എണ്ണം കൂട്ടണമെന്നല്ലാതെ അസംഭവ്യമായ ഒന്നും ഈ കണക്കിൽ വന്നു ചേരാനില്ല. പക്ഷെ ഇതൊക്കെയുണ്ടോ കണക്കു മാഷിന് അറിയുന്നു. വിവരമില്ലാത്ത വിഷയത്തിൽ ആളാവാൻ കുറച്ചു കണക്കുമായി ഇറങ്ങിയതാണ് ടിയാൻ.
ഒരു പരമ്പര ഒരു വട്ടം കേട്ട് മനപാഠമാക്കാൻ മാത്രം കഴിവൊക്കെ ഉണ്ടായിരുന്നു എന്ന് ചരിത്രകാരന്മാർ വിശേഷിപ്പിച്ച ഇമാം ബുഖാരിക്കുണ്ടൊ ഈ എൽ.പി സ്കൂൾ അദ്ധ്യാപകന്റെ കണക്കനുസരിച്ച് ഹദീസ് പഠിക്കാൻ സമയം !
“ഒരു ദിവസം ശരാശരി 51 ഹദീസ്. റിപ്പോർട്ടറെയും സാക്ഷിയെയും അന്വേഷിച്ചു കണ്ടെത്താൻ ഒരു 20 മിനിറ്റ്…” (വിമർശകൻ)
കണക്കു മാഷിന്റെ ഈ അമ്പരപ്പിക്കുന്ന കണ്ടെത്തൽ പ്രകാരം ഓരോ ഹദീസും വ്യത്യസ്ഥരായ റിപ്പോർറർമാരിൽ നിന്നാവണം. എങ്കിലല്ലെ അവരെ കണ്ടെത്താനുള്ള 20 മിനുട്ട് ഓരോ ഹദീസിലും കൂട്ടാൻ മാഷിന് കഴിയു. ഒരു റിപ്പോർട്ടറിൽ നിന്ന് 51 ഹദീസ് പഠിച്ചാലെന്താ…? എന്ന് പിള്ളേര് ചോദിക്കരുത്.
ബുഖാരി, അബ്ദുല്ലാഹിബ്നു യൂസുഫ് അത്തുനൈസിയിൽ നിന്ന് മുന്നൂറിലേറെ ഹദീസുകളും അലിയ്യിബ്നുൽ മദീനിയിൽ നിന്ന് ഇരുന്നൂറിലേറെ ഹദീസുകളും സ്വഹീഹുൽ ബുഖാരിയിൽ എഴുതിയിട്ടുണ്ടെന്നൊന്നും മാഷിനറിയില്ലല്ലൊ. അഥവാ ഒരൊറ്റ റിപ്പോർട്ടറിൽ നിന്ന് മാത്രം മൂന്നുറിലേറെയും ഇരുന്നൂറിലേറെയും ഹദീസുകൾ ബുഖാരി തന്റെ സ്വഹീഹിൽ രേഖപ്പെടുത്തി എന്ന് തെളിയുന്നതോടെ മാഷിന്റെ മറ്റൊരു കണക്ക് ചമ്മന്തി പരിവത്തിലാവുന്നു. വിവരമില്ലാത്ത വിഷയത്തിൽ ആളാവാൻ കുറച്ചു കണക്കുമായി ഇറങ്ങിയതാണ് ടിയാൻ.
ഇമാം ബുഖാരി ചെറുപ്പം മുതൽ ഹദീസ് പഠനമാരംഭിച്ചിട്ടുണ്ട് എന്ന് നാം സൂചിപ്പിച്ചുവല്ലൊ. അതിൽ ഒരുപാട് ഹദീസുകൾ ഹദീസ് പണ്ഡിതന്മാരുടെ ഗ്രന്ഥങ്ങളിൽ നിന്ന് വായിച്ച് പഠിച്ചതാണ് എന്നും നിവേദകന്മാരെ കണ്ടെത്തി പഠിച്ചതല്ലെന്നും നാം സൂചിപ്പിച്ചു. അപ്പോൾ 3 ലക്ഷം ഹദീസുകളോട് ഗുണിക്കാനായി നീക്കിവെക്കുന്ന -റിപ്പോർട്ടർമാരെ കണ്ടെത്താൻ എടുക്കുന്ന – 20 മിനുറ്റ്, ഗ്രന്ഥങ്ങളിൽ നിന്ന് വായിച്ച് പഠിച്ച ഹദീസുകളുടെ കാര്യത്തിൽ ഒഴിവാക്കേണ്ടതല്ലെ ?. അത്തരം ഹദീസുകൾ എത്രയാണെന്ന് കണക്കുകൂട്ടി 3 ലക്ഷത്തിൽ നിന്ന് കുറക്കാത്തിടത്തോളം മാഷിന്റെ കണക്കുകൾ ചീട്ടു കൊട്ടാരം പോലെ നിലം പതിക്കുന്നു.
തന്റെ ഗുരുക്കളിൽ നിന്ന് പഠിക്കുന്നതിനും ഹദീസ് ശേഖരിക്കുന്നതിനും പല മാർഗങ്ങളും ഇമാം ബുഖാരി അവലംബിച്ചിട്ടുണ്ടെന്ന് അദ്ദേഹം തന്നെ വ്യക്തമാക്കിയതായി അദ്ദേഹത്തിന്റെ ജീവചരിത്രത്തിൽ കാണാം. അതിലൊന്ന് ഗുരു ശേകരിച്ച ഗ്രന്ഥങ്ങളിൽ നിന്ന് വായിച്ച് അദ്ദേഹത്തിന്റെ സമ്മഞ്ഞോടെ തന്റെ ഹദീസ് ശേഖരത്തിൽ ഉൾപ്പെടുത്തുക എന്നതാണ്.
.سمعت محمد بن إسماعيل يقول : كان إسماعيل بن أبي أويس إذا انتخبت من كتابه نسخ تلك الأحاديث
ബുഖാരി പറഞ്ഞു: ഞാൻ എന്റെ ഗുരുവായ ഇസ്മാഈൽ ഇബ്നു അബീ ഉവൈസിന്റെ ഗ്രന്ഥത്തിൽ നിന്ന് അദ്ദേഹത്തിന്റെ സമ്മതത്തോടെ ഹദീസുകൾ തിരഞ്ഞെടുക്കുമായിരുന്നു… (സിയറു അഅ്ലാമിന്നുബലാഅ്: 12:408)
ഗുരുക്കന്മാരിൽ നിന്ന് ഹദീസ് സ്വീകരിക്കുന്നതിലെ മറ്റൊരു മാർഗമാണ് അവരുടെ ഗ്രന്ഥത്തിലെ ഹദീസ് അവർക്കു തന്നെ വായിച്ച് കൊടുത്ത് അവർ അത് അംഗീകരിക്കുമ്പോൾ, അവരിൽ നിന്നും ഹദീസുകൾ (ആ ഗ്രന്ഥത്തിലുള്ള) പഠിച്ചതായി പരിഗണിക്കൽ. ഇതിനെ ഹദീസ്- കർമ്മശാസ്ത്ര സാങ്കേതിക ഭാഷയിൽ ഇംലാഅ്, ഇക്റാഅ് എന്നൊക്കെയാണ് പറയുക. ഇമാം ബുഖാരി തന്നെ ഇംലാഇലൂടെയും ഇക്റാഇലൂടെയും ഹദീസുകൾ പഠിക്കുകയും പഠിപ്പിക്കുകയും ചെയ്തിട്ടുണ്ടെന്ന് അദ്ദേഹത്തിന്റെ ജീവചരിത്രത്തിലുണ്ട്. ثم خرجت من الكتاب بعد العشر ، فجعلت أختلف إلى الداخلي وغيره . فقال يوما فيما كان يقرأ للناس (സിയറു അഅ്ലാമിന്നുബലാഅ്: 12:392)
ഇമാം ബുഖാരിയുടെ ഗുരുക്കളായിരുന്നു ഹുമൈദി, ഇബ്നു റാഹൂയ, അഹ്മദിബ്നു ഹമ്പൽ എന്നിവർ. ഇവർക്ക് ഓരോരുത്തർക്കും മുസ്നദ് എന്ന പേരിൽ ഹദീസ് ഗ്രന്ഥങ്ങളുണ്ട്. അഹ്മദിബ്നു ഹമ്പലിന്റെ മുസ്നദിൽ മാത്രം 40000 ഹദീസുകൾ നമുക്ക് കാണാം. ഈ മൂന്ന് ഗുരുക്കന്മാരുടേയും മുസ്നദുകളിലെ 40000 ഹദീസുകൾ വീതം വായിച്ചു പഠിച്ചാൽ തന്നെ അദ്ദേഹത്തിന് ഒരു ലക്ഷത്തി ഇരുപതിനായിരം ഹദീസുകൾ പഠിക്കാം. റിപ്പോർട്ടർമാരെ അന്വേഷിച്ച് കണ്ടെത്താനുള്ള സമയം ചെലവഴിക്കേണ്ടി വരുന്നില്ല.
മാഷിന്റെ അമ്പരപ്പിക്കുന്ന മറ്റൊരു കണ്ടെത്തൽ: “300000 ഹദീസ് 5840 ദിവസം കൊണ്ട് ശേഖരിക്കുമ്പോൾ ഒരു ദിവസം ശരാശരി 51 ഹദീസ്… വുളു എടുക്കാനും, നമസ്കരിക്കാനും, കേൾക്കാനും, എഴുതാനും ഒരു 15 മിനിറ്റ്… ” (വിമർശകൻ)
പക്ഷെ താൻ പഠിച്ച 3 ലക്ഷം ഹദീസുകൾ പഠിക്കുമ്പോഴും “വുളു എടുക്കാനും, നമസ്കരിക്കാനും, കേൾക്കാനും, എഴുതാനും” നിൽക്കുമായിരുന്നു എന്നത് നുണയല്ലെ മാഷേ ? സ്വഹീഹുൽ ബുഖാരിയിൽ ഒരോ ഹദീസും എഴുതി ചേർക്കുമ്പോൾ താൻ നമസ്ക്കരിച്ചിരുന്നു എന്നല്ലെ ബുഖാരി പറഞ്ഞിട്ടുള്ളു. (ഉംദത്തുൽകാരി: 1:5, ശർഹുൽ കസ്തല്ലാനി: 1:29, തഗ്ലീക്കു തഅ്ലീക്: 5:5554, അതവ്ളീഹ് ഫി ശർഹിൽ ജാമിഉ സ്വഹീഹ് : 2:28, ഇശ്റൂന ഹദീസ്: 1:14, തഹ്ദീബുൽ അസ്മാഅ്: നവവി: 1:74)
ബുഖാരിയിൽ ആവർത്തനങ്ങൾ ഉൾപ്പെടെ ഏഴായിരത്തോളം ഹദീസും ആവർത്തനം ഒഴിവാക്കിയിൽ 2602 ഹദീസുകളുമാണുള്ളത്. (മുഖദ്ദിമത്തു ഫത്ഹുൽ ബാരി) 2602 ഹദീസുകളിൽ ഓരോന്നിനും വേണ്ടി വുളു എടുക്കാനും, നമസ്കരിക്കാനും, കേൾക്കാനും, എഴുതാനും 15 മിനിറ്റ് എടുത്താൽ ആകെ വരുന്നത് 39030 മിനുറ്റ് അഥവാ 650.5 മണിക്കൂറാണ്. രണ്ട് വർഷത്തിൽ 730 ദിവസം കൂട്ടിയാൽ, വർഷത്തിലെ ഓരോ ദിവസവും ഒരു മണിക്കൂർ ഈ വുളു എടുക്കാനും നമസ്കരിക്കാനും, കേൾക്കാനും, എഴുതാനും വേണ്ടി ഉപയോഗിച്ചാൽ തന്നെ 2 വർഷം തികയുന്നതിന് മുമ്പ് തന്നെ സ്വഹീഹുൽ ബുഖാരിയിലെ ഹദീസ് മൊത്തം ഇമാം ബുഖാരിക്ക് എഴുതി തീർക്കാം. !
സ്ത്രീകളുടെ യോനീസ്രവം മഞ്ഞ നിറത്തിലുള്ളതാണെന്നും അതാണ് കുഞ്ഞിന്റെ രൂപീകരണത്തിൽ പങ്കാളിയാകുന്നത് എന്നുമെല്ലാം വ്യക്തമാക്കുന്ന നിരവധി ഹദീഥുകളുണ്ട്. പ്രത്യുത്പാദനത്തിൽ പങ്കെടുക്കുന്ന സ്ത്രീയുടെ സ്രവം യോനിയിൽ പുറത്തേക്ക് കാണാൻ സാധ്യമല്ല. സ്ത്രീയുടെ സ്ഖലനത്തെയും അതിന്റെ നിറത്തെയുമെല്ലാം കുറിച്ച പരാമർശങ്ങൾ മുഹമ്മദ് നബിയുടെ തെറ്റിദ്ധാരണയിൽ നിന്നുണ്ടായതാണെന്നതല്ലേ ശരി? യോനീഭാഗത്തെ അണുബാധ നിമിത്തം യോനീസ്രവം ചിലപ്പോൾ മഞ്ഞ നിറത്തിലാകാറുണ്ട്. അത് കണ്ട് തെറ്റിദ്ധരിച്ച മുഹമ്മദ് നബി പറഞ്ഞപ്പോൾ വന്ന അബദ്ധമല്ലേ ഹദീഥുകളിലെ ഈ മഞ്ഞ ദ്രാവകം ??
സിൽഷിജ്.K
അല്ല. കുഞ്ഞിന്റെ നിർമ്മാണത്തിൽ പങ്കെടുക്കുന്ന സ്ത്രീദ്രാവകത്തിന് മഞ്ഞ നിറമാണെന്ന് പറഞ്ഞ മുഹമ്മദ് നബി(സ)ക്ക് തെറ്റുകളൊന്നും പറ്റിയിട്ടില്ല. ദൈവികബോധനത്തിന്റെ അടിസ്ഥാനത്തിൽ നബി (സ) പറഞ്ഞ കാര്യങ്ങൾ തെറ്റാണെന്ന് തെളിയിക്കാൻ ഒരു ശാസ്ത്രത്തിനും കഴിയില്ല. ഈ രംഗത്തെ പുതിയ ഗവേഷണങ്ങൾ വ്യക്തമാക്കുന്നത് ഈ സത്യമാണ്.
ഥൗബാനിൽ(റ) നിന്ന് ഇമാം മുസ്ലിം തന്റെ സ്വഹീഹിൽ നിവേദനം ചെയ്ത, ജൂത പണ്ഡിതന്റെ ചോദ്യങ്ങള്ക്കുള്ള പ്രവാചകന്റെ(സ) ഉത്തരത്തെപ്പറ്റി വിശദീകരിക്കുന്ന ദീര്ഘമായ ഒരു ഹദീഥുണ്ട്. ആ ഹദീഥിൽ ശിശുവിന്റെ സൃഷ്ടിയെക്കുറിച്ച ചോദ്യത്തിനുള്ള വിശദമായ ഉത്തരം ആരംഭിക്കുന്നത് 'പുരുഷസ്രവം വെളുത്തനിറത്തിലുള്ളതും സ്ത്രീസ്രവം മഞ്ഞനിറത്തിലുള്ളതുമാണ്' എന്നു പറഞ്ഞുകൊണ്ടാണ്. ജൂത ചോദ്യങ്ങള്ക്കെല്ലാം മറുപടി പറഞ്ഞ ശേഷം 'അയാള് എന്നോട് ചോദിച്ച കാര്യങ്ങളെക്കുറിച്ചൊന്നും അല്ലാഹു അറിയിച്ചുതരുന്നതുവരെ എനിക്ക് യാതൊരു വിവരവുമുണ്ടായിരുന്നില്ല' എന്ന് പറഞ്ഞതായുള്ള ഥൗബാനി(റ)ന്റെ പരാമര്ശം ശ്രദ്ധേയമാണ്. സ്വന്തം സ്രവത്തെക്കുറിച്ച് അറിയാത്ത സ്ത്രീകള്ക്കടക്കം നിങ്ങളുടെ സ്രവം മഞ്ഞനിറത്തിലുള്ളതാണ് എന്ന് പ്രവാചകന് (സ) പറഞ്ഞുകൊടുത്തത് വ്യക്തമായ ദൈവബോധനത്തിന്റെ അടിസ്ഥാനത്തിലാണെന്ന് വ്യക്തമാക്കുന്നതാണീ പ്രവാചകപരാമര്ശം. ബാഹ്യമായി കാണുന്ന ഏതെങ്കിലും സ്രവത്തെക്കുറിച്ചുള്ളതായിക്കൊള്ളണമെന്നില്ല ഈ പ്രവാചകപരാമർശമെന്ന് ഇതിൽ നിന്ന് വ്യക്തമാവുന്നുണ്ട്. ഇതിൽ നിന്ന് വ്യത്യസ്തമായി ബാഹ്യമായി കാണുന്ന ദ്രാവകത്തെക്കുറിച്ച് തന്നെയാണ് മഞ്ഞദ്രാവകമെന്ന് പ്രവാചകൻ(സ) പറഞ്ഞതെന്ന് ഖണ്ഡിതമായി മനസ്സിലാക്കിത്തരുന്ന നബിവചനങ്ങളൊന്നും തന്നെയില്ല.
ഏതാണീ മഞ്ഞ ദ്രാവകം? കുഞ്ഞിന്റെ സൃഷ്ടിയില് പങ്കെടുക്കുന്ന പുരുഷസ്രവത്തിന്റെ നിറം 'അബ്യദ്വ്' (വെള്ള) ആണെന്നു പറഞ്ഞതിനുശേഷമാണ് സ്ത്രീ സ്രവത്തിന്റെ നിറം 'അസ്വ്ഫര്' (മഞ്ഞ) ആണെന്ന് പ്രവാചകന് (സ) പറഞ്ഞത്. രണ്ടും കൂടിച്ചേര്ന്നാണ് കുഞ്ഞുണ്ടാകുന്നതെന്നും അതിനുശേഷം അദ്ദേഹം വ്യക്തമാക്കി. വെള്ള നിറത്തിലുള്ള പുരുഷസ്രവത്തെപോലെതന്നെ ബീജസങ്കലനത്തില് പങ്കെടുക്കുന്ന സ്ത്രീസ്രവത്തിന്റെ നിറം മഞ്ഞയാണെന്നാണ് പ്രവാചകന് (സ) ഇവിടെ പഠിപ്പിക്കുന്നതെന്നുറപ്പാണ്. സ്ത്രീശരീരത്തില്നിന്ന് നിര്ഗളിക്കു ന്ന ഏതു സ്രവത്തിനാണ് മഞ്ഞനിറമുള്ളതെന്ന കാര്യത്തില് കര്മശാസ്ത്ര പണ്ഡിതന്മാര് ഏറെ ചര്ച്ച ചെയ്തതായി കാണാന് കഴിയും. സ്ത്രീജനനേന്ദ്രിയത്തില്നിന്ന് നിര്ഗളിക്കുന്ന കാണാനാവുന്ന സ്രവങ്ങള്ക്കൊന്നും തന്നെ മഞ്ഞ നിറമില്ലെന്ന വസ്തുതയാണ് വിശാലമായ ഇത്തരം ചര്ച്ചകളുടെ ഉല്ഭവത്തിന് നിമിത്തമായത്.
സ്ത്രീകളുടെ ജനനേന്ദ്രിയത്തില്നിന്ന് പുറത്തുവരുന്ന സ്രവങ്ങള് മൂന്നെണ്ണമാണ്. തന്റെ ശരീരം ലൈംഗികബന്ധത്തിന് സജ്ജമായിയെന്ന് അറിയിച്ചുകൊണ്ട് സ്ത്രീജനനേന്ദ്രിയത്തില്നിന്ന് കിനിഞ്ഞിറങ്ങുന്ന ബര്ത്തോലിന് സ്രവം (Bartholin fluid) ആണ് ഒന്നാമത്തേത്. യോനീമുഖത്തിനകത്തായി സ്ഥിതി ചെയ്യുന്ന പയര്വിത്തിന്റെ വലിപ്പത്തിലുള്ള രണ്ട് ബര്ത്തോലിന് ഗ്രന്ഥികള് സ്ത്രീശരീരം ലൈംഗികമായി ഉത്തേജിപ്പിക്കപ്പെടുമ്പോള് പുറപ്പെടുവിക്കുന്ന ഈ സ്രവത്തിന് നിറമില്ല. രതിമൂര്ച്ചയുടെ അവസരത്തില് ചില സ്ത്രീകളുടെ ജനനേന്ദ്രിയത്തില്നിന്ന് പുറത്തുവരുന്ന പാരായുറിത്രല് സ്രവമാണ് (Para urethral fluid) രണ്ടാമത്തെ യോനീ സ്രവം. യോനിയുടെ ആന്തരികഭിത്തിയില് സ്ഥിതി ചെയ്യുന്ന പാരായുറിത്രല് ഗ്രന്ഥികളില്നിന്നു വളരെ ചെറിയ അളവില്മാത്രം പുറത്തുവരുന്ന ഈ സ്രവം താരതമ്യേന കട്ടിയുള്ളതും വെള്ള നിറത്തിലുള്ളതുമായിരിക്കും. സ്ത്രീ ജനനേന്ദ്രിയത്തെ എല്ലായ്പ്പോഴും വരളാതെ സൂക്ഷിക്കുന്ന സെര്വിക്കല് ശ്ലേഷ്മ (Cervical mucus) ആണ് മൂന്നാമത്തെ യോനീ സ്രവം. അണ്ഡോല്സര്ജനസമയമല്ലെങ്കില് ഈ സ്രവം വഴുവഴുപ്പുള്ളതും നല്ല വെളുത്ത ക്രീം നിറത്തിലുള്ളതുമായിരിക്കും. അണ്ഡോല്സര്ജനത്തോടടുക്കുമ്പോള് വെള്ളനിറം മങ്ങുകയും വഴുവഴുപ്പ് കുറയുകയും ചെയ്യുന്ന ഈ സ്രവം ഉല്സര്ജനസമയമാകുമ്പോഴേക്ക് ജലത്തെപ്പോലെ വര്ണരഹിതമാവുകയും മുട്ടയുടെ വെള്ളക്കരുവിനെപ്പോലെയായിത്തീരുകയും ചെയ്യും. അണുബാധയുണ്ടാകുമ്പോള് മാത്രമാണ് സെല്വിക്കല് ശ്ലേഷ്മത്തിന് മങ്ങിയ മഞ്ഞനിറമുണ്ടാകുന്നത്. സ്ത്രീജനനേന്ദ്രിയത്തില്നിന്ന് സാധാരണഗതിയില് നിര്ഗളിക്കപ്പെടുന്ന മൂന്ന് സ്രവങ്ങളും വെളുത്തതോ നിറമില്ലാത്തതോ ആണെന്നും ഹദീഥുകളില് പറഞ്ഞ മഞ്ഞസ്രവമല്ല ഇവയെന്നും വ്യക്തമാണ്. ഇവയ്ക്കൊന്നുംതന്നെ കുഞ്ഞിന്റെ രൂപീകരണത്തില് നേരിട്ട് പങ്കൊന്നുമില്ലതാനും.
കുഞ്ഞിന്റെ രൂപീകരണത്തിന് നിമിത്തമാകുന്ന സ്രവമെന്താണ് എന്ന ചോദ്യത്തിന് ഉത്തരം കാണാന് ശ്രമിക്കുമ്പോഴാണ് ഹദീഥുകളില് പറഞ്ഞ മഞ്ഞ സ്രവമേതാണെന്ന് നമുക്ക് മനസ്സിലാവുക. ആര്ത്തവചക്രത്തിന്റെ പതിനാലാം ദിവസം അണ്ഡാശയത്തിനകത്തെ പൂര്ണ വളര്ച്ചയെത്തിയ ഫോളിക്കിളില് പ്രത്യക്ഷപ്പെടുന്ന ദ്വാരത്തിലൂടെ പ്രായപൂര്ത്തിയെത്തിയ അണ്ഡത്തെവഹിച്ചുകൊണ്ട് ഫോളിക്കുളാര് ദ്രവവും ക്യൂമുലസ് കോശങ്ങളും പുറത്തേക്ക് തെറിച്ച് ഫലോപ്പിയന് നാളിയുടെ അറ്റത്തുള്ള ഫിംബ്രയകളില് പതിക്കുന്നതിനാണ് അണ്ഡോല്സര്ജനം (Ovulation) എന്നു പറയുന്നത്. രതിമൂര്ച്ചയോടനുബന്ധിച്ച് പുരുഷശരീരത്തില് നടക്കുന്ന ശുക്ലസ്ഖലന(Ejaculation)ത്തിന് തുല്യമായി സ്ത്രീശരീരത്തില് നടക്കുന്ന പ്രക്രിയയാണ് ഇതെങ്കിലും ഒരു ആര്ത്തവചക്രത്തില് ഒരു തവണ മാത്രമാണ് ഇത് സംഭവിക്കുന്നത്. ശുക്ല സ്ഖലനവും അണ്ഡോല്സര്ജനവുമാണ് കുഞ്ഞിന്റെ സൃഷ്ടിക്ക് നിദാനമായി പുരുഷശരീരത്തിലും സ്ത്രീശരീരത്തിലും സംഭവിക്കുന്ന രണ്ട് പ്രക്രിയകള്. പുരുഷബീജങ്ങളെ വഹിക്കുന്ന ശുക്ലദ്രാവകത്തെപ്പോലെ സ്ത്രീയുടെ അണ്ഡത്തെ വഹിക്കുന്ന ഫോളിക്കുളാര് ദ്രവവും കുഞ്ഞിന്റെ നിര്മാണത്തിന് നിമിത്തമാകുന്ന ദ്രാവകമാണ്. ഹദീഥുകളില് പറഞ്ഞ കുഞ്ഞിന്റെ സൃഷ്ടിക്ക് കാരണമായ സ്ത്രീസ്രവം അണ്ഡത്തെ വഹിക്കുന്ന ഫോളിക്കുളാര് ദ്രാവകമാണെന്നാണ് ഇത് വ്യക്തമാക്കുന്നത്. അങ്ങനെയാണെങ്കില് പുരുഷദ്രാവകം വെളുത്തതും സ്ത്രീദ്രാവകം മഞ്ഞയുമെന്ന് പരാമര്ശത്തിന്റെ വെളിച്ചത്തില് പരിശോധിക്കുമ്പോള് ഫോളിക്കുളാര് ദ്രാവകത്തിന്റെ നിറം മഞ്ഞയായിരിക്കണം. എന്നാല് എന്താണ് വസ്തുത?
പ്രായപൂര്ത്തിയെത്തുന്നതിനുമുമ്പുള്ള അണ്ഡാവസ്ഥയായ അണ്ഡത്തെ (Oocyte) സംരക്ഷിക്കുകയും വളര്ത്തിക്കൊണ്ടുവന്ന് ബീജസങ്കലനത്തിന് പറ്റിയ അണ്ഡമാക്കിത്തീര്ക്കുകയും ചെയ്യുകയാണ് ഫോളിക്കിളിന്റെ ധര്മം. പെണ്കുഞ്ഞ് ജനിക്കുമ്പോള് തന്നെ അവളുടെ അണ്ഡാശയത്തിലുള്ള പ്രായപൂര്ത്തിയെത്താത്ത അണ്ഡകങ്ങളെ പൊതിഞ്ഞ് ആദിമ ഫോളിക്കിളുകളുണ്ടാവും (Primordial follicles). അവള് പ്രായപൂര്ത്തിയാകുന്നതോടെ ഇതില് ചില ഫോളിക്കിളുകള് വളര്ന്നുവരികയും ഓരോ ആര്ത്തവചക്രത്തിന്റെയും ശരാശരി 14-16 ദിവസങ്ങള് കഴിഞ്ഞ് പൊട്ടി പൂര്ണവളര്ച്ചയെത്തിയ അണ്ഡത്തെ (Ovum) പുറത്തുവിടുന്നതോടെ അവയുടെ ധര്മം അവസാനിക്കുകയും ചെയ്യുന്നു. ജനനസമയത്തുള്ള ഏകദേശം 1,80,000 ഫോളിക്കിളുകളില് നാനൂറെണ്ണത്തോളം മാത്രമാണ് അണ്ഡോല്സര്ജനത്തിനുമുമ്പത്തെ വളര്ച്ചയെത്തുവാനുള്ള ഭാഗ്യമുണ്ടാകുന്നത്. പ്രസ്തുത വളര്ച്ചയ്ക്ക് വ്യത്യസ്തങ്ങളായ ഘട്ടങ്ങളുണ്ട്. ഇതിലെ ഓരോ ഘട്ടങ്ങളിലും അതു കടന്നുപോകാന് കഴിയാത്ത ഫോളിക്കിളുകള് മരിച്ചുപോകുന്നുണ്ട്. ഓരോ ആര്ത്തവചക്രത്തിലും ഇരുപതോളം ഫോളിക്കിളുകള് വളര്ച്ചയെത്തുന്നുവെങ്കിലും ഒരെണ്ണത്തിന് മാത്രമാണ് ഫോളിക്കിള് മരണമായ അട്രീഷ്യ(atresia)യില്നിന്ന് രക്ഷപ്പെട്ട് അണ്ഡോല്സര്ജനത്തിന് കഴിയുന്നത്. അട്രീഷ്യയില് നിന്ന് രക്ഷപ്പെട്ട് അണ്ഡോല്സര്ജനത്തിന് കഴിയുന്ന ഫോളിക്കിളുകള് രണ്ട് ദശകളിലൂടെയാണ് കടന്നു പോകുന്നത്. അണ്ഡോല്സര്ജനത്തിലൂടെ അവസാനിക്കുന്ന ഒന്നാമത്തെ ദശയെ ഫോളിക്കുളാര് ദശ (follicular phase) എന്നും അതിനുശേഷ മുള്ള ദശയെ ലൂടിയല് ദശ (luteal phase) എന്നുമാണ് വിളിക്കുക. ആര്ത്തവം മുതല് അണ്ഡോല്സര്ജനം വരെയുള്ള ഫോളിക്കുളാര് ദശയില് അണ്ഡകം പൂര്ണവളര്ച്ചയെത്തിയ അണ്ഡമായിത്തീരുന്നതിനും യഥാരൂപത്തിലുള്ള അണ്ഡോല്സര്ജനം നടക്കുന്നതിനും വേണ്ടി വ്യത്യസ്തങ്ങളായ പ്രക്രിയകള് നടക്കേണ്ടതുണ്ട്. ഈ പ്രക്രിയകളുടെ അവസാനമായി ശരീരത്തിലെ ഈസ്ട്രജന് നില പരമാവധി ഉയരുകയും ലൂറ്റിനൈസിംഗ് ഹോര്മോണ് (LH), ഫോളിക്കിള് സ്റ്റിമുലേറ്റിംഗ് ഹോര്മോണ് (FSH) എന്നീ ഹോര്മോണുകളെ ഇതിന്റെ ഫലമായി ഉത്പാദിപ്പിക്കുകയും ചെയ്യുന്നു. 24 മുതല് 36 വരെ മണിക്കൂറുകള് നീണ്ടുനില്ക്കുന്ന ഈ പ്രക്രിയയുടെ അന്ത്യം കുറിച്ചുകൊണ്ടാണ് അണ്ഡം വഹിക്കുന്ന പൂര്ണവളര്ച്ചയെത്തിയ ഫോളിക്കിളില് (Ovarian follicle) സ്റ്റിഗ്മയെന്ന് പേരുള്ള ദ്വാരമുണ്ടാവുകയും അത് പൊട്ടി അണ്ഡത്തെ വഹിച്ചുകൊണ്ട് ഫോളിക്കുളാര് ദ്രവം പുറത്തേക്ക് തെറിക്കുകയും ചെയ്യുന്നത്. ഈ പുറത്തേക്കു തെറിക്കല് പ്രക്രിയക്കാണ് അണ്ഡോല്സര്ജനം (Ovulation) എന്നു പറയുക
ഫോളിക്കുളാര് ദശയിലുടനീളം നടക്കുന്ന അണ്ഡവളര്ച്ചയ്ക്കും അതിന് ഉല്സര്ജിക്കാനാവശ്യമായസംവിധാനങ്ങളൊരുക്കുന്നതിനും നിമിത്തമാകുന്നത് FSHന്റെ പ്രവര്ത്തനങ്ങളാണ്. പ്രസ്തുത ഉത്പാദനത്തോടനുബന്ധിച്ചാണ് ഹൈപ്പോതലാമസില്നിന്നുള്ള ഗൊണാടോട്രോ പിന് റിലീസിംഗ് ഹോര്മോണിന്റെ (GnRH) പ്രേരണയാല് പിറ്റിയൂട്ടറിയില്നിന്ന് LHന്റെ ഉത്പാദനം നടക്കുന്നത്. ഈ ഹോര്മോണ് ഉത്പാദിപ്പിക്കുന്ന പ്രോട്ടീന് വിഘാടക രസങ്ങളായ പ്രോട്ടിയോലിറ്റിക് എന്സൈമുകളാണ് (Proteolytic enzymes) ഫോളിക്കിളിലുണ്ടാവുന്ന ദ്വാരമായ സ്റ്റിഗ്മക്ക് കാരണമാകുന്നത്. അണ്ഡോല്സര്ജനത്തിനുശേഷമുള്ള ഫോളിക്കിള് അവശിഷ്ടങ്ങളെ നിയന്ത്രിക്കുന്നതും പ്രധാനമായി ഈ ഹോര്മോണാണ്. ലൂട്ടിയല് ദശയില് അണ്ഡം നഷ്ടപ്പെട്ട ഫോളിക്കിള് അവശിഷ്ടങ്ങള് കോര്പസ് ലൂടിയം (Lorpus Luteum) ആയിത്തീരുകയും മാതൃസ്വഭാവങ്ങളെ ഉദ്ദീപിക്കുന്ന പ്രോജസ്റ്ററോണ് (Progesterone) ഹോര്മോണിന്റെ വര്ധിതമായ ഉത്പാദനത്തിന് നിമിത്തമാവുകയും ചെയ്യുന്നു.
എന്താണീ ലൂറ്റിനൈസിംഗ് ഹോര്മോണ്? മഞ്ഞയെന്ന് അര്ത്ഥം വരുന്ന ലൂറ്റിയസ് (Luteus) എന്ന ലാറ്റിന് പദത്തിന്റെ നപുംസകരൂപമായ ലൂറ്റിയത്തില്നിന്നാണ് (Luteum) ലൂറ്റിനൈസ് (Luteinize) എന്ന ക്രിയയുണ്ടായിരിക്കുന്നത്. കോര്പ്പസ് ലൂടിയത്തിന്റെ നിര്മിതിക്ക് നിമിത്ത മായ പ്രവര്ത്തനങ്ങള്ക്കാണ് സാങ്കേതികമായി ലൂറ്റിനൈസ് എന്ന് പറയുന്നതെങ്കിലും പദപരമായി അതിനര്ത്ഥം 'മഞ്ഞയാക്കുന്നത്' എന്നാണ്. ലൂറ്റിനൈസിംഗ് ഹോര്മോണിന്റെ പ്രവര്ത്തനഫലമായാണ് ഫോളിക്കുളാര് ദശ പിന്നിട്ട ഫോളിക്കിള് അവശിഷ്ടങ്ങള് കോര് പസ് ലൂടിയം ആയിത്തീരുന്നത്. കോര്പസ് ലൂടിയം എന്ന പദദ്വയത്തിനര്ത്ഥം മഞ്ഞ വസ്തുവെന്നാണ് (Yellow body). ലൂടിയല് ദശയിലേക്ക് കടന്ന അണ്ഡം നഷ്ടപ്പെട്ട ഫോളിക്കിള് അവശിഷ്ടങ്ങളെല്ലാം കൂടി രണ്ടു മുതല് അഞ്ചു സെന്റീമീറ്റര് വരെ വ്യാസത്തില് ശരീരത്തില് ഏതാ നും ദിവസങ്ങള് കൂടി അവശേഷിക്കും. മനുഷ്യരില് ഇത് ഓറഞ്ചു നിറത്തിലാണ് കാണപ്പെടുന്നത്. അണ്ഡോല്സര്ജനത്തിന്റെ അവസാനഘട്ടത്തില് ഉത്പാദിപ്പിക്കപ്പെടുന്ന LH അതിന്റെ പ്രവര്ത്തനമാരംഭിക്കുകയും ഫോളിക്കുളാര് ദ്രവത്തെ മഞ്ഞവല്ക്കരിക്കുകയും ചെയ്യും. ഫോളിക്കിളിലെ സ്റ്റിഗ്മ പൊട്ടി അണ്ഡത്തോടെ പുറത്തേക്ക് തെറിക്കുന്ന ഫോളിക്കുളാര് ദ്രാവകത്തിന്റെ നിറം മഞ്ഞയായിരിക്കും. പുരുഷ ശുക്ലവുമായി താരതമ്യം ചെയ്യുമ്പോള് കട്ടിയില്ലാത്തതും മഞ്ഞ നിറത്തിലുള്ളതുമായ ദ്രാവകമാണ് ഫോളിക്കിള് പൊട്ടി പുറത്തേക്കൊഴുകുന്ന കുഞ്ഞിന്റെ നിര്മാണത്തിന് നിമിത്തമാകുന്ന സ്ത്രീസ്രവം എന്നര്ത്ഥം.
മഞ്ഞ നിറത്തിലുള്ള സ്ത്രീയുടെ സ്രവമാണ് വെളുത്ത നിറത്തിലുള്ള പുരുഷന്റെ സ്രവവുമായി ചേർന്ന് കുഞ്ഞുണ്ടാവുന്നത് എന്ന പ്രവാചകന്റെ പരാമർശം തന്നെയാണ് ശരിയെന്ന് ഇവ വ്യക്തമാക്കുന്നു. ഈ നൂറ്റാണ്ടിന്റെ തുടക്കത്തിൽ മാത്രം ഭ്രൂണശാസ്ത്രജ്ഞർ മനസ്സിലാക്കിയ കാര്യങ്ങൾ പതിനാലു നൂറ്റാണ്ടുകൾക്ക് മുമ്പ് പ്രസ്താവിച്ചതിനു ശേഷം നബി (സ) വ്യക്തമാക്കിയ കാര്യം ഇവിടെ എടുത്ത് പറയേണ്ടതുണ്ടെന്ന് തോന്നുന്നു. "അയാള് എന്നോട് ചോദിച്ച കാര്യങ്ങളെക്കുറിച്ചൊന്നും അല്ലാഹു അറിയിച്ചുതരുന്നതുവരെ എനിക്ക് യാതൊരു വിവരവുമുണ്ടായിരുന്നില്ല”.
വിഷയവുമായി ബന്ധപ്പെട്ട വീഡിയോ
പാരമ്പര്യ സ്വഭാവങ്ങളുടെ സംപ്രേഷണത്തെപ്പറ്റി ക്വുര്ആനില് വ്യക്തമായ പരാമര്ശങ്ങളൊന്നുമില്ല. സ്വഹീഹു മുസ്ലിമിലെ കിതാബുല് ഹയ്ദിലുള്ള സ്ത്രീയുടെ സ്രവത്തെക്കുറിച്ച ഒരു നബിവചനത്തില് പാരമ്പര്യത്തെക്കുറിച്ച കൃത്യവും വ്യക്തവുമായ സൂചനകളുണ്ട്. പ്രസ്തുത ഹദീഥിന്റെ സാരം ഇങ്ങനെയാണ്:
'സ്രവം കാരണമായിട്ടാണ് കുട്ടിക്ക് സാദൃശ്യമുണ്ടാകുന്നത്. സ്ത്രീയുടെ സ്രവം പുരുഷന്റെ സ്രവത്തിന് മുകളില് വന്നാല് കുട്ടിക്ക് മാതൃ സഹോദരന്മാരോട് സാദൃശ്യമുണ്ടാകും. പുരുഷന്റെ സ്രവം സ്ത്രീയുടെ സ്രവത്തിന് മുകളില് വന്നാല് കുട്ടിക്ക് അവന്റെ പിതൃവ്യന്മാ രോട് സാദൃശ്യമുണ്ടാകും.''(സ്വഹീഹു മുസ്ലിം, കിതാബുല് ഹൈദ്വ്, ബാബു വുജുബില് ഗസ്ലി അലല് മര്അത്തി ബി ഖുറൂജില് മനിയ്യി മിന്ഹ, ഹദീഥ് 314.)
പുരുഷന്റെയും സ്ത്രീയുടെയും സ്രവങ്ങളാണ് കുഞ്ഞിലേക്ക് പാരമ്പര്യ സ്വഭാവങ്ങള് പകര്ത്തുന്നെന്ന് വ്യക്തമാക്കുന്ന ഈ ഹദീഥ് ജനി തക സംപ്രേഷണവുമായി ബന്ധപ്പെട്ട പുതിയ വിവരങ്ങളുമായി പൂര്ണമായും പൊരുത്തപ്പെടുന്നതാണെന്ന വസ്തുത അത്ഭുതകരമാണ്. ഓരോ അവയവങ്ങളില് നിന്നും ഊര്ന്നിറങ്ങുന്നതാണ് ബീജമെന്നും ആണില് നിന്നോ പെണ്ണില് നിന്നോ ആരില്നിന്നാണോ ശക്തബീജമുണ്ടാ കുന്നത് അവരുടെ സവിശേഷതയായിരിക്കും കുഞ്ഞിലേക്ക് പകര്ത്തപ്പെടുന്നെന്നും ആര്ജ്ജിത സ്വഭാവങ്ങള് കുഞ്ഞിലേക്കു പകരുമെന്നു മുള്ള ഹിപ്പോക്രാറ്റസ് മുതല് ഡാര്വിന് വരെയുള്ളവരുടെ വീക്ഷണങ്ങളെ ഈ ഹദീഥ് അനുകൂലിക്കുന്നില്ല. രക്തത്തിലൂടെയാണ് പാരമ്പ ര്യത്തിന്റെ സംപ്രേഷണം നടക്കുന്നതെന്ന അരിസ്റ്റോട്ടിലിന്റെ വീക്ഷണത്തെ ഇത് നിരാകരിക്കുകയും ചെയ്യുന്നു. സ്ത്രീപുരുഷസ്രവങ്ങളു ടെ പ്രത്യക്ഷീകരണമാണ് കുഞ്ഞിന്റെ സവിശേഷതകള് നിര്ണയിക്കുന്നതെന്ന ഈ ഹദീഥ് മുന്നോട്ടുവെക്കുന്ന ആശയം ആധുനികകാലം വരെയുള്ള ശാസ്ത്രജ്ഞരൊന്നും മനസ്സിലാക്കിയിട്ടില്ലാത്തതാണ്. അതിശക്തമായ സൂക്ഷ്മദര്ശനികളുടെ സഹായത്താല് നടത്തിയ ഗവേഷ ണങ്ങള് വെളിപ്പെടുത്തിയ യാഥാര്ത്ഥ്യങ്ങളുമായി ഈ നബിവചനം യോജിച്ചുവരുന്നുവെന്ന വസ്തുത എന്തുമാത്രം ആശ്ചര്യകരമല്ല!
ഈ ഹദീഥില് പുരുഷന്റെ സ്രവത്തെ സ്ത്രീയുടെ സ്രവം അതിജയിച്ചാല് എന്നു പരിഭാഷപ്പെടുത്തിയിരിക്കുന്നത് 'ഇദാ അലാ മാഉഹാ മാഉര് റജൂലി'യെന്ന അറബി വചനത്തെയാണ്. പെണ്സ്രവം പുരുഷസ്രവത്തെ അതിജയിക്കുന്നതിന് ഇവിടെ 'അലാ'യെന്നാണ് പ്രയോഗി ച്ചിരിക്കുന്നത്. ഒന്നിനു മുകളില് മറ്റൊന്ന് ആധിപത്യം പുലര്ത്തുന്നതിനാണ് 'അലാ'യെന്നു പ്രയോഗിക്കുകയെന്ന് സൂറത്തുല് മുഅ്മിനൂ നിലെ 91-ാം വചനത്തില് നിന്ന് നാം നേരത്തെ മനസ്സിലാക്കിയിട്ടുള്ളതാണ്. പ്രത്യക്ഷീകരണ(dominance)ത്തെ ദ്യോതിപ്പിക്കുന്ന കൃത്യമായ പദമാണിത്. പുരുഷസ്രവം പെണ്സ്രവത്തിനുമേല് പ്രത്യക്ഷീകരിക്കുമ്പോള് പിതൃസഹോദരങ്ങളോടും, പെണ്സ്രവമാണ് പ്രത്യക്ഷീകരി ക്കുന്നതെങ്കില് മാതൃസഹോദരങ്ങളോടുമായിരിക്കും കുഞ്ഞിനു സാദൃശ്യമെന്നാണ് ഈ ഹദീഥ് പഠിപ്പിക്കുന്നത്. ഏതെങ്കിലുമൊരു സവി ശേഷതയുമായി ബന്ധപ്പെട്ട പെണ്സ്രവത്തിലെ ജീനാണ് പ്രത്യക്ഷമാവുന്നതെങ്കില് മാതൃസഹോദരങ്ങളിലാരുടെയെങ്കിലും സവിശേഷ തയാണ് കുഞ്ഞിനുണ്ടാവുകയെന്നും ആണ്സ്രവത്തിലെ ജീനാണ് പ്രത്യക്ഷമാവുന്നതെങ്കില് പിതൃസഹോദരങ്ങളില് ആരുടെയെങ്കിലും സവിശേഷതയാണ് കുഞ്ഞിനുണ്ടാവുകയെന്നുമുള്ള വസ്തുതകള് -ഇരുപതാം നൂറ്റാണ്ടിന്റെ തുടക്കത്തില് മാത്രം നാം മനസ്സിലാക്കിയ സത്യങ്ങള്- എത്ര കൃത്യമായാണ് ഈ ഹദീഥില് പ്രസ്താവിക്കുന്നത്!
ഹദീഥില് പിതൃസഹോദരങ്ങള് എന്നു പരിഭാഷപ്പെടുത്തിയിരിക്കുന്നത് 'അഅ്മാം' എന്ന പദത്തെയും മാതൃസഹോദരങ്ങള് എന്നു പരി ഭാഷപ്പെടുത്തിയിരിക്കുന്നത് 'അഖ്ലാല്' എന്ന പദത്തെയുമാണ്. 'അമ്മി'ന്റെ ബഹുവചനമാണ് 'അഅ്മാം'; 'ഖാലി'ന്റേത് 'അഖ്ലാലും'. പിതൃസഹോദരങ്ങളെ മൊത്തത്തില് അഅ്മാം എന്നും, മാതൃസഹോദരങ്ങളെ മൊത്തത്തില് അഖ്ലാല് എന്നും വിളിക്കുന്നു. പുരുഷ സ്രവം പെണ് സ്രവത്തെ അതിജയിച്ചാല് പിതാവിന്റെയും പെണ്സ്രവമാണ് അതിജയിക്കുന്നതെങ്കില് മാതാവിന്റെയും സാദൃശ്യമാണ് കുഞ്ഞിനുണ്ടാവുകയെന്നായിരുന്നു ഈ ഹദീഥിലുള്ളതെങ്കില് പാരമ്പര്യത്തെക്കുറിച്ച പുതിയ വിവരങ്ങളുമായി അത് വൈരുദ്ധ്യം പുലര്ത്തുന്നുവെന്ന് പറയാന് കഴിയുമായിരുന്നു; എന്നാല് സദൃശ്യപ്പെടാനുള്ള സാധ്യത പിതാവിലോ മാതാവിലോ പരിമിതപ്പെടുത്തു ന്നില്ല. ഏതെങ്കിലുമൊരു ജീനിന്റെ പ്രത്യക്ഷീകരണം നടക്കുമ്പോള് അത് പിതാവില് പ്രത്യക്ഷമായതു തന്നെയാകണമെന്നില്ലെന്നും പിതൃസ ഹോദരങ്ങളിലാരിലെങ്കിലും പ്രത്യക്ഷമായതാകാമെന്നുമാണല്ലോ ജനിതകം നമ്മെ പഠിപ്പിക്കുന്നത്. മാതൃസഹോദരങ്ങള്, പിതൃസഹോദ രങ്ങള് തുടങ്ങിയ ബഹുവചന പ്രയോഗങ്ങളിലൂടെ ഓരോ സവിശേഷതകളുടെയും ജീനുകള് പ്രത്യക്ഷീകരിക്കുന്നതിനുള്ള സാധ്യതകള് ഈ ഹദീഥില് തുറന്നിട്ടിരിക്കുകയാണ്. മാതൃശരീരത്തില് നിന്നുള്ള ജീനാണ് കുഞ്ഞില് പ്രത്യക്ഷമാകുന്നതെങ്കില് അതേ ജീന് മാതാവില് പ്രത്യ ക്ഷമല്ലെങ്കിലും മാതൃസഹോദരങ്ങളില് ആരിലെങ്കിലും പ്രത്യക്ഷമായിരിക്കുമെന്നും പിതാവില് നിന്നുള്ളതാണെങ്കില് പിതൃസഹോദരന് മാരിലാരിലെങ്കിലും അത് പ്രത്യക്ഷമായിരിക്കുമെന്നുമുള്ള ജനിതക ശാസ്ത്രം നമുക്ക് നല്കുന്ന അറിവുകള് എത്ര സമര്ത്ഥമായാണ് ഈ ഹദീഥിലെ പരാമര്ശങ്ങള്ക്കിടയില് ഒളിപ്പിച്ചുവെച്ചിരിക്കുന്നത്! പാരമ്പര്യത്തെക്കുറിച്ച പുതിയ വിവരങ്ങളുമായി പൂര്ണമായും പൊരുത്തപ്പെടുന്നതാണീ ഹദീഥ്. ഇതിലെ പദപ്രയോഗങ്ങളുടെ കൃത്യത ആരെയും അത്ഭുതപ്പെടുത്തുന്നതാണ്.
ലിംഗനിര്ണയവുമായി ബന്ധപ്പെട്ട ക്വുര്ആന് പരാമര്ശങ്ങള് ശ്രദ്ധിക്കുക.
''ആണ്, പെണ് എന്നീ രണ്ട് ഇണകളെ അവനാണ് സൃഷ്ടിച്ചതെന്നും. ഒരു ബീജം സ്രവിക്കപ്പെടുമ്പോള് അതില് നിന്ന്.'' (53: 45-46) (1)
''പിന്നെ അവന് ഒരു ഭ്രൂണമായി. എന്നിട്ട് അല്ലാഹു (അവനെ) സൃഷ്ടിച്ചു സംവിധാനിച്ചു. അങ്ങനെ അതില് നിന്ന് ആണും പെണ്ണുമാകുന്ന രണ്ടു ഇണകളെ അവന് ഉണ്ടാക്കി. അങ്ങനെയുള്ളവന് മരിച്ചവരെ ജീവിപ്പിക്കാന് കഴിവുള്ളവനല്ലെ?'' (75: 38-40)(2)
ഹദീഥുകളിലാണ് ലിംഗനിര്ണയത്തെപ്പറ്റി കുറേക്കൂടി വ്യക്തമായ പരാമര്ശമുള്ളത്.
- അനസില് നിന്ന്: പ്രവാചകന് മദീനയില് വന്ന വിവരം അബ്ദുല്ലാഹിബ്നു സലാമിനു കിട്ടി. അദ്ദേഹം നബിയുടെ അടുത്തുവന്ന് പറഞ്ഞു: 'ഒരു പ്രവാചകനു മാത്രം അറിയാവുന്ന മൂന്നു കാര്യങ്ങള് ഞാന് താങ്കളോട് ചോദിക്കുകയാണ്..... ഇനി കുട്ടിക്ക് സാദൃശ്യം ലഭിക്കുന്ന കാര്യം; പുരുഷന് സ്ത്രീയുമായി വേഴ്ച നടത്തുന്ന വേളയില് അവന്റെ സ്രവം അവളുടെ സ്രവത്തെ അതിജയിച്ചാല് കുട്ടിക്ക് സാദൃശ്യം അയാളോടായി. അവളുടെ സ്രവം അവന്റെ സ്രവത്തെയാണ് അതിജയിക്കുന്നതെങ്കില് അവളോടും.' അബ്ദുല്ല പറഞ്ഞു: 'താങ്കള് അല്ലാഹുവിന്റെ ദൂതനാണെന്നു ഞാന് സാക്ഷ്യപ്പെടുത്തുന്നു.'(3)
- അനസ് ബ്നുമാലികി(റ)ല് നിന്ന്: പുരുഷന് സ്വപ്നസ്ഖലനമുണ്ടാവുന്നതുപോലെ സ്ത്രീക്കും സ്വപ്നസ്ഖലനമുണ്ടായാല് അവള് എന്താണ് ചെേയ്യണ്ടത് എന്നതിനെ സംബന്ധിച്ച് ഉമ്മുസുലൈം പ്രവാചകനോട് ചോദിച്ചു. ...........നിശ്ചയമായും പുരുഷന്റെ‚ഇന്ദ്രിയം വെളുത്തതും കട്ടിയുള്ളതുമാണ്. സ്ത്രീയുടെ ഇന്ദ്രിയം മഞ്ഞനിറമുള്ളതും നേര്മയുള്ളതുമാണ്. ഏത് മുകളില് വരുന്നുവോ അല്ലെങ്കില് മുന്കടക്കുന്നുവോ അതിനോടാണ് കുട്ടിക്ക് സാദൃശ്യമുണ്ടാവുക.'(4)
- നബി (സ) സ്വാതന്ത്ര്യം നല്കിയ ഥൗബാനി(റ)ല് നിന്ന്: ഞാന് നബി(സ)യുടെ അടുക്കല് നില്ക്കുമ്പോള്ƒജൂത പണ്ഡിതന്മാരില് നിന്നുള്ള ഒരു പണ്ഡിതന് വരികയും 'അസ്സലാമു അലൈക്ക യാ മുഹമ്മദ് (മുഹമ്മദ്, നിനക്ക് സമാധാനമുണ്ടാകട്ടെ)' എന്ന് പറയുകയും ചെയ്തു. ............ അയാള് തുടര്ന്നു പറഞ്ഞു: 'ഭൂനിവാസികളില്നിന്നും ഒരു പ്രവാചകനോ അല്ലെങ്കില് ഒന്നോ രണ്ടോ ആളുകള്ക്കോ അല്ലാതെ മറ്റൊരാക്കും അറിയാത്ത ഒരു കാര്യത്തെ സംബന്ധിച്ച് ചോദിക്കുവാനാണ് ഞാന് വന്നിട്ടുള്ളത്.' നബി (സ) ചോദിച്ചു: 'ഞാനത് പറഞ്ഞാ ല് നിനക്കത് ഉപകരിക്കുമോ?'. 'ഞാന് എന്റ ചെവികള് കൊണ്ട് കേള്ക്കും'. അയാള് പറഞ്ഞു: '(പ്രസവിക്കപ്പെടുന്ന) ശിശുവിനെക്കു റിച്ച് ചോദിക്കുവാനാണ് ഞാന് വന്നത്' നബി (സ) പറഞ്ഞു: 'പുരുഷന്റെ‚ ഇന്ദ്രിയം വെളുത്ത നിറത്തിലുളളതും സ്ത്രീയുടെ ഇന്ദ്രിയം മഞ്ഞനിറത്തിലുള്ളതുമാണ്. അത് രണ്ടും ഒരുമിച്ച് ചേരുകയും പുരുഷ ഇന്ദ്രിയം സ്ത്രീ ഇന്ദ്രിയത്തെ അതിജയിക്കുകയും ചെയ്താല് അല്ലാഹുവിന്റെ അനുമതിയോടെ അത് ആണ് കുട്ടിയായിതീരുന്നു. സ്ത്രീയുടെ ഇന്ദ്രിയം പുരുഷ ഇന്ദ്രിയത്തെ അതിജയിച്ചാല് അല്ലാഹു വിന്റെ‚അനുമതിയോടെ അത് പെണ്കുട്ടിയായി തീരുന്നു.' ജൂതന് പറഞ്ഞു: 'തീര്ച്ചയായും താങ്കള് പറഞ്ഞത് സത്യമാണ്. തീര്ച്ചയായും താങ്കള് ഒരു പ്രവാചകന് തന്നെയാണ്'. പിന്നെ അയാള് തിരിച്ചുപോയി. അപ്പോള് നബി (സ) പറഞ്ഞു: 'അയാള് എന്നോടു ചോദിച്ച കാര്യങ്ങളെക്കുറിച്ചൊന്നും അല്ലാഹു അറിയിച്ചുതരുന്നതുവരെ എനിക്ക് യാതൊരു വിവരവും ഉണ്ടായിരുന്നില്ല.'(5)
- ഹുദൈഫത്ത് ബ്നുഅസീദി(റ)ണ് നിന്ന്: നബി (സ) പറഞ്ഞു: 'ഗര്ഭാശയത്തിണ് ബീജം നാല്പത് ദിവസം അല്ലെങ്കില് നാല്പത്തഞ്ച് ദിവസം ആയിത്തീരുമ്പോള് അതിന്മേല് ഒരു മലക്ക് പ്രവേശിക്കും. എന്നിട്ടവന് ചോദിക്കും: രക്ഷിതാവേ, ദൗര്ഭാഗ്യവാനോ അതോ സൗഭാഗ്യവാനോ? എന്നിട്ട് അത് രേഖപ്പെടുത്തും. പിന്നെ ചോദിക്കും: രക്ഷിതാവേ, ആണോ അതോ പെണ്ണോ? എന്നിട്ട് അതും രേഖപ്പെ ടുത്തും. അവന്റെ കര്മവും അവന്റെ‚ ഫലവും, അവന്റെ‚അവധിയും, അവന്റെ‚ ഉപജീവനവും എഴുതപ്പെടും. പിന്നീട് ഏടുകള് ചുരുട്ടപ്പെടും. അതില് ഒന്നും വര്ദ്ധിപ്പിക്കപ്പെടുകയില്ല; ഒന്നും ചുരുട്ടപ്പെടുകയുമില്ല.'(6)
- അബ്ദാഹി ബ്നുമസ്ഊദി(റ)ല് നിന്ന്: നബി (സ) പറയുന്നത് ഞാന് കേട്ടു: 'ബീജത്തിന്മേല് നാല്പത്തിരണ്ട് ദിവസം കഴിഞ്ഞാല് അല്ലാഹു ഒരു മലക്കിനെ നിയോഗിക്കും. എന്നിട്ട് അവന് അതിനെ രൂപപ്പെടുത്തുകയും, അതിന് കേള്വിയും കാഴ്ചയും ചര്മവും മാംസവും അസ്ഥിയും രൂപപ്പൈടുത്തുകയും ചെയ്യും. പിന്നീട് ആ മലക്ക് ചോദിക്കും: രക്ഷിതാവേ, ആണോ അതോ പെണ്ണോ? അപ്പോള് നിന്റെ രക്ഷിതാവ് അവന് ഉദ്ദേശിക്കുന്നത് വിധിക്കും. മലക്ക് അത് രേഖപ്പെടുത്തും. പിന്നീട് മലക്ക് ചോദിക്കും: രക്ഷിതാവേ ഇവന്റെ അവധി? അപ്പോള് നിന്റെ രക്ഷിതാവ് അവന് ഉദ്ദേശിച്ചത് പറയുകയും മലക്ക് അത് രേഖപ്പെടുത്തുകയും ചെയ്യും. പിന്നെ മലക്ക് ചോദി ക്കും: രക്ഷിതാവേ, ഇവന്റെ ഉപജീവനം? അപ്പോള് നിന്റെ രക്ഷിതാവ് അവന് ഉദ്ദേശിച്ചത് വിധിക്കുകയും മലക്ക് അത് രേഖപ്പെടുത്തു കയും ചെയ്യും. പിന്നീട് മലക്ക് തന്റെ‚കയ്യില് ആ ഏടുമായി പോകും. കല്പിക്കപ്പെട്ടതിനേക്കാള് വര്ദ്ധിപ്പിക്കുകയോ ചുരുക്കുകയോ ഇല്ല.'(7)
- അനസ് ബ്നുമാലികില് (റ) നിന്ന്: നബി (സ) പറഞ്ഞു: 'പ്രതാപവാനും മഹാനുമായ അല്ലാഹു ഗര്ഭാശയത്തിന്റെ കാര്യം ഒരു മലക്കിനെ ഏല്പിച്ചിട്ടുണ്ട്. ആ മലക്ക് പറയും: രക്ഷിതാവേ, ബീജമാണ്. രക്ഷിതാവേ സിക്താണ്ഡമാണ്. രക്ഷിതാവേ മാംസപിണ്ഡമാണ്. അല്ലാഹു ഒരു സൃഷ്ടിയില് വിധിക്കാന് ഉദ്ദേശിച്ചാല് മലക്ക് പറയും: രക്ഷിതാവേ, ആണോ പെണ്ണോ? ദൗര്ഭാഗ്യവാനോ അതോ സൗഭാഗ്യവാനോ? ഉപജീവനം എങ്ങനെയാണ്? അവധി എത്രയാണ്? അങ്ങനെ അവയെല്ലാം തന്റെ മാതാവിന്റെ വയറ്റിലായിരിക്കെ തന്നെ രേഖപ്പെടുത്ത പ്പെടും.(8)
- (നബി(സ)യോട് ചോദിക്കപ്പെട്ടു:) സ്വപ്നസ്ഖലനമുണ്ടായാല് സ്ത്രീ കുളിക്കേണ്ടതുണ്ടോ? അപ്പോള് നബി(സ) പറഞ്ഞു: 'അതെ; അവള് ഇന്ദ്രിയം കണ്ടാല്'. അപ്പോള് ഉമ്മുസുലൈം (റ) ചോദിച്ചു: 'സ്ത്രീക്ക് സ്ഖലനമുണ്ടാകുമോ?' അപ്പോള് അദ്ദേഹം പറഞ്ഞു: 'എന്തൊരു കഷ്ടം! പിന്നെ? എങ്ങനെയാണ് കുട്ടിക്ക് അവളോട് സാദൃശ്യം ഉണ്ടാകുന്നത്?' മറ്റൊരു നിവേദനത്തില് ആഇശ (റ) ഉമ്മുസുലൈം(റ)യോട് 'ഛെ! സ്ത്രീക്ക് അതുണ്ടാകുമോ?' എന്ന് ചോദിച്ചുവെന്നാണുള്ളത്. മറ്റൊരു റിപ്പോര്ട്ടില്, ഈ ഹദീഥിന്റെ‚അവസാന ഭാഗത്ത് ഇങ്ങനെ കൂടിയുണ്ട്. 'ഇന്ദ്രിയം കാരണമായിട്ടാണ് കുട്ടിക്ക് സാദൃശ്യമുണ്ടാകുന്നത്. സ്ത്രീയുടെ ഇന്ദ്രിയം പുരുഷന്റെ ഇന്ദ്രിയത്തിന് മുകളില് വന്നാല് കുട്ടിക്ക് മാതൃ സഹോദരന്മാരോട് സാദൃശ്യമുണ്ടാകും. പുരുഷന്റെ‚ ഇന്ദ്രിയം സ്ത്രീയുടെ ഇന്ദ്രിയത്തിന് മുകളില് വന്നാല് കുട്ടിക്ക് അവന്റെ പിതൃവ്യന്മാരോട് സാദൃശ്യമുണ്ടാകും.'(9)
മുകളില് പറഞ്ഞ അല്ലാഹുവിന്റെയും അവന്റെ ദൂതന്റെയും വചനങ്ങളിലൊന്നും തന്നെ ആശാസ്ത്രീയമായ പരാമര്ശങ്ങളൊന്നുമില്ല. ലിംഗനിര്ണയത്തെപ്പറ്റിയുള്ള ഏറ്റവും പുതിയ വിവരങ്ങളുമായിപ്പോലും അവ പൂര്ണമായും യോജിച്ചു വരുന്നുവെന്നത് അത്ഭുതകരം തന്നെയാണ്.
- സ്രവിക്കപ്പെടുന്ന ശുക്ലത്തില് നിന്നാണ് ആണും പെണ്ണുമുണ്ടാകുന്നതെന്ന് സൂറത്തുല് ഖിയാമയിലെ 38 മുതല് 40 വരെയുള്ള വചനങ്ങ ളില് പറയുന്നു. ശുക്ലദ്രാവകത്തിലെ പുരുഷബീജം X ക്രോമസോം വഹിക്കുന്നതാണെങ്കില് അത് അണ്ഡവുമായി ചേര്ന്നാല് പെണ്കു ഞ്ഞും Y ക്രോമസോം വഹിക്കുന്നതാണെങ്കില് അത് അണ്ഡവുമായി ചേര്ന്നാല് ആണ്കുഞ്ഞുമുണ്ടാകുന്നു. ശുക്ലദ്രാവകമാണ് കുഞ്ഞ് ആണോ പെണ്ണോ എന്നു തീരുമാനിക്കുന്നത് എന്നര്ത്ഥം.
- സ്രവിക്കപ്പെടുന്ന ബീജത്തില് തന്നെ ആണ്, പെണ് എന്നിവയുണ്ടെന്നും അതാണ് ആണ്-പെണ് ഇണകളുടെ ഉല്പത്തിക്ക് കാരണമാകുന്ന തെന്നും സൂറത്തുന്നജ്മിലെ 45,46 വചനങ്ങള് വ്യക്തമാക്കുന്നു. സ്രവിക്കപ്പെടുന്ന ബീജത്തില് തന്നെ ആണ് ക്രോമസോമായ Yയെ വഹി ക്കുന്ന ബീജാണുക്കളും പെണ്ക്രോമസോമായ Xനെ വഹിക്കുന്ന ബീജാണുക്കുളുമുണ്ട്. ബീജദ്രാവകത്തിലെ Y ആണ്ബീജം അണ്ഡവു മായി ചേര്ന്നാല് ആണ്കുട്ടിയും X പെണ്ബീജമാണ് അണ്ഡവുമായി ചേരുന്നതെങ്കില് പെണ്കുട്ടിയുമാണുണ്ടാവുക.
- അനസില് നിന്ന് ബുഖാരി നിവേദനം ചെയ്ത അബ്ദുല്ലാഹിബ്നു സലാമുമായി പ്രവാചകന് (സ) നടത്തിയ സംഭാഷണത്തെക്കുറിച്ച് വിവരിക്കുന്ന ഹദീഥിലും അദ്ദേഹത്തില് നിന്നുതന്നെ മുസ്ലിം നിവേദനം ചെയ്ത സ്വപ്നസ്ഖലനത്തെക്കുറിച്ച ഹദീഥിലും ഥൗബാ നി(റ)ല് നിന്ന് മുസ്ലിം നിവേദനം ചെയ്ത ജൂതപുരോഹിതനു നല്കിയ മറുപടികയെക്കുറിച്ച് വിശദീകരിക്കുന്ന ഹദീഥിലും പുരുഷ ബീജം സ്ത്രീബീജത്തെ അതിജയിച്ചാല് ആണ്കുഞ്ഞും, സ്ത്രീബീജം പുരുഷബീജത്തെയാണ് അതിജയിക്കുന്നതെങ്കില് പെണ്കുട്ടിയുമാ ണുണ്ടാവുകയെന്ന് പ്രവാചകന് (സ) പറഞ്ഞതായി ഉദ്ധരിച്ചിരിക്കുന്നു. ഈ പരാമര്ശത്തെ സുരതക്രിയയില് പുരുഷനാണ് ആദ്യം സ്ഖലിക്കുന്നതെങ്കില് ആണ്കുട്ടിയും സ്ത്രീക്കാണ് ആദ്യം സ്ഖലിക്കുകയെങ്കില് പെണ്കുട്ടിയുമാണുണ്ടാവുകയെന്നാണ് പല പണ്ഡിത ന്മാരും വ്യാഖ്യാനിച്ചിരിക്കുന്നത്. രതിമൂര്ച്ചയോടനുബന്ധിച്ച് ചില സ്ത്രീകള്ക്ക് പാരായൂറിത്രല് നാളിയില് നിന്ന് പുറത്തേക്കുവ രുന്ന ദ്രാവകത്തിന് കുഞ്ഞിന്റെ ജനനത്തില് യാതൊരു പങ്കുമില്ല എന്ന് ഇന്ന് നമുക്കറിയാം. പെണ്ണിന്റെ സ്ഖലനത്തിന് കുഞ്ഞിന്റെ ഉല്പത്തി പ്രക്രിയയില് യാതൊരു പങ്കും വഹിക്കുവാനില്ലെങ്കില് അതോടനുബന്ധിച്ചുണ്ടാകുന്ന ദ്രാവകം ആദ്യമോ പിന്നെയോ ഉണ്ടാകുന്നതെന്നത് ലിംഗനിര്ണയത്തെ ബാധിക്കുവാന് സാധ്യതയൊന്നുമില്ല. ഈ ഹദീഥുകളില് ബീജത്തിന്റെ അധീശത്വത്തെക്കുറിക്കു വാന് പ്രയോഗിച്ചിരിക്കുന്നത് 'സബഖ'യെന്നും 'അലാ' എന്നുമുള്ള ക്രിയകളാണ്. ഒന്നിനുമേല് മറ്റൊന്ന് മുന്കടക്കുന്നതിനോ ആദ്യമാകു ന്നതിനോ വിജയിക്കുന്നതിനോ അധികാരം സ്ഥാപിക്കുന്നതിനോ ആണ് 'സബഖ'യെന്നു പറയുകയെന്ന് അംഗീകൃത ഭാഷാ നിഘണ്ടുക്കള് പരിശോധിച്ചാല് വ്യക്തമാകും(10).
ഒന്നിനുമുകളില് മറ്റൊന്ന് ആധിപത്യം സ്ഥാപിക്കുന്നതിനാണ് 'അലാ'യെന്ന് പ്രയോഗിക്കുകയെന്ന് ക്വുര്ആനില്നിന്നു തന്നെ വ്യക്തമാകു ന്നുണ്ട്. സൂറത്തുല് മുഅ്മിനൂനിലെ 91-ാം വചനം നോക്കുക.
''അല്ലാഹു യാതൊരു സന്താനത്തെയും സ്വീകരിച്ചിട്ടില്ല. അവനോടൊപ്പം യാതൊരു ദൈവവുമുണ്ടായിട്ടില്ല. അങ്ങനെയായിരുന്നുവെങ്കില് ഓരോ ദൈവവും താന് സൃഷ്ടിച്ചതുമായി പോയിക്കളയുകയും, അവരില് ചിലര് ചിലരെ അടിച്ചമര്ത്തുകയും ചെയ്യുമായിരുന്നു. അവര് പറഞ്ഞുണ്ടാക്കുന്നതില് നിന്നെല്ലാം അല്ലാഹു എത്ര പരിശുദ്ധന്!'' (23: 91)(11)
ഈ വചനത്തില് 'ചിലര് ചിലരെ അടിച്ചമര്ത്തുകയും ചെയ്യുമായിരുന്നു'വെന്നു പരിഭാഷപ്പെടുത്തിയിരിക്കുന്നത് 'വ ലഅലാ ബഅദുഹും അലാ ബഅദിന്' എന്ന പ്രയോഗത്തെയാണ്. 'അലാ'യെന്നാല് ആധിപത്യം സ്ഥാപിക്കുക, അടിച്ചമര്ത്തുക എന്നിങ്ങനെയാണ് യഥാര്ത്ഥത്തിലുള്ള സാരമെന്നര്ത്ഥം.
പുരുഷബീജത്തിലെ Y പെണ്ബീജത്തിലെ Xനുമേല് ആധിപത്യം സ്ഥാപിക്കുമ്പോഴാണ് ആണ്കുഞ്ഞുണ്ടാകുന്നത് എന്നും പെണ്ബീജത്തിലെ X പുരുഷബീജത്തിലെ Yക്കുമേല് ആധിപത്യം സ്ഥാപിക്കുമ്പോഴാണ് പെണ്കുഞ്ഞുണ്ടാകുന്നത് എന്നുമുള്ള ജനിതകശാസ്ത്ര വസ്തുതകളു മായി ഈ ഹദീഥുകള് പൂര്ണമായും പൊരുത്തപ്പെടുന്നു. മധ്യകാലഘട്ടത്തിലുണ്ടായിരുന്ന അറിവിന്റെ അടിസ്ഥാനത്തില് ഹദീഥ് മനസ്സിലാക്കിയവര് ആണ്സ്ഖലനം ആദ്യം നടന്നാല് ആണ്കുഞ്ഞും പെണ്സ്ഖലനം നടന്നാല് പെണ്കുഞ്ഞുമുണ്ടാകുമെന്ന് ഇതില്നിന്ന് മനസ്സിലാക്കിയെന്നത് നബിവചനത്തിന്റെ ആശാസ്ത്രീയതയല്ല, അറിവിന്റെ കാലനിബന്ധതയെയാണ് വെളിപ്പെടുത്തുന്നത്. 'സബഖ' യെന്ന ക്രിയയെ വ്യാഖ്യാനിച്ചാല് ആദ്യമുണ്ടാകുന്നത് ഏത് ദ്രവമാണോ അതിന്റെ അടിസ്ഥാനത്തിലാണ് ലിംഗനിര്ണയമെന്ന് വേണമെ ങ്കില് പറയാനാകുമെങ്കിലും 'അലാ'യെന്ന പ്രയോഗം അത്തരമൊരു വ്യാഖ്യാനത്തിന് പഴുതുകളൊന്നും നല്കുന്നില്ല. ഈ ഹദീഥുകളെ ഒന്നിച്ചു പരിഗണിച്ചുകൊണ്ട്, നിലനില്ക്കുന്ന അറിവിന്റെ അടിസ്ഥാനത്തില് വ്യാഖ്യാനിച്ചപ്പോഴാണ് പുരുഷ-പെണ് സ്ഖലനങ്ങളുടെ ക്രമമാണ് ലിംഗനിര്ണയത്തിന് നിദാനമെന്നാണ് ഈ ഹദീഥുകള് പഠിപ്പിക്കുന്നതെന്ന നിഗമനത്തില് വ്യാഖ്യാതാക്കള് എത്തിച്ചേര്ന്നത്. ഹദീഥുകളെ മൊത്തത്തിലെടുത്ത് പരിശോധിച്ചാല് ഒരു ദ്രവത്തിനു മേലുള്ള മറ്റേ ദ്രവത്തിന്റെ ആധിപത്യം തന്നെയാണ് അവയില് വ്യക്തമാക്കപ്പെട്ടിരിക്കുന്നതെന്ന് മനസ്സിലാകും. കഴിഞ്ഞ നൂറ്റാണ്ടിന്റെ പകുതിയില് മാത്രം ശാസ്ത്രലോകത്തിന് മനസ്സിലായ ബീജത്തി ന്റെ ആധിപത്യമാണ് ലിംഗനിര്ണയത്തിന് കാരണമാകുന്നതെന്ന വസ്തുത എത്ര കൃത്യമായാണ് ഈ ഹദീഥുകള് വരച്ച് കാണിക്കുന്നത്!
- മുസ്ലിം ഹുദൈഫത്തു ബ്നു അസീദില് (റ) നിന്നും അബ്ദുല്ലാഹിബ്നു മസ്ഊദില് (റ) നിന്നും നിവേദനം ചെയ്ത രണ്ട് വ്യത്യസ്ത ഹദീഥുകളില് നിന്ന് ഗര്ഭസ്ഥശിശുവിലുളള ലിംഗമാറ്റത്തിനുവേണ്ടിയുള്ള മലക്ക് പ്രത്യക്ഷപ്പെടുന്നതും കുട്ടി ആണോ പെണ്ണോയെന്ന് ആത്യന്തികമായി തീരുമാനിക്കപ്പെടുന്നതും ബീജസങ്കലനത്തിന് ശേഷം നാല്പത് ദിവസങ്ങള്ക്കും നാല്പത്തിയഞ്ച് ദിവസങ്ങള്ക്കുമിടയിലാണെന്ന് വ്യക്തമാവുന്നു.
SRY ജീന് പ്രവര്ത്തനക്ഷമമാകുന്നത് ആറാമത്തെ ആഴ്ചയാണെന്ന വിവരം നമുക്ക് ലഭിച്ചത് മൂന്നു പതിറ്റാണ്ടുകള്ക്ക് മുമ്പു മാത്രമാണ്. XX സിക്താണ്ഡമാണെങ്കിലും XY സിക്താണ്ഡമാണെങ്കിലും അപൂര്വമായുണ്ടാകുന്ന സിക്താണ്ഡങ്ങളാണെങ്കിലുമെല്ലാം അവയുടെ ലിംഗമെന്താ ണെന്ന് ആത്യന്തികമായി തീരുമാനിക്കപ്പെടുക SRY ജീന് പ്രവര്ത്തനക്ഷമമാകുന്നതിന്റെ അടിസ്ഥാനത്തിലാണ്. ആറാമത്തെ ആഴ്ചയാണ് SRY ജീന് പ്രവര്ത്തനക്ഷമമാവുന്നതെന്ന ഭ്രൂണശാസ്ത്രം 1985ല് മാത്രം നമുക്കു പറഞ്ഞുതന്ന വിവരവും നാല്പതു ദിവസങ്ങള്ക്കും നല്പത്തിയഞ്ച് ദിവസങ്ങള്ക്കുമിടയിലാണ് ലിംഗതീരുമാനവുമായി മലക്ക് നിയോഗിക്കപ്പെടുന്നതെന്ന പതിനാലു നൂറ്റാണ്ടുകള്ക്ക് മുമ്പ് നബി (സ) പറഞ്ഞ വിവരവും എത്ര ക്രൃത്യമായാണ് ഇവിടെ യോജിച്ചുവരുന്നത്! എന്തുകൊണ്ടാണ് ഹദീഥുകളിലെ പരാമര്ശങ്ങള് ഇത്ര യും കൃത്യമാകുന്നതെന്ന ചോദ്യത്തിന് ക്വുര്ആന് തന്നെ ഉത്തരം നല്കിയിട്ടുണ്ട്.
''നിങ്ങളുടെ കൂട്ടുകാരന് വഴിതെറ്റിയിട്ടില്ല. ദുര്മാര്ഗിയായിട്ടുമില്ല. അദ്ദേഹം തന്നിഷ്ടപ്രകാരം സംസാരിക്കുന്നുമില്ല. അത് അദ്ദേഹത്തിന് ദിവ്യസമ്പേശമായി നല്കപ്പെടുന്ന ഒരു ഉല്ബോധനം മാത്രമാകുന്നു.'' (53: 2-4)
കുറിപ്പുകൾ
- ക്വുര്ആന് 53: 45-46
- ക്വുര്ആന് 75: 38-40
- സ്വഹീഹുല് ബുഖാരി, കിതാബു അഹാദീഥുല് അംബിയാഅ്, ബാബു ഖല്ഖി ആദം വ ദുര്റിയ്യത്തിഹി, ഹദീഥ്
- സ്വഹീഹു മുസ്ലിം, കിതാബുല് ഹൈദ്വ്, ബാബു വുജുബില് ഗസ്ലി അലല് മര്അത്തി ബി ഖുറൂജില് മനിയ്യി മിന്ഹ, ഹദീഥ്
- സ്വഹീഹു മുസ്ലിം, കിതാബുല് ഹൈദ്വ്, ബാബു ബയാനി സ്വിഫത്തില് മനിയിര്റജുലി വല് മര്അത്തി വ അന്നല് വലദ മഖ്ലൂഖുന് മിന് മാഇ.
- സ്വഹീഹു മുസ്ലിം, കിതാബുല് ക്വദ്ര്, ബാബു കൈഫിയ്യത്തില് ഖല്ബില് ആദമിയ്യി ഫീ ബദനി ഉമ്മിഹി വ കിതാബത്തി രിസ്കിഹി വ അജലിഹി, വ അമലിഹി വ ശകാവത്തിഹി വ സഅദത്തിഹി, ഹദീഥ്
- സ്വഹീഹു മുസ്ലിം, കിതാബുല് ക്വദ്ര്, ബാബു കൈഫിയ്യത്തില് ഖല്ബില് ആദമിയ്യി ഫീ ബദനി ഉമ്മിഹി വ കിതാബത്തി രിസ്കിഹി വ അജലിഹി, വ അമലിഹി വ ശകാവത്തിഹി വ സഅദത്തിഹി, ഹദീഥ്
- സ്വഹീഹു മുസ്ലിം, കിതാബുല് ക്വദ്ര്, ബാബു കൈഫിയ്യത്തില് ഖല്ബില് ആദമിയ്യി ഫീ ബദനി ഉമ്മിഹി വ കിതാബത്തി രിസ്കിഹി വ അജലിഹി, വ അമലിഹി വ ശകാവത്തിഹി വ സഅദത്തിഹി, ഹദീഥ്
- സ്വഹീഹു മുസ്ലിം, കിതാബുല് ഹൈദ്വ്, ബാബു വുജുബില് ഗസ്ലി അലല് മര്അത്തി ബി ഖുറൂജില് മനിയ്യി മിന്ഹ, ഹദീഥ്
- Edward William Lane : Arabic-English Lexicon, London, 1863, Book 1, Page 1300.
- ക്വുര്ആന് 23: 91
ഭ്രൂണവളര്ച്ചയുടെ ആദ്യത്തെ നാല്പതു ദിവസങ്ങള് തികച്ചും ജൈവികവും യാന്ത്രികവുമായ മൂന്നു ഘട്ടങ്ങളുടേതാണെന്നും അതിനുശേ ഷമാണ് അല്ലാഹുവിന്റെ അനുഗ്രഹങ്ങളുമായി മലക്കുകളുടെ നിയോഗമുണ്ടാവുകയും വൈയക്തികമായ സവിശേഷതകള് തീരുമാനിക്ക പ്പെടുകയും ചെയ്യുന്നതെന്ന് വ്യക്തമാക്കുന്ന നിരവധി ഹദീഥുകളുണ്ട്.
- അബ്ദുല്ലാഹിബ്നു മസ്ഊദില് (റ) നിന്ന്: സത്യസന്ധനും വിശ്വസ്തനുമായ അല്ലാഹുവിന്റെ ദൂതന് ഞങ്ങളെ അറിയിച്ചു: നിങ്ങളിലുള്ള ഒരാളുടെ സൃഷ്ടികര്മം തന്റെ മാതാവിന്റെ ഉദരത്തില് സംയോജിക്കുന്നത് നാല്പതു ദിവസങ്ങളിലായാണ്. പിന്നെ, അതേപോലെത്ത ന്നെ അത് അലഖയായിത്തീരുന്നു. അതേപോലെ പിന്നീടത് മുദ്വ്അയായി മാറുന്നു. അനന്തരം അല്ലാഹു ഒരു മലക്കിനെ അയക്കുകയും നാലുകാര്യങ്ങള് രേഖപ്പെടുത്താന് കല്പിക്കുകയും ചെയ്യുന്നു. അവന്റെ കര്മവും ആഹാരവും ആയുസ്സും സൗഭാഗ്യവാനോ നിര്ഭാഗ്യവാനോ എന്നീ കാര്യങ്ങള്. ശേഷം അവനിലേക്ക് ആത്മാവ് ഊതപ്പെടുന്നു.(1)
- ഹുദൈഫത്തുബ്നു ഉസൈദ് (റ)യില് നിന്ന്: നുത്വ്ഫ ഗര്ഭാശയത്തിലെത്തി നാല്പതോ നാല്പത്തിയഞ്ചോ ദിവസങ്ങള് കഴിഞ്ഞാല് അതിന്മേല് ഒരു മലക്ക് എത്തുകയും ഇങ്ങനെ ചോദിക്കുകയും ചെയ്യുന്നു. 'നാഥാ, സൗഭാഗ്യവാനോ നിര്ഭാഗ്യവാനോ?' അത് രേഖപ്പെ ടുത്തുന്നു. അവന്റെ കര്മങ്ങളും പ്രവൃത്തികളും അന്ത്യവും ആഹാരവുമെല്ലാം രേഖപ്പെടുത്തുന്നു. പിന്നെ ആ രേഖ ചുരുട്ടപ്പെടുന്നു. ശേഷം അതില് കൂട്ടിച്ചര്ക്കലുകളോ കിഴിക്കലുകളോ ഇല്ല.(2)
- അബ്ദുല്ലാഹിബ്നു മസ്ഊദില് നിന്ന്: അല്ലാഹുവിന്റെ ദൂതന് ഇങ്ങനെ പറയുന്നത് താന് കേട്ടതായി ഹുദൈഫത്തുബ്നു ഉസൈദുല് ഗിഫാരി (റ) പറഞ്ഞു: നുത്വ്ഫയെത്തി നാല്പത്തിരണ്ടു ദിവസങ്ങള് കഴിഞ്ഞാല് അല്ലാഹു ഒരു മലക്കിനെ അയക്കുകയും അവന് ആകൃതി നല്കുകയും ചെയ്യുന്നു. ശേഷം അവന് കേള്വിശക്തിയും കാഴ്ചശക്തിയും ത്വക്കും മാംസപേശികളും അസ്ഥികളുമെല്ലാം നല്കുന്നു. അങ്ങനെ മലക്ക് ചോദിക്കുന്നു: നാഥാ, പുരുഷനോ സ്ത്രീയോ? നിന്റെ നാഥന് ഉദ്ദേശിക്കുന്നത് തീരുമാനിക്കുകയും മലക്ക് അത് രേഖപ്പെടുത്തുകയും ചെയ്യുന്നു. പിന്നെ മലക്ക് ചോദിക്കുന്നു: നാഥാ, അവന്റെ അന്ത്യമെന്നാണ്? നിന്റെ നാഥന് ഉദ്ദേശിക്കുന്നത് പറയുകയും അത് രേഖപ്പെടുത്തുകയും ചെയ്യുന്നു. പിന്നെ മലക്ക് ചോദിക്കുന്നു: അവന്റെ ഉപജീവനമെങ്ങനെയാണ്? നിന്റെ നാഥന് ഇച്ഛിക്കുന്ന പോലെ തീരുമാനിക്കുകയും മലക്ക് അത് രേഖപ്പെടുത്തുകയും ചെയ്യുന്നു. ശേഷം മലക്ക് തന്റെ കയ്യില് ചുരുട്ടിയ രേഖയു മായി പോകുന്നു. പിന്നെ അതില് കൂട്ടിച്ചേര്ക്കലുകളോ കിഴിക്കലുകളോ ഇല്ല.(3)
- അബൂ തുഫൈലി(റ)ല് നിന്ന്: ഞാന് ഹുദൈഫത്തുബ്നു ഉസൈദ് അല് ഗിഫാരി(റ)യെ സന്ദര്ശിച്ചപ്പോള് അദ്ദേഹം പറഞ്ഞു: അല്ലാഹുവി ന്റെ ദൂതന് (സ) ഇങ്ങനെ പറയുന്നതായി ഞാനെന്റെ രണ്ടു ചെവികള് കൊണ്ടു കേട്ടിട്ടുണ്ട്. നുത്വ്ഫ ഗര്ഭാശയത്തിലെത്തി നാല്പതു രാവുകള് കഴിഞ്ഞാല് പിന്നെ ഒരു മലക്കെത്തി അതിന് രൂപം നല്കും. സുഹൈര് (റ) പറഞ്ഞു: അദ്ദേഹം ഇങ്ങനെകൂടി പറഞ്ഞുവെന്ന് ഞാന് കരുതുന്നു: കണക്കാക്കുകയും രൂപപ്പെടുത്തുകയും ചെയ്യുന്നവനോട് അവന് ചോദിക്കും: നാഥാ, സ്ത്രീയോ പുരുഷനോ? അങ്ങനെ അല്ലാഹു അതിനെ ആണോ പെണ്ണോ ആക്കിത്തീര്ക്കുന്നു. പിന്നെ അവന് ചോദിക്കുന്നു: നാഥാ, വൈകല്യമുള്ളവനോ ഇല്ലാത്ത വനോ? അങ്ങനെ അല്ലാഹു അവനെ പൂര്ണനോ വൈകല്യമുള്ളവനോ ആക്കുന്നു. പിന്നെ അവന് ചോദിക്കുന്നു: നാഥാ, അവന്റെ ഉപജീ വനമെന്താണ്? അന്ത്യമെന്നാണ്? പ്രകൃതമെന്താണ്? അങ്ങനെ അവന് സൗഭാഗ്യവാനോ നിര്ഭാഗ്യവാനോയെന്ന് അല്ലാഹു തീരുമാനി ക്കുന്നു.(4)
നുത്വ്ഫയുണ്ടായി നാല്പതുരാവുകള്ക്കു ശേഷമാണ് ഒരാളുടെ അവയവങ്ങള് രൂപീകരിക്കപ്പെടുകയും വ്യക്തിത്വം തീരുമാനിക്കപ്പെ ടുകയും ചെയ്യുന്നതെന്ന് വ്യക്തമാക്കുന്നതാണ് ഈ ഹദീഥുകള്. നാല്പത് ദിവസങ്ങള് കഴിഞ്ഞശേഷം സംഭവിക്കുന്നതായി ഈ ഹദീഥുകള് വ്യക്തമാക്കുന്ന കാര്യങ്ങള് താഴെ പറയുന്നവയാണ്:-
- ലിംഗ നിര്ണയം
- വ്യക്തിത്വ നിര്ണയം
- അവയവ രൂപീകരണം
- വൈകല്യങ്ങളുണ്ടെങ്കില് അവയുടെ പ്രകടനം
- വിധി നിര്ണയത്തിന്റെ രേഖീകരണം
മുദ്വ്അഃയെന്നു ക്വുര്ആന് വിളിക്കുന്ന ചര്വ്വിതമാംസത്തിന്റെ പൂര്ണരൂപം പ്രാപിക്കുന്ന ആറാമത്തെ ആഴ്ചയുടെ അവസാനത്തിലുള്ള ഭ്രൂണത്തെയും അതിന്റെ രൂപത്തെയും അതിലുള്ള അവയവാടയാളങ്ങളെയും മുകുളങ്ങളെയും കുറിച്ചെല്ലാം നാം മനസ്സിലാക്കിക്കഴിഞ്ഞ താണ്. ആറാം ആഴ്ചക്കു മുമ്പുനടക്കുന്ന മൂന്ന് ഘട്ടങ്ങളും ക്രമപ്രവൃദ്ധമായി സംഭവിക്കുന്നതാണെന്നും നാം കണ്ടു. ഈ ഘട്ടങ്ങളുടെ നൈര ന്തര്യം സൂചിപ്പിക്കുന്ന ഒരു ഹദീഥുമുണ്ട്.
അനസുബ്നു മാലിക് (റ)യില് നിന്ന്: അത്യുന്നതനായ അല്ലാഹു ഗര്ഭാശയത്തിന്റെ സംരക്ഷകനായി ഒരു മലക്കിനെ നിയോഗിക്കും. അപ്പോള് മലക്ക് പറയും: നാഥാ, നുത്വ്ഫ; നാഥാ, അലഖഃ; മുദ്വ്അഃ; അതിന്റെ സൃഷ്ടി പൂര്ത്തിയാക്കുവാന് അല്ലാഹു ഉദ്ദേശിച്ചിട്ടുണ്ടെ ങ്കില് മലക്ക് ചോദിക്കും: നാഥാ, ആണോ അതല്ല പെണ്ണോ? സൗഭാഗ്യവാനോ അതല്ല നിര്ഭാഗ്യവാനോ? ഉപജീവനമെന്താണ്? അന്ത്യമെങ്ങനെയാണ്? മാതാവിന്റെ ഗര്ഭാശയത്തില് വെച്ചുതന്നെ അതെല്ലാം എഴുതപ്പെടും.(5)
ഈ ഹദീഥുകളില് നിന്നെല്ലാം തന്നെ നുത്വ്ഫ, അലഖഃ; മുദ്വ്അഃ എന്നിങ്ങനെ ക്വുര്ആന് പരാമര്്വശിച്ച മൂന്ന് ഭ്രൂണഘട്ടങ്ങളും നടക്കു ന്നത് ആദ്യത്തെ ആറ് ആഴ്ചകളിലാണെന്നു വ്യക്തമാണ്. എന്നാല് മുകളില് ആദ്യമായി ഉദ്ധരിച്ചിരിക്കുന്ന ബുഖാരിയും മുസ്ലിമും അബ്ദുല്ലാഹിബ്നു മസ്ഊദില് (റ) നിന്ന് നിവേദനം ചെയ്തിരിക്കുന്ന ഹദീഥില് നിന്ന് ഭ്രൂണത്തിന്റെ നുത്വ്ഫ ഘട്ടവും അലഖ ഘട്ടവും മുദ്വ്അ ഘട്ടവും നാല്പത് ദിവസം വീതം ദിവസങ്ങളുള്ക്കൊള്ളുന്നതാണെന്ന ഒരു ധാരണ ആദ്യകാല ഹദീഥ് പണ്ഡിതന്മാര് മുതല് തന്നെ വെച്ചുപുലര്ത്തിയിരുന്നതായി നമുക്ക് കാണാന് കഴിയും. പ്രസ്തുത ധാരണ എടുത്തുദ്ധരിച്ചുകൊണ്ട് ക്വുര്ആനിലും ഹദീഥുകളിലും പ്രതിപാദിച്ചിട്ടുള്ള നുത്വ്ഫ, അലഖ, മുദ്വ്അ പരാമര്ശങ്ങളെല്ലാം അശാസ്ത്രീയവും അബദ്ധജഡിലവുമാണെന്ന് വിമര്ശിക്കുന്നവരുണ്ട്. ഭ്രൂണഘടനയെക്കുറിച്ച് പരാമര്ശിക്കുന്ന ക്വുര്ആന് വചനങ്ങളും മുഴുവന് ഹദീഥുകളും താരതമ്യം ചെയ്തുകൊണ്ട് പരിശോധിച്ചാല് ഈ വിമര്ശനങ്ങളിലൊന്നും യാതൊരുവിധ കഴമ്പുമില്ലെന്നു മനസ്സിലാകും. താഴെ പറയുന്ന വസ്തുതകള് ശ്രദ്ധിക്കുക.
- ഹദീഥുകള് പ്രവാചകനില് നിന്നുള്ളതാണെന്ന് ഉറപ്പാണെങ്കില് അതിലെ ആശയങ്ങള് അല്ലാഹുവില് നിന്നുള്ളതും അതുകൊണ്ടു തന്നെ അമാനുഷികവുമായിരിക്കും. എന്നാല് ക്വുര്ആനിലെ പദങ്ങള്ക്കുള്ളതുപോലെ ഹദീഥുകളുടെ പദങ്ങള്ക്ക് അമാനുഷികതയൊന്നുമില്ല. അല്ലാഹുവില് നിന്നുള്ള ബോധനത്തിന്റെ വെളിച്ചത്തില് പ്രവാചകന് (സ) പറഞ്ഞ കാര്യങ്ങള് അതുകേട്ട സ്വഹാബിമാര് അവരുടെ ഭാഷയില് ആവിഷ്കരിച്ചതാണ് ഹദീഥുകളിലെ പ്രവാചകവചനങ്ങള്. മാറ്റാന് പാടില്ലെന്ന് ദൈവദൂതരാല് നിഷ്കര്ഷിക്കപ്പെട്ട പ്രാര് ത്ഥനകളെപ്പോലെയുള്ളവ യാതൊരുവിധ മാറ്റവുമില്ലാതെ നിവേദനം ചെയ്ത സ്വഹാബിമാര് തന്നെ പ്രവാചകനില് (സ) നിന്ന് ലഭിച്ച ആശയങ്ങള് തങ്ങളുടേതായ ഭാഷയില് മറ്റുള്ളവര്ക്ക് പറഞ്ഞുകൊടുത്തതായി കാണാന് കഴിയും. പ്രവാചകന് ഉപയോഗിച്ച പദങ്ങ ള്ക്ക് അമാനുഷികതയില്ലെന്നും അതിലെ ആശയങ്ങള് മാത്രമാണ് ദൈവപ്രോക്തമെന്നുമായിരുന്നു അവര് മനസ്സിലാക്കിയിരുന്നതെന്ന് ഇതില് നിന്ന് വ്യക്തമാണ്. പ്രവാചകനില് (സ) നിന്നുകേട്ട ഒരേകാര്യം തന്നെ വ്യത്യസ്ത സ്വഹാബിമാര് വ്യത്യസ്ത പദങ്ങളും ശൈലിയു മുപയോഗിച്ച് അടുത്ത തലമുറക്ക് പറഞ്ഞുകൊടുത്തതിന് നിരവധി ഉദാഹരണങ്ങളുണ്ട്. ഹദീഥുകളില് ഉപയോഗിക്കപ്പെട്ടിരിക്കുന്ന പദങ്ങള് പലപ്പോഴും സ്വഹാബിമാരുടേതായിരിക്കുമെന്നും അവയുള്ക്കൊള്ളുന്ന ആശയം മാത്രമാണ് ദൈവികമെന്നുമാണ് ഇസ്ലാം പഠിപ്പിക്കുന്നതെന്ന വസ്തുത മനസ്സിലാക്കാതെയാണ് ഇതടക്കമുള്ള പല വിമര്ശനങ്ങളും ഉന്നയിക്കപ്പെടുന്നത്.
- നുത്വ്ഫ, അലഖ, മുദ്വ്അ തുടങ്ങിയ ഘട്ടങ്ങളിലോരോന്നും നാല്പത് ദിവസം വീതം ഉള്ക്കൊള്ളുന്നുവെന്ന് സ്വഹീഹുല് ബുഖാരി, സ്വഹീഹുമുസ്ലിം, സുനനു അബൂദാവൂദ്, ജാമിഉത്തിര്മിദി, സുനനു ഇബ്നു മാജ, മുസ്നദ് അഹ്മദ് തുടങ്ങിയ ഹദീഥ് ഗ്രന്ഥങ്ങളിലൊ ന്നും തന്നെയില്ല. പ്രസ്തുത ഗ്രന്ഥങ്ങളിലെല്ലാം അബ്ദുല്ലാഹിബ്നു മസ്ഊദില് (റ) നിന്ന് നിവേദനം ചെയ്യുന്ന ഹദീഥ് തുടങ്ങുന്നത് ഇങ്ങ നെയാണ്: 'നിങ്ങളിലുള്ള ഒരാളുടെ സൃഷ്ടികര്മം തന്റെ മാതാവിന്റെ ഉദരത്തില് സംയോജിക്കുന്നത് നാല്പതു ദിവസങ്ങളിലായാണ്. പിന്നെ അതേപോലെ അത് അലഖയാകുന്നു; പിന്നെ അതേപോലെ അത് മുദ്വ്അയാകുന്നു. ഇവിടെ നാല്പതു ദിവസത്തില് നടക്കുമെന്ന് പറഞ്ഞത് മാതൃശരീരത്തിലെ സംയോജനമാണ് (ജംഉല് ഖല്ഖ്); അത് നുത്വ്ഫയല്ല. ജംഉല് ഖല്ഖ് എന്നുപറഞ്ഞത് നുത്വ്ഫയെ ഉദ്ദേശി ച്ചുകൊണ്ടാണെന്ന തെറ്റിദ്ധാരണ കൊണ്ടാണ് ഈ ഹദീഥിന്റെ അടിസ്ഥാനത്തില് നുത്വ്ഫ, അലഖ, മുദ്വ്അ എന്നീ മൂന്നുഘട്ടങ്ങള്ക്കും നാല്പതുവീതം ദിവസങ്ങളാണെന്ന നിഗമനത്തില് ഹദീഥ് പണ്ഡിതന്മാരില് ചിലര് എത്തിച്ചേര്ന്നത്.
- സ്വഹീഹുല് ബുഖാരിയിലും സ്വഹീഹുല് മുസ്ലിമിലും നിരവധി തവണ ഉദ്ധരിക്കപ്പെട്ടതാണ് ഈ ഹദീഥ്. സ്വഹീഹുല് ബുഖാരിയില് തന്നെ കിതാബു ബദ്ഉല് ഖല്ഖ്, കിതാബു അഹാദീഥുല് അന്ബിയാഅ്, കിതാബുല് ഖദ്റ്, കിതാബുത്തൗഹീദ് എന്നീ നാല് അധ്യായങ്ങ ളില് ഈ ഹദീഥ് ഉദ്ധരിക്കപ്പെട്ടിട്ടുണ്ട്. സ്വഹീഹു മുസ്ലിമിലെ കിതാബുല് ഖദ്റില് ഉദ്ധരിക്കപ്പെട്ട ഇതേ ഹദീഥിന് ഇമാം നവവി നല്കു ന്ന വ്യാഖ്യാനത്തിലാണ് നാല്പത്, നാല്പത്തിരണ്ട്, നാല്പത്തിയഞ്ച് ദിവസങ്ങള് കഴിഞ്ഞാല് നുത്വ്ഫക്കുമേല് മലക്ക് വരുമെന്ന് പ്രസ്താവിക്കുന്ന നടേ ഉദ്ധരിക്കപ്പെട്ട രണ്ടു മുതല് നാലുവരെയുള്ള ഹദീഥുകളിലെ ആശയങ്ങളെയും ഈ ഹദീഥിലെ ആശയത്തെയും കൂട്ടിയോജിപ്പിച്ചുകൊണ്ട് ആദ്യത്തെ നാല്പതുദിവസത്തില് നടക്കുമെന്ന് പറഞ്ഞ ജംഉല് ഖല്ഖ് കൊണ്ടു വിവക്ഷിക്കുന്നത് നുത്വ്ഫ യാണെന്നും നാല്പതു ദിവസങ്ങള്കൂടി കഴിഞ്ഞ് അലഖയും വീണ്ടും നാല്പതു ദിവസങ്ങള്കൂടി കഴിഞ്ഞ് മുദ്വ്അയുമുണ്ടാവുകയാണ് ചെയ്യുന്നതെന്നും അതുകൊണ്ടുതന്നെ ആത്മാവ് ഊതുന്നത് നാല് മാസങ്ങള്ക്കുശേഷമാണ് എന്നുമുള്ള അഭിപ്രായങ്ങളുന്നയിക്കുന്നത്.(6) ഈ അഭിപ്രായം പ്രവാചകന്റേതായി നിവേദനം ചെയ്യപ്പെട്ടതല്ല എന്നതുകൊണ്ടുതന്നെ അത് നിര്ബന്ധമായും സ്വീകരിക്കേണ്ടതായി മുസ്ലിംകളാരും കരുതുന്നില്ല. ഗര്ഭാശയത്തിലെന്ത് നടക്കുന്നുവെന്നറിയാന് ശാസ്ത്രീയമായ മാര്ഗങ്ങളൊന്നുമില്ലാതിരുന്ന കാലത്ത് ഹദീഥിലെ പദങ്ങളുടെ വിവക്ഷയെക്കുറിച്ച് നടത്തിയ ഊഹങ്ങള് മാത്രമാണ് ഇമാം നവവിയുടേത്. പ്രസ്തുത ഊഹം തെറ്റാണെന്ന് ശാസ്ത്രീയമായ പഠനങ്ങളിലൂടെ മനസ്സിലായാല് അത് സ്വീകരിക്കുവാന് മുഹമ്മദ് നബി(സ)യെ പിന്പറ്റുന്ന മുസ്ലിംകള്ക്ക് യാതൊരു വൈമനസ്യവുമുണ്ടാവില്ല.
- ഭ്രൂണഘട്ടങ്ങളുടെ നാല്പത് ദിവസത്തെക്കുറിച്ച് പറയുന്ന മുകളിലെ ഹദീഥുകളും ഭ്രൂണപരിണാമത്തിന്റെ ഘട്ടങ്ങളെ പ്രതിപാദിക്കുന്ന ക്വുര്ആന് ആയത്തുകളും മുന്നില്വെച്ചുകൊണ്ട് പരിശോധിച്ചാല് നുത്വ്ഫ മുതല് മുദ്വ്അ വരെയുള്ള മൂന്നുഘട്ടങ്ങളും നാല്പതു ദിവസത്തിനകം പൂര്ണമാകുന്നതാണ് എന്നു മനസ്സിലാകും.
- മുദ്വ്അയില് നിന്നാണ് ഇദ്വാം അഥവാ അസ്ഥികള് ഉണ്ടാകുന്നതെന്നാണ് ക്വുര്ആന് പറയുന്നത് (23:14). നാല്പത്തിരണ്ട് ദിവസങ്ങള് ക്കുശേഷമാണ് ഭ്രൂണത്തിന് അസ്ഥിയുണ്ടാകുന്നതെന്ന് ഹുദൈഫ(റ)യില് നിന്ന് അബ്ദുല്ലാഹിബ്നു മസ്ഊദ് (റ) നിവേദനം ചെയ്തതായി സ്വഹീഹു മുസ്ലിമിലുള്ള ഹദീഥിലുണ്ട് (മുകളിലത്തെ മൂന്നാമത്തെ ഹദീഥ്). ഈ ഹദീഥ് മുഹദ്ദിസുകളായ അബൂദാവൂദും ത്വബ്റാനി യും തങ്ങളുടെ ഹദീഥ് ഗ്രന്ഥങ്ങളില് ഉദ്ധരിച്ചിട്ടുമുണ്ട്. നുത്വ്ഫ മുതല് മുദ്വ്അ വരെയുള്ള ഓരോ ഘട്ടങ്ങള്ക്കും നാല്പതു ദിവസങ്ങള് വീതമുണ്ടെങ്കില് ക്വുര്ആന് വചനപ്രകാരം നാലുമാസങ്ങള്ക്കുശേഷമാണ് അസ്ഥിയുണ്ടാവുക. ഈ ഹദീഥിലാണെങ്കില് നാല്പത്തി രണ്ടു രാത്രികള്ക്കുശേഷം അസ്ഥികളുണ്ടാവാന് തുടങ്ങുന്നുവെന്നാണുള്ളത്. നാല്പത്തിരണ്ടു ദിവസങ്ങള്ക്കു മുമ്പ് മുദ്വ്അയെന്ന ഘട്ടം കഴിഞ്ഞുപോയിയെന്നാണ് ഇതിനര്ത്ഥം. ഈ ഹദീഥും ആയത്തും യോജിപ്പിച്ചാല് ഇക്കാര്യം വ്യക്തമായി മനസ്സിലാകും.
- മുദ്വ്അയുണ്ടായതിനു ശേഷമാണ് അവയവ രൂപീകരണങ്ങളെല്ലാം നടക്കുന്നതെന്ന് മുകളിലെ ഹദീഥുകള് വ്യക്തമാക്കുന്നു. മുദ്വ്അയാ യിത്തീര്ന്ന് നാല്പതു ദിവസം കഴിഞ്ഞാണ് അവയവ രൂപീകരണം നടക്കുന്നതെന്നു പറഞ്ഞാല് നാലു മാസങ്ങള്ക്കുശേഷമാണ് അവയ വ രൂപീകരണം നടക്കുന്നതെന്നാണ് അതിനര്ത്ഥം. മൂന്നുമാസം പ്രായമായ സന്ദര്ഭത്തില് ഗര്ഭം അലസിപ്പോകുമ്പോള് ആ പ്രായത്തി ലുള്ള കുഞ്ഞുങ്ങള്ക്ക് ഒരുവിധം എല്ലാ ബാഹ്യാവയവങ്ങളും രൂപീകരിക്കപ്പെട്ടിട്ടുണ്ടെന്ന് പ്രവാചകകാലത്ത് ജീവിച്ചിരുന്നവര്ക്ക് മനസ്സിലാവുമായിരുന്നു. നൂറ്റിയിരുപത് ദിവസങ്ങള്ക്കുശേഷമാണ് അവയവ രൂപീകരണമെന്ന് അവര് പ്രവാചകനില് (സ) നിന്ന് മനസ്സിലാക്കിയിരുന്നുവെങ്കില് തങ്ങളുടെ അനുഭവത്തിന്റെ അടിസ്ഥാനത്തില് അവര് അദ്ദേഹത്തോട് സംശയമുന്നയിക്കുമായിരുന്നു. ഇക്കാര്യത്തില് അനുയായികളാരെങ്കിലും സംശയമുന്നയിച്ചതായോ എതിരിളികളാരെങ്കിലും വിമര്ശനമുന്നയിച്ചതായോ യാതൊരു വിധ നിവേദനങ്ങളുമില്ല. നൂറ്റിയിരുപത് ദിവസങ്ങള് കഴിഞ്ഞ ശേഷമേ മുദ്വ്അ ഘട്ടം കഴിയൂവെന്ന് അവരാരും പ്രവാചകനില് (സ) നിന്ന് മനസ്സിലാക്കിയിട്ടില്ലെന്നാണ് ഇതിനര്ത്ഥം.
- ബുഖാരിയിലും മുസ്ലിമിലുമുള്ള ഈ ഹദീഥിലെ വാചകങ്ങളില് പ്രധാനപ്പെട്ട ഒരു വ്യത്യാസമുണ്ട്. നിങ്ങളോരോരുത്തരും മാതൃവ യറ്റില് നാല്പതു ദിവസം കൊണ്ടാണ് സംയോജിക്കുന്നത് എന്നുപറഞ്ഞശേഷം അങ്ങനെ അതേപോലെത്തന്നെ അലഖയാകുന്നു; അങ്ങ നെ അതേപോലെത്തന്നെ മുദ്വ്അയാകുന്നു' (ഥുമ്മ യകൂനു അലഖത്തുന് മിഥ്ല ദാലിക്ക; ഥുമ്മ യകൂനു മുദ്വ്അത്തുന് മിഥ്ല ദാലിക്ക) എന്നാണ് ബുഖാരിയിലുള്ളത്. മുസ്ലിമിലാകട്ടെ 'അങ്ങനെ അതേപോലെത്തന്നെ അതില്വെച്ചുതന്നെ അലഖയാകുന്നു; അങ്ങനെ അതേ പോലെത്തന്നെ അതില്വെച്ചുതന്നെ മുദ്വ്അയാകുന്നു' (ഥുമ്മ യകൂനു ഫീദാലിക അലഖത്തുന് മിഥ്ല ദാലിക്ക; ഥുമ്മ യകൂനു മുദ്വ്അത്തു ന് ഫീദാലിക മിഥ്ല ദാലിക്ക)(7) എന്നാണുള്ളത്. രണ്ടുതവണ 'ഫീദാലിക്ക'യെന്ന് ആവര്ത്തിച്ചു വന്നിട്ടുണ്ട്, ഇവിടെ. 'അതില് വെച്ചുത ന്നെ'യെന്നു പരിഭാഷപ്പെടുത്തിയിരിക്കുന്ന ഫീദാലിക്കെയന്ന് ഉപയോഗിച്ചത് നാല്പതു ദിവസത്തെക്കുറിക്കുവാനാകാനാണ് കൂടുതല് സാധ്യതയെന്നാണ് വാചകത്തിന്റെ ഘടനയില് നിന്ന് മനസ്സിലാവുക. അങ്ങനെയാണെങ്കില് ഈ ഹദീഥിന്റെ നേര്ക്കുനേരെയുള്ള വിവ ക്ഷ നാല്പതു ദിവസങ്ങള്ക്കകത്താണ് അലഖ, മുദ്വ്അ എന്നീ ഘട്ടങ്ങള് എന്നായിരിക്കും. മുസ്ലിമിലുള്ള ഈ ഹദീഥിന്റെ വാചകഘട നയോടും മറ്റുസമാനമായ ഹദീഥുകളിലെ ആശയങ്ങളോടും നിരീക്ഷിച്ച് മനസ്സിലാക്കാന് കഴിയുന്ന വസ്തുതകളോടും പൊരുത്തപ്പെടുന്ന അര്ത്ഥമതാണ്.
- പരാമര്ശിക്കപ്പെട്ട ഹദീഥിലെ മിഥ്ല ദാലിക്ക (അതേപോലെത്തന്നെ)യെന്ന അലഖയെയും മുദ്വ്അയെയും കുറിച്ച പരാമര്ശങ്ങള്ക്കു ശേഷം ആവര്ത്തിക്കപ്പെട്ടിരിക്കുന്ന പദപ്രയോഗം, 'നാല്പതു ദിവസം' എന്ന ആദ്യഭാഗത്തിന്റെ ആവര്ത്തനത്തെയാണ് കുറിക്കുന്ന തെന്ന വീക്ഷണത്തില് നിന്നാണ് അലഖ, മുദ്വ്അ എന്നീ ഘട്ടങ്ങളില് ഓരോന്നിനും നാല്പതു ദിവസങ്ങള് വീതം ഉണ്ടെന്ന നിഗമനത്തിലെ ത്തുവാന് ഇമാം നവവിയെപ്പോലെയുള്ള ഹദീഥ് വിശാദരന്മാരെ പ്രേരിപ്പിച്ചത്. എന്നാല് ഈ വിഷയം പറയുന്ന മറ്റു ഹദീഥുകളുമാ യി താരതമ്യം ചെയ്യുകയും വാചകഘടനയെ സൂക്ഷ്മമായി വിശകലനം നടത്തുകയും ചെയ്താല് അത് മാതൃശരീരത്തിലെ സംയോജന ത്തെയാണ് (ജംഉല് ഖല്ഖ്) കുറിക്കുന്നതെന്നാണ് മനസ്സിലാവുകയെന്ന് പല പണ്ഡിതന്മാരും സൂചിപ്പിച്ചിട്ടുണ്ട്. ക്രിസ്താബ്ദം പതി മൂന്നാം നൂറ്റാണ്ടില് ജീവിച്ച കമാല് അബ്ദുല് വാഹിദ് ബ്നു അബ്ദുല് കരീം അസ്സംലക്കാനിയെന്ന ക്വുര്ആന് പണ്ഡിതന് തന്റെ അല് ബുര്ഹാനല് കാശിഫ് അന്ഇഅ്ജാസുല് ക്വുര്ആന്(8) എന്ന കൃതിയില് ഇക്കാര്യം സമര്ത്ഥിക്കുന്നുണ്ട്. ഭ്രൂണശാസ്ത്ര വസ്തുതകള് മനസ്സിലാക്കിയതിനുശേഷം ഇസ്ലാമിക പ്രബോധകര് നടത്തുന്ന ദുര്വ്യാഖ്യാനമല്ല ഇതെന്നര്ത്ഥം.
ആറ് ആഴ്ചകള്ക്കുശേഷമാണ് ഭ്രൂണത്തിന്റെ ലിംഗനിര്ണയവും വ്യക്തിത്വത്തിന്റെ പാരമ്പര്യനിര്ണയവും അവയവ രൂപീകരണവും വൈകല്യങ്ങളുണ്ടെങ്കില് അവയുടെ സ്ഥിരീകരണവും നടക്കുകയെന്ന ഹദീഥുകള് സൂചിപ്പിക്കുന്ന കാര്യങ്ങളെല്ലാം കൃത്യമാണെന്ന് ആധു നിക ശാസ്ത്രം നമുക്ക് പറഞ്ഞുതരുന്നു. ബീജസങ്കലനം, പ്രതിഷ്ഠാപനം, സോമൈറ്റ് രൂപീകരണം എന്നീ ഭ്രണവളര്ച്ചയുടെ പ്രാഥമികമായ മൂന്ന് ഘട്ടങ്ങളും ക്രമപ്രവൃദ്ധമായി നടക്കുന്നത് ആദ്യത്തെ നാല്പതു ദിവസങ്ങള്ക്കുള്ളിലാണെന്ന് ഭ്രൂണശാസ്ത്രം പറയുമ്പോള് അത് ഹദീഥുകളുടെ സത്യതയുടെ പ്രഘോഷണമാണെന്നതാണ് സത്യം; ക്വുര്ആന് വിമര്ശകര് എത്രതന്നെ ശക്തമായി നിഷേധിച്ചാലും അതാണ് വസ്തുത. സത്യാന്വേഷികള്ക്കെല്ലാം അത് സുതരാം മനസ്സിലാവും, തീര്ച്ച.
കുറിപ്പുകള്:
- സ്വഹീഹുല് ബുഖാരി, കിതാബുല് ഖദ്ര്, ഹദീഥ്
- സ്വഹീഹ് മുസ്ലിം, കിതാബുല് ഖദ്ര്, ഹദീഥ്
- സ്വഹീഹ് മുസ്ലിം, കിതാബുല് ഖദ്ര്, ഹദീഥ്
- സ്വഹീഹ് മുസ്ലിം, കിതാബുല് ഖദ്ര്, ഹദീഥ്
- സ്വഹീഹുല് ബുഖാരി, കിതാബുല് ഖദ്ര്, ഹദീഥ് 6595, സ്വഹീഹ് മുസ്ലിം, കിതാബുല് ഖദ്ര്, ഹദീഥ്
- ഇമാം നവവി: സ്വഹീഹ് മുസ്ലിം http://hadith.al-islam.com/.
- സ്വഹീഹ് മുസ്ലിം, കിതാബുല് ഖദ്ര്, ഹദീഥ്
- Abdul-Majeed A. Zindani, Mustafa A. Ahmed, Joe Leigh Simpson: Embryogenesis and Human Development in the first 40 days in Abdul-Majeed A. Zindani: Human Development as Described in the Quran and Sunnah, Riyadh, 1983, Page 122.
ജൂത പണ്ഡിതന്റെ ചോദ്യങ്ങള്ക്കുള്ള പ്രവാചകന്റെ(സ) ഉത്തരത്തെപ്പറ്റി വിശദീകരിക്കുന്ന ഥൗബാന്(റ) നിവേദനം ചെയ്ത ദീര്ഘമായ ഹദീഥിലും ശിശുവിന്റെ സൃഷ്ടിയെക്കുറിച്ച ചോദ്യത്തിനുള്ള വിശദമായ ഉത്തരം ആരംഭിക്കുന്നത് 'പുരുഷസ്രവം വെളുത്തനിറത്തിലു ള്ളതും സ്ത്രീസ്രവം മഞ്ഞനിറത്തിലുള്ളതുമാണ്' എന്നു പറഞ്ഞുകൊണ്ടാണ്.(സ്വഹീഹ്മുസ്ലിം) ജൂത ചോദ്യങ്ങള്ക്കെല്ലാം മറുപടി പറഞ്ഞ ശേഷം 'അയാള് എന്നോട് ചോദിച്ച കാര്യങ്ങളെക്കുറിച്ചൊന്നും അല്ലാഹു അറിയിച്ചുതരുന്നതുവരെ എനിക്ക് യാതൊരു വിവരവുമുണ്ടാ യിരുന്നില്ല' എന്ന് പറഞ്ഞതായുള്ള ഥൗബാനി (റ)ന്റെ പരാമര്ശം ശ്രദ്ധേയമാണ്. സ്വന്തം സ്രവത്തെക്കുറിച്ച് അറിയാത്ത സ്ത്രീകള്ക്കടക്കം നിങ്ങളുടെ സ്രവം മഞ്ഞനിറത്തിലുള്ളതാണ് എന്ന് പ്രവാചകന്(സ) പറഞ്ഞുകൊടുത്തത് വ്യക്തമായ ദൈവബോധനത്തിന്റെ അടിസ്ഥാന ത്തിലാണെന്ന് വ്യക്തമാക്കുന്നതാണീ പ്രവാചകപരാമര്ശം.
ഏതാണീ മഞ്ഞ ദ്രാവകം? കുഞ്ഞിന്റെ സൃഷ്ടിയില് പങ്കെടുക്കുന്ന പുരുഷസ്രവത്തിന്റെ നിറം 'അബ്യദ്വ്' ആണെന്നു പറഞ്ഞതിനുശേഷ മാണ് സ്ത്രീ സ്രവത്തിന്റെ നിറം 'അസ്വ്ഫര്' (മഞ്ഞ) ആണെന്ന് പ്രവാചകന് (സ) പറഞ്ഞത്. രണ്ടും കൂടിച്ചേര്ന്നാണ് കുഞ്ഞുണ്ടാകുന്ന തെന്നും അതിനുശേഷം അദ്ദേഹം വ്യക്തമാക്കി. വെള്ള നിറത്തിലുള്ള പുരുഷസ്രവത്തെപോലെതന്നെ ബീജ സങ്കലനത്തില് പങ്കെടുക്കുന്ന സ്ത്രീസ്രവത്തിന്റെ നിറം മഞ്ഞയാണെന്നാണ് പ്രവാചകന് (സ) ഇവിടെ പഠിപ്പിക്കുന്നതെന്നുറപ്പാണ്. സ്ത്രീശരീരത്തില്നിന്ന് നിര്ഗളിക്കു ന്ന ഏതു സ്രവത്തിനാണ് മഞ്ഞനിറമുള്ളതെന്ന കാര്യത്തില് കര്മശാസ്ത്ര പണ്ഡിതന്മാര് ഏറെ ചര്ച്ച ചെയ്തതായി കാണാന് കഴിയും. സ്ത്രീജനനേന്ദ്രിയത്തില്നിന്ന് നിര്ഗളിക്കുന്ന കാണാനാവുന്ന സ്രവങ്ങള്ക്കൊന്നും തന്നെ മഞ്ഞനിറമില്ലെന്ന വസ്തുതയാണ് വിശാലമായ ഇത്തരം ചര്ച്ചകളുടെ ഉല്ഭവത്തിന് നിമിത്തമായത്.
സ്ത്രീകളുടെ ജനനേന്ദ്രിയത്തില്നിന്ന് പുറത്തുവരുന്ന സ്രവങ്ങള് മൂന്നെണ്ണമാണ്. തന്റെ ശരീരം ലൈംഗികബന്ധത്തിന് സജ്ജമായിയെന്ന് അറിയിച്ചുകൊണ്ട് സ്ത്രീജനനേന്ദ്രിയത്തില്നിന്ന് കിനിഞ്ഞിറങ്ങുന്ന ബര്ത്തോലിന് സ്രവം (Bartholin fluid) ആണ് ഒന്നാമത്തേത്. യോനീമുഖ ത്തിനകത്തായി സ്ഥിതി ചെയ്യുന്ന പയര്വിത്തിന്റെ വലിപ്പത്തിലുള്ള രണ്ട് ബര്ത്തോലിന്ഗ്രന്ഥികള് സ്ത്രീശരീരം ലൈംഗികമായി ഉത്തേ ജിപ്പിക്കപ്പെടുമ്പോള് പുറപ്പെടുവിക്കുന്ന ഈ സ്രവത്തിന് നിറമില്ല. രതിമൂര്ച്ചയുടെ അവസരത്തില് ചില സ്ത്രീകളുടെ ജനനേന്ദ്രിയത്തി ല്നിന്ന് പുറത്തുവരുന്ന പാരായുറിത്രല് സ്രവമാണ് (Para urethral fluid) രണ്ടാമത്തെ യോനീ സ്രവം. യോനിയുടെ ആന്തരികഭിത്തിയില് സ്ഥിതി ചെയ്യുന്ന പാരായുറിത്രല് ഗ്രന്ഥികളില്നിന്നു വളരെ ചെറിയ അളവില്മാത്രം പുറത്തുവരുന്ന ഈ സ്രവം താരതമ്യേന കട്ടിയുള്ള തും വെള്ള നിറത്തിലുള്ളതുമായിരിക്കും. സ്ത്രീ ജനനേന്ദ്രിയത്തെ എല്ലായ്പ്പോഴും വരളാതെ സൂക്ഷിക്കുന്ന സെര്വിക്കല് ശ്ലേഷ്മ (Cervical mucus) ആണ് മൂന്നാമത്തെ യോനീ സ്രവം. അണ്ഡോല്സര്ജനസമയമല്ലെങ്കില് ഈ സ്രവം വഴുവഴുപ്പുള്ളതും നല്ല വെളുത്ത ക്രീം നിറത്തിലു ള്ളതുമായിരിക്കും. അണ്ഡോല്സര്ജനത്തോടടുക്കുമ്പോള് വെള്ളനിറം മങ്ങുകയും വഴുവഴുപ്പ് കുറയുകയും ചെയ്യുന്ന ഈ സ്രവം ഉല്സ ര്ജനസമയമാകുമ്പോഴേക്ക് ജലത്തെപ്പോലെ വര്ണരഹിതമാവുകയും മുട്ടയുടെ വെള്ളക്കരുവിനെപ്പോലെയായിത്തീരുകയും ചെയ്യും. (Fritz K. Beller&Gebhard F.B. Schumacher (Editors): Biology of the Fluids of the Female Genital Tract, Amsterdam, 1979 Pages 312-388) അണുബാധയുണ്ടാ കുമ്പോള് മാത്രമാണ് സെല്വിക്കല് ശ്ലേഷ്മത്തിന് മങ്ങിയ മഞ്ഞനിറമുണ്ടാകുന്നത്. സ്ത്രീജനനേന്ദ്രിയത്തില്നിന്ന് സാധാരണഗതിയില് നിര്ഗളിക്കപ്പെടുന്ന മൂന്ന് സ്രവങ്ങളും വെളുത്തതോ നിറമില്ലാത്തതോ ആണെന്നും ഹദീഥുകളില് പറഞ്ഞ മഞ്ഞസ്രവമല്ല ഇവയെന്നും വ്യക്തമാണ്. ഇവയ്ക്കൊന്നുംതന്നെ കുഞ്ഞിന്റെ രൂപീകരണത്തില് നേരിട്ട് പങ്കൊന്നുമില്ലതാനും.
കുഞ്ഞിന്റെ രൂപീകരണത്തിന് നിമിത്തമാകുന്ന സ്രവമെന്താണ് എന്ന ചോദ്യത്തിന് ഉത്തരം കാണാന് ശ്രമിക്കുമ്പോഴാണ് ഹദീഥുകളില് പറഞ്ഞ മഞ്ഞ സ്രവമേതാണെന്ന് നമുക്ക് മനസ്സിലാവുക. ആര്ത്തവചക്രത്തിന്റെ പതിനാലാം ദിവസം അണ്ഡാശയത്തിനകത്തെ പൂര്ണ വളര്ച്ചയെത്തിയ ഫോളിക്കിളില് പ്രത്യക്ഷപ്പെടുന്ന ദ്വാരത്തിലൂടെ പ്രായപൂര്ത്തിയെത്തിയ അണ്ഡത്തെവഹിച്ചുകൊണ്ട് ഫോളിക്കുളാര് ദ്രവവും ക്യൂമുലസ് കോശങ്ങളും പുറത്തേക്ക് തെറിച്ച് ഫലോപ്പിയന് നാളിയുടെ അറ്റത്തുള്ള ഫിംബ്രയകളില് പതിക്കുന്നതിനാണ് അണ്ഡോല്സര്ജനം (Ovulation) എന്നു പറയുന്നത്. രതിമൂര്ച്ചയോടനുബന്ധിച്ച് പുരുഷശരീരത്തില് നടക്കുന്ന ശുക്ലസ്ഖലന (Ejaculation) ത്തിന് തുല്യമായി സ്ത്രീശരീരത്തില് നടക്കുന്ന പ്രക്രിയയാണ് ഇതെങ്കിലും ഒരു ആര്ത്തവചക്രത്തില് ഒരു തവണ മാത്രമാണ് ഇത് സംഭവി ക്കുന്നത്. ശുക്ല സ്ഖലനവും അണ്ഡോല്സര്ജനവുമാണ് കുഞ്ഞിന്റെ സൃഷ്ടിക്ക് നിദാനമായി പുരുഷശരീരത്തിലും സ്ത്രീശരീരത്തിലും യഥാക്രമം സംഭവിക്കുന്ന രണ്ട് പ്രക്രിയകള്. പുരുഷബീജങ്ങളെ വഹിക്കുന്ന ശുക്ലദ്രാവകത്തെപ്പോലെ സ്ത്രീയുടെ അണ്ഡത്തെ വഹിക്കുന്ന ഫോളിക്കുളാര് ദ്രവവും കുഞ്ഞിന്റെ നിര്മാണത്തിന് നിമിത്തമാകുന്ന ദ്രാവകമാണ്. ഹദീഥുകളില് പറഞ്ഞ കുഞ്ഞിന്റെ സൃഷ്ടിക്ക് കാരണമായ സ്ത്രീസ്രവം അണ്ഡത്തെ വഹിക്കുന്ന ഫോളിക്കുളാര് ദ്രാവകമാണെന്നാണ് ഇത് വ്യക്തമാക്കുന്നത്. അങ്ങനെയാണെങ്കില് പുരുഷദ്രാവകം വെളുത്തതും സ്ത്രീദ്രാവകം മഞ്ഞയുമെന്ന് പരാമര്ശത്തിന്റെ വെളിച്ചത്തില് പരിശോധിക്കുമ്പോള് ഫോളിക്കുളാര് ദ്രാവകത്തിന്റെ നിറം മഞ്ഞയായിരിക്കണം. എന്നാല് എന്താണ് വസ്തുത?
പ്രായപൂര്ത്തിയെത്തുന്നതിനുമുമ്പുള്ള അണ്ഡാവസ്ഥയായ അണ്ഡത്തെ (Oocyte) സംരക്ഷിക്കുകയും വളര്ത്തിക്കൊണ്ടുവന്ന് ബീജസങ്കല നത്തിന് പറ്റിയ അണ്ഡമാക്കിത്തീര്ക്കുകയും ചെയ്യുകയാണ് ഫോളിക്കിളിന്റെ ധര്മം. പെണ്കുഞ്ഞ് ജനിക്കുമ്പോള് തന്നെ അവളുടെ അണ്ഡാശയത്തിലുള്ള പ്രായപൂര്ത്തിയെത്താത്ത അണ്ഡകങ്ങളെ പൊതിഞ്ഞ് ആദിമ ഫോളിക്കിളുകളുണ്ടാവും (Primordial follicles). അവള് പ്രായപൂര്ത്തിയാകുന്നതോടെ ഇതില് ചില ഫോളിക്കിളുകള് വളര്ന്നുവരികയും ഓരോ ആര്ത്തവചക്രത്തിന്റെയും ശരാശരി 14-16 ദിവസങ്ങള് കഴിഞ്ഞ് പൊട്ടി പൂര്ണവളര്ച്ചയെത്തിയ അണ്ഡത്തെ (Ovum) പുറത്തുവിടുന്നതോടെ അവയുടെ ധര്മം അവസാനിക്കുകയും ചെയ്യുന്നു. ജനനസമയത്തുള്ള ഏകദേശം 1,80,000 ഫോളിക്കിളുകളില് നാനൂറെണ്ണത്തോളം മാത്രമാണ് അണ്ഡോല്സര്ജനത്തിനുമുമ്പത്തെ വളര്ച്ചയെത്തുവാനുള്ള ഭാഗ്യമുണ്ടാകുന്നത്. പ്രസ്തുത വളര്ച്ചയ്ക്ക് വ്യത്യസ്തങ്ങളായ ഘട്ടങ്ങളുണ്ട്. ഇതിലെ ഓരോ ഘട്ടങ്ങളിലും അതു കടന്നുപോകാന് കഴിയാത്ത ഫോളിക്കിളുകള് മരിച്ചുപോകുന്നുണ്ട്. ഓരോ ആര്ത്തവചക്രത്തിലും ഇരുപതോളം ഫോളിക്കിളുകള് വളര്ച്ചയെത്തുന്നുവെങ്കിലും ഒരെണ്ണത്തിന് മാത്രമാണ് ഫോളിക്കിള് മരണമായ അട്രീഷ്യ(atresia)യില്നിന്ന് രക്ഷപ്പെട്ട് അണ്ഡോല്സര്ജന ത്തിന് കഴിയുന്നത്. അട്രീഷ്യയില് നിന്ന് രക്ഷപ്പെട്ട് അണ്ഡോല്സര്ജനത്തിന് കഴിയുന്ന ഫോളിക്കിളുകള് രണ്ട് ദശകളിലൂടെയാണ് കടന്നു പോകുന്നത്. അണ്ഡോല്സര്ജനത്തിലൂടെ അവസാനിക്കുന്ന ഒന്നാമത്തെ ദശയെ ഫോളിക്കുളാര് ദശ (follicular phase) എന്നും അതിനുശേഷ മുള്ള ദശയെ ലൂടിയല് ദശ (luteal phase) എന്നുമാണ് വിളിക്കുക. ആര്ത്തവം മുതല് അണ്ഡോല്സര്ജനം വരെയുള്ള ഫോളിക്കുളാര് ദശയില് അണ്ഡകം പൂര്ണവളര്ച്ചയെത്തിയ അണ്ഡമായിത്തീരുന്നതിനും യഥാരൂപത്തിലുള്ള അണ്ഡോല്സര്ജനം നടക്കുന്നതിനും വേണ്ടി വ്യത്യ സ്തങ്ങളായ പ്രക്രിയകള് നടക്കേണ്ടതുണ്ട്. ഈ പ്രക്രിയകളുടെ അവസാനമായി ശരീരത്തിലെ ഈസ്ട്രജന് നില പരമാവധി ഉയരുകയും ലൂറ്റിനൈസിംഗ് ഹോര്മോണ് (LH), ഫോളിക്കിള് സ്റ്റിമുലേറ്റിംഗ് ഹോര്മോണ് (FSH) എന്നീ ഹോര്മോണുകളെ ഇതിന്റെ ഫലമായി ഉത്പാദിപ്പിക്കുകയും ചെയ്യുന്നു. 24 മുതല് 36 വരെ മണിക്കൂറുകള് നീണ്ടുനില്ക്കുന്ന ഈ പ്രക്രിയയുടെ അന്ത്യം കുറിച്ചുകൊണ്ടാണ് അണ്ഡം വഹിക്കുന്ന പൂര്ണവളര്ച്ചയെത്തിയ ഫോളിക്കിളില് (Ovarian follicle) സ്റ്റിഗ്മയെന്ന് പേരുള്ള ദ്വാരമുണ്ടാവുകയും അത് പൊട്ടി അണ്ഡത്തെ വഹിച്ചുകൊണ്ട് ഫോളിക്കുളാര് ദ്രവം പുറത്തേക്ക് തെറിക്കുകയും ചെയ്യുന്നത്. ഈ പുറത്തേക്കു തെറിക്കല് പ്രക്രിയക്കാണ് അണ്ഡോല്സര്ജനം (Ovulation) എന്നു പറയുക.(Anand Kumar&Amal K. Mukhopadhyay (Editors): Follicular Growth, Ovulation And Fertilization: Molecular And Clinical Basis, New Delhi, 2002 Pages 207-265)
ഫോളിക്കുളാര് ദശയിലുടനീളം നടക്കുന്ന അണ്ഡവളര്ച്ചയ്ക്കും അതിന് ഉല്സര്ജിക്കാനാവശ്യമായസംവിധാനങ്ങളൊരുക്കുന്നതിനും നിമി ത്തമാകുന്നത് FSHന്റെ പ്രവര്ത്തനങ്ങളാണ്. പ്രസ്തുത ഉത്പാദനത്തോടനുബന്ധിച്ചാണ് ഹൈപ്പോതലാമസില്നിന്നുള്ള ഗൊണാടോട്രോ പിന് റിലീസിംഗ് ഹോര്മോണിന്റെ (GnRH) പ്രേരണയാല് പിറ്റിയൂട്ടറിയില്നിന്ന് LHന്റെ ഉത്പാദനം നടക്കുന്നത്. ഈ ഹോര്മോണ് ഉത്പാദിപ്പിക്കുന്ന പ്രോട്ടീന് വിഘാടക രസങ്ങളായ പ്രോട്ടിയോലിറ്റിക് എന്സൈമുകളാണ് (Proteolytic enzymes) ഫോളിക്കിളിലുണ്ടാവുന്ന ദ്വാരമായ സ്റ്റിഗ്മക്ക് കാരണമാകുന്നത്. അണ്ഡോല്സര്ജനത്തിനുശേഷമുള്ള ഫോളിക്കിള് അവശിഷ്ടങ്ങളെ നിയന്ത്രിക്കുന്നതും പ്രധാനമായി ഈ ഹോര്മോണാണ്. ലൂട്ടിയല് ദശയില് അണ്ഡം നഷ്ടപ്പെട്ട ഫോളിക്കിള് അവശിഷ്ടങ്ങള് കോര്പസ് ലൂടിയം (Lorpus Luteum) ആയിത്തീരു കയും മാതൃസ്വഭാവങ്ങളെ ഉദ്ദീപിക്കുന്ന പ്രോജസ്റ്ററോണ് (Progesterone) ഹോര്മോണിന്റെ വര്ധിതമായ ഉത്പാദനത്തിന് നിമിത്തമാവുകയും ചെയ്യുന്നു.
എന്താണീ ലൂറ്റിനൈസിംഗ് ഹോര്മോണ്? മഞ്ഞയെന്ന് അര്ത്ഥം വരുന്ന ലൂറ്റിയസ് (Luteus) എന്ന ലാറ്റിന് പദത്തിന്റെ നപുംസകരൂപമായ ലൂറ്റിയത്തില്നിന്നാണ് (Luteum) ലൂറ്റിനൈസ് (Luteinize) എന്ന ക്രിയയുണ്ടായിരിക്കുന്നത്. കോര്പ്പസ് ലൂടിയത്തിന്റെ നിര്മിതിക്ക് നിമിത്ത മായ പ്രവര്ത്തനങ്ങള്ക്കാണ് സാങ്കേതികമായി ലൂറ്റിനൈസ് എന്ന് പറയുന്നതെങ്കിലും പദപരമായി അതിനര്ത്ഥം 'മഞ്ഞയാക്കുന്നത്' എന്നാണ്. ലൂറ്റിനൈസിംഗ് ഹോര്മോണിന്റെ പ്രവര്ത്തനഫലമായാണ് ഫോളിക്കുളാര് ദശ പിന്നിട്ട ഫോളിക്കിള് അവശിഷ്ടങ്ങള് കോര് പസ് ലൂടിയം ആയിത്തീരുന്നത്. കോര്പസ് ലൂടിയം എന്ന പദദ്വയത്തിനര്ത്ഥം മഞ്ഞ വസ്തുവെന്നാണ് (Yellow body). ലൂടിയല് ദശയിലേക്ക് കടന്ന അണ്ഡം നഷ്ടപ്പെട്ട ഫോളിക്കിള് അവശിഷ്ടങ്ങളെല്ലാം കൂടി രണ്ടു മുതല് അഞ്ചു സെന്റീമീറ്റര് വരെ വ്യാസത്തില് ശരീരത്തില് ഏതാ നും ദിവസങ്ങള് കൂടി അവശേഷിക്കും. മനുഷ്യരില് ഇത് ഓറഞ്ചു നിറത്തിലാണ് കാണപ്പെടുന്നത്.(Jno C Dalton Jr.: Prize Essay on the Corpus Luteum, Charleston, South Carolina, 2009, Pages 45-67) അണ്ഡോല്സര്ജനത്തിന്റെ അവസാനഘട്ടത്തില് ഉത്പാദിപ്പിക്കപ്പെടുന്ന LH അതിന്റെ പ്രവര്ത്തനമാരംഭിക്കുകയും ഫോളിക്കുളാര് ദ്രവത്തെ മഞ്ഞവല്ക്കരിക്കുകയും ചെയ്യും. ഫോളിക്കിളിലെ സ്റ്റിഗ്മ പൊട്ടി അണ്ഡത്തോടെ പുറത്തേക്ക് തെറിക്കുന്ന ഫോളിക്കുളാര് ദ്രാവകത്തിന്റെ നിറം മഞ്ഞയായിരിക്കും. പുരുഷ ശുക്ലവുമായി താരതമ്യം ചെയ്യുമ്പോള് കട്ടിയില്ലാത്തതും മഞ്ഞ നിറത്തിലുള്ളതുമായ ദ്രാവകമാണ് ഫോളിക്കിള് പൊട്ടി പുറത്തേക്കൊഴുകുന്ന കുഞ്ഞിന്റെ നിര്മാണത്തിന് നിമിത്തമാകുന്ന സ്ത്രീസ്രവം എന്നര്ത്ഥം.
ഹദീഥുകളിൽ പറഞ്ഞതെത്ര ശരി !!
ഇല്ല. മുഹമ്മദ് നബിക്കു (സ) ശേഷം രചിക്കപ്പെട്ടതാണെന്ന് വ്യക്തമാക്കുന്ന പരാമർശങ്ങളും ശാസ്ത്രീയമായ അബദ്ധങ്ങളും ഹദീഥുകളിലുണ്ടെന്ന് സ്ഥാപിക്കുവാൻ വേണ്ടിയുള്ള ശ്രമങ്ങൾ നടന്നിട്ടുണ്ട് എന്നത് ശരിയാണ്. അങ്ങനെ ഉണ്ടെന്നത് അടിസ്ഥാനരഹിതമായ ഒരു ആരോപണമാണ്.
ചരിത്രവിമര്ശന രീതിയില് മത്നിനെ അപഗ്രഥിച്ച് അത് നബി(സ) പറഞ്ഞതല്ലെന്ന് സ്ഥാപിക്കണമെങ്കില് അതില് കാലാനുക്രമ പ്രമാദ (anarchonism) ങ്ങളെന്തെങ്കിലുമുണ്ടാവണം. നബി(സ)ക്കുശേഷം നിലവില് വന്ന എന്തെങ്കി ലും കാര്യങ്ങളെക്കുറിച്ച പരാമര്ശങ്ങളുണ്ടാവുകയും പ്രസ്തുത പരാമര്ശങ്ങള് നബി(സ)ക്കു ശേഷം രചിക്കപ്പെട്ടവയാണ് അതെന്ന് വ്യക്തമാക്കുകയും ചെയ്യുമ്പോഴാണ് അവയെ കാലാനുക്രമ പ്രമാദങ്ങളായി പരിഗണിക്കുന്നത്. അങ്ങനെയുള്ള എന്തെങ്കിലും സ്വഹീഹായ ഹദീഥുകളിലുള്ളതായി തെളിയിക്കുവാന് വിമര്ശകര്ക്കൊന്നും കഴിഞ്ഞിട്ടില്ല. അത്തരം വല്ല പരാമര്ശങ്ങളും മത്ന് ഉള്ക്കൊള്ളുന്നു ണ്ടോയെന്ന പരിശോധന കൂടി കഴിഞ്ഞ ശേഷമാണ് ഒരു ഹദീഥ് സ്വഹീഹാണെന്ന് തീരുമാനിക്കുന്നത് എന്നതിനാലാണിത്. കാലാനുക്രമ പ്രമാദങ്ങളെ പരതിപ്പരിശോധിച്ച് ബൈബിളിനെപ്പോലെയുള്ള രചനകളുടെ ചരിത്രപരതയെ ചോദ്യം ചെയ്യുന്നതു പോലെ ഹദീഥുകളുടെ ചരിത്രപരതയെ നിഷേധിക്കുവാന് ചരിത്ര വിമര്ശകര്ക്ക് കഴിയില്ല. ചരിത്ര വിമര്ശന രീതിയുടെ വക്താക്കള് കാലാനുക്രമപ്രമാദങ്ങളെ പഠനവിധേയമാക്കി ഏതെങ്കിലുമൊരു പുരാതന സ്രോതസ്സിന്റെ ചരിത്രപരത നിര്ണയിക്കുവാനുള്ള സങ്കേതങ്ങള് കണ്ടെത്തുന്നതിന് നൂറ്റാണ്ടുകള്ക്കു മുമ്പുതന്നെ പ്രസ്തുത സങ്കേതങ്ങള്ക്ക് പിടികൊടുക്കാത്തവണ്ണം ഹദീഥുകളെ അന്യൂനമാക്കുവാന് ഹദീഥ് നിദാന ശാസ്ത്ര ജ്ഞര്ക്ക് കഴിഞ്ഞിരുന്നുവെന്നര്ഥം.
നബി(സ)യില് നിന്നുള്ളതാണെന്ന രീതിയില് നിവേദനം ചെയ്യപ്പെട്ട ഹദീഥുകളിലൊന്നും കാലാനുക്രമ പ്രമാദങ്ങളുണ്ടായിരുന്നില്ല എന്ന് ഇതിനര്ഥമില്ല. അന്തിമ പ്രവാചകന്റെ പേരില് ആരോപിക്കപ്പെട്ടിരുന്ന വ്യാജ ഹദീഥുകളുടെ മത്നുകളില് അത്തരം പ്രമാദങ്ങളുണ്ടാ യിരുന്നു. നബി(സ)ക്ക് പതിറ്റാണ്ടുകള്ക്കു ശേഷം രചിക്കപ്പെടുകയും നബി(സ)യുടെ പേരില് ആരോപിക്കപ്പെടുകയും ചെയ്യുന്ന വൃത്താന്തങ്ങളില് അത് സ്വാഭാവികമാണ്. വ്യാജഹദീഥുകള് നിര്മിച്ചയാളുടെ ചരിത്രപരമായ അജ്ഞതയുടെ തോതനുസരിച്ച് അത്തരം പ്രമാദ ങ്ങളുടെ എണ്ണത്തില് ഏറ്റക്കുറച്ചിലുകളുണ്ടാകുമെന്നേയുള്ളൂ. 'ഇരുനൂറു വര്ഷങ്ങള്ക്കു ശേഷമാണ് ലോകാവസാനത്തിന്റെ അടയാള ങ്ങള് പ്രത്യക്ഷപ്പെടുക' എന്ന ഹദീഥ് തള്ളുന്നതിന് ഇരുനൂറു വര്ഷം കഴിഞ്ഞിട്ടും അത്തരം ലക്ഷണങ്ങളൊന്നും കണ്ടിട്ടില്ല എന്ന കാരണം കൂടി ഇമാം ബുഖാരി പറഞ്ഞതായി ഇമാം ദഹബി വ്യക്തമാക്കുന്നുണ്ട്.(അദ്ദഹബി: മീസാനുല് ഇഅ്തിദാല്, വാല്യം 3, പുറം 306) 'പ്രചാരത്തിലിരിക്കുന്ന മുസ്ലിം നാണയങ്ങളെ പൊട്ടിക്കുന്നത് പ്രവാചകന്(സ) വിലക്കിയിരിക്കുന്നു' വെന്ന ഹദീഥ് നിവേദനം ചെയ്ത മുഹമ്മദ് ബിന് ഫദാ സ്വീകരിക്കുവാന് കൊള്ളാത്തവനാണെന്ന് വിധിക്കുവാനുള്ള കാരണങ്ങളിലൊന്നായി അമവിയ്യാക്കളുടെ കാലത്ത് മാത്രമാണ് മുസ്ലിംകള് നാണയങ്ങള് അടിച്ചിറക്കാനാരംഭിച്ചത് എന്ന വസ്തുത എടുത്തുപറയുകയും നബി(സ)യുടെ കാലത്ത് നാണയം നിലനിന്നിരുന്നില്ലെന്നതിനാല് അദ്ദേഹം അസ്വീകാര്യനാണെന്ന് ഇമാം ബുഖാരി വ്യക്തമാക്കുകയും ചെയ്തിട്ടുണ്ട്.(ഇമാം ബുഖാരിയുടെ അത്താരീഖുല് ഔസത്തില് നിന്ന് (വാല്യം 2, പുറം 110 -119) ഖൈബറിലെ യഹൂദന്മാരോട് നികുതിവാങ്ങിയതിന് സാക്ഷി സഅദ് ബ്നു മുആദും (റ)കരാര് എഴുതിയത് മുആവിയയേുമായിരുന്നു' എന്ന ഹദീഥ് തള്ളിക്കളയുന്നതിനു ള്ള കാരണമായി ഖൈബര് യുദ്ധകാലത്ത് കപ്പം വാങ്ങുന്ന സമ്പ്രദായമുണ്ടായിരുന്നില്ലെന്നും സഅദ്ബ്നു മുആദ് (റ)പ്രസ്തുത യുദ്ധത്തിന് മുമ്പു തന്നെ മരണപ്പെട്ടിട്ടുണ്ടെന്നും മുആവിയ (റ)മുസ്ലിമായത് യുദ്ധത്തിന് ശേഷവുമാണെന്ന വസ്തുതകള് കൂടി പണ്ഡിതന്മാര് എടുത്തു പറഞ്ഞതായി നടേ വ്യക്തമാക്കിയിട്ടുണ്ട്. ഇത്തരം പ്രമാദങ്ങള് കണ്ടെത്തുകയും അവയെ അപഗ്രഥിച്ച് മാറ്റിവെക്കുകയും ചെയ്ത ശേഷ മാണ് ഹദീഥ് നിദാന ശാസ്ത്രജ്ഞന്മാര് സ്വഹീഹായ ഹദീഥുകളെ വേര്തിരിച്ച് രേഖപ്പെടുത്തിയത്. സ്വഹീഹായ ഹദീഥുകളുടെ മത്നില് ഇനിയുമൊരു ചരിത്ര വിമര്ശനാപഗ്രഥനം ആവശ്യമില്ലെന്നര്ഥം.
സ്വഹീഹായ ഹദീഥുകളില് ആരോപിക്കപ്പെടുന്ന ശാസ്ത്രാബദ്ധങ്ങളുടെ സ്ഥിതിയും ഇതുതന്നെയാണ്. ഖണ്ഡിതമായി തെളിയിക്കപ്പെട്ട ശാസ്ത്രവസ്തുതകളൊന്നും സ്വഹീഹായ ഹദീഥുകള് നിഷേധിക്കുന്നില്ല. എന്നാല് ശാസ്ത്രീയമായി തെളിയിക്കപ്പെടാത്ത കാര്യങ്ങളെക്കു റിച്ച പരാമര്ശങ്ങള് അവയിലുണ്ടാവാം. നിലനില്ക്കുന്ന അറിവിന്റെ മാത്രം അടിസ്ഥാനത്തില് അത്തരം പരാമര്ശങ്ങളെ അബദ്ധമായി ഗണിച്ച് ഹദീഥുകള് അസ്വീകാര്യമാണെന്ന് വിധിക്കുന്നത് അപകടകരമാണ്. മനുഷ്യന്റെ വൈജ്ഞാനിക മണ്ഡലം ഇനിയെത്ര വളരാനിരി ക്കുന്നു?! പ്രാപഞ്ചിക പ്രതിഭാസങ്ങളെക്കുറിച്ച അറിവിന്റെ ചുരുളുകള് ഇനിയുമെത്ര നിവരാനിരിക്കുന്നു?! വിജ്ഞാനമഹാസാഗര ത്തിലെ തുള്ളികള് മാത്രം ആസ്വദിക്കാന് അവസരം ലഭിച്ച മനുഷ്യര്ക്കെങ്ങനെയാണ് പ്രസ്തുത സാഗരത്തിന്റെ ഉടമസ്ഥനായ അല്ലാഹു വിന്റെ ബോധനത്തെ തനിക്കു ലഭിച്ച ചെറിയ അറിവിന്റെ വെളിച്ചത്തില് തള്ളിക്കളയാനാവുക? നമുക്ക് മനസ്സിലായിക്കഴിഞ്ഞിട്ടില്ലാത്ത കാര്യങ്ങളെക്കുറിച്ച് സ്വഹീഹായ ഹദീഥുകള് എന്തെങ്കിലും പറയുന്നുവെങ്കില്, നമ്മുടെ വൈജ്ഞാനിക മണ്ഡലം അവിടെയെത്തിക്കഴി ഞ്ഞിട്ടില്ലെന്ന് കരുതി കാത്തിരിക്കുന്നതല്ലേ ശരി?! സ്വഹീഹായ ഹദീഥുകളില് അശാസ്ത്രീയത പരതുന്നവര്ക്ക്, അറിയാനുള്ളതെല്ലാം അറിഞ്ഞുകഴിഞ്ഞവരാണ് തങ്ങളെന്ന് അഭിപ്രായമുണ്ടോ? ഇല്ലെങ്കില്, എങ്ങനെയാണ് ഹദീഥുകളെ തള്ളിക്കളയാന് ശാസ്ത്രത്തിന്റെ ഉപകരണങ്ങള് ഉപയുക്തമാവുക?
ഹദീഥുകളിലെ ആശയപ്രധാനമായ ഭാഗമായ മത് നിനെ ഉസൂലുൽ ഹദീഥിന്റെ പണ്ഡിതന്മാർ വിമർശിക്കുകയും യുക്തിക്ക് വിരുദ്ധമായവ തള്ളിക്കളയുകയും ചെയ്തിട്ടുണ്ടല്ലോ. നബിയിൽ നിന്ന് സ്വീകാര്യമായ പരമ്പര(സനദ്)യോടെ നിവേദനം ചെയ്യപ്പെട്ട ഹദീഥുകളിൽ തന്നെ യുക്തിവിരുദ്ധമായ കാര്യങ്ങളുണ്ടെന്ന് പറയുമ്പോൾ അത് ഹദീഥുകളുടെ സ്വീകാര്യതയെ സംശയാസ്പദമാക്കുന്നില്ലേ ?
ഹദീഥ് നിദാനശാസ്ത്രജ്ഞന്മാര് മത്ന് വിമര്ശനം നടത്തിയെന്ന് പറയുമ്പോള് തങ്ങളുടെ ബുദ്ധിക്കോ യുക്തിക്കോ യോജിക്കാത്തവയും ക്വുര്ആനിനെതിരാണെന്ന് തങ്ങള്ക്ക് തോന്നുന്നതുമായ ഹദീഥുകളെല്ലാം തള്ളിക്കളയണമെന്ന് ധരിച്ചു കൂടാത്തതാണ്. യുക്തിക്കോ ബുദ്ധി ക്കോ എതിരാണെന്ന് ഒറ്റനോട്ടത്തില് തോന്നുന്ന ഹദീഥുകളില് പലതിനെയും അവഗാഢമായി അപഗ്രഥിച്ചാല് അതില് യുക്തിവിരുദ്ധമായി യാതൊന്നുമില്ലെന്ന് കാണാനാവും. ക്വുര്ആനിക തത്ത്വങ്ങള്ക്കെതിരാണെന്ന് പ്രത്യക്ഷത്തില് തോന്നുന്ന ചില ഹദീഥുകളുടെ സ്ഥിതിയും ഇതു തന്നെയാണ്. ആയത്ത് അവതരിപ്പിക്കപ്പെട്ട സന്ദര്ഭവും ഹദീഥിലെ സംഭവത്തിന്റെ പശ്ചാത്തലവും മനസ്സിലാക്കി വ്യാഖ്യാനിച്ചാല് ഇങ്ങനെ ആരോപിക്കപ്പെട്ട പല ഹദീഥുകളിലുമുള്ളത് ക്വുര്ആനിന് പൂരകമായ കാര്യങ്ങളാണെന്ന് ബോധ്യപ്പെടും. അതുകൊണ്ടു തന്നെ പണ്ഡിതന്മാര്ക്ക് മത്ന് വിമര്ശനം വഴി മാത്രമായി ഹദീഥുകളൊന്നും തള്ളിക്കളയേണ്ടി വന്നിട്ടില്ല. ഇസ്നാദ് സ്വഹീഹാണെങ്കില് മത്നും സ്വീകാര്യമാണെന്ന നിലപാടിനെ ചോദ്യം ചെയ്യാനാകുന്ന തരത്തിലുള്ള മത്നകളൊന്നും തന്നെ സ്വഹീഹായ ഹദീഥുകള് ഉള്ക്കൊള്ളുന്നില്ല.
തങ്ങള്ക്ക് ഗ്രഹിക്കാന് കഴിയാത്ത കാര്യങ്ങളേതെങ്കിലും ഹദീഥുകളിലുണ്ടെങ്കില് കണ്ണടച്ച് അത് യുക്തിക്കെതിരാണെന്നും തള്ളിക്കളയേണ്ട താണെന്നും പറയുന്നവരായിരുന്നില്ല ഹദീഥ് നിദാനശാസ്ത്രജ്ഞര് എന്നതാണ് ശരി. നബി(സ)യുടെ വ്യക്തിത്വത്തെയും ദൗത്യത്തെയും കുറിച്ച് കൃത്യമായ അവബോധം അവര്ക്കുണ്ടായിരുന്നു എന്നതുകൊണ്ടായിരുന്നു ഇത്. ദൈവികബോധനത്തിന്റെ അടിസ്ഥാനത്തിലുള്ള നബിമൊഴികളില് യുക്തിക്ക് വിരുദ്ധമായ വല്ലതുമുണ്ടെന്ന് തങ്ങള്ക്ക് തോന്നിയാല് അത് തങ്ങളുടെ യുക്തിയുടെ കുഴപ്പമായിക്കൂടേയെ ന്നായിരിക്കും അവര് ആദ്യം പരിശോധിക്കുക. സാമാന്യയുക്തിയുമായി ഒരര്ഥത്തിലും പൊരുത്തപ്പെടാത്തതാണെങ്കില് മാത്രമായിരിക്കും അവരത് തള്ളിക്കളയുക. അറിയപ്പെടുന്ന യാഥാര്ഥ്യങ്ങള്ക്കെതിരാണ് നബിമൊഴികളെന്ന് അവര്ക്ക് തോന്നിയാല് അതിലെ പരാമര്ശ ങ്ങള് ഇന്ന് മനസ്സിലാക്കാന് സാധ്യതയില്ലാത്തതും ഭാവിയില് അറിയാന് കഴിയുന്നതുമാണോയെന്നായിരിക്കും അവര് പരിശോ ധിക്കുക; ക്വുര്ആനിനോ അറിയപ്പെടുന്ന ഹദീഥുകള്ക്കോ എതിരാണെന്ന് തോന്നുന്ന പരാമര്ശങ്ങളുള്ള ഒരു ഹദീഥ് ലഭിച്ചാല് എതിരാണെന്നത് പരിമിതമായ അറിവുപയോഗിച്ചുള്ള തന്റെ വിലയിരുത്തല് മാത്രമാണോ അതല്ല യാഥാര്ഥ്യം തന്നെയാണോയെന്നായിരിക്കും അവര് അപഗ്രഥിക്കുക. മത്ന് വിമര്ശനത്തിന്റെ വെളിച്ചത്തില് മാത്രമായി ഹദീഥുകള് തള്ളിക്കളയാറുണ്ടായിരുന്നില്ലെന്ന് സാരം.
സ്വീകാര്യമായ സനദോടു കൂടിയുള്ളവയാണെങ്കില് മത്നിലെ അസ്വീകാര്യമെന്നു തോന്നുന്ന കാര്യങ്ങള് വ്യാഖ്യാനിച്ച് യോജിപ്പിക്കാന് ശ്രമിക്കുന്ന ഹദീഥ് നിദാനശാസ്ത്രജ്ഞരുടെ പരിശ്രമങ്ങള് അവരുടെ ആത്മാര്ഥതയില്നിന്നുല്ഭൂതമായവയായിരുന്നു. ബുദ്ധിയുടെയും യുക്തിയുടെയും ഉന്നതവിതാനത്തിലാണ് തങ്ങളെന്ന അഹങ്കാരം അവര്ക്കുണ്ടായിരുന്നില്ല. അല്ലാഹുവിന്റെ തീരുമാനത്തിനു മുമ്പില് തങ്ങളുടെ യുക്തിബോധത്തെ സമര്പ്പിക്കുവാന് സന്നദ്ധരായിരുന്നു അവര്. പ്രവാചകനില്നിന്നുള്ളവയാണെന്ന് ബാഹ്യമായ തെളിവുക ളാല് സ്ഥാപിക്കപ്പെട്ട വചനങ്ങളിലെ പരാമര്ശങ്ങള് തങ്ങള്ക്ക് ഉള്ക്കൊള്ളാനാവുന്നില്ലെങ്കില് അത് തങ്ങളുടെ പരിമിതിയാണെന്ന് മനസ്സിലാക്കുവാനുള്ള വിനയം അവര്ക്കുണ്ടായിരുന്നു. അതുകൊണ്ട് തന്നെ മത്ന് വിമര്ശനത്തെക്കാള് അവര് പ്രാധാന്യം നല്കിയത് സനദ് അപഗ്രഥനത്തിനാണ്. തങ്ങളുടെ രീതി മാത്രമാണ് ശരിയെന്ന് അഹങ്കരിക്കുന്നവര്ക്ക് സമര്പ്പണത്തില് നിന്നുല്ഭൂതമായ യുക്തിയുടെ സാഷ്ടാംഗ പ്രണാമം മനസ്സിലായിക്കൊള്ളണമെന്നില്ല. അത്യുന്നതനായ നാഥനെക്കുറിച്ച അറിവില് നിന്ന് ഉല്ഭൂതമാകുന്ന വിനയം ജീവിത സപര്യയാക്കാന് കഴിയുമ്പോള് മാത്രമെ മനുഷ്യയുക്തി സ്രഷ്ടാവിനു മുന്നില് പ്രണാമം ചെയ്യുന്നതിന്റെ യുക്തിയെയും സമര്പ്പണത്തിന്റെ സൗന്ദര്യത്തെയും ആസ്വദിക്കാനാവൂ. അതിന്നുള്ള ഭാഗ്യം ഹദീഥ് നിഷേധികള്ക്ക് ഉണ്ടായിട്ടില്ലെന്ന് കരുതുന്നുതാവും ശരി!
തെറ്റായ ഒരു ആരോപണമാണിത്. തങ്ങളുടെ ദൗത്യത്തിന് മത്ന് വിമര്ശനം കൊണ്ട് കാര്യമില്ലെന്ന് മനസ്സിലാക്കിയവരായിരുന്നു ഹദീഥ് നിദാനശാസ്ത്രജ്ഞര് എന്നതിനാല് തങ്ങളുടെ പ്രധാനപ്പെട്ട മേഖലയായി അവര് കണ്ടത് സനദിന്റെ അപഗ്രഥനമായിരുന്നുവെന്നത് ശരി യാണ്. എന്നാല് നബി(സ)യില് നിന്നുള്ളതല്ലെന്ന് ഉറപ്പു നല്കുന്ന രിതിയിലുള്ള മത്നുകളുള്ള ഹദീഥുകള് അവര് അസ്വീകാര്യമായി വിലയിരുത്തിയിരുന്നുവെന്ന വസ്തുത നിഷേധിക്കാനാവാത്തതാണ്. തങ്ങളുടെ ദൗത്യത്തില് മത്ന് വിമര്ശനത്തിന് ചെറിയൊരു ധര്മം മാത്രമെ നിര്വഹിക്കാനാവൂയെന്നതിനാല് ഇസ്നാദുകള് പരിശോധിക്കുവാനും അതിന്റെ സത്യതയും നൈരന്തര്യവും ഉറപ്പുവരുത്തു വാനും അവരുപയോഗിച്ച സമയവും ഊര്ജവുമായി താരതമ്യം ചെയ്യുമ്പോള് മത്ന് വിമര്ശനത്തിന് വളരെ കുറച്ച് സമയവും ഊര്ജവും മാത്രമെ അവര് ചെലവഴിച്ചിരുന്നുള്ളൂ. അതിന്റെ അടിസ്ഥാനത്തില് ഹദീഥ് നിദാന ശാസ്ത്രജ്ഞന്മാര് മത്ന് വിമര്ശനത്തെ അവഗണിച്ചു വെന്ന് പറയുന്നത് അടിസ്ഥാന രഹിതമാണ്. ദുര്ബല ഹദീഥുകളുടെ കൂട്ടത്തിലുള്ള ശാദ്ദ്, മുദ്റജ് എന്നീ ഇനങ്ങളും വ്യാജ ഹദീഥുകളെ നിരീ ക്ഷിക്കാനുള്ള അടയാളങ്ങളായി പണ്ഡിതന്മാര് വിശദീകരിച്ച കാര്യങ്ങളും മത്നിനെ അടിസ്ഥാനമാക്കിയുള്ള നിരൂപണത്തിന്റെ ഭാഗമാണ്
മത്ന് വിമര്ശനത്തില് ഉസൂലുല് ഹദീഥിന്റെ പണ്ഡിതന്മാര് സ്വീകരിച്ച മാനദണ്ഡങ്ങള് താഴെ പറയുന്നവയാണ്.
- സാഹിത്യത്തിലും വാചകശുദ്ധിയിലും ഉന്നത നിലവാരം പുലര്ത്തിയിരുന്ന നബി(സ)യില് നിന്നുള്ളതാണെന്ന് ഉറപ്പു നല്കുന്ന വാചക ഘടനയിലുള്ളതും നിലവാരം കുറഞ്ഞ പദപ്രയോഗങ്ങള് ഉള്ക്കാള്ളാത്തതുമാവുക.
- വ്യാഖ്യാനിക്കാന് പറ്റാത്തവിധം പ്രാഥമിക ബുദ്ധിക്കോ തെളിയിക്കപ്പെട്ട വസ്തുതകള്ക്കോ അംഗീകരിക്കപ്പെട്ട സ്വഭാവ മൂല്യങ്ങള് ക്കോ എതിരായ പരാമര്ശങ്ങള് ഉള്ക്കൊള്ളാത്തതാവുക.
- ക്വുര്ആനോ സ്വീകരിക്കപ്പെട്ട ഹദീഥോ പണ്ഡിതന്മാരുടെ ഏകകണ്ഠമായ അഭിപ്രായമായ ഇജ്മാഓ ഉള്ള കാര്യത്തിനെതിരായി വ്യഖ്യാനിച്ച് യോജിപ്പിക്കാന് കഴിയാത്ത പരാമര്ശങ്ങളില്ലാത്തതാവുക.
- നബി(സ)യുടെ കാലത്തെ അറിയപ്പെട്ട ചരിത്രയാഥാര്ഥ്യങ്ങള്ക്കെതിരായ പരാമര്ശങ്ങള് ഇല്ലാതിരിക്കുക.
- തര്ക്കമുള്ള കാര്യങ്ങളില് നിവേദകന്റെ പക്ഷത്തെ ന്യായീകരിക്കുന്ന പ്രസ്താവനകളില് നിന്ന് മുക്തമാവുക.
- ജനക്കൂട്ടത്തില് വെച്ചു പരസ്യമായി നടന്ന ഒരു കാര്യമാണെങ്കില് ധാരാളം പേര് നിവേദനം ചെയ്യാന് സാധ്യതയുണ്ടായിട്ടും ഒരാള് മാത്രം നിവേദനം ചെയ്തത് അല്ലാതിരിക്കുക.
- തുച്ഛമായ കര്മങ്ങള്ക്ക് അതിരു കവിഞ്ഞ പ്രതിഫലങ്ങള് വാഗ്ദാനം ചെയ്യുന്നതോ ഭയങ്കരമായ ശിക്ഷകളെക്കുറിച്ച് മുന്നറിയിപ്പു നല്കുന്നതോ ആയ പരാമര്ശങ്ങള് ഇല്ലാതിരിക്കുക.
- ഇസ്ലാമിക ശരീഅത്തിന്റെ അടിസ്ഥാന തത്ത്വങ്ങള്ക്ക് വിരുദ്ധമായ നീചമായ കാര്യങ്ങള്ക്ക് പ്രേരണയോ പ്രോല്സാഹനമോ നല് കുന്ന പരാമര്ശങ്ങള് ഇല്ലാതിരിക്കുക.
നബി(സ)യില് നിന്നുള്ളതല്ലെന്ന് ഉറപ്പിക്കാനാവുന്ന മത്ന് ഉള്ക്കൊള്ളുന്ന ഹദീഥുകള് അസ്വീകാര്യമായവയാണെന്ന് വിധിക്കുകയും അതി നെതിരെ ബോധവല്ക്കരിക്കുകയും ചെയ്ത നിരവധി സംഭവങ്ങളുണ്ട്. 'നൂഹ് നബി(അ)യുടെ കപ്പല് കഅ്ബയെ ഏഴു തവണ പ്രദക്ഷിണം ചെയ്യുകയും മഖാമു ഇബ്റാഹീമില് നമസ്കരിക്കുകയും ചെയ്തു', 'സുന്ദരമായ മുഖത്തേക്ക് നോക്കുന്നത് കണ്ണിന് കാഴ്ച തെളിയിക്കും', 'ഉനജ്ബ്നു ഉനുബ് എന്ന ഒരാളുടെ നീളം മൂവായിരം മുഴമായിരുന്നു; നൂഹ് നബിയുടെ കാലത്തെ പ്രളയം അയാളുടെ ഞെരിയാണിവ രെയേ എത്തിയിരുന്നുള്ളൂ', 'പൂവന് കോഴി എന്റെ സുഹൃത്താണ്; എന്റെ സുഹൃത്തിന്റെ സുഹൃത്ത് ജിബ്രീലാണ്' തുടങ്ങിയ പരാമ ര്ശങ്ങളുള്ള ഹദീഥുകള് പ്രാഥമിക ബുദ്ധിക്കും അംഗീകരിക്കപ്പെട്ട സ്വഭാവമൂല്യങ്ങള്ക്കുമെതിരാണെന്ന് പണ്ഡിതന്മാര് വ്യക്തമാക്കി യിട്ടുണ്ട്. 'വ്യഭിചാരത്തില് ജനിച്ച സന്താനം ഏഴു മക്കള്വരേക്കും സ്വര്ഗത്തില് പ്രവേശിക്കുകയില്ല' എന്ന ഹദീഥ് ഒരാളുടെ കുറ്റം മറ്റൊ രാള് വഹിക്കുകയില്ലെന്ന ക്വുര്ആനിന്റെ സ്പഷ്ടമായ ആശയത്തിനെതിരാണെന്നും 'സത്യവുമായി പൊരുത്തപ്പെടുന്ന ഒരു വചനം എന്നെപ്പറ്റി പറയപ്പെട്ടാല്, ഞാന് അത് പറഞ്ഞിട്ടുണ്ടെങ്കിലും ഇല്ലെങ്കിലും നിങ്ങളത് പിന്പറ്റുക'യെന്ന ഹദീഥ് നബി(സ)യെപ്പറ്റി കളവു പറയരുതെന്ന് വിലക്കുന്ന സുപ്രസിദ്ധമായ ഹദീഥിന് വിരുദ്ധമാണെന്നും 'സന്താനത്തിന് മുഹമ്മദ് എന്നു പേരിട്ടാല് അവനും അവന്റെ സന്താനവും സ്വര്ഗത്തിലായിരിക്കും' എന്ന ഹദീഥ് നാമമല്ല കര്മമാണ് സ്വര്ഗപ്രാപ്തിക്ക് നിദാനമെന്ന സ്ഥിരപ്പെട്ട ഇസ്ലാമിക തത്ത്വ ത്തിന് എതിരാണെന്നും ഖൈബറിലെ യഹൂദന്മാരില്നിന്ന് നികുതിവാങ്ങി, അതിന് സാക്ഷി നിന്നത് സഅദ്ബ്നു മുആദും (റ) കരാര് എഴുതിയത് മുആവിയയേുമായിരുന്നുവെന്ന ഹദീഥ്, കപ്പം വാങ്ങുന്ന സമ്പ്രദായം അന്ന് നിലവിലുണ്ടായിരുന്നില്ല, ഖൈബറിന് മുന്പ് ഖന്ദഖ് യുദ്ധം കഴിഞ്ഞ ഉടനെ മരണപ്പെട്ടയാളാണ് സഅദ്, മുആവിയ ഇസ്ലാം സ്വീകരിച്ചത് ഖൈബറിന് മാസങ്ങള്ക്ക് ശേഷം നടന്ന മക്കാവിജയത്തിനു ശേഷമാണ് എന്നീ കാരണങ്ങളാല് മാത്രമായിത്തന്നെ അസ്വീകാര്യമായി ഗണിക്കാവുന്നതാണെന്നും നബി(സ)യെ കുളി മുറിയില് വെച്ച് കണ്ടതായി അനസ് (റ) നിവേദനം ചെയ്ത ഹദീഥ് അക്കാലത്ത് ഹിജാസില് കുളിമുറിയുണ്ടാക്കുന്ന സമ്പ്രദായമില്ലെന്ന കാരണത്താല് തന്നെ തള്ളിക്കളയാവുന്നതാണെന്നും ഹദീഥ് നിദാനശാസ്ത്രജ്ഞന്മാര് എടുത്തു പറഞ്ഞിട്ടുണ്ട്. (ഡോക്ടര് മുസ്തഫ സ്സ ബാഇയുടെ സുന്നത്തും ഇസ്ലാം ശരീഅത്തില് അതിന്റെ സ്ഥാനവും എന്ന ഗ്രന്ഥത്തില് ഉദ്ധരിക്കപ്പെട്ടവയാണീ ഹദീഥുകള്.)
എന്നാല് ഇവയൊന്നും തന്നെ മത്ന് വിമര്ശനം വഴി മാത്രം മാറ്റിനിര്ത്തപ്പെട്ടവയല്ല. പ്രത്യുത, അവയുടെ ഇസ്നാദുകള് കൂടി പരിശോധിച്ച ശേഷം അവ ദുര്ബലമാണെന്ന് ബോധ്യപ്പെട്ട ശേഷം മാത്രം തള്ളപ്പെട്ട ഹദീഥുകളാണ്. ഇസ്നാദുകള് പ്രബലമായ ഹദീഥുകളെയൊന്നും മത്ന് വിമര്ശനം വഴി പണ്ഡിതന്മാര്ക്ക് തള്ളിക്കളയേണ്ടിവന്നിരുന്നില്ല എന്ന് പറയുന്നതാവും ശരി; നിഷ്കൃഷ്ടമായ ഇസ്നാദ് പരിശോധന യുടെ അരിപ്പയിലൂടെ നബി(സ)യില് നിന്നുള്ളതാണെന്ന് ഉറപ്പുള്ള ഹദീഥുകള് മാത്രമേ പുറത്ത്വന്നിരുന്നുള്ളു. അവയുടെ മത്നിന് കുഴ പ്പങ്ങളെന്തെങ്കിലുമുള്ളതായി പണ്ഡിതന്മാര്ക്ക് തോന്നിയിരുന്നില്ല. സ്വഹീഹായ സനദോടുകൂടി നിവേദനം ചെയ്യപ്പെട്ട ഹദീഥുകളുടെ മത്നുകളിലേതിലെങ്കിലും നടേ പറഞ്ഞ തകരാറുകള് കണ്ടെത്തുവാന് അവര്ക്ക് കഴിഞ്ഞിരുന്നിരുന്നില്ല എന്നര്ഥം.
ഹദീഥുകളുടെ ധര്മത്തെയും ദൗത്യത്തെയും പറ്റി വേണ്ടത്ര മനസ്സിലാക്കാത്തതുകൊണ്ട് ഉയര്ന്നുവരുന്നതാണീ അഭിപ്രായം. ചരിത്രപരമെന്നതിലേറെ ധാര്മികമായ ദൗത്യമാണ് ഹദീഥുകള് നിര്വഹിക്കുന്നത്. മുഹമ്മദ് നബി(സ) ഒരു ചരിത്രപുരുഷനായിരുന്നുവെന്ന് സ്ഥാപിക്കുകയല്ല ഹദീഥുകളുടെ ധര്മം. ആ ജീവിതത്തെക്കുറിച്ച് കൃത്യമായ വിവരങ്ങള് നല്കി അദ്ദേഹത്തെ അനുധാവനം ചെയ്യാന് മുസ്ലിംകളെ പര്യാപ്തരാക്കിത്തീര്ക്കുകയാണ് ഹദീഥുകള് ചെയ്യുന്നത്. കേവലമൊരു ചരിത്രപുരുഷനോ ആത്മീയാചാര്യനോ ഭരണാ ധികാരിയോ അല്ല മുഹമ്മദ് നബി(സ). സര്വശക്തനില്നിന്ന് ബോധനം ലഭിച്ചുകൊണ്ടിരുന്ന പ്രവാചകനാണ്; ആ ബോധനപ്രകാരം സംസാരിക്കുകയും പ്രവര്ത്തിക്കുകയും അനുവദിക്കുകയും ചെയ്ത ദൈവദൂതനാണ്. വാക്കും പ്രവൃത്തിയും അനുവാദവുമെല്ലാം ദൈവികബോധന പ്രകാരം ചിട്ടപ്പെടുത്തിയ അന്തിമപ്രവാചകന്റെ ജീവിതത്തില് സംഭവിച്ചതും അദ്ദേഹം പറഞ്ഞതും ചെയ്തതും അനുവദിച്ചതുമായ കാര്യങ്ങളുടെ രേഖീകരണമാണല്ലോ ഹദീഥ്. ദൈവിക ബോധനങ്ങളാല് സ്ഥിരപ്പടുത്തപ്പെടുന്ന കാര്യങ്ങള് ശാസ്ത്രീയ മായ അപഗ്രഥനത്തിനതീതമാണ് എന്ന കാര്യത്തില് സംശയമില്ല. അങ്ങനെയാകുമ്പോള് നബി(സ)യുടെ ജീവിതത്തിന്റെ രേഖീകരണമായ ഹദീഥുകളെ ശാസ്ത്രീയ വിശകലനത്തിന് വിധേയമാക്കുന്നതെങ്ങനെയാണ്?
നബിജീവിതത്തിലെ അമാനുഷിക സംഭവങ്ങള് ഉദാഹരണമായെടുക്കുക. ഒരു അത്ഭുതം കാണിക്കാനാവശ്യപ്പെട്ട മക്കാമുശ്രിക്കുകളുടെ മുന്നില് വെച്ച് ചന്ദ്രന് പിളര്ന്നതായി വ്യക്തമാക്കുന്ന സ്വഹീഹായ ഹദീഥുകളുണ്ട്.( സ്വഹീഹുല് ബുഖാരി, കിതാബു മനാക്വിബില് അന്സ്വാര്; സ്വഹീഹു മുസ്ലിം, കിതാബു സ്വിഫാത്തില് ക്വിയാമഃ വല്ജന്നഃ വന്നാര്; കിതാബുല് ഈമാന്.) ക്വുര്ആനിലും ഇക്കാര്യം സ്ഥിരീകരിക്കപ്പെട്ടിട്ടുണ്ട്. (54:1,2). ഒരൊറ്റ രാത്രികൊണ്ട് നബി(സ) മക്കയിലുള്ള മസ്ജിദുല് ഹറമില് നിന്ന് ഫലസ്തീനിലുള്ള മസ്ജിദുല് അഖ്സയിലേക്കും അവിടെനിന്ന് ആകാശലോകങ്ങളിലേക്കും യാത്രചെയ്തതായി സ്ഥിരികരിക്കപ്പെട്ട ഹദീഥുകള് വ്യക്തമാക്കുന്നു.( സ്വഹീ ഹുല് ബുഖാരി, കിതാബു മനാഖിബുല് അന്സ്വാര്; സ്വഹീഹു മുസ്ലിം, കിതാബുസ്സ്വഹാബ.) മസ്ജിദുല് ഹറമില്നിന്ന് അഖ്സയിലേക്കുള്ള നിശാപ്രയാണം ക്വുര്ആനും ശരിവെക്കുന്നുണ്ട്. (17:1)
ഇങ്ങനെ നിരവധി അത്ഭുതസംഭവങ്ങള് നബിജീവിതത്തിലുണ്ടായിട്ടുണ്ട്. ഇവ അത്ഭുതങ്ങളാവുന്നതു തന്നെ ശാസ്ത്രത്തിന് വിശദീകരിക്കു വാന് കഴിയാത്തതുകൊണ്ടാണ്. ഇവയുള്ക്കൊള്ളുന്ന ഹദീഥുകളെ ചരിത്രവിമര്ശന രീതിയില് അപഗ്രഥിക്കുന്നതെങ്ങനെയാണ്? അത്തരമൊരു അപഗ്രഥനത്തിന് വിധേയമാക്കിയാല് ഇങ്ങനെയൊന്നും സംഭവിച്ചിട്ടില്ലെന്ന് പറയാനേ ചരിത്രവിമര്ശകര്ക്ക് കഴിയൂ. അവരുടെ പരിശോധനാ സങ്കേതങ്ങളുടെ വരുതിയിലൊതുങ്ങുന്നതല്ല ഈ സംഭവങ്ങള് എന്നു പറയുന്നതാവും ശരി. ചരിത്രവിമര്ശന ത്തിന്റെ പരീക്ഷണനാളിക്ക് ഉള്ക്കൊള്ളാനാവാത്ത വിധം ബൃഹത്തായ വിഷയങ്ങളാണ് ഹദീഥുകള് കൈകാര്യം ചെയ്യുന്നതെന്നര്ഥം.
നബി(സ) പറയുകയോ ചെയ്യുകയോ അനുവദിക്കുകയോ ചെയ്തുവെന്ന് ഉറപ്പുള്ള കാര്യങ്ങള് ചോദ്യം ചെയ്യാതെ അനുധാവനം ചെയ്യു ന്നവനാണ് മുസ്ലിം. അത് ഉറപ്പിക്കുകയാണ് ഹദീഥ് നിദാനശാസ്ത്രത്തിന്റെ ധര്മം. മത്നിനെ ചരിത്രവിമര്ശനത്തിന്റെ രീതിയില് അപഗ്രഥിച്ചാല് പരമാവധി പറയാന് കഴിയുക നബി(സ) അത് ചെയ്തിട്ടില്ലെന്നോ ചെയ്തിരിക്കാന് സാധ്യതയുണ്ടെന്നോ മാത്രമാണ്. ചെയ്തിട്ടില്ലെന്ന് ഉറപ്പിക്കുവാനല്ലാതെ ചെയ്തിട്ടുണ്ടെന്ന് ഉറപ്പിക്കാനാവശ്യമായ സങ്കേതങ്ങള് ചരിത്രവിമര്ശകന്മാര്ക്ക് വികസിപ്പിച്ചെ ടുക്കുവാന് കഴിഞ്ഞിട്ടില്ല. ഹദീഥ് നിദാനശാസ്ത്രത്തിന്റെ ദൗത്യം നബി(സ) ചെയ്യുകയോ പറയുകയോ അനുവദിക്കുകയോ ചെയ്തിട്ടു ണ്ടെന്ന് ഉറപ്പിക്കുകയാണ്. അതിന് മത്ന് വിമര്ശനം തീരെ അപര്യാപ്തമാണ്. നബി(സ)യില് നിന്നുണ്ടായതല്ലെന്ന് ഉറപ്പുള്ള കാര്യങ്ങള് മത്നില് ഉണ്ടെങ്കില് അത് അസ്വീകാര്യമാണെന്ന് ഹദീഥ് നിദാനശാസ്ത്രജ്ഞര് വിധിച്ചിട്ടുമുണ്ട്.
വിശ്വാസം, കര്മം, സ്വഭാവം, മര്യാദ, നിയമം തുടങ്ങിയ കാര്യങ്ങളാണ് ഹദീഥുകള് കൈകാര്യം ചെയ്യുന്ന വിഷയങ്ങള്. ദൈവിക ബോധ നപ്രകാരമുള്ള നബി നിര്ദേശങ്ങള് അപ്പടി സ്വീകരിക്കുകയാണ് ഇത്തരം വിഷയങ്ങളില് മുസ്ലിം ചെയ്യേണ്ടതെന്നാണ് പഠിപ്പിക്കപ്പെട്ടി രിക്കുന്നത്. തന്നിഷ്ട പ്രകാരമല്ല, ദൈവികബോധനത്തിന്റെ അടിസ്ഥാനത്തിലാണ് നബി(സ) സംസാരിച്ചതെന്ന് ഖുര്ആന് (53:3,4) വ്യക്തമാ ക്കുന്നുണ്ട്. ഒരു പ്രത്യേക കര്മം മുഹമ്മദ് നബി(സ) പഠിപ്പിച്ചിട്ടുണ്ടോ ഇല്ലേയെന്ന് മനസ്സിലാക്കാന് ചരിത്രവിമര്ശന രീതിയില് മത്നിനെ അപഗ്രഥന വിധേയമാക്കിയാല് കഴിയുമോ? വുദുവെടുക്കുമ്പോള് ഏതെല്ലാം അവയവങ്ങള് എത്ര തവണ വീതമാണ് നബി(സ) കഴുകിയ തെന്ന് പഠിപ്പിക്കുന്ന ഹദീഥിനെ അപഗ്രഥിച്ച് അത് നബി(സ)യില് നിന്നുള്ളതാണോ അല്ലേയെന്ന് വ്യക്തമാക്കുവാന് ചരിത്രവിമര്ശന രീതി യുടെ അപ്രമാദിത്വത്തില് അഹങ്കരിക്കുന്നവര്ക്ക് കഴിയുമോ? ഈ ചോദ്യങ്ങള്ക്കെല്ലാമുള്ള ഉത്തരം 'ഇല്ല'യെന്നാണ്. മത്ന് വിമര്ശനം വഴി ഹദീഥ് നിദാനശാസ്ത്രത്തിന് അതിന്റെ ദൗത്യം നിര്വഹിക്കാനാവില്ലെന്നര്ഥം.
നബിജീവിതത്തിന്റെ സത്യസന്ധമായ ആവിഷ്കാരമാണ് തങ്ങള്ക്ക് ലഭിക്കേണ്ടത് എന്നതിനാല് അതിന് ഉപയുക്തമായ ഏറ്റവും ശാസ്ത്രീ യമായ രീതിയാണ് ഹദീഥ് നിദാനശാസ്ത്രജ്ഞര് വികസിപ്പിച്ചെടുത്തത്. മത്ന് വിമര്ശനം വഴി തങ്ങള്ക്കാവശ്യമായത് ലഭിക്കില്ലെന്ന് അവര്ക്കറിയാമായിരുന്നു. സ്വീകാര്യമായ മത്നുകള് ഏതൊക്കെയാണെന്നായിരുന്നു അവര് അന്വേഷിച്ചത്. അപഗ്രഥിക്കുവാനാകാത്ത വിവരങ്ങളുള്ക്കൊള്ളുന്നതും തങ്ങള് നിര്ബന്ധമായും പിന്തുടരേണ്ടതുമായ മത്നുകള് വേര്തിരിച്ചു മനസ്സിലാക്കുവാനാണ് അവര് സനദിനെ നിഷ്കൃഷ്ടമായി അപഗ്രഥിച്ചത്. അവര്ക്കാവശ്യമായത് അതില് നിന്ന് അവര്ക്ക് ലഭിക്കുകയും ചെയ്തു. ഹദീഥ് നിദാനശാ സ്ത്രത്തിന്റെ ധര്മത്തെയും ദൗത്യത്തെയും കുറിച്ച് മനസ്സിലാകാത്തുകൊണ്ടാണ് സനദ് അപഗ്രഥനമല്ല മത്ന് വിമര്ശനമാണ് ശാസ്ത്രീയം എന്ന് ചിലര് കരുതാനുള്ള കാരണമെന്ന് പറയുന്നത് അതുകൊണ്ടാണ്.
ഹദീഥുകളുടെ സ്വീകാര്യതയെ കുറിക്കുന്ന പ്രയോഗങ്ങളാണ് സ്വഹീഹ്, ദ്വഈഫ്, ഹസൻ എന്നിവ. അസ്വീകാര്യമായ ഹദീഥുകളെ കുറിക്കുവാന് ആദ്യകാലത്ത് ഉപയോഗിക്കപ്പെട്ട രണ്ടു പ്രയോഗങ്ങളായിരുന്നു 'മുന്കര്' (അസ്വീകാര്യം), 'ലയ്സ ലഹു അസ്ല്' (അതിന് അടിത്തറയൊന്നുമില്ല) എന്നിവ. ഇമാം മാലിക്കിന്റെ കാലം മുതല് തന്നെ സ്വഹീഹ് (പ്രാമാണികം), ദ്വഈഫ് (ദുര്ബലം) എന്നീ ശബ്ദങ്ങളുപയോഗിച്ച് ഹദീഥുകളെ വര്ഗീകരിക്കാനാരംഭിച്ചിരുന്നു. ഹദീഥുകളുടെ ദൃഢീകരണത്തിന്റെ അടിസ്ഥാനത്തില് മശ്ഹൂര് (സുപ്രസിദ്ധം) എന്നും മുന്കര് (അസ്വീകാര്യം) എന്നും തിരിച്ചു കൊണ്ടുള്ള വര്ഗീകരണവും അക്കാലത്തു തന്നെ നിലവിലുണ്ടായിരുന്നു.
സ്വഹീഹ്, മശ്ഹൂര് എന്നീ പ്രയോഗങ്ങള് സ്വീകാര്യതയെയും ദ്വഈഫ്, മുന്കര് എന്നിവ അസ്വീകാര്യത യെയും കുറിക്കുന്നു. ഒരു ഋജുവായ (ആദില്) നിവേദകന് അതേപോലെത്തന്നെ സത്യസന്ധനായ നിവേദകനില് നിന്ന് എന്ന രൂപത്തില് പ്രവാചകന് വരെ നീളുന്ന മുറിയാത്ത സനദോടു കൂടിയത് എന്നാണ് സ്വഹീഹായ ഹദീഥിന് ഇമാം ഇബ്നു ഖുസൈമ തന്റെ സ്വഹീ ഹില് നല്കിയിട്ടുള്ള നിര്വചനം. സ്വഹീഹായ ഹദീഥിനെക്കുറിച്ച് ഇമാം ശാഫി പറയുന്നത് ഇങ്ങനെയാണ്: ''ഓരോ നിവേദകനും അയാളുടെ മതത്തില് ആത്മാര്ഥതയുള്ളവനാകണം; നിവേദനത്തില് സത്യസന്ധനും. എന്താണ് നിവേദനം ചെയ്യുന്നതെന്ന് വ്യക്തമായി അറിയുന്നവനും വ്യത്യസ്ത പ്രയോഗങ്ങള് വഴി ഭാഷയിലുണ്ടാകുന്ന അര്ഥവ്യത്യാസത്തെക്കുറിച്ച് ബോധവാനും അക്ഷരം പ്രതി ഉദ്ധരി ക്കുന്നവനുമായിരിക്കണം അയാള്. വ്യത്യസ്ത പ്രയോഗങ്ങള്വഴി ഭാഷയിലുണ്ടാകുന്ന അര്ഥവ്യത്യാസത്തെക്കുറിച്ച് മനസ്സിലാകാത്തയാ ളാണെങ്കില് തന്റെ പ്രയോഗങ്ങള് വഴി താന് അനുവദനീയമായതിനെ വിരോധിക്കുന്നുണ്ടോയെന്നോ നിഷിദ്ധമായതിനെ അനുവദനീയമാ ക്കുന്നുണ്ടോയെന്നോ അറിയാന് അയാള്ക്ക് കഴിയില്ലെന്നതു കൊണ്ടാണിത്. ഹദീഥില് നിന്ന് താന് മനസ്സിലാക്കിയതെന്തോ അതല്ല, താന് എന്ത് കേട്ടോ അത് അയാള് നിവേദനം ചെയ്യുമ്പോള് ഹദീഥില് അര്ഥവ്യത്യാസമുണ്ടാവുകയില്ല. തന്റെ ഓര്മയില് നിന്നെടുത്ത് നിവേദനം ചെയ്യുന്നയാളാണെങ്കില് നല്ല ഓര്മശക്തിയുള്ളയാളും രേഖകളില് നിന്ന് ഉദ്ധരിക്കുന്നയാളാണെങ്കില് രേഖാസംരക്ഷണത്തില് അതീവശ്രദ്ധ യുള്ളയാളുമാകണം അയാള്. അറിയപ്പെട്ട ഹദീഥ് നിവേദകന്മാരുടെ നിവേദനത്തില് പരാമര്ശിക്കപ്പെട്ട വിഷയമാണ് അയാള് നിവേദനം ചെയ്ത ഹദീഥിലുള്ളതെങ്കില് അതുമായി വൈരുധ്യം പുലര്ത്താത്ത വിധം യോജിപ്പുള്ളതാവണം. താന് നേര്ക്കു നേരെ കേട്ടിട്ടില്ലാത്തത് കേട്ടുവെന്ന് വരുത്തിത്തീര്ത്ത് നിവേദനം ചെയ്യുന്ന മുദല്ലിസോ പ്രവാചകനില് നിന്ന് വിശ്വസ്തമായ പരമ്പരയോടു കൂടി നിവേദനം ചെയ്യ പ്പെട്ട വചനങ്ങള്ക്ക് വിരുദ്ധമായ കാര്യങ്ങള് നിവേദനം ചെയ്യുന്നയാളോ ആകരുത് അയാള്. ഇവിടെ പറഞ്ഞ രീതിയിലുള്ള നിവേദകന് മാര് മാത്രമുള്ള നബി(സ) വരെയെത്തുന്ന മുറിയാത്ത ശൃംഖലയോടു കൂടിയ ഇസ്നാദുള്ള ഹദീഥുകളാണ് സ്വഹീഹ്''
ആദ്യകാലത്തെ ഹദീഥ് വിഭജനത്തില് സ്വഹീഹ്, ദ്വഈഫ് എന്നിങ്ങനെ മാത്രമെയുണ്ടായിരുന്നുള്ളൂ. നടേപറഞ്ഞ ഗുണഗണങ്ങളുള്ളവ സ്വഹീഹും അല്ലാത്തവ ദ്വഈഫും എന്ന രൂപത്തിലായിരുന്നു വര്ഗീകരിക്കപ്പെട്ടിരുന്നത്. ഇസ്നാദിന്റെ നിഷ്കൃഷ്ടമായ പരിശോധനയില് ചെറിയ പ്രശ്നങ്ങളുള്ളവ പോലും ദ്വഈഫായാണ് വിലയിരുത്തപ്പെട്ടിരുന്നത്. അതുകൊണ്ടാണ് പ്രബലമായ മറ്റു തെളിവുകള് ലഭ്യമല്ലെ ങ്കില് ദ്വഈഫായ ഹദീഥുകളുടെ അടിസ്ഥാനത്തില് മതവിധി നിര്ണയിക്കാമെന്ന് ഇമാം അഹ്മദ് ബിന് ഹന്ബല് അഭിപ്രായപ്പെട്ടത്. സ്വഹീ ഹായ ഇസ്നാദില്ലെങ്കിലും മതവിധി നിര്ണയിക്കാന് ഉപയുക്തമായ വിധം വിശ്വസനീയമായത്, പൂര്ണമായും അസ്വീകാര്യമായതും ഒഴിവാക്കപ്പെടേണ്ടതുമായത് എന്നിങ്ങനെ രണ്ടുതരം ദ്വഈഫുകളുണ്ടായിരുന്നുവെന്ന് ഇമാം ഇബ്നു തൈമിയ വ്യക്തമാക്കുന്നുണ്ട്.
ഹിജ്റ 279ല് അന്തരിച്ച, ഇമാം ബുഖാരിയുടെ ശിഷ്യനും പ്രസിദ്ധമായ ആറ് ഹദീഥ് ഗ്രന്ഥങ്ങളിലൊന്നിന്റെ കര്ത്താവുമായ അബൂഈസാ മുഹമ്മദ്ബിനു ഈസാ അത്തിര്മിദിയാണ് സ്വഹീഹിന്റെ മാനദണ്ഡങ്ങള് പാലിക്കുന്നില്ലെങ്കിലും മതവിധി നിര്ണയിക്കുവാനായി ഉപയോഗിക്കാനാവുന്ന ഹദീഥുകളെ ഹസന് (കുഴപ്പമില്ലാത്തത്) എന്ന പേരില് ആദ്യമായി വിളിച്ചത്. തന്റെ ഹദീഥ് സമാഹാരത്തിന്റെ ആമുഖത്തില് എന്താണ് ഹസനെന്നും എങ്ങനെയുള്ള ഹദീഥുകളെയാണ് ഹസനായി പരിഗണിക്കാനാവുകയെന്നും അദ്ദേഹം വിശദമായി വിവരിക്കുന്നുണ്ട്.‘കളവോ വ്യാജനിര്മിതിയോ ആരോപിക്കപ്പെടാത്തവര് മാത്രം ഉള്ക്കൊള്ളുന്ന സനദോടു കൂടിയതും യോഗ്യതയു ള്ളവരുടെ നിവേദനത്തിന് വിരുദ്ധമായത് (ശാദ്ദ്) അല്ലാത്തതും ഒന്നിലധികം ശൃംഖലയോടെ നിവേദനം ചെയ്യപ്പെട്ടതുമായ ഹദീഥാണ് 'ഹസന്'എന്നാണ് അദ്ദേഹത്തിന്റെ നിര്വചനം. ഹസനായ ഹദീഥുകള് രണ്ടുതരമാണെന്നും അശ്രദ്ധരും അമിതമായി അബദ്ധങ്ങള് പിണയുന്നവരും കളവു പറഞ്ഞേക്കാമെന്ന് സംശയിക്കപ്പെടുന്നവരുമല്ലെങ്കിലും അര്ഹതയെക്കുറിച്ച് ശരിക്കും അറിയപ്പെട്ടിട്ടില്ലാത്ത ഒരാള് സനദില് ഉള്പെട്ടിരിക്കുവാന് സാധ്യതയുള്ളതും അതേപ്രകാരമോ അതിനോട് സമാനമായ രീതിയിലോ വേറെവഴിക്ക് നിവേദനം ചെയ്യപ്പെട്ടതുമായ ഹദീഥുകളും സത്യസന്ധതയിലും വിശ്വസ്തതയിലും പ്രസിദ്ധനാണെങ്കിലും മനഃപാഠത്തിലും സൂക്ഷ്മതയിലും സ്വഹീ ഹിന്റെ സ്ഥാനം കൈവരിച്ചിട്ടില്ലാത്ത നിവേദകനിലൂടെ കടന്നുവന്നതും ആക്ഷേപവിധേയമാകാത്ത ഇസ്നാദോടുകൂടിയതും വിശാസ യോഗ്യ നിവേദനങ്ങള്ക്ക് വിരുദ്ധമാകാത്തതും കേടുപാടുകളില്ലാത്തതുമായ 'മത്ന്' ഉള്ക്കൊള്ളുന്ന ഹദീഥുകളുമാണ് 'ഹസന്' ആയി പരിഗണിക്കപ്പെടുന്ന രണ്ട് വിഭാഗങ്ങളെന്ന് ഹദീഥ് പണ്ഡിതനായ ഇബ്നുസ്സ്വലാഹ് വിശദീകരിച്ചിട്ടുണ്ട്.
ഇസ്നാദുകള് പരിശോധിച്ചതോടൊപ്പം തന്നെ ഹദീഥിന്റെ ആശയപ്രധാന ഭാഗമായ മത്നും ഹദീഥ് പണ്ഡിതന്മാരുടെ അപഗ്രഥനത്തിന് വിധേയമായിട്ടുണ്ട്.
ഭാഷാസാഹിത്യത്തിന് യോജിക്കാത്തവിധം താഴ്ന്ന നിലവാരത്തിലുള്ള പദപ്രയോഗങ്ങള് ഉള്ക്കൊള്ളുന്നതോ വ്യാഖ്യാനത്തിന് സാധ്യമല്ലാത്ത വിധം പ്രാഥമികബുദ്ധിക്ക് ഉള്ക്കൊള്ളാനാവാത്തതോ അനുഭവത്തിനും സാക്ഷ്യത്തിനും എതിരായതോ നിയമങ്ങളിലും സ്വഭാവഗുണങ്ങളിലുമുള്ള പൊതുതത്ത്വങ്ങള്ക്ക് വിരുദ്ധമായതോ ശാസ്ത്രീയമായി തെളിയിക്കപ്പെട്ട സ്പഷ്ടമായ കാര്യ ങ്ങളോട് യോജിക്കാത്തതോ ഇസ്ലാമിക നിയമ വ്യവസ്ഥയുടെ ആത്മാവിന് നിരയ്ക്കാത്തവിധം നീചമായ കാര്യങ്ങള്ക്ക് പ്രേരണനല്കു ന്നതോ അല്ലാഹുവിന്റ നടപടിക്രമങ്ങള്ക്ക് എതിരായ പരാമര്ശങ്ങളുള്ക്കൊള്ളുന്നതോ മാന്യന്മാര്ക്ക് ചെയ്യാന് മടിയുള്ള നികൃഷ്ട ഗുണങ്ങള് ഉള്ക്കൊള്ളുന്നതോ ക്വുര്ആനിനോടോ സ്ഥിരീകരിക്കപ്പെട്ട സുന്നത്തിനോടോ പണ്ഡിതന്മാരുടെ ഐകകണ്ഠേനയുള്ള അഭിപ്രാ യമായ ഇജ്മാഇനോടോ വ്യാഖ്യാനത്തിന് സാധ്യമല്ലാത്തവിധം എതിരായതോ നബി(സ)യുടെ കാലത്തെ ചരിത്രത്തിന് വിരുദ്ധമായതോ ചെറുതും നിസ്സാരവുമായ കര്മങ്ങള്ക്ക് വളരെ വലിയ പ്രതിഫലമോ കഠിനശിക്ഷയോ ഉണ്ടെന്ന് വിളംബരം ചെയ്യുന്നതോ ആയ ഹദീഥുക ളെ അസ്വീകാര്യമായവയുടെ ഗണത്തിലാണ് ആദ്യകാലം മുതല് തന്നെ പണ്ഡിതന്മാര് ഉള്പ്പെടുത്തിപ്പോന്നിട്ടുള്ളത്. അഥവാ ഇവയൊക്കെ വ്യാജ ഹദീഥുകളുടെ ലക്ഷണങ്ങളായി കണ്ടിരുന്നുവെന്ന് സാരം.
എന്നാല് കേവലബുദ്ധിയുടെയോ യുക്തിയിടെയോ മാത്രം അടിസ്ഥാനത്തി ലുള്ള നടപടിയായിരുന്നില്ല ഇത്. നബി(സ)ക്ക് ദിവ്യബോധനമായി ലഭിക്കുന്ന അറിവുകളെ മനുഷ്യയുക്തിയുടെ ചട്ടകള്ക്കുള്ളില് ഒതു ക്കാന് കഴിയില്ലെന്ന് അവര്ക്കറിയാമായിരുന്നു. അതുകൊണ്ട് തന്നെ 'മത്ന്' അപഗ്രഥിച്ചുകൊണ്ട് ഹദീഥുകളുടെ സ്വീകാര്യതയെപ്പറ്റി അഭിപ്രായം പറയുന്നതിന് മുമ്പ് തങ്ങള് മനസ്സിലാക്കിയതല്ലാത്ത അര്ഥങ്ങളെന്തെങ്കിലും അതിനുണ്ടോയെന്നും വ്യാഖ്യാനിക്കുവാന് പഴുതുകളെന്തെങ്കിലും അവശേഷിക്കുന്നുണ്ടോയെന്നും വിശദമായി അവര് പരിശോധിച്ചിരുന്നു. മത്ന് വിമര്ശനത്തിലൂടെ മാത്രമായി ഹദീഥുകള് തള്ളിക്കളയുകയെന്നതിലുപരിയായി അവയുടെ ഇസ്നാദുകള് കൂടി പരിശോധിക്കുകയും അവ അസ്വീകാര്യമാണെന്ന് ബോധ്യപ്പെടുകയും ചെയ്തശേഷം മാത്രമാണ് അത്തരം ഹദീഥുകള് സ്വീകരിക്കാതെ മാറ്റിനിര്ത്തപ്പെട്ടത്. മത്നില് തകരാറുള്ളതുകൊണ്ട് സ്വീകരിക്കാതിരുന്ന ഹദീഥുകള്ക്കുള്ള ഉദാഹരണമായി പറയപ്പെടുന്നവയെല്ലാം ദുര്ബലമായ ഇസ്നാദോടുകൂടിയവയാണ്. പ്രബലമായ ഇസ്നാദോടെ നിവേദനം ചെയ്യപ്പെട്ട ഹദീഥുകളിലൊന്നിലും തള്ളപ്പെടേണ്ട തരത്തിലുള്ള മത്നുകളുള്ളതായി ഹദീഥ് പണ്ഡിതന്മാര് കരുതിയിരുന്നില്ലന്നര്ഥം.
ഇസ്നാദ് പരിശോധനയും നിവേദകന്മാരെക്കുറിച്ച നിഷ്കൃഷ്ടമായ അപഗ്രഥനവും കഴിഞ്ഞാൽ,നിവേദകപരമ്പരയെ ബലപ്പെടുത്തുന്ന മറ്റു തെളിവുകൾ കണ്ടെത്തുന്നതിനു വേണ്ടിയുള്ള പരിശ്രമമാണ് നടക്കുന്നത്.നിവേദകപരമ്പരയെ ബലപ്പെടുത്തുന്ന മറ്റു തെളിവുകള് കണ്ടെത്തുന്നതിന് ദൃഢീകരണം (ഇഅ്തിബാര്) എന്നാണ് പറയുക.
ഇസ്നാദിലു ള്ള ഓരോ നിവേദകനെയും ബലപ്പെടുത്തുന്ന തെളിവുകളുണ്ടോയെന്ന അന്വേഷണമാണിത്. ഒരു ഗുരുവില് നിന്ന് ഒരേയൊരു ശിഷ്യന് മാത്രം ഒരു ഹദീഥ് നിവേദനം ചെയ്യുകയും പ്രസ്തുത ഹദീഥ് പറഞ്ഞു കൊടുത്തിട്ടുണ്ടെങ്കില്, അങ്ങനെ ചെയ്യുമ്പോള് സദസ്സിലുണ്ടായിരു ന്നിരിക്കേണ്ട മറ്റൊരാളും അത് നിവേദനം ചെയ്യാതിരിക്കുകയും പ്രസ്തുത ചോദ്യങ്ങള്ക്ക് തൃപ്തികരമായ ഉത്തരം കണ്ടെത്താന് കഴി ഞ്ഞിട്ടില്ലെങ്കില് നിവേദകന്റെ വിശ്വാസ്യതയാണ് തകരുന്നത്; ഒപ്പം ഹദീഥ് ദുര്ബലമായി വിലയിരുത്തപ്പെടുകയും ചെയ്യുന്നു.
ഇസ്നാദിലെ നിവേദകന്മാരെ ദൃഢീകരിക്കുന്നത് രണ്ടു രൂപത്തിലാണ്. ഒരു സ്വഹാബിയില് നിന്ന് ഉദ്ധരിക്കപ്പെട്ട ഒരു ഹദീഥിന്റെ നിവേ ദക പരമ്പരയില് എവിടെയെങ്കിലും ഒന്നിലധികം നിവേദകന്മാരുണ്ടെങ്കില് അവരിലൂടെ മറ്റൊരു ഇസ്നാദില് അതേ ഹദീഥ് നിവേദനം ചെയ്യപ്പെട്ടിട്ടുണ്ടോ എന്ന അന്വേഷണമാണ് ഒന്നാമത്തേത്. അങ്ങനെയുണ്ടെങ്കില് അതിന് പൊരുത്തം (മുതാബഅ) എന്നു പറയുന്നു. ഒരു സ്വഹാബിയില് നിന്ന് ഒരു പ്രത്യേകമായ ഇസ്നാദോടുകൂടി നിവേദനം ചെയ്തിട്ടുള്ള ഹദീഥ് മറ്റൊരു സ്വഹാബിയില് നിന്ന് മറ്റൊരു ഇസ് നാദോടുകൂടി നിവേദനം ചെയ്തിട്ടുണ്ടോയെന്ന അന്വേഷണമാണ് രണ്ടാമത്തേത്. അങ്ങനെയുണ്ടെങ്കില് ഒന്നാമത്തെ ഹദീഥിന്റെ സാക്ഷി (ശാഹിദ്) ആണ് രണ്ടാമത്തെ ഹദീഥ് എന്ന് പറയാവുന്നതാണ്. മുതാബഅ നിവേദക പരമ്പരയെയും ശാഹിദ് ഹദീഥിനെയും ബലപ്പെടു ത്തുന്നുവെന്നാണ് ഹദീഥ് നിദാനശാസ്ത്രജ്ഞര് പറയുക. ഇസ്നാദിന്റെ ന്യൂനതകള് പരിഹരിക്കാവുന്ന യാതൊരു തെളിവുകളുമില്ലെ ങ്കില് അത്തരം ഹദീഥുകളെ അസ്വീകാര്യമായാണ് ആദ്യകാല ഹദീഥ് പണ്ഡിതന്മാര് കണ്ടിരുന്നത്. 'സ്വീകരിക്കാന് പറ്റാത്തത്' എന്ന അര്ഥ ത്തില് അവര് അവയെ 'മുന്കര്' എന്നു വിളിച്ചു മാറ്റിവെച്ചു. ദൃഢീകരിക്കുന്ന തെളിവുകളൊന്നുമില്ലെങ്കിലും ഒരു ഹദീഥ് സ്വീകാര്യമായ മറ്റു നിവേദകന്മാരുടെ ഹദീഥിലെ ആശയവുമായി പൊരുത്തപ്പെടുന്നുണ്ടെങ്കില് അത് സ്വീകാര്യമാണെന്നാണ് പണ്ഡിതന്മാര് വിധിച്ചത്. എന്നാല് പ്രസിദ്ധനല്ലാത്ത ഒരു നിവേദകന് ഇമാം സുഹ്രിയെപ്പോലെയുള്ള പ്രസിദ്ധനും പ്രഗല്ഭനുമായ ഒരു ഹദീഥ് പണ്ഡിതനില് നിന്ന് ഒരു ഹദീഥ് നിവേദനം ചെയ്യുകയും അത് ധാരാളം വരുന്ന അദ്ദേഹത്തിന്റെ ശിഷ്യഗണങ്ങളൊന്നും അറിയാതെ പോവുകയും ചെയ്തിട്ടു ണ്ടെങ്കില് അത് മുന്കറിന്റെ ഗണത്തിലാണ് ഉള്പ്പെടുക.(സ്വഹീഹു മുസ്ലിം, മുഖദ്ദിമ)
ഒരു നിവേദകനിലൂടെ നിരവധി ഹദീഥുകള് ഉദ്ധരിക്കപ്പെടുകയും അവയിലധികവും ദൃഢീകരിക്കപ്പെടുന്ന തെളിവുകളാല് സമൃദ്ധവുമാ ണെങ്കില് അയാളിലൂടെയുള്ള ദൃഢീകരിക്കപ്പെടാത്ത ഹദീഥുകളും സ്വീകരിക്കാമെന്നാണ് പണ്ഡിതാഭിപ്രായം. ഇമാമുമാര് സുഹ്രി, മാലിക്ക്, ഇബ്നുല് മുബാറക്, ഖുതൈബതുബ്നു സഈദ് എന്നിവരിലൂടെ നിവേദനം ചെയ്യപ്പെട്ട ഹദീഥുകളെ ഇമാം ബുഖാരിയെയും ഇബ്നു ആമിയെയും പോലെയുള്ള പണ്ഡിതന്മാര് അവഗാഢമായ അപഗ്രഥനത്തിന് വിധേയമാക്കുകയും അവരിലൂടെയുള്ള ദൃഢീകരി ക്കപ്പെടാത്ത ഹദീഥുകളും സ്വീകാര്യമാണെന്ന തീരുമാനത്തിലെത്തിച്ചേരുകയും ചെയ്തിട്ടുണ്ട്. നിഷേധിക്കാനാവാത്ത തെളിവുകളാല് സ്വീകാര്യമെന്ന് നിദേവകന്മാരിലൂടെ ഉദ്ധരിക്കപ്പെട്ട ദൃഢീകരിക്കപ്പെടാത്ത ഹദീഥുകളെ 'സ്വീകാര്യമായ അപൂര്വ' (സ്വഹീഹ് ഗരീബ്) ഹദീഥുകള് എന്നാണ് വിളിക്കുന്നത്. നിവേദക പരമ്പരയില് മുഴുവന് ഘട്ടങ്ങളിലോ ചിലതിലോ ഒരാള് മാത്രമായിപ്പോകുന്ന ഹദീഥുകള് ക്കാണ് 'ഗരീബ്' എന്നു പറയുക. ദൈവദൂതന് ശിരോകവചം ധരിച്ച് മക്കയില് പ്രവേശിക്കുകയും മുസ്ലിംകളുടെ ഗൂഢശത്രുവായിരുന്ന ഇബ്നുഖത്താലിനെ വധിക്കുവാന് കല്പിക്കുകയും ചെയ്തു(ജാമിഉത്തിര്മിദി, കിതാബുല് ജിഹാദ്, ബാബ് മാജാഅ ഫില് മിഗ്ഫാര്) വെന്ന ഹദീഥ് ഉദാഹരണം. ഇതിന് അനസ്ബ്നു മാലിക് aസുഹ്രി aമാലിക് ബ്നുഅനസ് എന്ന ഒരേയൊരു ഇസ്നാദ് മാത്രമെയുള്ളുവെ ങ്കിലും ഈ ശൃംഖലയിലുള്ള മൂന്നുപേരും ദൃഢീകരണം ആവശ്യമില്ലാത്ത വിധം പ്രസിദ്ധരായതിനാല് അത് സ്വീകാര്യമാണെന്നാണ് പണ്ഡി തമതം. എന്നാല് ഹദീഥുകള് നിവേദനം ചെയ്യുന്ന കാര്യത്തില് സൂക്ഷമതയില്ലാത്തവരായ ഒരാളെങ്കിലും ഇസ്നാദിലുണ്ടാവുകയും അതിന് ദൃഢീകരിക്കാനാവുന്ന മറ്റു തെളിവുകള് ഇല്ലാതിരിക്കുകയും ചെയ്താല് ഹദീഥ് അസ്വീകാര്യമാണെന്നാണ് (മുന്കര്) വിധി.
സംശയം ജനിപ്പിക്കാത്ത ഇസ്നാദോടുകൂടി നിവേദനം ചെയ്യപ്പെട്ട ഹദീഥുകളെപ്പോലും നിഷ്കൃഷ്ടമായ അപഗ്രഥനത്തിന് വിധേയമാക്കു വാന് ഹദീഥ് നിദാനശാസ്ത്രജ്ഞര് സന്നദ്ധമായിട്ടുണ്ട്. ഒരേ ഹദീഥിന്റെ വ്യത്യസ്ത നിവേദനങ്ങളെ താരതമ്യം ചെയ്ത് നിവേദകര്ക്ക് സംഭവിച്ച സ്വാഭാവികവും മാനുഷികവുമായ പാളിച്ചകളെപ്പോലും പുറത്തുകൊണ്ടുവരുവാനുള്ള അവരുടെ കഠിനാധ്വാനം വിലമതി ക്കാനാവാത്തതാണ്. ഇത്തരം പാളിച്ചകളെയാണ് 'ഇലല്'(ന്യൂനതകള്) എന്നു പറയുക. ഹിജ്റ 385ല് അന്തരിച്ച ഇമാം അബുല് ഹസന് അലിയ്യിബിന് ഉമര് അല് ദാറഖുത്നിയുടെ പതിനൊന്ന് വാല്യങ്ങളുള്ള ഇലല് ഗ്രന്ഥമാണ് ഇലലുകളെക്കുറിച്ച് വിശദമായി അപഗ്രഥിക്കു ന്നവയില് ഏറ്റവും പ്രസിദ്ധമായത്.
നബി(സ)യിൽ നിന്ന് നാലും അഞ്ചും പേരിലൂടെ കടന്നു വന്ന് ബുഖാരിയുടെയും മുസ്ലിമിന്റെയുമെല്ലാം അടുത്തെത്തുന്ന ഹദീഥുകൾ, ഈ നിവേദകരെല്ലാം സത്യസന്ധരാണെങ്കിലും, ഇവ സംപ്രേക്ഷണം ചെയ്യുന്നതിൽ അബദ്ധങ്ങൾ വരാനുള്ള സാധ്യത നിഷേധിക്കാനാവുമോ? ഇവരെല്ലാം പരസ്പരം കേട്ടതിനു ശേഷമാണ് ഹദീഥുകൾ നിവേദനം ചെയ്തതെന്ന് ഉറപ്പു വരുത്തുന്നതെങ്ങനെ?
നിവേദകന്മാരെക്കുറിച്ച അപഗ്രഥനമാണ് ഹദീഥ്പരിശോധനയുടെ ഒന്നാം പടി. നബിയിൽ നിന്ന് സൂക്ഷ്മവും സത്യസന്ധവുമായി നിവേദനം ചെയ്യപ്പെട്ടതാണോ ഹദീഥ് എന്ന അന്വേഷണമാണത്. നിവേദകരെല്ലാം സത്യസന്ധരും സ്വീകാര്യരുമാണെന്ന് മനസ്സിലാക്കിയാലും ഒരു ഹദീഥിന്റെ സ്വീകാര്യത ആത്യന്തികമായി തീരുമാനിക്കപ്പെടുന്നില്ല. അതിന് നിവേദനത്തിന്റെ നൈരന്തര്യം (അല് ഇത്തിസാല്) കൂടി പരിശോധിക്കപ്പെ ടേണ്ടതുണ്ട്. മുഹമ്മദ് നബി(സ)യില് നിന്ന് തുടങ്ങി ഹദീഥ് ശേഖരിക്കുന്നയാള്വരെ ഇസ്നാദിലുള്ള വ്യക്തികളെല്ലാം പരസ്പരം കാണുക യോ ഹദീഥ് കേള്ക്കുകയോ ചെയ്തിട്ടുണ്ടോയെന്ന അന്വേഷണമാണിത്. ഈ അന്വേഷണത്തിന്, ഇസ്നാദിന്റെ ശൃംഖലയിലുള്ള ആരെങ്കി ലും പരസ്പരം കണ്ടുമുട്ടുകയോ ഹദീഥ് കൈമാറുകയോ ചെയ്തിട്ടില്ലെന്ന് മനസ്സിലായാല് ആ ഇസ്നാദ് പരമ്പരമുറിഞ്ഞതാണെന്ന് (മുന് ക്വത്വിഅ്) വിധിക്കുകയും അസ്വീകാര്യമാണെന്ന് തീരുമാനിക്കുകയും ചെയ്യുന്നു. ഒരു ഹദീഥിന്റെ ഇസ്നാദ് നബി(സ) →A →B →C →D എന്നിങ്ങനെയാണെങ്കില് നബി(സ)യെ Aയും Aയെ Bയും Bയെ Cയും Cയെ Dയും കാണുകയോ സമകാലികരാണെന്ന് സ്ഥാപിക്കപ്പെടു കയോ ചെയ്യുകയും അവര് ഹദീഥ് കൈമാറിയിട്ടുണ്ടെന്ന് ഉറപ്പാവുകയും ചെയ്യുമ്പോള് മാത്രമെ പ്രസ്തുത ഇസ്നാദ് അവിച്ഛിന്നമാണെന്ന് (മുത്തസ്വില്) തീരുമാനിക്കുകയും ഹദീഥ് സ്വീകരിക്കപ്പെടുകയും ചെയ്യുകയുള്ളൂ.
കളവ് പറയുകയില്ലെന്ന് അദാലത്ത് പരിശോധന വഴി ബോധ്യപ്പെട്ട നിവേദകന്മാരുടെ നൈരന്തര്യം തീരുമാനിക്കാന് അവരുടെ പദപ്ര യോഗങ്ങളെയാണ് പ്രാഥമികമായി പഠനവിധേയമാക്കുന്നത്. നിവേദകന്മാര് പൊതുവായി തങ്ങള്ക്ക് ഹദീഥ് ലഭിച്ചതിനെ സൂചിപ്പിക്കു മ്പോള് പറയാറുള്ളത് 'ഇന്നയാള് എന്നോട് നിവേദനം ചെയ്തു' (ഹദ്ദഥനീ) വെന്നോ 'ഇന്നയാള് എന്നെ അറിയിച്ചു' (അഖ്ബറനീ) യെന്നോ 'ഇന്നയാളില്നിന്ന് ഞാന് കേട്ടു' (സമിഅ്ത്തുമിന്)വെന്നോ 'ഇന്നയാള് പ്രകാരം' (അന്) എന്നോ ആണ്. ഇതിലെ ആദ്യത്തെ മൂന്നു പ്രയോഗ ങ്ങളും നേര്ക്കുനേരെയുള്ള സംപ്രേഷണത്തെയാണ് കുറിക്കുന്നത്. ഒരാളുടെ പേരുപറഞ്ഞുകൊണ്ട് ഹദ്ദഥനീയെന്നോ, അഖ്ബറനീയെന്നോ, സമിഅ്ത്തുമിന് എന്നോ സത്യസന്ധനായ ഒരു നിവേദകന് പറയുകയാണെങ്കില് അയാളില്നിന്ന് നേര്ക്കുനേരെ നിവേദനകന് ഈ ഹദീഥ് കേള്ക്കുകയോ മനസ്സിലാക്കുകയോ ചെയ്തിട്ടുണ്ടെന്നാണ് അതിനര്ഥം.
എന്നാല് നാലാമത്തെ പ്രയോഗമായ 'അന്' നേര്ക്കു നേരെയുള്ള സംപ്രേക്ഷണം ഉറപ്പുവരുത്തുന്നില്ല. ഒരാള് പറഞ്ഞതായി മറ്റൊരാളില്നിന്ന് അറിഞ്ഞാലും 'അന്' എന്ന് പ്രയോഗിക്കാവുന്നതാണ്. അത്തരം പ്രയോഗങ്ങളുള്ള ഇസ്നാദുകളുള്ക്കൊള്ളുന്ന ഹദീഥുകള് മുത്തസ്വിലാണെന്ന് ഉറപ്പിക്കുവാനാവുകയില്ല. അങ്ങനെ പറഞ്ഞ നിവേദകനും (ശിഷ്യന്) അയാള് ആരില്നിന്നാണോ അത് ഉദ്ധരിക്കുന്നത് അയാളും (ഗുരു) പരസ്പരം കാണുകയും സംസാരിക്കുകയും ചെയ്തിട്ടുേണ്ടായെന്നുകൂടി പരിശോധിച്ചതിനുശേഷമാണ് അത്തരം ഹദീഥുകളുടെ സ്വീകാര്യത നിര്ണയിക്കുക. അതിനായി അവര് രണ്ടു പേരുടെയും ജീവിതകാലവും ജനന-മരണത്തീയതികളും ജീവിച്ച സ്ഥലങ്ങളും പഠനസമ്പ്രദായങ്ങളുമെല്ലാം അപഗ്രഥിക്കപ്പെടുന്നു. ഗുരു വും ശിഷ്യനും സമകാലികരാണെങ്കില് ഒരാളില്നിന്ന് മറ്റേയാള് കേട്ടിരിക്കുവാന് സാധ്യതയുണ്ടെന്ന് മനസ്സിലാക്കി, അവരുടെ സത്യസന്ധ തകൂടി കണക്കിലെടുത്ത് അവയെ മുത്തസ്വിലായി പരിഗണിക്കുകയും അല്ലെങ്കില് മുന്ക്വത്വിഅ് ആയി മാറ്റിനിര്ത്തപ്പെടുകയുമാണ് ചെയ്യുന്നത്.
ഒരാള് മറ്റൊരാളില് നിന്ന് കേട്ടുവെന്ന് പറയുമ്പോള് രണ്ടു പേരും അല്പകാലമെങ്കിലും ഒന്നിച്ചുണ്ടാവണമെന്നതുകൊണ്ടാണ് പരസ്പരം കണ്ടുമുട്ടിയിട്ടില്ലെന്ന് ഉറപ്പുള്ളവര് ഒരു ഹദീഥ് സനദിന്റെ ശൃംഖലയില് അടുത്ത കണ്ണികളായുണ്ടെങ്കില് അത്തരം ഹദീഥുകളെ മുറിഞ്ഞ ഇസ്നാദോടുകൂടിയുള്ളതായി പരിഗണിച്ച് മാറ്റിനിര്ത്തപ്പെടുന്നത്. നിവേദകന്മാര് ജീവിച്ചിരുന്ന കാലവും ബന്ധപ്പെടാനുള്ള സാധ്യതയും മാത്രമല്ല, അവര് യഥാര്ഥത്തില് ഹദീഥ് കൈമാറിയിട്ടുണ്ടോ എന്നു കൂടി സൂക്ഷ്മമായി പരിശോധിക്കുവാന് പണ്ഡിതന്മാര് പരിശ്രമിച്ചി ട്ടുണ്ട്. ഇതിന്റെ ഫലമായിട്ടാണ് 'തമസ്കരണ'ത്തെ (തദ്ലീസ്)ക്കുറിച്ച ചര്ച്ചകളുണ്ടായത്. ഒരു നിവേദകന് ഇന്നയാള് പറഞ്ഞു(ക്വാല)വെ ന്നോ ഇന്നയാളിന് പ്രകാരം (അന്) എന്നോ പറഞ്ഞുകൊണ്ട് പറഞ്ഞ വ്യക്തിയില് നിന്ന് താന് അത് കേട്ടിട്ടുണ്ടെന്ന് വരുത്തിത്തീര്ക്കുകയും യഥാര്ഥത്തില് അയാള് പറഞ്ഞത് മറ്റൊരാള് ഉദ്ധരിച്ചതാണ് താന് കേട്ടതെന്ന വസ്തുത മറച്ചുവെക്കുകയും ചെയ്യുന്നതിനാണ് 'തദ്ലീസ്' എന്നു പറയുക. C നിവേദനം ചെയ്യുന്നത് A പറഞ്ഞുവെന്നാണ്; പക്ഷേ, C കേട്ടിരിക്കുന്നത് Aയില് നിന്ന് നേരിട്ടല്ല; പ്രത്യുത A പറഞ്ഞതായി B യില്നിന്നാണ്. Bയുടെ പേര് മറച്ചുവെച്ചുകൊണ്ട് A യില്നിന്ന് താന് കേട്ടുവെന്ന രീതിയില് C പറയുമ്പോള് അത് തദ്ലീസായിത്തീരുന്നു. തദ്ലീസ് ചെയ്യുന്നവരെ മുദല്ലിസ് എന്നാണ് വിളിക്കുന്നത്. പൊതുവെ വെറുക്കപ്പെട്ടതാണ് തദ്ലീസ്. താന് നേരിട്ട് കേട്ട വ്യക്തിയുടെ പേര് മറച്ചുവെക്കുന്നത് അയാള്ക്ക് എന്തെങ്കിലും ന്യൂനതയുള്ളതുകൊണ്ടായിരിക്കുമല്ലോ. ന്യൂനത മറച്ചുവെച്ചുകൊണ്ട് ഹദീഥിനെ സ്വീകരി പ്പിക്കുവാനുള്ള ശ്രമമുള്ളതിനാലാണ് തദ്ലീസ് വെറുക്കപ്പെട്ടതാവുന്നത്.
എന്നാല് തെറ്റായ ലക്ഷ്യങ്ങളോടെയല്ലാതെയും തദ്ലീസ് ചെയ്യാന് സാധ്യതയുള്ളതിനാല് മുദല്ലിസുകളെയെല്ലാം അസ്വീകാര്യരായ നിവേദകരുടെ ഗണത്തില് പണ്ഡിതന്മാര് ഉള്പ്പെടുത്തിയിട്ടില്ല. ഗുരുവിന് കീഴില് ഹദീഥ് അഭ്യസിച്ചുകൊണ്ടിരിക്കെ പ്രാഥമിക ആവശ്യത്തിനായി പോയ ഒരു ശിഷ്യന് ആ ഗുരു പറഞ്ഞുകൊടുത്ത ഹദീഥ് നേര്ക്കു നേരെ കേള്ക്കാള് കഴിഞ്ഞിട്ടില്ലെങ്കിലും തന്റെ സഹപാഠികളുടെ സാക്ഷ്യത്തില്നിന്ന് അത് ഗുരു പറഞ്ഞുവെന്ന് അയാള് മനസ്സിലാക്കു കയും ഗുരുവില്നിന്നാണെന്ന രൂപത്തില് തന്നെ അയാള് നിവേദനം ചെയ്യുന്ന അവസ്ഥയുണ്ടാവാം. തെറ്റായ ലക്ഷ്യത്തിനുവേണ്ടിയല്ലാതെ യുള്ള തദ്ലീസിനുള്ള ഉദാഹരണമാണിത്. അതുകൊണ്ടുതന്നെ തദ്ലീസ് ചെയ്യുന്ന വ്യക്തിയെയും സന്ദര്ഭത്തെയും അപഗ്രഥിച്ചുകൊണ്ടു മാത്രമെ മുദല്ലിസ് സ്വീകാര്യനാണോ അല്ലേയെന്ന് തീരുമാനിക്കപ്പെടുകയുള്ളൂ.
ഹദീഥുകൾ നബിയിൽ നിന്നുള്ളത് തന്നെയാണെന്ന് ഉറപ്പു വരുത്തുന്നതിന് വേണ്ടി എത്രത്തോളം നിഷ്കൃഷ്ടമായ പരിശോധനകളാണ് പണ്ഡിതന്മാർ നടത്തിയതെന്ന ഇതിൽ നിന്ന് മനസ്സിലാക്കാം. ഈ പരിശോധനകൾ കഴിഞ്ഞ സ്വീകാര്യമെന്ന തീരുമാനിക്കപ്പെട്ട ഹദീഥുകൾ നബിയിൽ നിന്നുള്ളത് തന്നെയാണെന്ന് ഉറപ്പിച്ച് പറയാൻ കഴിയും.
ഹദീഥ്ഗ്രൻഥങ്ങൾ ക്രോഡീകരിക്കപ്പെട്ടത് നബിക്കു ശേഷമുള്ള നാലാം തലമുറയിലും അതിനു ശേഷവുമാണല്ലോ. ഓരോ ഹദീഥുകളും ഗ്രന്ഥകർത്താക്കളുടെ അടുത്തെത്തുന്നത് നിരവധി നിവേദകരിലൂടെയാണ്. ഈ നിവേദകരെല്ലാം സത്യസന്ധരായാൽ മാത്രമാണ് പ്രസ്തുത ഹദീഥ് നബിയിൽ നിന്നുള്ളതാണെന്ന് ഉറപ്പിച്ച് പറയാൻ കഴിയുക. നിവേദകരുടെ സത്യസന്ധതയെക്കുറിച്ച കേവലം ഊഹങ്ങളല്ലാതെ ശാസ്ത്രീയമായ വല്ല തെളിവും ഹദീഥ് നിദാനശാസ്ത്രം നൽകുന്നുണ്ടോ?
ഹദീഥ് നിവേദനങ്ങൾ സത്യസന്ധം തന്നെയാണെന്ന് ഉറപ്പിക്കുവാൻ തികച്ചും ശാസ്ത്രീയവും വസ്തുനിഷ്ഠവുമായ രീതി തന്നെ ഹദീഥ് നിദാനശാസ്ത്രജ്ഞന്മാർ വികസിപ്പിച്ചിട്ടുണ്ട്. ആ രീതിക്കാണ് അൽജർഹു വ ത്തഅദീൽ എന്ന് പറയുക.
പ്രവാചകൻ മുതല് ഹദീഥുകള് ശേഖരിക്കുന്ന വ്യക്തിവരെ ആരിലൂടെയൊക്കെയാണ് ഒരു ഹദീഥ് കടന്നുവന്നിട്ടുള്ളതെന്ന് മനസ്സിലാക്കുകയാണ് ഹദീഥ് നിദാനശാസ്ത്രത്തിലെ ഇസ്നാദ് പരിശോധനയെന്ന ഒന്നാം ഘട്ടം. അത് മനസ്സിലാക്കിക്കഴിഞ്ഞാല് പിന്നെ ആ കടന്നുവന്ന വ്യക്തികളുടെ വിശ്വാസ്യതയെക്കുറിച്ച് പഠിക്കുകയും അവരിലോരോരുത്തര്ക്കും അതിനു നേരെ മുമ്പു ള്ള വ്യക്തിയില് നിന്നു തന്നെയാണോ പ്രസ്തുത ഹദീഥ് കിട്ടിയതെന്ന് പരിശോധിക്കുകയും ചെയ്യുകയാണ് ഉസ്വൂലുല് ഹദീഥിന്റെ രണ്ടാ മത്തെ അപഗ്രഥനഘട്ടം. ഇസ്നാദിലുള്ള ഓരോരുത്തരെയും കൃത്യമായി അപഗ്രഥിക്കുകയും അവര് വിശ്വസ്തരാണോയെന്ന് പരിശോധി ക്കുകയും ചെയ്യുക മാത്രമല്ല, നിവേദനത്തില് എവിടെയെങ്കിലും വിശ്വസ്തരല്ലാത്ത ആരുടെയെങ്കിലും പങ്കാളിത്തമുണ്ടോ എന്നുകൂടി ഈ ഘട്ടത്തില് വിലയിരുത്തപ്പെടുന്നു. നിവേദകനെക്കുറിച്ച അപഗ്രഥനവും നിവേദനത്തിന്റെ നൈരന്തര്യവും ഈ ഘട്ടത്തില് പരിശോധിക്ക പ്പെടേണ്ടതുണ്ട്. പ്രസ്തുത പരിശോധനയ്ക്ക് ശേഷം മാത്രമെ ഹദീഥ് സ്വീകാര്യമാണോയെന്ന് തീരുമാനിക്കുകയുള്ളൂ. ഇസ്നാദുകളുടെ പരിശോധനവഴി ഹദീഥ് പണ്ഡിതന്മാര് നിര്വഹിച്ച ദൗത്യമിതാണ്.
ഹദീഥ് നിവേദകന്മാരെക്കുറിച്ച അപഗ്രഥിച്ചുള്ള പഠനം 'വിമര്ശനവും അംഗീകാരവും' (അല്ജര് ഹു വ ത്തഅ്ദീല്) എന്ന സാങ്കേതികശബ്ദം കൊണ്ടാണ് പരിചയപ്പെടുത്തപ്പെടാറുള്ളത്. നിവേദകന്റെ വ്യക്തിത്വത്തിന്റെ വിവിധ വശ ങ്ങളെക്കുറിച്ച്, ഒരു കുറ്റാന്വേഷകന്റെ സൂക്ഷ്മതയോടെ ചോദ്യം ചെയ്യുകയും അംഗീകരിക്കാനാവുന്നവരെ മാത്രം സ്വീകരിക്കുകയും ചെയ്യുന്ന ഉസ്വൂലുല് ഹദീഥിലെ സുപ്രധാനമായ ഒരു ഘട്ടമാണിത്. നിവേദകന്റെ വ്യക്തിത്വത്തിന്റെ പൂര്ണമായ അപഗ്രഥനമാണിത്; അയാള് എത്രത്തോളം സ്വീകാര്യമായ വ്യക്തിത്വത്തിന്റെ ഉടമയാണ് (അദാലത്ത്) എന്നും അദ്ദേഹത്തിലൂടെയുള്ള നിവേദനങ്ങള് എത്ര ത്തോളം കൃത്യമാണ് (ദ്വബ്ത്) എന്നുമുള്ള അന്വേഷണം.
സ്വഹാബികള്ക്കു ശേഷമുള്ള തലമുറയായ താബിഉകളുടെ കാലത്ത് വിശദമായ രീതിയിലല്ലെങ്കിലും ഹദീഥുകളിലെ നെല്ലും പതിരും വേര് തിരിക്കുന്നതിനു വേണ്ടിയുള്ള പ്രവര്ത്തനങ്ങള്ക്ക് തുടക്കം കുറിക്കപ്പെട്ടിരുന്നു. അടുത്ത തലമുറകളിലും ഈ ശ്രമം തുടർന്നു. ഹിജ്റ രണ്ടാം നൂറ്റാണ്ടിനും നാലാം നൂറ്റാണ്ടിനുമിടയിൽ ഹദീഥ് പഠന-ഗവേഷണ രംഗത്തെ സുവര്ണകാലമായി അറിയപ്പെടുന്ന കാലത്ത് ഹദീഥ് നിദാനശാസ്ത്രത്തിന് മഹത്തായ സംഭാവനകളര്പ്പിച്ച നിരവധി മഹാപ്രതിഭകൽ ജീവിച്ചിരുന്നിട്ടുണ്ട്. . എങ്ങനെയാണ് ഈ മഹാപ്രതിഭകള് ഹദീഥ് നിവേദകന്മാരുടെ സ്വീകാര്യത പരിശോധിച്ചതെന്ന് മനസ്സിലാക്കുമ്പോള് ആധുനിക കുറ്റാന്വേ ഷകരുടേതിനെക്കാള് കുറ്റമറ്റ രീതിയിലായിരുന്നു അവരുടേത് എന്ന വസ്തുത ആര്ക്കും അംഗീകരിക്കേണ്ടിവരും.
ഒരു ഹദീഥിന്റെ നിവേ ദകന്മാര് ആരൊക്കെയാണെന്ന് പരിശോധിക്കുകയും അവരെക്കുറിച്ച് ലഭ്യമായ അറിവുകളെല്ലാം ശേഖരിക്കുകയുമാണ് ഒന്നാമതായി ചെയ്യുന്നത്. നിവേദകന്മാരായി അറിയപ്പെടുന്നവരില് എല്ലാവരും ജീവിച്ചിരുന്നുവെന്നും അവര് ഹദീഥുകള് നിവേദനം ചെയ്തിട്ടുണ്ടെ ന്നും ഉറപ്പുവരുത്തുകയാണ് അടുത്തപടി. അവരില് ഓരോരുത്തരെയും പ്രസിദ്ധരായ ഹദീഥ് നിവേദകര്ക്ക് പരിചയമുണ്ടെങ്കില് മാത്രമെ അവരിലൂടെയുള്ള ഹദീഥുകള് പരിശോധനക്കായി പരിഗണിക്കുകയുള്ളൂ. അങ്ങനെയല്ലെങ്കില് നിവേദകന് അജ്ഞാതനാണെന്ന് (മജ്ഹൂല്) പറഞ്ഞ് പ്രസ്തുത ഹദീഥ് മാറ്റിവെക്കുകയാണ് ചെയ്യുക. ഓരോ നിവേദകനെയും ഈ തലത്തില് പരിശോധിച്ച ശേഷമാണ് അടു ത്തഘട്ടത്തിലേക്ക് കടക്കുക.
ഓരോ നിവേദകനും വ്യത്യസ്ത ഗുരുക്കന്മാരില്നിന്ന് നിവേദനം ചെയ്ത ഹദീഥുകളെ താരതമ്യത്തിന് വിധേ യമാക്കുകയാണ് അടുത്ത ഘട്ടം. തന്റെ ഗുരുവില്നിന്ന് ഹദീഥ് നിവേദനം ചെയ്ത ഒരാള് എത്രമാത്രം പരിഗണനാര്ഹമാണെന്ന് തീരുമാനി ക്കുന്നതിന് അയാളല്ലാത്ത അതേ ഗുരുവിന്റെ മറ്റു ശിഷ്യന്മാരില് എത്രപേര് പ്രസ്തുത ഹദീഥ് നിവേദനം ചെയ്തിട്ടുണ്ടെന്നാണ് പ്രധാന മായും പരിശോധിക്കുക. ഗുരുവിന്റെ ശിഷ്യന്മാരില് നല്ലൊരുശതമാനമാളുകള് പ്രസ്തുത ഹദീഥ് നിവേദനം ചെയ്തിട്ടുണ്ടെങ്കില് മാത്ര മെ അയാള് സ്വീകാര്യനായി വിലയിരുത്തപ്പെടുകയുള്ളൂ. 'ഒരാള് നിവേദനം ചെയ്ത ഹദീഥുകളില് ഭൂരിഭാഗവും സത്യസന്ധരും സൂക്ഷ്മാ ലുക്കളുമെന്ന് തെളിയിക്കപ്പെട്ട നിവേദനകന്മാരുടെ ഹദീഥുകളുമായി യോജിക്കുന്നവയല്ലെങ്കില് അയാളെ ദുര്ബലനായി (ദ്വഈഫ്) പരിഗ ണിക്കപ്പെടു'മെന്നാണ്(സ്വഹീഹു മുസ്ലിം, മുഖദ്ദിമ.) ഇമാം മുസ്ലിം തന്റെ ഹദീഥ് സമാഹാരത്തിന്റെ മുഖവുരയില് വ്യക്തമാക്കുന്നത്.
അറിയപ്പെടുന്നവനും പരിഗണാര്ഹനുമായ നിവേദകനാണെങ്കിലും അയാളുടെ ഹദീഥുകള് സ്വീകാര്യമാകണമെങ്കില് വളരെ പ്രധാനപ്പെട്ട അടുത്ത ഘട്ടം കൂടി കടന്നുപോകേണ്ടതുണ്ട്. അയാളുടെ വ്യക്തിത്വം എത്രത്തോളം സ്വീകാര്യമാണെന്ന പരിശോധനയാണത്. ഋജുത്വ (അദാലത്ത്) പരിശോധനയെന്ന് ഈ ഘട്ടത്തെ വിളിക്കാം.
ഈ ഘട്ടത്തില് നിവേദകനെ പ്രതിക്കൂട്ടില് നിര്ത്തി അന്വേഷകന് ചോദിക്കുന്ന ചോദ്യങ്ങള് ഇവയാണ്.
(1) നബി(സ)യുടെ പേരില് കളവു പറയുന്നവനാണോ?
(2) സാധാരണ സംസാരങ്ങളില് കളവു പറയുന്നവനാണോ?
(3) മതത്തില്നിന്ന് പുറത്തു പോകുന്നതരത്തിലുള്ള അനാചാരങ്ങളുടെ(ബിദ്അത്ത്) വക്താവാണോ?
(4) കക്ഷിത്വത്തിനനുകൂലമായി ഹദീഥ് നിവേദനം ചെയ്യുന്നയാളാണോ?
(5) മതവിരോധിയാണോ?
(6) ദുര്നടപ്പുകാരനാണോ?
(7) കാര്യബോധവും മര്യാദയും മാന്യതയുമില്ലാത്തവനാണോ?
(8) താന് പറയുന്നതെന്തെന്ന് ഗ്രഹിക്കാനാവാത്ത ഭോഷനാണോ?
ഈ ചോദ്യങ്ങള്ക്കെല്ലാം 'അല്ല'യെന്ന ഉത്തരമുണ്ടെങ്കില് മാത്രമെ അയാളിലൂടെയുള്ള നിവേദനം ഋജുത്വ പരിശോധനയുടെ അരിപ്പയിലൂടെ കടന്നുപോവുകയുള്ളൂ. അങ്ങനെ കടന്നുപോയ ഹദീഥുകള് മാത്രമാണ് അടുത്ത ഘട്ടത്തിലേക്ക് നീങ്ങുന്നത്. നിവേദകന്മാരുടെ വ്യക്തിത്വ വിമര്ശനത്തിന് (അദാലത്ത്) ശേഷം നടക്കുന്നത് ഹദീഥിന്റെ കൃത്യതാ പരിശോധനയാണ് (ദ്വബ്ത്ത്). ഋജുവും സത്യസന്ധനുമാണെങ്കിലും നിവേദകന് ഹദീഥ് നിവേദനത്തില് കൃത്യത പാലിക്കുവാന് കഴിഞ്ഞിട്ടുണ്ടോയെന്ന അന്വേഷണമാണത്. ഈ ഘട്ടത്തിലും നിവേദകന്മാര് അന്വേഷകന്റെ പ്രതിക്കൂട്ടില് നില്ക്കേണ്ടിവരും. അയാള് നേരിടേണ്ട ചോദ്യങ്ങള് ഇവയാണ്.
(1) നിവേദനത്തില് അബദ്ധം പിണയാറുള്ളയാളാണോ?
(2) മറവി അധികമായുള്ളയാളാണോ?
(3) വാര്ധ്യക്യത്താല് ഓര്മശക്തി കുറഞ്ഞ് തെറ്റു സംഭവിക്കാന് സാധ്യതയുള്ളപ്പോഴാണോ ഹദീഥ് നിവേദനം ചെയ്തത്?
(4) ഹൃദിസ്ഥമാക്കുവാനുള്ള കഴിവ് കുറഞ്ഞയാളാണോ?
(5) വിശ്വസ്തരായ നിവേദകരിലൂടെ വന്ന ഹദീഥുകളിലെ ആശയങ്ങള്ക്കെതിരെയുള്ള ഹദീഥുകള് നിവേദനം ചെയ്യുന്നയാളാണോ?
(6) ബലപ്പെട്ടവരെന്നോ അല്ലാത്തവരെന്നോ പരിശോധിക്കാതെ എല്ലാവരില്നിന്നുമായി ഹദീഥുകള് നിവേദനം ചെയ്യുന്നയാളാണോ?
(7) തന്റെ ആശയങ്ങള്ക്കനുകൂലമായി ഹദീഥുകള് വളച്ചൊടിക്കുന്നയാളാണോ?
ഇവയ്ക്കെല്ലാം 'അല്ല'യെന്ന ഉത്തരമാണ് കൃത്യതാ പരിശോധകന് ലഭിക്കുന്നതെങ്കില് മാത്രമെ 'ദ്വബ്ത്തു'ള്ള(കൃത്യതയുള്ള) ഹദീഥായി അതിനെ പരിഗണിക്കുകയുള്ളൂ. ഈ പരിശോധന കൂടി കഴിഞ്ഞാല് നിവേദകന് സ്വീകാര്യനാണെന്ന് സ്ഥിരീകരിക്കപ്പെട്ടുകഴിഞ്ഞു. ഇനി അയാളിലൂടെയുള്ള ഹദീഥുകള് സ്വീകരിക്കാവുന്നതാണ്. നിവേദകരുടെ സ്വീകാര്യത നിര്ണയിക്കുന്നതിനു വേണ്ടി പണ്ഡിതന്മാര്ക്ക് ആയിരക്കണക്കിന് നിവേദകരുടെ ജീവിതത്തെ നിഷ്കൃഷ്ടമായി അപഗ്രഥിക്കേണ്ടിവന്നിട്ടുണ്ട്. 'വിമര്ശനവും അംഗീകാരവും' (അല്ജര്ഹു വത്തഅ്ദീല്) എന്ന പദത്തിന്റെ ഏതു മാനത്തിലൂടെ നോക്കിയാലും അതിനെ അന്വര്ഥമാക്കുന്ന രീതിയിലുള്ളതായിരുന്നു പണ്ഡിതന്മാ രുടെ ഈ രംഗത്തെ പരിശ്രമങ്ങളെന്ന് കാണാം. ശാസ്ത്രീയതയുടെ ഏതു മാനദണ്ഡമുപയോഗിച്ചാണ് അല്ജര്ഹുവത്തഅ്ദീല് അശാസ്ത്രീ യമാണെന്നു പറയാനാവുക? ഭൂതകാലത്ത് ജീവിച്ച ഒരാളുടെ ജീവിതത്തില് ആരോപിക്കപ്പെടുന്ന കാര്യങ്ങളിലെ മിഥ്യയും യാഥാര്ഥ്യവും വേര്തിരിക്കുവാന് ഇതിനെക്കാള് ശാസ്ത്രീയമായ രീതികളെന്തെങ്കിലും നിര്ദേശിക്കുവാന് വിമര്ശകര്ക്കു കഴിയുമോ?
ഹദീഥ് നിവേദകരെ വിമര്ശിക്കുകയും അംഗീകരിക്കാനാവുന്നവരെ അംഗീകരിക്കുകയും (അല്ജര്ഹു വത്തഅ്ദീല്) ചെയ്യുന്നതിനു വേണ്ടി ഒരു വിജ്ഞാനീയം തന്നെ ഹദീഥ് നിദാനശാസ്ത്രത്തിന്റെ ശാഖയായി വളര്ന്നു വികസിക്കുകയുണ്ടായി. വ്യക്തി വിജ്ഞാനീയം (ഇല്മുര്രിജാല്) എന്നാണ് പ്രസ്തുത വൈജ്ഞാനികശാഖ അറിയപ്പെടുന്നത്. കേവലമായ ഊഹങ്ങളുടെ അടിസ്ഥാനത്തിലല്ല, വസ്തുനിഷ്ഠവും ശാസ്ത്രീയവുമായ വിവരങ്ങളുടെ അടിസ്ഥാനത്തിലാണ് ഒരു ഹദീഥ് സ്വീകാര്യമാണോയെന്ന് നിശ്ചയിക്കുന്നത് എന്നർത്ഥം.
ഇസ്നാദ് പരിശോധന വഴി ഹദീഥുകൾ മുഹമ്മദ് നബി(സ)യിൽ നിന്നുള്ളതാണോയെന്ന് ഉറപ്പു വരുത്തുന്ന സമ്പ്രദായം എത്രത്തോളം ശാസ്ത്രീയമാണ് ?
മുഹമ്മദ് നബി(സ)യില് നിന്നുള്ളതാണെന്ന രൂപത്തില് ഉദ്ധരിക്കപ്പെടുന്ന വര്ത്തമാനങ്ങള് അപഗ്രഥിച്ച് അതിലെ നേരും നുണയും ചികയു ന്നതിന് ഏറ്റവും അനുയോജ്യമായ രീതിയാണ് യഥാർത്ഥത്തിൽ ഇസ്നാദ് പരിശോധന. പ്രവാചകചര്യയെ സംരക്ഷിക്കുന്നതിനും വരും തലമുറകൾക്കു വേണ്ടി കൈമാറുന്നതിനുമായി മൂന്ന് ഘട്ടങ്ങളായുള്ള ഒരു അരിപ്പ സമ്പ്രദായമാണ് ഉസ്വൂലുല് ഹദീഥിന്റെ പണ്ഡിതന്മാര് വികസിപ്പിച്ചെടുത്തത്. അവ ഇങ്ങനെയാണ്:
1) നബിക്കുറിച്ച് ഉള്ളതാണെന്ന രൂപത്തിൽ നിവേദനം ചെയ്യപ്പെടുന്ന വൃത്താന്തങ്ങളുടെ സ്രോതസ്സ് ആവശ്യപ്പെടുക.
2) സ്രോതസ്സിനെ അപഗ്രഥിച്ച് അത് എത്രത്തോളം വിശ്വാസ്യയോഗ്യമാണെന്ന് കണ്ടെത്തുകയും അതിന്റെ നൈരന്തര്യം ഉറപ്പുവരുത്തുക യും ചെയ്യുക.
3) സ്രോതസ്സിനെ ബലപ്പെടുത്തുന്നതിന് ഉപോല്ബലകമായ മറ്റു തെളിവുകള് കണ്ടെത്തുകയും അതിനെ ദൃഢീകരിക്കുകയും ചെയ്യുക.
ഇതിൽ നബി വൃത്താന്തങ്ങളുടെ സ്രോതസ്സ് ആവശ്യപ്പെടുകയെന്ന ഒന്നാം ഘട്ടമാണ് ഇസ്നാദ് പരിശോധന.
ആര്ക്കും ആരെക്കുറിച്ചും എന്തും പറയാം. ആ പറയലിന് ആധികാരികതയുണ്ടാവണമെങ്കില് അതിന്റെ വിശ്വാസ്യത പരിശോധിക്കു കയും ബോധ്യപ്പെടുകയും വേണം. ഒരാള് പറഞ്ഞുവെന്നോ ചെയ്തുവെന്നോ മറ്റൊരാള് പറയുമ്പോള് അതിന്റെ വിശ്വാസ്യത ഉറപ്പിക്കു ന്നതിന്റെ ഒന്നാമത്തെ പടി അതിന്റെ സ്രോതസ്സ് ആവശ്യപ്പെടുകയാണ്. ആരെക്കുറിച്ചാണോ പറഞ്ഞത് അയാളോടുതന്നെ ചോദിച്ചു മനസ്സിലാക്കുകയോ അല്ലെങ്കില് അയാളുമായി അടുത്ത ബന്ധമുള്ളവരില്നിന്ന് കാര്യത്തിന്റെ യാഥാര്ഥ്യമറിയുകയോ ചെയ്യാവുന്നതാണ്. അയാള് ജീവിച്ചിരിക്കുന്നില്ലെങ്കില് രണ്ടാമത്തെ മാര്ഗം മാത്രമെ അന്വേഷകന്റെ മുന്നില് അവശേഷിക്കുന്നുള്ളൂ. അയാളുമായി ബന്ധപ്പെട്ട ആളില്നിന്ന് വിവരങ്ങള് ശേഖരിക്കുമ്പോള് പ്രസ്തുത വിവരങ്ങള് സത്യസന്ധമാണോയെന്ന് പരിശോധിക്കേണ്ട ബാധ്യത അന്വേഷക നുണ്ട്. തനിക്ക് വിവരം നല്കുന്നയാള്ക്ക് നടേ പറഞ്ഞ വ്യക്തിയുമായുള്ള ബന്ധം അന്വേഷിക്കുകയും അയാള് ചെയ്തതോ പറഞ്ഞതോ ആയി നിവേദനം ചെയ്യപ്പെടുന്ന കാര്യം അയാള് അറിഞ്ഞതെങ്ങനെയെന്ന് പരിശോധിക്കുകയും ചെയ്യുകയാണ് വാര്ത്തയുടെ സത്യതയെ അറിയാന് ശ്രമിക്കുന്നവര് ഒന്നാമതായി ചെയ്യേണ്ടത്. ഇത് തന്നെയാണ് ഇസ്നാദ് പരിശോധന വഴി ഹദീഥ് പണ്ഡിതന്മാർ നിർവഹിച്ചത്.
നബി(സ)യെക്കുറിച്ച് പറയപ്പെടുന്ന വിവരം അത് പറയുന്ന വ്യക്തിയില് എത്തിച്ചേ ര്ന്നതെങ്ങനെയെന്നാണ് ഇസ്നാദുകളെക്കുറിച്ച പഠനം പരിശോധിക്കുന്നത്. ''പ്രവര്ത്തനങ്ങള്ക്ക് അവയുടെ ഉദ്ദേശമനുസരിച്ചാണ് പ്രതി ഫലം ലഭിക്കുക''യെന്ന സ്വഹീഹുല് ബുഖാരിയിലെ ഒന്നാമത്തെ ഹദീഥ് ഉദാഹരണമായെടുക്കുക. 'ദൈവദൂതന് ഇങ്ങനെ പറഞ്ഞതായി ഞാന് കേട്ടു' (സമിഅ്ത്തു റസൂലല്ലാഹി(സ) യക്വൂലു)വെന്ന് പറഞ്ഞുകൊണ്ടാണ് ഉമര്(സ) പ്രസ്തുത ഹദീഥ് ഉദ്ധരിച്ചിരിക്കുന്നത്. പ്രവാച കന്(സ) ഇതു പറയുന്നത് ഉമര്(റ) നേരിട്ടു കേട്ടതാണെന്നര്ഥം. പ്രമുഖ സ്വഹാബിയായിരുന്ന ഉമറുബ്നുല് ഖത്ത്വാബില്നിന്ന് അല്ക്വമതു ബ്നുവക്വാസും അദ്ദേഹത്തില് നിന്ന് മുഹമ്മദ്ബ്നു ഇബ്റാഹീമത്തമീമിയും അദ്ദേഹത്തില്നിന്ന് യഹ്യബ്നു സഈദില് അന്സ്വാരി യും അദ്ദേഹത്തില് നിന്ന് സുഫ്യാനുബ്നു ഉയയ്നയും അദ്ദേഹത്തില്നിന്ന് അബ്ദുല്ലാഹിബ്നു സുബൈര് അല്ഹുമൈദിയും അദ്ദേഹത്തി ല് നിന്ന് ഞാനും കേട്ടുവെന്ന് പറഞ്ഞുകൊണ്ടാണ് ഇമാം മുഹമ്മദ്ബ്നു ഇസ്മായില് അല് ബുഖാരി ഈ ഹദീഥ് ഉദ്ധരിക്കുന്നത്. മുഹമ്മദ് നബി → ഉമറുബ്നുല് ഖത്ത്വാബ് → അല്ക്വമത്തുബ്നു വക്വാസ് → മുഹമ്മദ്ബ്നു ഇബ്റാഹീമത്തമീമി → യഹ്യബ്നുസഈദ് അല് അന് സ്വാരി മസുഫ്യാനുബ്നു ഉയയ്ന മഅബ്ദുല്ലാഹിബ്നു സുബൈര് അല്ഹൂമൈദി എന്നതാണ് ഈ ഹദീഥിന്റെ ഇസ്നാദ്. ഈ ശൃംഖല കൃത്യമായുണ്ടോയെന്ന് പരിശോധിക്കുകയാണ് ഒരു ഹദീഥ് സ്വീകാര്യമാണോയെന്ന പരിശോധനയുടെ പ്രാഥമിക നടപടി.
എത്ര നല്ല ആശയമാണെങ്കിലും അത് നബി(സ)യോട് ചേര്ത്ത് വ്യവഹരിക്കണമെങ്കില് ഇസ്നാദോടു കൂടിത്തന്നെ അത് നിവേദനം ചെയ്യപ്പെ ട്ടതാകണമെന്ന് പണ്ഡിതന്മാര്ക്ക് നിര്ബന്ധമുണ്ടായിരുന്നു. ഇമാം ശാഫിഈയുടെ ഗുരുവര്യന്മാരിലൊരാളായ അബ്ദുല്ലാഹിബ്നുല് മുബാറക് (റ)പറയുന്നതായി ഇമാം മുസ്ലിം ഉദ്ധരിക്കുന്നു: ''മതത്തില്പെട്ടതാണ് ഇസ്നാദ്. അത് ഇല്ലായിരുന്നുവെങ്കില് ഹദീഥില് വേണ്ട വര്ക്ക് വേണ്ടതെന്തും പറയാന് പറ്റുന്ന അവസ്ഥയുണ്ടാകുമായിരുന്നു.'' പ്രമുഖ കര്മശാസ്ത്ര പണ്ഡിതനായിരുന്ന ഇമാം ശാഫി പറഞ്ഞ തിങ്ങനെയാണ്: ''ഇത് എവിടെനിന്നു ലഭിച്ചുവെന്ന് ചോദിച്ച് ഇസ്നാദ് മനസ്സിലാക്കാതെ വിജ്ഞാനം സമ്പാദിക്കുന്നവന് രാത്രിയില് വിറകു മരത്തടികള് ശേഖരിക്കുന്നവനെപ്പോലെയാണ്. തന്റെ ചുമലില് ശേഖരിച്ചുവെച്ച് താങ്ങി നടക്കുന്ന മരത്തടിക്കെട്ടിനകത്ത് അണലി ഒളി ഞ്ഞു കിടക്കുന്നുണ്ടാവാം. അത് അവനെത്തന്നെ കടിക്കുകയും ചെയ്യാം.'' പ്രമുഖ ഹദീഥ് നിവേദന ശാസ്ത്രജ്ഞനായ ശുഅ്ബത്തുബ്നുല് ഹജ്ജാജ് പറഞ്ഞതിങ്ങനെയാണ്: 'അദ്ദേഹം എന്നോട് പറഞ്ഞുവെന്നോ അദ്ദേഹം എന്നോട് നിവേദനം ചെയ്തുവെന്നോ ഉള്ള (ഇസ്നാദി ന്റെ മൂലകങ്ങളായ) പരാമര്ശങ്ങളുള്ക്കൊള്ളാത്ത എല്ലാ മതവിജ്ഞാനങ്ങളും വാലറ്റവയാണ്'.
ഹദീഥ് പരിശോധനയ്ക്ക് വേണ്ടി രൂപപ്പെട്ട ഇസ്നാദ് പരിശോധനാരീതി അറബി സാഹിത്യത്തെയും ചരിത്രത്തെയും ഭൂമിശാസ്ത്രത്തെ യുമെല്ലാം കുറിച്ച പഠനങ്ങള്ക്ക് പില്ക്കാലത്ത് പ്രയോജനീഭവിച്ചതായി വ്യക്തമാക്കുന്ന രേഖകളുണ്ട്. പ്രവാചക ശിഷ്യന്മാരുടെ കാലം മുതലുള്ള മുറിയാത്ത ശൃംഖലയോടുകൂടി നിവേദനം ചെയ്യപ്പെട്ട ഹദീഥുകളിലൂടെ സ്ഥിരീകരിക്കപ്പെട്ടതാണ് മുഹമ്മദ് നബി(സ)യുടെ ജീവിതവൃത്താന്തങ്ങളെന്ന വസ്തുത അംഗീകരിക്കാതിരിക്കുവാന് ഇസ്നാദുകളെപ്പറ്റി അല്പമെങ്കിലും പഠിച്ചവര്ക്കൊന്നും സാധ്യമല്ല. നബി(സ)ചര്യയുടെ ചരിത്രപരതയ്ക്ക് തെളിവുകള് അന്വേഷിക്കുന്നവര്ക്ക്, അവരുപയോഗിക്കുന്ന മാനദണ്ഡങ്ങളെ തൃപ്തമാക്കുവാന് മുറി യാത്ത ശൃംഖലയുള്ള ഹദീഥുകളുടെ ഇസ്നാദുകള് മാത്രം മതി. നബി(സ) ജീവിച്ചത് കണ്ടവരുടെയും അടുത്തതും അതിനടുത്തതുമായ തല മുറകളിലെ നൂറുകണക്കിന് സത്യസന്ധരായ വ്യക്തികളുടെയും സാക്ഷ്യം പോരേ, അദ്ദേഹത്തിന്റെ ചര്യയുടെ ചരിത്രപരത തെളിയിക്കുവാൻ? എന്നാല് ഹദീഥ് നിദാന ശാസ്ത്രം ഇവിടെ നിര്ത്തുന്നില്ല. മുഹമ്മദ് നബി(സ) യെന്ന ഒരാള് ജീവിച്ചിരുന്നുവെന്ന് തെളിയിക്കുകയല്ല ഹദീഥുക ളുടെ ദൗത്യമെന്നതിനാല് ഇസ്നാദ് സമര്പ്പിച്ചുകൊണ്ട് നിര്ത്തുന്നതിന് പകരം ആ ജീവിതത്തില് എന്തൊക്കെ സംഭവിച്ചിട്ടുണ്ടെന്ന സൂക്ഷ്മ വും കൃത്യവും സത്യസന്ധവുമായ അപഗ്രഥനം കൂടി ഹദീഥ് നിവേദന ശാസ്ത്രജ്ഞന്മാര് നടത്തുന്നുണ്ട്.
ഭൂതകാല സംഭവാഖ്യാനത്തിലെ നെല്ലും പതിരും വേർതിരിക്കുന്നതിനു വേണ്ടി ഓറിയന്റലിസ്റ്റുകൾ വികസിപ്പിച്ചെടുത്ത ചരിത്രവിമർശനരീതിയനുസരിച്ച് ഹദീഥ് നിദാനശാസ്ത്രം തികച്ചും അശാസ്ത്രീയമായ ഒരു രീതിയാണ്. ഇങ്ങനെ അശാസ്ത്രീയമായ രീതിയനുസരിച്ച് നബിയുടെ ജീവിതത്തെക്കുറിച്ച് മനസിലാക്കാമെന്ന് കരുതുന്നത് അബദ്ധമല്ല ?
യൂറോപ്യന് മാനദണ്ഡങ്ങള് പ്രകാരമുള്ള അപഗ്രഥനം മാത്രമെ ശാസ്ത്രീയമാവൂയെന്ന യൂറോ കേന്ദ്രീകൃത ലോകവീക്ഷണത്തിന്റെ (eurocentrism) വക്താക്കള്ക്ക് ഹദീഥ് നിദാനശാസ്ത്രത്തിന്റെ രീതി ഉള്ക്കൊള്ളാന് കഴിയുക പ്രയാസകരമാണ്. ബുദ്ധി മുഴുവന് യൂറോപ്പി ന്റേതാണെന്ന വെളുത്ത അഹങ്കാരത്തിന്റെ കണ്ണിലൂടെ നോക്കുന്നവര്ക്ക് ഹദീഥ് നിദാനശാസ്ത്രം മൊത്തത്തില് തന്നെ അസംബന്ധമായി ത്തോന്നാനും സാധ്യതയുണ്ട്. ഭൂതകാല രചനകളിലെ നെല്ലും പതിരും വേര്തിരിക്കുവാന് യൂറോപ്പ് ആവിഷ്കരിച്ച ചരിത്രാഖ്യാനശാ സ്ത്രം (historiography), ചരിത്ര വിമര്ശനരീതി (histori-cal critical method) അഥവാ ഉന്നത വിമര്ശനം (higher criticism) എന്നിവയെക്കാള് എന്തു കൊണ്ടും ഉത്തമമാണ് ഉസ്വൂലുല് ഹദീഥ് അഥവാ ഹദീഥ് നിദാനശാസ്ത്രം എന്നതാണ് വസ്തുത.
യൂറോപ്യന് അഹങ്കാരം മസ്തിഷ്കത്തെ കീഴ്പ്പെടുത്തിയിട്ടില്ലാത്ത ചില ഓറിയന്റലിസ്റ്റുകളെങ്കിലും ഇക്കാര്യം തുറന്നു സമ്മതിച്ചിട്ടുണ്ട്. വാഷിംഗ്ടണ് സര്വകലാശാലയിലെ ഇസ്ലാമിക പഠന വിഭാഗം അസിസ്റ്റന്റ് പ്രൊഫസറും ഓക്സ്ഫോര്ഡ് എന്സൈക്ലോപീഡിയ ഓഫ് ഇസ്ലാമിക് ലോയുടെ മുഖ്യപത്രാധി പരുമായ ഡോ: ജോനാഥന് എ.സി. ബ്രൗണ് ഒരു പ്രഭാഷണത്തില് പറയുന്നത് ഇങ്ങനെയാണ്. ''ചരിത്രത്തിലുള്ള മറ്റാരുടെയും ജീവിതം, മുസ്ലിം ഹദീഥ് പണ്ഡിതന്മാരുടെ ജീവിതത്തോളം എന്റെ മനസ്സിനെ സ്വാധീനിച്ചിട്ടില്ല. ഹദീഥുകളെക്കുറിച്ച് പഠിക്കാനാരംഭിച്ചപ്പോള് അവയെല്ലാം വെറുതെ എഴുതിയുണ്ടാക്കിയ ചവറുകളാണെന്നും കൃത്രിമമാണെന്നുമായിരുന്നു എന്റെ വിചാരം. എന്നാല് കൂടുതലായി പഠിക്കാന് ശ്രമിക്കുന്തോറും അവരുടെ ബുദ്ധിസാമര്ഥ്യത്തെ ഞാന് തിരിച്ചറിയാന് തുടങ്ങി. ആയിരക്കണക്കിന് ഗ്രന്ഥങ്ങള് ഹൃദിസ്ഥമാക്കു വാനും ആവശ്യമുള്ളപ്പോള് അവ ഓര്മയില്നിന്ന് ചികഞ്ഞെടുക്കുവാനും വിഷയാധിഷ്ഠിതമായി അവ ക്രമീകരിച്ചശേഷം അവയുടെ സ്വീകാര്യത പരിശോധിക്കുവാനും അവയുടെ അടിസ്ഥാനത്തില് വിധികള് നിര്ണയിക്കുവാനും അവര്ക്ക് സാധിച്ചുവെന്നതാണ് ഞാന് അര്ഥമാക്കുന്നത്. ഇലക്ട്രോണിക് പദസഞ്ചയവും കംപ്യൂട്ടറുകളുമെല്ലാം ഉപലബ്ധമായ ഇന്ന് ഹദീഥുകളെക്കുറിച്ച് അവര് നിര്വഹിച്ച ദൗത്യം പരതിയെടുക്കുവാന് തന്നെ ഞാന് പ്രയാസപ്പെടുകയാണ്. ഇത് അവിശ്വസീയം തന്നെയാണ്; ഇത് അവിശ്വസനീയം തന്നെയാണ്; അവര് എഴുതിവെച്ച ഗ്രന്ഥങ്ങള് നമ്മുടെ മുന്നിലില്ലായിരുന്നുവെങ്കില് ഞാന് തീര്ച്ചയായും അവര്ക്കിതിന് സാധിച്ചുവെന്ന് വിശ്വസിക്കുക യില്ലാരുന്നു.''(Dr. Jonathan AC Brown: A Brief history of Hadith Collection and Criticism (www.youtube.com/watch?v=cxuebxgixhs)
ചരിത്രാഖ്യാന ശാസ്ത്രത്തിന്റെയും ചരിത്രവിമര്ശന രീതിയുടെയും മാനദണ്ഡങ്ങള് ഹദീഥ് നിദാന ശാസ്ത്രത്തെ പരിശോധിക്കുവാന് തീരെ അപര്യാപ്തമാണ്. രണ്ടും തികച്ചും വിരുദ്ധമായ രണ്ട് രീതി ശാസ്ത്രങ്ങളിലുള്ള അപഗ്രഥനരീതികളാണ് എന്നതുകൊണ്ടാണത്. നിലവിലുള്ള ഒരു ചരിത്രസ്രോതസ്സിനെ സംശയിച്ചുകൊണ്ടാണ് ചരിത്രവിമര്ശന രീതിയുടെ തുടക്കം. പ്രസ്തുത സ്രോതസ്സ് യഥാര്ഥത്തില് അത് എഴുതിയതെന്ന് വിശ്വസിക്കപ്പെടുന്നയാളുടെ രചനതന്നെയാണോയെന്നാണ് അത് അന്വേഷിക്കുന്നത്. അല്ലയെന്ന് സ്ഥാപിക്കുന്നതില് മാത്രമെ ചരിത്ര വിമര്ശകര്ക്ക് താല്പര്യമുള്ളൂ. അയാളുടേതല്ലെങ്കില് പിന്നെയാരുടേത് എന്ന ചോദ്യത്തിന് അവരുടെ പക്കല് ഉത്തരമില്ല. പരമ്പരാഗത ധാരണകളെ തകര്ക്കുന്നതില് മാത്രമാണവരുടെ താല്പര്യം. ഉസ്വൂലുല്ഹദീഥിന്റെ പണ്ഡിതന്മാര് പരമ്പരാഗത ധാരണകളെ തകര്ക്കുകയല്ല, പ്രത്യുത പരിശോധിച്ച് സ്ഥാപിക്കാന് ശ്രമിക്കുകയാണ് ചെയ്യുന്നത്. മുഹമ്മദ് നബി(സ)യില് നിന്നുള്ളതാണ് എന്ന രൂപ ത്തില് സമൂഹത്തില് പ്രചാരത്തിലുള്ള ഹദീഥുകള് അദ്ദേഹത്തില് നിന്നുള്ളവ തന്നെയാണോയെന്ന് പരിശോധിക്കുകയും ഉറപ്പുവരുത്തു കയും ചെയ്യുകയാണ് അവരുടെ ദൗത്യം. ഈ പരിശോധനയില് നബി(സ)യില് നിന്നുള്ളതല്ലെന്ന് ഉറപ്പുള്ളവ വേര്തിരിക്കപ്പെടുകയും മാറ്റി നിര്ത്തപ്പെടുകയും ചെയ്യുമെന്നത് ശരിയാണ്. പക്ഷേ, അങ്ങനെ മാറ്റി നിര്ത്തുകയല്ല അവരുടെ ലക്ഷ്യം. പ്രത്യുത നബി(സ)യില് നിന്നുതന്നെ യാണെന്ന് ഉറപ്പുവരുത്തി സ്വീകരിക്കുവാന് കഴിയുന്നവയെല്ലാം സ്വീകരിക്കുകയാണ്. ചരിത്രവിമര്ശനരീതി നിഷേധത്തില്നിന്നു തുടങ്ങു മ്പോള് ഉസ്വൂലുല് ഹദീഥ് അംഗീകാരത്തില് നിന്നാണ് ആരംഭിക്കുന്നത്.
അബൂഹുറൈറ(റ) യുടെ യഥാർത്ഥ പേരെന്താണെന്നറിയില്ല; ഇസ്ലാം സ്വീകരണത്തിന് മുമ്പുള്ള അദ്ദേഹത്തിന്റെ ചരിത്രവും അറിയില്ല. അങ്ങനെയുള്ള ഒരാളുടെ ഇസ്ലാം ആശ്ലേഷം സത്യസന്ധമായിരുന്നുവെന്ന് എങ്ങനെ കരുത്താനാവും? സ്വഹാബിമാരിൽ പലരും അദ്ദേഹത്തെ വിമർശച്ചതായി കാണാനും കഴിയും. അക്ഷരജ്ഞാനം പോലുമില്ലാത്ത ഒരാളെ ഒരാളെ വിശ്വസിച്ചുകൊണ്ട് അയാൾ പറഞ്ഞതെല്ലാം നബി(സ)യിൽ നിന്നുള്ളവയാണെന്ന് എങ്ങനെ കരുതാനാവും ?
അബൂഹുറയ്റയുടെ(റ) യഥാര്ഥ നാമത്തെക്കുറിച്ചുള്ള അഭിപ്രായവ്യത്യാസങ്ങളും ഇസ്ലാം സ്വീകരണത്തിനുമുമ്പുള്ള അദ്ദേഹത്തിന്റെ ചരിത്രത്തെക്കുറിച്ചുള്ള അജ്ഞതയും അദ്ദേഹത്തിന്റെ സ്വീകാര്യതയെക്കുറിച്ച് സംശയങ്ങളുണ്ടാക്കുന്ന കാര്യങ്ങളായി എടുത്തുപറയുന്ന വിമർശിക്കുന്നവർക്ക് മറ്റു സ്വഹാബിമാരുടെ യഥാര്ഥനാമവും ചരിത്രവും എത്രത്തോളം അറിയാമെന്നാണ് തിരിച്ചു ചോദിക്കാനുള്ളത്. നബി(സ)യുടെ സന്തതസഹചാരിയും ആദ്യ ഖലീഫയുമായിരുന്ന അബൂബക്കര്െ(റ)ന്റ യഥാര്ഥ പേരിനെക്കുറിച്ച് ചരിത്രകാരന്മാ ര്ക്കിടയില് അഭിപ്രായാന്തരങ്ങളുണ്ടെന്നത് അദ്ദേഹത്തിന്റെ വിശ്വാസ്യതയെ ബാധിക്കുമെന്ന് ഏതെങ്കിലും അബൂഹൂറയ്റാ വിമര്ശകന് മാര് വാദിച്ചുകണ്ടിട്ടില്ല. അബൂഉബൈദ(റ), അബൂദുജാന(റ), അബൂദര്റുല് ഗിഫ്ഫാരി(റ), അബൂദര്ദാഅ് (റ)തുടങ്ങിയ നാമങ്ങളില് അറി യപ്പെടുന്ന പ്രമുഖരായ സ്വഹാബികളുടെ യഥാര്ഥ പേരെന്തായിരുന്നുവെന്ന് ചരിത്രകാരന്മാരില് പലര്ക്കും അജ്ഞാതമാണ്.സ്വഹാബി മാരില് പലരുടെയും ഇസ്ലാം സ്വീകരണത്തിന് മുമ്പുള്ള ചരിത്രത്തെക്കുറിച്ചും കൂടുതല് അറിവൊന്നുമില്ല. അറബികള്ക്കിടയില് പ്രശ സ്തമായിരുന്ന ബനൂദൗസ് ഗോത്രക്കാരനായ അബൂഹുറയ്റയേുടെ ചരിത്രവും കുടുംബ വേരുകളും മറ്റു പല സ്വഹാബികളുടേതിലുമ പേക്ഷിച്ച് അറിയപ്പെടുന്നവയാണ്.
അദ്ദേഹത്തിന്റെ ചരിത്രം തീരെ അജ്ഞാതമാണെന്ന് വന്നാല്പോലും അതിന്റെ അടിസ്ഥാനത്തില് അബൂഹുറയ്റ(റ)യുടെ സത്യസന്ധതയെ ചോദ്യം ചെയ്യാന് എങ്ങനെ സാധിക്കും? ജാഹിലിയ്യാ ചരിത്രം അജ്ഞാതമാണെന്ന കാരണത്താല് അസ്വീകാര്യമായി വിലയിരുത്തുകയാണെങ്കില് സ്വഹാബിമാരില് മിക്കവരും അസ്വീകാര്യരായിത്തീരുമെന്നതാണ് വസ്തുത. അബൂഹുറ യ്റെ(റ)യെ തകര്ത്ത് അതിലൂടെ സ്വഹാബിമാരെയും ഹദീഥുകളെയുമെല്ലാം അസ്വീകാര്യമായി മുദ്രകുത്തി നബി(സ)യുടെ ചരിത്രപരത യെത്തന്നെ നിഷേധിക്കുന്നതിനുള്ള പാതയൊരുക്കിയവര് നല്കിയ 'തെളിവുകളു' പയോഗിക്കുന്നവര് യഥാര്ഥത്തില് തകര്ക്കുവാന് ശ്രമി ക്കുന്നത് ഇസ്ലാമിനെത്തന്നെയാണെന്ന വസ്തുതയാണിവിടെ അനാവൃതമാവുന്നത്.
ഓറിയന്റലിസ്റ്റുകള് നല്കിയ 'തെളിവുകളു'പയോഗിച്ച് അബൂഹുറയ്റ(റ)യുടെ ഇസ്ലാം ആശ്ലേഷണം ആത്മാര്ഥമായിരുന്നില്ലെന്ന് വാദിക്കുന്നവരുടെ പക്കല് ആത്മാര്ഥതയെ അളക്കുന്നതിനുള്ള മാനദണ്ഡമെന്താണ്? നബി(സ)യുടെ കൂടെ അദ്ദേഹത്തോടൊപ്പം നാലു വര് ഷത്തിലധികം ജീവിച്ചിട്ട് അബൂഹുറയ്റയുടെ (റ)ആത്മാര്ഥതയില് എന്തെങ്കിലുമൊരു സംശയം നബി(സ) പ്രകടിപ്പിച്ചതായി തെളിയി ക്കുന്ന ഒരു സംഭവം പോലും ഉദ്ധരിക്കുവാന് അബൂഹുറയ്റാവിമര്ശകര്ക്ക് കഴിഞ്ഞിട്ടില്ല. വിദൂര ദേശത്തുനിന്ന് നബി(സ) ജീവിതത്തെ ക്കുറിച്ച് പഠിക്കുവാനായി മദീനയിലേക്ക് പലായനം ചെയ്തെത്തിയ അദ്ദേഹത്തിന്റെ ആത്മാര്ഥതയില് മുഹാജിറുകളിലോ അന്സ്വാരി കളിലോ പെട്ട ആരെങ്കിലും സംശയം പ്രകടിപ്പിച്ചതായി യാതൊരു രേഖയുമില്ല. സകാത്തിന്റെ സ്വത്തു സൂക്ഷിക്കാന് നബി(സ)ഏല്പ്പിച്ചി രുന്നത് അബൂഹുറയ്റെ(റ)യെ ആയിരുന്നു.(സ്വഹീഹുല് ബുഖാരി, കിതാബു സ്സകാത്ത്) നബിജീവിതത്തെ നിരീക്ഷിക്കുന്നതിനായി പള്ളിവരാന്തയില് കഴിഞ്ഞിരുന്നവരെ ക്വുര് ആന് പ്രശംസിക്കുകയാണ് ചെയ്തിട്ടുള്ളത് (2:273). അവരില് കപട വിശ്വാസികളുള്ളതായി ക്വുര്ആന് യാതൊരു സൂചനയും നല്കുന്നില്ല. അല്ലാഹുവിനോ റസൂലിനോല സ്വഹാബികള്ക്കോ മനസ്സിലാകാതിരുന്ന അബൂഹുറയ്റ(റ)യുടെ 'കാപട്യം' തങ്ങള്ക്കാണ് തിരിച്ചറിയാന് കഴിഞ്ഞതെന്ന് ഗര്വ് നടിക്കുന്നവര്, ഓറിയന്റലിസ്റ്റുകള് വെട്ടിവെടിപ്പാക്കിയ വഴിയിലൂടെ പോയി ക്വുര്ആനിനെയും നബി(സ)യെയു മാണ് യഥാര്ഥത്തില് തള്ളിപ്പറയുന്നത്.
അക്ഷരജ്ഞാനമില്ലാതിരുന്ന അബൂഹുറയ്റെ(റ)ക്കെങ്ങനെയാണ് ഇത്രയധികം നബിചര്യകള് ഓര്മിച്ചുവെക്കാനായതെന്ന് ആശ്ചര്യപ്പെട്ട് അദ്ദേഹത്തെ കളിയാക്കാനൊരുമ്പെടുന്നവര് യഥാര്ഥത്തില് പരിഹസിക്കുന്നത് നബി(സ)യെയും ക്വുര്ആനിനെയുമാണ്. വിശുദ്ധ ക്വുര് ആന് അവതരിപ്പിക്കപ്പെട്ടത് അക്ഷരാഭ്യാസമില്ലാത്ത മുഹമ്മദ് നബി(സ)ക്കായിരുന്നു. അദ്ദേഹവും അനുചരന്മാരില് ചിലരും ക്വുര്ആന് പൂര്ണമായും ഹൃദിസ്ഥമാക്കിയിരുെന്നന്ന വസ്തുത വിമര്ശകര്പോലും അംഗീകരിക്കുന്നതാണ്. മനഃപാഠമാക്കുവാനുള്ള, അറബിക ളുടെ ശേഷിയെക്കുറിച്ച് ഓറിയന്റലിസ്റ്റുകളില് പലരും വാചാലരായിട്ടുണ്ട്. നബി(സ)യില് നിന്ന് പഠിച്ച കാര്യങ്ങള് പരസ്പരം പറ ഞ്ഞ് പരിശോധിച്ച് ഹൃദിസ്ഥമാക്കിയിട്ടുണ്ടോയെന്ന് ഉറപ്പുവരുത്തുന്ന പതിവ് സ്വഹാബിമാര്ക്കുണ്ടായിരുന്നുവെന്ന് ഡോ: മുഹമ്മദ് മുസ്തഫ അല് അഅ്ദമി തെളിവുകളുദ്ധരിച്ചുകൊണ്ട് സമര്ഥിക്കുന്നുണ്ട്. നിവേദക പരമ്പരകളോടെ ഇമാം ബുഖാരി മൂന്ന് ലക്ഷവും ഇമാം അഹ്മദ് ആറ് ലക്ഷവും ഹദീഥുകള് മനഃപാഠമാക്കിയിരുന്നുവെന്ന വസ്തുത ഹൃദിസ്ഥമാക്കുവാനുള്ള അറബികളുടെ കഴിവ് വ്യക്ത മാക്കുന്ന തെളിവുകളാണ്. പ്രവാചകനില്നിന്ന് താന് പഠിച്ച മൂവായിരത്തോളം വസ്തുതകള് മനസ്സില് സൂക്ഷിക്കുകയും പില്കാലക്കാ ര്ക്ക് പകര്ന്നു നല്കുകയും ചെയ്ത അബൂഹുറയ്റ(റ)യുടെയും മനഃപാഠമാക്കുവാനുള്ള കഴിവ് മികച്ചതായിരുന്നു; അറബികള്ക്ക് അതൊരിക്കലും അസാധ്യമായിരുന്നില്ല.
മറ്റു സ്വഹാബിമാരെക്കാള് നബി(സ)യുടെ ഹദീഥുകള് മനഃപാഠമാക്കിയിരുന്നത് അബൂഹുറയ്റ യോണെന്ന് അബൂദുല്ലാഹിബ്നു അംറ് (റ)പറഞ്ഞതായി ഇമാം ഇബ്നുകഥീര് രേഖപ്പെടുത്തുന്നുണ്ട്. അമവീ ഖലീഫയായിരുന്ന മര്വാനു ബ്നുല് ഹകം അബൂഹുറയ്റയേുടെ ഓര്മശക്തി പരിശോധിച്ച് ഉറപ്പുവരുത്തിയതായും അദ്ദേഹം ഉദ്ധരിക്കുന്നുണ്ട്. അക്ഷരാഭ്യാസമില്ലെങ്കിലും ഹൃദിസ്ഥമാക്കുന്നതില് സമര്ഥനായിരുന്ന അബൂഹുറയ്റ (റ)നബിജീവിതത്തെപ്പറ്റി താന് അറിഞ്ഞ കാര്യങ്ങള് മനസ്സില് സൂക്ഷിക്കുകയും അടുത്ത തലമുറക്ക് പകര്ന്നു നല്കുകയുമാണ് ചെയ്തതെന്ന് സാരം.
സ്വഹാബീപ്രമുഖരായ ഉമര്(റ), ഉഥ്മാന്(റ), അലി(റ), ആയിശ(റ) തുടങ്ങിയവര് അബൂഹുറയ്റെ(റ)യെ വിമര്ശിക്കുകയും നിഷേധിക്കു കയും അദ്ദേഹത്തിന്റെ സത്യസന്ധതയില് സംശയിക്കുകയും ചെയ്തിരുന്നുവെന്ന് സ്ഥാപിക്കാനായി വിമര്ശകര് ഉദ്ധരിക്കുന്ന സംഭവ ങ്ങള് വ്യാജമായി നിര്മിക്കപ്പെട്ടവയോ സന്ദര്ഭത്തില് നിന്നടര്ത്തി മാറ്റി തെറ്റായി വ്യാഖ്യാനിക്കപ്പെട്ടവയോ ആണെന്ന് തെളിവുകളുദ്ധ രിച്ചുകൊണ്ട് പണ്ഡിതന്മാര് വ്യക്തമാക്കിയിട്ടുണ്ട്. പ്രവാചകന്റെല നിര്ദേശാനുസരണം ഇസ്ലാമിലെ ആദ്യത്തെ ഹജ്ജിന് നേതൃത്വം നല്കിയ അബൂബക്കര് (റ)തന്നോടൊപ്പമുണ്ടായിരുന്ന അബൂഹുറയ്റ(റ)യെയാണ് അടുത്ത വര്ഷം മുതല് ബഹുദൈവാരാധകര്ക്ക് ഹജ്ജിന് വരാന് അനുവാദമുണ്ടാവുകയില്ലെന്ന് പ്രഖ്യാപിക്കുവാന് ചുമതലപ്പെടുത്തിയതെന്ന വസ്തുത അബൂബക്കര് (റ)അബൂഹുറ യ്റെ(റ)യെ അംഗീകരിച്ചിരുന്നുവെന്ന് വ്യക്തമാക്കുന്നു.പച്ചകുത്തലുമായി ബന്ധപ്പെട്ട് നബി(സ) എന്തെങ്കിലും പറഞ്ഞിട്ടുണ്ടോയെന്ന ഉമറിന്റെ (റ)ചോദ്യത്തിന് അബൂഹുറയ്റ (റ)നല്കിയ മറുപടി ചോദ്യം ചെയ്യാതെ അദ്ദേഹം സ്വീകരിച്ചതുംഹസ്സാനുബ്നു സാബിത്തി ന്റെ കവിതകളെ നബി(സ) പുകഴ്ത്തിയതായുള്ള അബൂഹുറയ്റയുടെ സാക്ഷ്യം ഉമര് (റ)അപ്പടി സ്വീകരിച്ചതുമായ സംഭവങ്ങള് ഉമര് (റ)അദ്ദേഹത്തെ അംഗീകരിച്ചിരുന്നുവെന്നതിനുള്ള വ്യക്തമായ തെളിവുകളാണ്. ഉഥ്മാനോ (റ)അലിയോ (റ)ആയിശയോ അബൂഹുറയ് റെ(റ)യെ സംശയിക്കുകയോ വിമര്ശിക്കുകയോ ചെയ്തതായി വിശ്വാസയോഗ്യമായ തെളിവുകളെന്തെങ്കിലും ഉദ്ധരിക്കുവാന് വിമര്ശക ര്ക്ക് കഴിഞ്ഞിട്ടില്ല. ഹദീഥുകള് നിവേദനം ചെയ്യുമ്പോള് അബൂഹുറയ്റ(റ)യോടൊപ്പമുണ്ടായിരുന്ന പ്രസിദ്ധ സ്വഹാബിയായ അബൂസ ഈദില് ഖുദ്രി (റ)അദ്ദേഹത്തെ ശരിവെക്കുകയും സത്യപ്പെടുത്തുകയും ചെയ്തതായി ഇമാം മുസ്ലിം ഉദ്ധരിക്കുന്നുണ്ട്. അബൂഹു റയ്റ (റ)നിവേദനം ചെയ്ത ഹദീഥുകളെ നിരവധി സന്ദര്ഭങ്ങളില് പ്രസിദ്ധ സ്വഹാബിയായ ജാബിറുബ്നു അബ്ദില്ല അനുകൂലി ക്കുകയും സത്യപ്പെടുത്തുകയും ചെയ്തതായി കാണാനാവും. സ്വഹാബിമാരെല്ലാം അബൂഹുറയ്റ(റ)യുടെ സത്യസന്ധത അംഗീകരിച്ചിരുന്നു വെന്ന് ഇവ വ്യക്തമാക്കുന്നു.
മക്കയിലും മദീനയിലുമെല്ലാംനബി(സ)യോടൊപ്പമുണ്ടായിരുന്ന അബൂബക്കറും(റ ) ഉമറും(റ) ഉഥ്മാനുമെല്ലാം (റ) ഏതാനും ഹദീഥുകൾ മാത്രം നിവേദനം ചെയ്തപ്പോൾ അബൂഹുറയ്റ(റ) ആയിരക്കണക്കിന് ഹദീഥുകൾ നിവേശനം ചെയ്തിട്ടുണ്ട്. ഇത് ഹദീഥുണ്ടാക്കുന്നതിൽ അബൂ ഹുറയ്റ(റ)അതിരുവിട്ട് ആവേശം കാണിച്ചിട്ടുണ്ടെന്നല്ലേ വ്യക്തമാക്കുന്നത്?
ആദ്യകാലത്തുതന്നെ ഇസ്ലാം സ്വീകരിക്കുകയും മക്കയിലും മദീനയിലുമെല്ലാംനബി(സ)യോടൊപ്പമുണ്ടാവുകയും ചെയ്ത അബൂബക്ക റും(റ) ഉമറും(റ)ഉഥ്മാനുമൊന്നും(റ)അബൂഹുറയ്റ(റ)നിവേദനം ചെയ്തതു പോലെ ധാരാളം ഹദീഥുകള് നിവേദനം ചെയ്തിട്ടില്ലെന്നത് അബൂ ഹുറയ്റ(റ)ഇക്കാര്യത്തില് അതിരുവിട്ട് ആവേശം കാണിച്ചിട്ടുണ്ടെന്ന് വ്യക്തമാക്കുന്നുണ്ടെന്നും അത് അദ്ദേഹത്തിന്റെ വിശ്വാസ്യ തയെ തകര്ക്കുന്നുണ്ടെന്നും വാദിക്കുന്നത് ഹദീഥ് ശേഖരണത്തിന്റെ ചരിത്രത്തെക്കുറിച്ച അജ്ഞതകൊണ്ടോ അജ്ഞത നടിച്ച് നിഷ്കളങ്കരെ തെറ്റു ധരിപ്പിക്കുവാനുള്ള വ്യഗ്രതകൊണ്ടോ ആണെന്ന് വ്യക്തമാണ്.
അബൂക്കറില്(റ)നിന്ന് 142ഉം ഉമറില്(റ)നിന്ന് 537ഉം ഉഥ്മാനിൽ(റ) നിന്ന് 146 ഉം അലിയില്(റ)നിന്ന് 586ഉം ഹദീഥുകള് ഉദ്ധരിക്കപ്പട്ടപ്പോള് അബൂഹുറയ്റേയില്നിന്ന് 5347ഉം അബ്ദുല്ലാഹിബ്നു ഉമറില്നിന്ന് 2630ഉം അനസ്ബ്നു മാലിക്കില്നിന്ന്(റ) 2300ഉം ആയിശയില്(റ) നിന്ന് 2200ഉം അബ്ദുല്ലാഹിബ്നു അബ്ബാസില്(റ)നിന്ന് 1665ഉം ജാബിറുബ്നു അബ്ദില്ല(റ) യില്നിന്ന് 1500ഉം ഹദീഥുകള് ഉദ്ധരിക്കപ്പെടാനുള്ള കാരണം, രണ്ടാമത് പറഞ്ഞവര്ക്കാണ് അടുത്ത തലമുറയുമായി സമ്പര്ക്ക ത്തിലാവാന് കൂടുതല് അവസരമുണ്ടായത് എന്നതാണ്.
ഹദീഥുകളുടെ നിവേദനവും സംപ്രേക്ഷണവും നടന്നതെങ്ങനെയെന്ന് മനസ്സിലാ യാല് ഇത്തരമൊ രു വിമര്ശനം തന്നെ അപ്രസക്തമാവുമെന്നുറപ്പാണ്. മുഹമ്മദ്നബി(സ)യില്നിന്ന് ഏതെങ്കിലുമൊരു കാര്യം മനസ്സിലാ ക്കിയ ഒരാള് അത് മനസ്സി ലാക്കാത്ത മറ്റൊരാള്ക്ക് പറഞ്ഞുകൊടുക്കുമ്പോഴാണ് അത് ഹദീഥ് ആയിത്തീരുന്നത്. മനസ്സിലാകാത്തവരുടെ എണ്ണവര്ധനവിനനുസരിച്ച് ഈ പറഞ്ഞുകൊടുക്കല് പ്രക്രിയയുടെ എണ്ണവും വര്ധിക്കുക സ്വാഭാവികമാണ്. പ്രവാചകവിയോഗത്തിന് 27 മാസങ്ങള് കഴിഞ്ഞ്, ഹിജ്റ 13 ജുമാദുല് ആഖിര് 13 തിങ്കളാഴ്ച മരണപ്പെട്ട അബൂബക്കറിന്റെ(റ)ജീവിതകാലത്ത്നബി(സ)യില് നിന്ന് കാര്യങ്ങള് നേര്ക്കുനേരെ മനസ്സിലാ ക്കിയവരുടെ എണ്ണം അങ്ങനെ മനസ്സിലാക്കാത്തവരുടെ എണ്ണത്തേക്കാള് വളരെ കൂടുതലായതിനാല് അദ്ദേഹത്തിലൂടെ നിവേദനം ചെയ്യ പ്പെട്ട ഹദീഥുകളുടെ എണ്ണവും കുറവാണെന്നത് സ്വാഭാവികം മാത്രം. ഹിജ്റ 23 ദുല്ഹിജ്ജ 26ന് മരണ പ്പെട്ട ഉമറും(റ)ഹിജ്റ 36 മുഹര്റം 16ന് മരണപ്പെട്ട ഉഥ്മാനും(റ)ഹിജ്റ 40 ശവ്വാല് 20ന് മരണപ്പെട്ട അലിയും(റ) ജീവിച്ചത് ധാരാളം സ്വഹാ ബിമാര് ജീവിച്ചിരുന്ന കാലത്തായി രുന്നതിനാലും അവര് ഭരണാധികാരികളും രാഷ്ട്രവ്യവഹാരത്തില് ശ്രദ്ധ കേന്ദ്രീകരിക്കേണ്ടവരും ആയതിനാലുമാണ് അവരിലൂടെ നിവേദ നം ചെയ്യപ്പെട്ട ഹദീഥുകളുടെ എണ്ണം താരതമ്യേന കുറവായത്. ഇവരെല്ലാം പ്രവാചകവിയോഗം കഴിഞ്ഞ് ആദ്യത്തെ നാലു പതിറ്റാണ്ടുകള് ക്കകം ജീവിച്ചവരാണ്. ധാരാളം സ്വഹാബിമാര് ജീവിച്ചിരുന്ന അക്കാലത്ത്,നബി(സ)യില് നിന്ന് നേര്ക്കുനേരെ കാര്യങ്ങള് മനസ്സിലാക്കിയവ ര്ക്ക് ഇവര് അതേ കാര്യത്തെക്കുറിച്ച് പറഞ്ഞു കൊടുക്കേണ്ടതില്ലായിരുന്നു.
എന്നാല് ഹിജ്റ നാലാമത്തെ പതിറ്റാണ്ടു കഴിഞ്ഞപ്പോഴേക്ക് സ്ഥിതിമാറി.നബി(സ)യില്നിന്ന് നേര്ക്കുനേരെ കാര്യങ്ങള് മനസ്സിലാക്കിയവരുടെ എണ്ണം കുറഞ്ഞുവന്നു. പുതിയ തലമുറക്ക് തങ്ങള്നബി(സ)യില് നിന്ന് മനസ്സിലാക്കിയ കാര്യങ്ങള് പറഞ്ഞുകൊടുക്കേണ്ടത് അന്നു ജീവിച്ചിരുന്ന സ്വഹാബിമാരുടെ ബാധ്യതയായിത്തീര്ന്നു. ഹിജ്റ 57ല് ത ന്റെ 78ാമത്തെ വയസ്സില് മരണപ്പെട്ട അബൂഹുറയ്റയും(റ)ഹിജ്റ 73ല് തന്റെ 80ാമത്തെ വയസ്സില് മരണപ്പെട്ട അബ്ദുല്ലാഹിബ്നു ഉമറും (റ)ഹിജ്റ 93ല് തന്റെ 103ാമത്തെ വയസ്സില് മരണപ്പെട്ട അനസ്ബ്നു മാലിക്കും (റ)ഹിജ്റ 58ല് തന്റെ 65ാമത്തെ വയസ്സില് മരണപ്പെട്ട പ്രവാചകപത്നി ആയിശയും ഹിജ്റ 68ല് തന്റെ 71ാമത്തെ വയസ്സില് മരണപ്പെട്ട അബ്ദുല്ലാഹിബ്നു അബ്ബാസും(റ)ഹിജ്റ 78ല് തന്റെ 94ാമത്തെ വയസ്സില് മരണപ്പെട്ട ജാബിറുബ്നു അബ്ദുല്ലയേും കൂടുതല് ഹദീഥുകള് നിവേദനം ചെയ്തത് തങ്ങളുടെ കാലത്ത്നബി(സ)യെ നേരില് കാണുകയോ കേള്ക്കുകയോ ചെയ്ത ആളുകള് താരതമ്യേന കുറവായതു കൊണ്ടായിരുന്നു.നബി(സ)യുടെ ജീവിതചര്യയെക്കുറിച്ച് പഠിപ്പി ക്കേണ്ടതിന്റെ ആവശ്യകത അദ്ദേഹത്തെ നേരില് അനുഭവിച്ചിട്ടില്ലാത്ത വരുടെ കാലത്ത് കൂടുതലായിരിക്കുമെന്ന സരളമായ വസ്തുത പോലും പരിഗണിക്കാതെയാണ് ഹദീഥ് നിവേദനത്തില് അത്യാവേശം കാണിച്ച് അബദ്ധങ്ങളെഴുന്നള്ളിച്ചയാളായി അബൂഹുറയ്റ(റ)യെ അവതരിപ്പിക്കുവാന് വിമര്ശകര് ധാര്ഷ്ട്യം കാണിക്കുന്നത്.
അല്ല. അബൂഹുറൈറ (റ) ഹദീഥുകളൊന്നും കെട്ടിയുണ്ടാക്കിയിട്ടില്ല. നബിജീവിതത്തിന്റെ അവസാനത്തെ നാല് വർഷങ്ങളിൽ നബിയോടൊപ്പം അദ്ദേഹത്തിന്റെ സ്വകാര്യനിമിഷങ്ങളിലൊഴിച്ച് മുഴുസമയവും ജീവിച്ച ആ മഹാത്യാഗിയുടെ ശ്രമഫലമായാണ് ഏറ്റവുമധികം ഹദീഥുകൾ അടുത്ത തലമുറക്ക് ലഭിച്ചത് എന്നതാണ് സത്യം . ഏറ്റവുമധികം ഹദീഥുകള് നിവേദനം ചെയ്ത അബൂഹുറയ്റ(റ) തന്നെയാണ് ഓറിയന്റലിസ്റ്റുകളുടെയും അവരുടെ ആശയങ്ങള്ക്ക് മുസ്ലിം സമുദായത്തില് വിലാസമുണ്ടാക്കുവാന് ശ്രമിക്കുന്നവരുടെയും ആക്ഷേപങ്ങള്ക്ക് വിധേയനായ പ്രധാനപ്പെട്ട സ്വഹാബി. ഹദീഥ് അപഗ്രഥനത്തില് അഗ്രഗണ്യരായി അറിയപ്പെടുന്ന ഇഗ്നാസ് ഗോള്ഡ് സീഹറും ഹരാള്ഡ് മോട്സ്കിയും ജോസഫ് സ്കാച്ച്ട്ടും ആല്ഫ്രഡ് ഗ്യുല്ല്യൂമുമൊന്നും അബൂഹുറയ്റ(റ)യെ വെറുതെ വിട്ടിട്ടില്ല. 'ഹദീഥുകള് ബോധപൂര്വം കെട്ടിയുണ്ടാ ക്കുന്നയാളായിരുന്നു അബൂഹുറയ്റ' യെന്ന് പറഞ്ഞത് ഇബ്നു ഹിശാമിന്റെ നബിചരിത്രം ഇംഗ്ലീഷിലേക്ക് വിവര്ത്തനം ചെയ്ത ആല്ഫ്രഡ് ഗ്യുല്ല്യൂം ആണ്. ഓറിയന്റലിസ്റ്റുകള് നല്കിയ ആയുധങ്ങളുപയോഗിച്ച് അബൂഹുറയ്റ(റ)യെ കണ്ണും മൂക്കും നോക്കാതെ ആക്രമിക്കുകയാണ് മോഡേണിസ്റ്റുകൾ ചെയ്തത്.
ഓറിയന്റലിസ്റ്റുകള് നല്കിയ ആയുധങ്ങളുപയോഗിച്ച് തത്ത്വദീക്ഷയില്ലാതെ അബൂഹുറയ്റയുടെ നേരെ ആക്രമണങ്ങള് അഴിച്ചുവിടുന്ന വര് തങ്ങള് ചെയ്യുന്നത് ആ മഹാസ്വഹാബിയെ തെറിപറയുക മാത്രമല്ല, നബി(സ)യുടെ ചരിത്രപരതയെ നിഷേധിക്കാനായി ഇസ്ലാമി ന്റെ ശത്രുക്കളുണ്ടാക്കിയ ആയുധത്തെ രാകി മൂര്ച്ചപ്പെടുത്തുകകൂടിയാണെന്ന വസ്തുത പലപ്പോഴും മനസ്സിലാക്കുന്നില്ല. നബിയോടൊപ്പം ജീവിച്ച് നബിജീവിതത്തെ സൂക്ഷ്മമായി നിരീക്ഷിക്കുകയും പഠിക്കുകയും ചെയ്ത് നബി(സ)യുടെ പ്രശംസയ്ക്ക് പാത്രീഭൂതനായ(ഫത്ഹുല്ബാരി) ഒരു അനുചരനെ ജൂതനും ജൂതചാരനുമെല്ലാമായി ചിത്രീകരിച്ച് ഹദീഥ്നിഷേധത്തിന് സൈദ്ധാന്തികമായ അടിത്തറയുണ്ടാക്കുവാന് ശ്രമിക്കുന്ന വര് യഥാര്ഥത്തില് വീണിരിക്കുന്നത് മുഹമ്മദ് നബി(സ) ഒരു ചരിത്രപുരുഷനല്ലെന്നും ഒരു മിത്തു മാത്രമാണെന്നും വരുത്തിത്തീര്ക്കുന്നതിന് ഇസ്ലാമിന്റെ ശത്രുക്കൾ കുഴിച്ച കുഴിയിലാണ്.
കേവലം മൂന്നുകൊല്ലം മാത്രം നബി(സ)യോടൊപ്പം സഹവസിച്ച അബൂഹുറയ്റ(റ)യാണ് ഏറ്റവുമധികം ഹദീഥുകള് നിവേദനം ചെയ്ത തെന്ന വസ്തുതയാണ് അദ്ദേഹത്തിന്റെ സത്യസന്ധതയില് സംശയിക്കുന്നവര് പ്രധാനമായി മുന്നോട്ടുവെക്കുന്ന ആക്ഷേപം. അബൂഹുറയ് റയില്(റ)നിന്ന് 5374 ഹദീഥുകള് നിവേദനം ചെയ്യപ്പെട്ടിട്ടുണ്ടെന്നതും അദ്ദേഹമാണ് ഏറ്റവുമധികം ഹദീഥുകള് നിവേദനം ചെയ്ത സ്വഹാ ബിയെന്നതും ശരിയാണ്. അബൂഹുറയ്റ(റ)യില്നിന്ന് എന്ന രൂപത്തില് നിവേദനം ചെയ്യപ്പെട്ട ഹദീഥുകളില് കെട്ടിയുണ്ടാക്കപ്പെട്ടവയാണെന്ന് ഉറപ്പുള്ളവയും ദുര്ബലമായ ഇസ്നാദോടു കൂടി ഉദ്ധരിക്കപ്പെട്ട അസ്വീകാര്യമായവയുമുണ്ടെന്നതും സത്യമാണ്. പക്ഷേ, അതെങ്ങനെയാണ് അബൂഹുറയ്റ(റ)യുടെ വിശ്വാസ്യതയെ ബാധിക്കുന്നത്? ഹദീഥ് നിവേദനത്തില് നിപുണനും സത്യസന്ധനെന്ന് സമൂഹം അംഗീകരിച്ചയാളുമായ അബൂഹുറയ്റ(റ)യുടെ പേരില് പില്ക്കാലത്തുള്ളവര് കെട്ടിയുണ്ടാക്കിയ ഹദീഥുകള് അദ്ദേഹത്തിന്റെ സത്യസന്ധതയെ ബാധിക്കുന്നതെങ്ങനെയാണെന്ന് വ്യക്തമാക്കു വാന് വിമര്ശകര്ക്ക് കഴിഞ്ഞിട്ടില്ല. യഥാര്ഥത്തില്, അബൂഹുറയ്റ(റ)യുടെ പേരിലാണ് ഏറ്റവുമധികം വ്യാജഹദീഥുകള് പ്രചരിച്ചിട്ടു ള്ളതെന്ന കാര്യം അദ്ദേഹത്തിന്റെ സത്യസന്ധതയെയും സ്വഹാബിമാര്ക്കും താബിഉകള്ക്കുമിടയിലുണ്ടായിരുന്ന അദ്ദേഹത്തിന്റെ സ്വീകാര്യതയുമല്ലേ വ്യക്തമാക്കുന്നത്? വ്യാജ ഹദീഥ് നിര്മാതാക്കള് തങ്ങളുടെ ഉല്പന്നം ജനങ്ങള്ക്കിടയില് സ്വീകാര്യമാവുന്നതിനായി സമൂഹത്തില് ഏറ്റവുമധികം ആദരിക്കപ്പെടുകയും അംഗീകരിക്കപ്പെടുകയും ചെയ്യുന്ന നിവേദകരിലേക്കായിരിക്കും ചേര്ത്തിപ്പറയുക യെന്നുറപ്പാണ്. അബൂഹുറയ്റ(റ)യില്നിന്ന് നിവേദനം ചെയ്യപ്പെട്ടതാണെങ്കില് അത് സമൂഹത്തില് വേഗം അംഗീകരിക്കപ്പെടുമെന്നതിനാ ലാണല്ലോ വ്യാജ ഹദീഥ് നിര്മാതാക്കള് അത് അദ്ദേഹത്തില്നിന്നാണെന്ന മട്ടില് ഉദ്ധരിക്കുന്നത്. അബൂഹുറയ്റ(റ)യുടെ പേരില് വര്ധമാന മായ തോതില് പ്രചരിപ്പിക്കപ്പെട്ട വ്യാജഹദീഥുകള് അദ്ദേഹത്തിന്റെ സത്യസന്ധതയെ ചോദ്യം ചെയ്യുകയല്ല, പ്രത്യുത സ്വീകാര്യതയെ ഉറപ്പിക്കുകയാണ് ചെയ്യുന്നതെന്ന് സാരം.
യമനിലെ ബനൂദൗസ് ഗോത്രക്കാരനായിരുന്ന അബൂഹുറയ്റ(റ) തന്റെ ഗോത്രക്കാരനും നബി(സ)യുടെ അടുത്തെത്തി ഇസ്ലാം സ്വീകരിച്ച യാളുമായ തുഫൈലുബ്നു അംറിന്റെ പ്രബോധനം വഴി ഹിജ്റക്ക് മുമ്പുതന്നെ ഇസ്ലാം സ്വീകരിച്ചിരുന്നുവെങ്കിലും ഹിജ്റ ഏഴാം വര് ഷം നടന്ന ഖൈബര് യുദ്ധത്തിന്റെ സന്ദര്ഭത്തിലാണ് നബി(സ)യുമായി സന്ധിച്ചത്. അന്നുമുതല് നബി(സ)യുടെ മരണം വരെ അദ്ദേഹം പൂര്ണമായും നബി(സ)യോടൊപ്പമായിരുന്നു. നബി(സ)യെ വിട്ടുപിരിയാതെ പള്ളിയുടെ തിണ്ണയില് കഴിഞ്ഞിരുന്നവരായ അസ്ഹാ ബു സ്സ്വുഫ്ഫയുടെ നേതാവായ അദ്ദേഹം പലപ്പോഴും ഭക്ഷണം കഴിച്ചിരുന്നത് പോലും നബി(സ)യോടൊപ്പമായിരുന്നു. എപ്പോഴെ ങ്കിലും കാണാതിരുന്നാല് നബി(സ) അദ്ദേഹത്തെ അന്വേഷിച്ചിരുന്നുവെന്നും അബൂഹുറയ്റെയെയും അദ്ദേഹത്തിന്റെ ഗോത്രത്തെയും നബി(സ) പുകഴ്ത്തിപ്പറഞ്ഞിരുന്നുവെന്നുമെല്ലാം ഹദീഥുകളില് നിന്ന് മനസ്സിലാവുന്നുണ്ട്. ഹിജ്റ ഏഴാം വര്ഷം സ്വഫര് മാസത്തില് ഖൈബറില് വെച്ച് നബി(സ)യോടൊപ്പം കൂടിയതിനുശേഷം പതിനൊന്നാം വര്ഷം റബീഉല് അവ്വല് മാസത്തില് നബി(സ) മരണപ്പെടുന്നതു വരെ യാത്രാസന്ദര്ഭങ്ങളൊഴിച്ച് ബാക്കി പൂര്ണമായും നബി(സ)യോടൊപ്പം തന്നെയായിരുന്ന അദ്ദേഹത്തിന് നബിജീവിതത്തെ കുറിച്ച് നന്നായി മനസ്സിലാക്കാന് കഴിഞ്ഞിരിക്കുമെന്നുറപ്പാണ്. ഈ നാല് വര്ഷങ്ങള്ക്കിടയില് യാത്രാ സമയത്തും മറ്റും വിട്ടുനിന്നതൊഴിച്ചാല് താന് നബി(സ)യുമായി പൂര്ണമായും ഒന്നിച്ചുനിന്നത് മൂന്നു വര്ഷമാണെന്ന് അബൂഹുറയ്റ(റ) തന്നെ വ്യക്തമാ ക്കിയിട്ടുണ്ട്. ഈ മൂന്നു വര്ഷം നബിജീവിതത്തിന്റെ അവസാനനാളുകളായിരുന്നുവെന്ന കാര്യം പ്രത്യേകം ഓര്ക്കേണ്ടതുണ്ട്. കര്മകാ ര്യങ്ങള് മറ്റുള്ളവര്ക്ക് പറഞ്ഞുകൊടുക്കുവാന് നബി(സ) ഏറെ സമയം കണ്ടെത്തിയിരുന്ന നാളുകളാണവ.
മക്കാവിജയവും ഖൈബര് യുദ്ധവും കഴിഞ്ഞതോടെ എതിരാളികളുടെ ശക്തി ക്ഷയിക്കുകയും ഇസ്ലാമിലേക്ക് ആളുകള് കൂട്ടം കൂട്ടമായി വന്നുകൊണ്ടിരിക്കുകയും ചെയ്തപ്പോള് മുസ്ലിംകള്ക്ക് ഇസ്ലാമിെനക്കുറിച്ച് പറഞ്ഞുകൊടുക്കുന്നതിലും കര്മകാര്യങ്ങള് വിശദീകരിച്ചുകൊടുക്കുന്നതിലും നബി(സ) കൂടുതല് ബദ്ധശ്രദ്ധനായി. തിണ്ണവാസിയായി നബി(സ)യോടൊപ്പമുണ്ടായിരുന്ന അബൂഹുറൈറ(റ) ഈ അവസരങ്ങള്ക്കെല്ലാം ദൃക്സാക്ഷിയായി. ഇസ്ലാമിക ജീവിതക്രമത്തെപ്പറ്റി നബി(സ)യില്നിന്ന് നേരിട്ടു പഠിക്കുവാന് അവസരം ലഭിച്ച അബൂഹുറയ്റ(റ) അത് ഓര്ത്തുവെക്കുവാനും അടുത്ത തലമുറക്ക് പറഞ്ഞുകൊടുക്കുവാനും ശ്രദ്ധിച്ചതിനാലാണ് അദ്ദേഹത്തിലൂടെ കൂടുതല് ഹദീഥുകള് നിവേ ദനം ചെയ്യപ്പെട്ടത്. നീണ്ട മൂന്നു വര്ഷക്കാലം നബി(സ)യുടെ വാക്കുകളും പ്രവര്ത്തനങ്ങളും പെരുമാറ്റങ്ങളും നിരന്തരമായി ശ്രദ്ധിക്കു കയും സൂക്ഷ്മമായി ഓര്ത്തുവെക്കുകയും ചെയ്ത ഒരാള്ക്ക് നിവേദനം ചെയ്യാനാവുന്നതിലും കൂടുതല് ഹദീഥുകള് അബൂഹുറയ്റ(റ) നിവേദനം ചെയ്തിട്ടില്ലെന്നതാണ് വാസ്തവം.
അബൂഹുറൈറ(റ)യിലൂടെ നിവേദനം ചെയ്യപ്പെട്ട ഹദീഥുകളില്നിന്ന് വ്യാജമെന്ന് ഉറപ്പുള്ളവ ഒഴിവാക്കി ഇമാം അഹ്മദ്(റഹ്) രേഖപ്പെടുത്തിയിട്ടുള്ളത് 3848 ഹദീഥുകളാണ്. അവയില് പലതും അബൂഹുറയ്റയില് നിന്ന് വ്യത്യ സ്ത ഇസ്നാദുകളില് നിവേദനം ചെയ്യപ്പെട്ട ഒരേ മത്നിന്റെ ആവര്ത്തനങ്ങളാണ്. ഇവയെയെല്ലാം സ്വതന്ത്ര ഹദീഥുകളായി പരിഗണി ച്ചാല് പോലും നബിജീവിതത്തെ സൂക്ഷ്മമായി നിരീക്ഷിച്ച ഒരാള്ക്ക് ഇത്രയും ഹദീഥുകള് നിവേദനം ചെയ്യാന് കഴിയില്ലെന്ന് പറയാന് എങ്ങനെ പറ്റും? മൂന്നു വര്ഷങ്ങളിലുള്ള ആയിരത്തിലധികം ദിവസവും നബി(സ)യെ നിരീക്ഷിച്ച ഒരാള്ക്ക് ഒരു ദിവസത്തില് നബി(സ) യില് നിന്ന് ശരാശരി നാല് കാര്യങ്ങളില് പഠിക്കാന് കഴിഞ്ഞുവെന്നത് വലിയൊരു അത്ഭുതമൊന്നുമല്ല. സംഭവബഹുലമായ നബിജീവി തത്തിന്റെ അവസാനനാളുകളില് നബി(സ)യുടെ സന്തതസഹചാരിയായി അദ്ദേഹത്തോടൊപ്പമുണ്ടായിരുന്ന ഒരാള്ക്ക് ആ ജീവിതത്തില് നിന്ന് ഒരു ദിവസം ശരാശരി നാലു കാര്യങ്ങള് പഠിക്കാന് കഴിഞ്ഞത് അസംഭവ്യമാണെന്ന് പറയാന് ഓറിയന്റലിസമുപ യോഗിക്കുന്ന മാനദണ്ഡമെന്താണ്? ശാസ്ത്രീയാപഗ്രഥനത്തിന്റെ അകമ്പടിയൊന്നുമില്ലാതെത്തന്നെ ആര്ക്കും സംഭവ്യമെന്ന് ബോധ്യപ്പെടുന്ന കാര്യങ്ങളെ പ്പോലും നിഷേധിക്കുവാന് ബുദ്ധിജീവനത്തിന്റെ മൊത്തക്കുത്തക അവകാശപ്പെടുന്നവരെ പ്രേരിപ്പിക്കുന്നതിന് പിന്നില് സത്യമതത്തോടു ള്ള അന്ധമായ വിരോധമല്ലാതെ മറ്റെന്താണ്?
അബൂഹുറൈറ(റ)യെ ഹദീഥ് നിര്മാതാവായി പരിചചയപ്പെടുത്തതാൻ ശ്രമിക്കുന്നവർ ചരിത്രത്തോടോ സത്യങ്ങളോടോ നീതി പുലർത്താത്തവരാണെന്ന് സാരം.
ടുന്ന ഇഗ്നാസ് ഗോള്ഡ് സീഹറും(1) ഹരാള്ഡ് മോട്സ്കി(2) യും ജോ സഫ് സ്കാച്ച്ട്ടും(3) ആല്ഫ്രഡ് ഗ്യുല്ല്യൂമുമൊന്നും(4) അബൂഹുറയ്റ(റ)യെ വെറുതെ വിട്ടിട്ടില്ല. 'ഹദീഥുകള് ബോധപൂര്വം കെട്ടിയുണ്ടാ ക്കുന്നയാളായിരുന്നു അബൂഹുറയ്റ'(5) യെന്നാണ് ഇബ്നു ഹിശാമിന്റെ നബിചരിത്രം ഇംഗ്ലീഷിലേക്ക് വിവര്ത്തനം ചെയ്തുകൊണ്ട് മുസ്ലിം ലോകത്തിന് ഏറെ വിലപ്പെട്ട സംഭാവന നല്കിയ വ്യക്തിത്വമായി കൊട്ടിഘോഷിക്കപ്പെടുന്ന ആല്ഫ്രഡ് ഗ്യുല്ല്യൂം എഴുതുന്നത്. ഓറിയന്റലിസ്റ്റുകള് നല്കിയ ആയുധങ്ങളുപയോഗിച്ച് അബൂഹുറയ്റ(റ)യെ കണ്ണും മൂക്കും നോക്കാതെ ആക്രമിച്ചത് യഥാര്ഥത്തില് മുസ്ലിം മോഡേണിസ്റ്റുകളായിരുന്നു. അബൂഹുറയ്റയെ തെറി വിളിക്കുന്ന കാര്യത്തിലും അദ്ദേഹത്തിനെതിരെ 'തെളിവുകള്' കണ്ടെത്തു ന്ന കാര്യത്തിലും അവര് ഓറിയന്റലിസ്റ്റുകളെ തോല്പിച്ചുവെന്ന് വേണമെങ്കില് പറയാം. തന്റെ മാസ്റ്റര് പീസായി അറിയപ്പെടുന്ന അദ്വ് വാഉന് അലാസുന്നത്തില് മുഹമ്മദിയ്യഃയെക്കൂടാതെ അബൂഹുറയ്റ(റ)യെ തെറിപറയാന് വേണ്ടിമാത്രം 'ശൈഖുല് മദീറ അബൂഹുറയ്റ' യെന്ന ഒരു ഗ്രന്ഥം കൂടി രചിച്ചിട്ടുണ്ട് ആധുനിക മുസ്ലിം ഹദീഥ് നിഷേധികളില് അഗ്രേസരനായി അറിയപ്പെടുന്ന മഹ്മൂദ് അബൂ റയ്യ(6). അദ്ദേഹത്തെ പിന്തുടര്ന്ന്, മോഡേണിസ്റ്റ് ചിന്തകള്ക്ക് അറബ്നാടുകളില് പ്രചാരം സിദ്ധിക്കുന്നതിനായി പരിശ്രമിച്ച മുഹമ്മദ് അമീനി ന്റെ ഫജ്റുല് ഇസ്ലാമിലും അബൂഹുറയ്റ(റ)യെ അതിരുകള് ലംഘിച്ച് വിമര്ശിക്കുകയും കളിയാക്കുകയും ചെയ്യുന്നുണ്ട്.(7)
ഓറിയന്റലിസ്റ്റുകള് നല്കിയ ആയുധങ്ങളുപയോഗിച്ച് തത്ത്വദീക്ഷയില്ലാതെ അബൂഹുറയ്റയുടെ നേരെ ആക്രമണങ്ങള് അഴിച്ചുവിടുന്ന വര് തങ്ങള് ചെയ്യുന്നത് ആ മഹാസ്വഹാബിയെ തെറിപറയുക മാത്രമല്ല, നബി(സ)യുടെ ചരിത്രപരതയെ നിഷേധിക്കാനായി ഇസ്ലാമി ന്റെ ശത്രുക്കളുണ്ടാക്കിയ ആയുധത്തെ രാകി മൂര്ച്ചപ്പെടുത്തുകകൂടിയാണെന്ന വസ്തുത പലപ്പോഴും മനസ്സിലാക്കുന്നില്ല. നബിയോടൊപ്പം ജീവിച്ച് നബിജീവിതത്തെ സൂക്ഷ്മമായി നിരീക്ഷിക്കുകയും പഠിക്കുകയും ചെയ്ത് നബി(സ)യുടെ പ്രശംസയ്ക്ക് പാത്രീഭൂതനായ(8) ഒരു അനുചരനെ ജൂതനും ജൂതചാരനുമെല്ലാമായി ചിത്രീകരിച്ച് ഹദീഥ്നിഷേധത്തിന് സൈദ്ധാന്തികമായ അടിത്തറയുണ്ടാക്കുവാന് ശ്രമിക്കുന്ന വര് യഥാര്ഥത്തില് വീണിരിക്കുന്നത് ഇഗ്നാസ് ഗോള്ഡ് സീഹര് എന്ന യഥാര്ഥ ജൂതനും അദ്ദേഹത്തിന്റെ പിന്ഗാമികളുംകൂടി കുഴിച്ച കുഴിയിലാണ്. മുഹമ്മദ് നബി(സ) ഒരു ചരിത്രപുരുഷനല്ലെന്നും ഒരു മിത്തു മാത്രമാണെന്നും വരുത്തിത്തീര്ക്കുന്നതിന് ഇസ്ലാമിന്റെ ശത്രുക്കള് കുഴിച്ച കുഴിയില് വീണുകിടക്കുമ്പോഴും തങ്ങള് ഇസ്ലാമിനും മുഹമ്മദ് നബി(സ)ക്കും സേവനം ചെയ്യുകയാണെന്നാണ് ഇവര് വിളിച്ചു പറഞ്ഞുകൊണ്ടിരിക്കുന്നത് എന്നതാണ് കഷ്ടം.
കേവലം മൂന്നുകൊല്ലം മാത്രം നബി(സ)യോടൊപ്പം സഹവസിച്ച അബൂഹുറയ്റ(റ)യാണ് ഏറ്റവുമധികം ഹദീഥുകള് നിവേദനം ചെയ്ത തെന്ന വസ്തുതയാണ് അദ്ദേഹത്തിന്റെ സത്യസന്ധതയില് സംശയിക്കുന്നവര് പ്രധാനമായി മുന്നോട്ടുവെക്കുന്ന ആക്ഷേപം. അബൂഹുറയ് റയില്(റ)നിന്ന് 5374 ഹദീഥുകള് നിവേദനം ചെയ്യപ്പെട്ടിട്ടുണ്ടെന്നതും(9) അദ്ദേഹമാണ് ഏറ്റവുമധികം ഹദീഥുകള് നിവേദനം ചെയ്ത സ്വഹാ ബിയെന്നതും ശരിയാണ്. ബുഖാരിയില് മാത്രം അബൂഹുറയ്റ(റ) നിവേദനം ചെയ്ത 446 ഹദീഥുകളുണ്ട്.(10) ബുഖാരിയും മുസ്ലിമും കൂടി ആകെ 609 ഹദീഥുകളും രണ്ടു ഗ്രന്ഥങ്ങളിലും ഒരേപോലെ 326 ഹദീഥുകളും അദ്ദേഹത്തില്നിന്ന് നിവേദനം ചെയ്യപ്പെട്ടിട്ടുണ്ട്. ഇമാം അഹ്മ ദ്ബ്നു ഹന്ബലിന്റെ മുസ്നദില് അബൂഹുറയ്റയില് നിന്ന് 3848 ഹദീഥുകള് നിവേദനം ചെയ്തിട്ടുണ്ട്.(11) അബൂഹുറയ്റ(റ)യില്നിന്ന് എന്ന രൂപത്തില് നിവേദനം ചെയ്യപ്പെട്ട 5374 ഹദീഥുകളില് കെട്ടിയുണ്ടാക്കപ്പെട്ടവയാണെന്ന് ഉറപ്പുള്ളവയും ദുര്ബലമായ ഇസ്നാദോടു കൂടി ഉദ്ധരിക്കപ്പെട്ട അസ്വീകാര്യമായവയുമുണ്ടെന്നത് ശരിയാണ്. പക്ഷേ, അതെങ്ങനെയാണ് അബൂഹുറയ്റ(റ)യുടെ വിശ്വാസ്യതയെ ബാധിക്കുന്നത്? ഹദീഥ് നിവേദനത്തില് നിപുണനും സത്യസന്ധനെന്ന് സമൂഹം അംഗീകരിച്ചയാളുമായ അബൂഹുറയ്റ(റ)യുടെ പേരില് പില്ക്കാലത്തുള്ളവര് കെട്ടിയുണ്ടാക്കിയ ഹദീഥുകള് അദ്ദേഹത്തിന്റെ സത്യസന്ധതയെ ബാധിക്കുന്നതെങ്ങനെയാണെന്ന് വ്യക്തമാക്കു വാന് വിമര്ശകര്ക്ക് കഴിഞ്ഞിട്ടില്ല. യഥാര്ഥത്തില്, അബൂഹുറയ്റ(റ)യുടെ പേരിലാണ് ഏറ്റവുമധികം വ്യാജഹദീഥുകള് പ്രചരിച്ചിട്ടു ള്ളതെന്ന കാര്യം അദ്ദേഹത്തിന്റെ സത്യസന്ധതയെയും സ്വഹാബിമാര്ക്കും താബിഉകള്ക്കുമിടയിലുണ്ടായിരുന്ന അദ്ദേഹത്തിന്റെ സ്വീകാര്യതയുമല്ലേ വ്യക്തമാക്കുന്നത്? വ്യാജ ഹദീഥ് നിര്മാതാക്കള് തങ്ങളുടെ ഉല്പന്നം ജനങ്ങള്ക്കിടയില് സ്വീകാര്യമാവുന്നതിനായി സമൂഹത്തില് ഏറ്റവുമധികം ആദരിക്കപ്പെടുകയും അംഗീകരിക്കപ്പെടുകയും ചെയ്യുന്ന നിവേദകരിലേക്കായിരിക്കും ചേര്ത്തിപ്പറയുക യെന്നുറപ്പാണ്. അബൂഹുറയ്റ(റ)യില്നിന്ന് നിവേദനം ചെയ്യപ്പെട്ടതാണെങ്കില് അത് സമൂഹത്തില് വേഗം അംഗീകരിക്കപ്പെടുമെന്നതിനാ ലാണല്ലോ വ്യാജ ഹദീഥ് നിര്മാതാക്കള് അത് അദ്ദേഹത്തില്നിന്നാണെന്ന മട്ടില് ഉദ്ധരിക്കുന്നത്. അബൂഹുറയ്റ(റ)യുടെ പേരില് വര്ധമാന മായ തോതില് പ്രചരിപ്പിക്കപ്പെട്ട വ്യാജഹദീഥുകള് അദ്ദേഹത്തിന്റെ സത്യസന്ധതയെ ചോദ്യം ചെയ്യുകയല്ല, പ്രത്യുത സ്വീകാര്യതയെ ഉറപ്പിക്കുകയാണ് ചെയ്യുന്നതെന്ന് സാരം.
യമനിലെ ബനൂദൗസ് ഗോത്രക്കാരനായിരുന്ന അബൂഹുറയ്റ(റ) തന്റെ ഗോത്രക്കാരനും നബി(സ)യുടെ അടുത്തെത്തി ഇസ്ലാം സ്വീകരിച്ച യാളുമായ തുഫൈലുബ്നു അംറിന്റെ പ്രബോധനം വഴി ഹിജ്റക്ക് മുമ്പുതന്നെ ഇസ്ലാം സ്വീകരിച്ചിരുന്നുവെങ്കിലും ഹിജ്റ ഏഴാം വര് ഷം നടന്ന ഖൈബര് യുദ്ധത്തിന്റെ സന്ദര്ഭത്തിലാണ് നബി(സ)യുമായി സന്ധിച്ചത്. അന്നുമുതല് നബി(സ)യുടെ മരണം വരെ അദ്ദേഹം പൂര്ണമായും നബി(സ)യോടൊപ്പമായിരുന്നു.(12) നബി(സ)യെ വിട്ടുപിരിയാതെ പള്ളിയുടെ തിണ്ണയില് കഴിഞ്ഞിരുന്നവരായ അസ്ഹാ ബു സ്സ്വുഫ്ഫ(13)യുടെ നേതാവായ അദ്ദേഹം പലപ്പോഴും ഭക്ഷണം കഴിച്ചിരുന്നത് പോലും നബി(സ)യോടൊപ്പമായിരുന്നു. എപ്പോഴെ ങ്കിലും കാണാതിരുന്നാല് നബി(സ) അദ്ദേഹത്തെ അന്വേഷിച്ചിരുന്നുവെന്നും(14) അബൂഹുറയ്റെയെയും അദ്ദേഹത്തിന്റെ ഗോത്രത്തെയും നബി(സ) പുകഴ്ത്തിപ്പറഞ്ഞിരുന്നുവെന്നുമെല്ലാം(15) ഹദീഥുകളില് നിന്ന് മനസ്സിലാവുന്നുണ്ട്. ഹിജ്റ ഏഴാം വര്ഷം സ്വഫര് മാസത്തില് ഖൈബറില് വെച്ച് നബി(സ)യോടൊപ്പം കൂടിയതിനുശേഷം പതിനൊന്നാം വര്ഷം റബീഉല് അവ്വല് മാസത്തില് നബി(സ) മരണപ്പെടുന്നതു വരെ യാത്രാസന്ദര്ഭങ്ങളൊഴിച്ച് ബാക്കി പൂര്ണമായും നബി(സ)യോടൊപ്പം തന്നെയായിരുന്ന അദ്ദേഹത്തിന് നബിജീവിതത്തെ കുറിച്ച് നന്നായി മനസ്സിലാക്കാന് കഴിഞ്ഞിരിക്കുമെന്നുറപ്പാണ്. ഹിജ്റ 7 മുതല് 11 വരെയുള്ള നാല് വര്ഷങ്ങള്ക്കിടയില് യാത്രാ സമയത്തും മറ്റും വിട്ടുനിന്നതൊഴിച്ചാല് താന് നബി(സ)യുമായി പൂര്ണമായും ഒന്നിച്ചുനിന്നത് മൂന്നു വര്ഷമാണെന്ന് അബൂഹുറയ്റ(റ) തന്നെ വ്യക്തമാ ക്കിയിട്ടുണ്ട്.(16) ഈ മൂന്നു വര്ഷം നബിജീവിതത്തിന്റെ അവസാനനാളുകളായിരുന്നുവെന്ന കാര്യം പ്രത്യേകം ഓര്ക്കേണ്ടതുണ്ട്. കര്മകാ ര്യങ്ങള് മറ്റുള്ളവര്ക്ക് പറഞ്ഞുകൊടുക്കുവാന് നബി(സ) ഏറെ സമയം കണ്ടെത്തിയിരുന്ന നാളുകളാണവ. മക്കാവിജയവും ഖൈബര് യുദ്ധവും കഴിഞ്ഞതോടെ എതിരാളികളുടെ ശക്തി ക്ഷയിക്കുകയും ഇസ്ലാമിലേക്ക് ആളുകള് കൂട്ടം കൂട്ടമായി വന്നുകൊണ്ടിരിക്കുകയും ചെയ്തപ്പോള് മുസ്ലിംകള്ക്ക് ഇസ്ലാമിെനക്കുറിച്ച് പറഞ്ഞുകൊടുക്കുന്നതിലും കര്മകാര്യങ്ങള് വിശദീകരിച്ചുകൊടുക്കുന്നതിലും നബി(സ) കൂടുതല് ബദ്ധശ്രദ്ധനായി. തിണ്ണവാസിയായി നബി(സ)യോടൊപ്പമുണ്ടായിരുന്ന അബൂഹുറൈറ(റ) ഈ അവസരങ്ങള്ക്കെല്ലാം ദൃക്സാക്ഷിയായി. ഇസ്ലാമിക ജീവിതക്രമത്തെപ്പറ്റി നബി(സ)യില്നിന്ന് നേരിട്ടു പഠിക്കുവാന് അവസരം ലഭിച്ച അബൂഹുറയ്റ(റ) അത് ഓര്ത്തുവെക്കുവാനും അടുത്ത തലമുറക്ക് പറഞ്ഞുകൊടുക്കുവാനും ശ്രദ്ധിച്ചതിനാലാണ് അദ്ദേഹത്തിലൂടെ കൂടുതല് ഹദീഥുകള് നിവേ ദനം ചെയ്യപ്പെട്ടത്. നീണ്ട മൂന്നു വര്ഷക്കാലം നബി(സ)യുടെ വാക്കുകളും പ്രവര്ത്തനങ്ങളും പെരുമാറ്റങ്ങളും നിരന്തരമായി ശ്രദ്ധിക്കു കയും സൂക്ഷ്മമായി ഓര്ത്തുവെക്കുകയും ചെയ്ത ഒരാള്ക്ക് നിവേദനം ചെയ്യാനാവുന്നതിലും കൂടുതല് ഹദീഥുകള് അബൂഹുറയ്റ(റ) നിവേദനം ചെയ്തിട്ടില്ലെന്നതാണ് വാസ്തവം. അദ്ദേഹത്തിലൂടെ നിവേദനം ചെയ്യപ്പെട്ട 5374 ഹദീഥുകളില്നിന്ന് വ്യാജമെന്ന് ഉറപ്പുള്ളവ ഒഴിവാക്കി ഇമാം അഹ്മദ്(റഹ്) രേഖപ്പെടുത്തിയിട്ടുള്ളത് 3848 ഹദീഥുകളാണ്.(17) അവയില് പലതും അബൂഹുറയ്റയില് നിന്ന് വ്യത്യ സ്ത ഇസ്നാദുകളില് നിവേദനം ചെയ്യപ്പെട്ട ഒരേ മത്നിന്റെ ആവര്ത്തനങ്ങളാണ്. ഇവയെയെല്ലാം സ്വതന്ത്ര ഹദീഥുകളായി പരിഗണി ച്ചാല് പോലും നബിജീവിതത്തെ സൂക്ഷ്മമായി നിരീക്ഷിച്ച ഒരാള്ക്ക് ഇത്രയും ഹദീഥുകള് നിവേദനം ചെയ്യാന് കഴിയില്ലെന്ന് പറയാന് എങ്ങനെ പറ്റും? മൂന്നു വര്ഷങ്ങളിലുള്ള ആയിരത്തിലധികം ദിവസവും നബി(സ)യെ നിരീക്ഷിച്ച ഒരാള്ക്ക് ഒരു ദിവസത്തില് നബി(സ) യില് നിന്ന് ശരാശരി നാല് കാര്യങ്ങളില് പഠിക്കാന് കഴിഞ്ഞുവെന്നത് വലിയൊരു അത്ഭുതമൊന്നുമല്ല. സംഭവബഹുലമായ നബിജീവി തത്തിന്റെ അവസാനനാളുകളില് നബി(സ)യുടെ സന്തതസഹചാരിയായി അദ്ദേഹത്തോടൊപ്പമുണ്ടായിരുന്ന ഒരാള്ക്ക് ആ ജീവിതത്തില് നിന്ന് ഒരു ദിവസം ശരാശരി നാലു കാര്യങ്ങള് പഠിക്കാന് കഴിഞ്ഞത് അസംഭവ്യമാണെന്ന് പറയാന് ഓറിയന്റലിസമുപ യോഗിക്കുന്ന മാനദണ്ഡമെന്താണ്? ശാസ്ത്രീയാപഗ്രഥനത്തിന്റെ അകമ്പടിയൊന്നുമില്ലാതെത്തന്നെ ആര്ക്കും സംഭവ്യമെന്ന് ബോധ്യപ്പെടുന്ന കാര്യങ്ങളെ പ്പോലും നിഷേധിക്കുവാന് ബുദ്ധിജീവനത്തിന്റെ മൊത്തക്കുത്തക അവകാശപ്പെടുന്നവരെ പ്രേരിപ്പിക്കുന്നതിന് പിന്നില് സത്യമതത്തോടു ള്ള അന്ധമായ വിരോധമല്ലാതെ മറ്റെന്താണ്?
കുറിപ്പുകൾ
- Ignaz Goldziher: Muslim Studies, Vol.2, Page 41-48.
- Harald Motzki: The Biography of Muhammed, The Issue of Sources, Brill, 2000.
- Joseph Schacht: The Origins of Muhammadan Juriprudance, ACLS, 2001 (www. historybook.org).
- Guillaume: Traditions of Islam: An Introduction to the studies of the Hadith Literature, Montana, 2003.
- Ibid Page 78.
- sunnidefence.com.
- hadithcommunity-wordpress.com.
- ഫത്ഹുല്ബാരി.
- തഖിയ്യിബ്നു മഖലദിന്റെ മുസ്നദില്നിന്ന് ഡോക്ടര് മുസ്തഫസ്സബാഈ ഉദ്ധരിച്ചത്. സുന്നത്തും ഇസ്ലാം ശീഅത്തില് അതിന്റെ സ്ഥാനവും, പുറം
- ഫത്ഹുല്ബാരി.
- മുസ്നദ് ഇമാം അഹ്മദ് (ekabakti.com).
- ഹാഫിദ് അഹ്മദ്ബ്നു അലിയുബ്നു ഹജറുല് അസ്ഖലാനി: അല് ഇസ്വാബതു ഫീ തംയീസി സ്സ്വഹാബഃ
- 'അസ്ഹാബുസ്സ്വുഫ്ഫ'യെന്നാല് തിണ്ണവാസികള് എന്നര്ഥം. മറ്റു ജോലികളില് വ്യാപൃതരാവാതെ മദീനയിലെ പള്ളിയുടെ തിണ്ണയില് കഴിഞ്ഞിരുന്നവരെ വിളിച്ചിരുന്ന പേരാണിത്. ക്വുര്ആനില് 'ഭൂമിയില് സഞ്ചരിച്ച് ഉപജീവനം നേടാന് കഴിയാത്തവിധം അല്ലാഹുവിന്റെ മാര്ഗത്തില് ബന്ധിതരായിരിക്കുന്നവര്' (2:273) എന്ന് പ്രശംസിച്ച് പറഞ്ഞിരിക്കുന്നത് ഇവരെക്കുറിച്ചാണെന്ന് വ്യാഖ്യാതാക്കള് പറഞ്ഞിട്ടുണ്ട്.
- സുനനു അബീദാവൂദ് (ekabakti.com).
- സ്വഹീഹുല് ബുഖാരി, കിത്താബുല് മഗാസി.
- സ്വഹീഹുല് ബുഖാരി, ഫദാഇലുര്റസൂലി വ സ്വഹാബത്തിഹി.
- മുസ്നദ് ഇമാം അഹ്മദ് (ekabakti.com).
മുഹമ്മദ് നബിയിൽ(സ) നിന്ന് സ്വഹാബിമാർ നിവേദനം ചെയ്ത കാര്യങ്ങളാണ് ഹദീഥുകൾ. സ്വഹാബിമാരൊന്നും നബിയുടെ (സ) പേരിൽ കള്ളം പറയില്ലെന്ന സങ്കല്പത്തിന്റെ അടിസ്ഥാനത്തിലാണ് ഹദീഥ് നിദാനശാസ്ത്രം അതിന്റെ അടിത്തറ പണിതിരിക്കുന്നത്. ഈ സങ്കൽപം തന്നെ അടിസ്ഥാനരഹിതമല്ലേ? സ്വഹാബിമാരുടെ സത്യസന്ധതയിൽ വിശ്വാസികൾക്ക് സംശയമില്ലായിരിക്കാം. എന്നാൽ തികച്ചും വ്യക്തിനിഷ്ഠവും ആത്മനിഷ്ഠവുമായ ഈ വിശ്വാസം എങ്ങനെയാണ് ശാസ്ത്രീയമായിത്തത്തീരുന്നത്?
ഖുര്ആന് ദൈവവചനവും മുഹമ്മദ് നബി(സ) ദൈവദൂതനുമാണെന്ന് മനസ്സിലാക്കുന്നവരെ സംബന്ധിച്ചിടത്തോളം സ്വഹാബിമാരുടെ സത്യസന്ധതയില് യാതൊരു സംശയവുമുണ്ടാകുവാന് തരമില്ല. മക്കയില് വെച്ച് നബി(സ)യില് വിശ്വസിക്കുകയും ത്യാഗങ്ങള് സഹിച്ച് പലായനം നടത്തുകയും ചെയ്ത മുഹാജിറുകളും മദീനയില് അവര്ക്ക് ആതിഥ്യമരുളുകയും അവിടെ ഒരു ഇസ്ലാമിക സമൂഹത്തിന് രൂപം നല്കാന് സഹായിക്കുകയും ചെയ്ത അന്സ്വാറുകളും ഇവരുടെ പിന്ഗാമികളായി ഇസ്ലാമിലെത്തിച്ചേര്ന്നവരുമടങ്ങുന്ന സ്വഹാ ബീസഞ്ചയത്തെ പ്രശംസിക്കുകയും അവരില് അല്ലാഹു സംപ്രീതനായിരിക്കുന്നുവെന്ന് പ്രഖ്യാപിക്കുകയും ചെയ്യുന്നുണ്ട്, ഖുര്ആന്: ''മുഹാജിറുകളില് നിന്നും അന്സ്വാറുകളില് നിന്നും ആദ്യമായി മുന്നോട്ട് വന്നവരും, സുകൃതം ചെയ്തുകൊണ്ട് അവരെ പിന്തുടര്ന്നവരും ആരോ അവരെപ്പറ്റി അല്ലാഹു സംതൃപ്തനായിരിക്കുന്നു. അവനെപ്പറ്റി അവരും സംതൃപ്തരായിരിക്കുന്നു. താഴ്ഭാഗത്ത് അരുവികള് ഒഴുകിക്കൊണ്ടിരിക്കുന്ന സ്വര്ഗത്തോപ്പുകള് അവര്ക്ക് അവന് ഒരുക്കിവെക്കുകയും ചെയ്തിരിക്കുന്നു. എന്നെന്നും അവരതില് നിത്യവാ സികളായിരിക്കും. അതത്രെ മഹത്തായ ഭാഗ്യം'' (9:100).
''വിശ്വസിക്കുകയും സ്വദേശം വെടിഞ്ഞ് പോകുകയും അല്ലാഹുവിന്റെ മാര്ഗത്തില് സമരത്തില് ഏര്പെടുകയും ചെയ്തവരും, അവ ര്ക്ക് അഭയം നല്കുകയും സഹായിക്കുകയും ചെയ്തവരും തന്നെയാണ് യഥാര്ഥത്തില് സത്യവിശ്വാസികള്. അവര്ക്ക് പാപമോചനവും മാന്യമായ ഉപജീവനവും ഉണ്ടായിരിക്കും'' (8:74).
അനുചരന്മാരെക്കുറിച്ച പ്രവാചക പരാമര്ശങ്ങളിലും അവര് സത്യസന്ധരും സന്മാര്ഗനിഷ്ഠരുമാണെന്ന വസ്തുത ഊന്നിപ്പറഞ്ഞത് കാണാനാവും. ''അബൂബുര്ദാ(റ)അബൂമൂസല് അശ്അരിയി(റ)വില് നിന്ന് നിവേദനം: നബി(സ)തന്റെ മുഖം ആകാശത്തേക്ക് ഉയര്ത്തി ക്കൊണ്ട് പറഞ്ഞു: ''നക്ഷത്രങ്ങള് ആകാശത്തിനുള്ള സുരക്ഷിതത്വമാണ്. നക്ഷത്രങ്ങള് നശിച്ചുകഴിഞ്ഞാല് ആകാശത്തിന് മുന്നറിയിപ്പ് നല് കപ്പെട്ടത് വന്നു ഭവിക്കുകയായി. ഞാന് എന്റെ അനുചരന്മാര്ക്കുള്ള സുരക്ഷിതത്വമാണ്. ഞാന് പോയിക്കഴിഞ്ഞാല് എന്റെ അനുചര ന്മാര്ക്ക് മുന്നറിയിപ്പ് നല്കപ്പെട്ടത് വന്നു ഭവിക്കുകയായി. എന്റെ അനുചരന്മാര് എന്റെ സമുദായത്തിനുള്ള സുരക്ഷിതത്വമാണ്. എന്റെ അനുചരന്മാര് പോയിക്കഴിഞ്ഞാല് എന്റെ സമുദായത്തിനും മുന്നറിയിപ്പ് നല്കപ്പെട്ടത് വന്നു ഭവിക്കുകയായി''(സ്വഹീഹുമുസ്ലിം, കിത്താബു ഫദാഇലി സ്സ്വഹാബ)
''അബ്ദുല്ലാ(റ)നിവേദനം: നബി(സ)പറഞ്ഞു: ''ജനങ്ങളില് ഏറ്റവും ഉത്തമര് എന്റെ തലമുറയാണ്. പിന്നീട് അവരെ തുടര്ന്ന് വരുന്നവരും പിന്നീട് അവരെ തുടര്ന്ന് വരുന്നവരും''(സ്വഹീഹുല് ബുഖാരി, കിത്താബു സ്സ്വഹാബ.)
''അബൂസഈദ് അല് ഖുദ്രി(റ)നിവേദനം: തിരുമേനി(സ)പറഞ്ഞു: ''എന്റെ അനുചരന്മാരെ നിങ്ങള് പഴി പറയരുത്. നിങ്ങളില് ഒരാള് ഉഹ്ദ് മലയോളം സ്വര്ണം ചെലവഴിച്ചാലും അവരിലൊരാള് ചെലവഴിച്ച ഒരു മുദ്ദിനോ (രണ്ട് കൈപ്പത്തികള് ചേര്ത്തുവെച്ചുകൊണ്ടുള്ള ഒരു വാരല്) അതിന്റെ പകുതിക്കുപോലുമോ എത്തുകയില്ല''(സ്വഹീഹുല് ബുഖാരി, കിത്താബു സ്സ്വഹാബ)
ക്വുര്ആനിന്റെയോ നബി വചനങ്ങളുടെയോ പ്രാമാണികത അംഗീകരിക്കാത്തവരെ സംബന്ധച്ചിടത്തോളം സ്വഹാബിക ളുടെ സത്യസന്ധതയ്ക്ക് അവ നല്കുന്ന സാക്ഷ്യം സ്വീകാര്യമാവില്ല.ഓറിയന്റലിസ്റ്റുകളുടെ ചരിത്രവിമര്ശനരീതി പ്രകാരം ഒരു കാലഘട്ടത്തിലെ ജനങ്ങളുടെയോ പ്രത്യേകമായ ഒരു ആദര്ശത്തിന്റെ വക്താക്കളുടെയോ സത്യസന്ധത നിര്ണയിക്കുവാനുള്ള മാനദണ്ഡമെന്താണെന്ന് ആരും വ്യക്തമാക്കിയിട്ടുമില്ല. വ്യക്തികളുടെ സത്യസന്ധത പരിശോധിച്ച് അവര് പറഞ്ഞ കാര്യങ്ങളിലെ നെല്ലും പതിരും വേര്തിരിക്കുന്ന ആത്മനിഷ്ഠമായ അപഗ്രഥന രീതി വസ്തുനിഷ്ഠവിശകലനത്തില് മാത്രം ശ്രദ്ധ കേന്ദ്രീകരിക്കുന്ന ചരിത്രവിമര്ശകന്മാര്ക്ക് പരിചയമുള്ളതല്ല.
എന്നാല് സ്വഹാബിമാരെപ്പറ്റി ഏതൊരാള്ക്കും മനസ്സിലാക്കാനാവുന്ന ചില വസ്തുതകളുണ്ട്. അവര് ജീവിച്ച സമൂഹം അവരുടെ സത്യസന്ധത യ്ക്ക് അന്യോന്യം സാക്ഷികളായിരുന്നുവെന്നതാണ് അതില് ഏറ്റവും പ്രധാനപ്പെട്ടത്. അവര് പരസ്പരം വിശ്വസിക്കുകയും പ്രവാചകനെ ക്കുറിച്ച് അവരില് ആരെങ്കിലുമൊരാള് എന്തെങ്കിലും പറഞ്ഞാല് അത് സത്യം തന്നെയാണെന്ന് കരുതുകയും ഇക്കാര്യത്തില് അവരെല്ലാ വരും പരസ്പരം സഹകരിക്കുകയും ചെയ്തിരുന്നു. 'എന്നെക്കുറിച്ച് ആരെങ്കിലും ബോധപൂര്വം കളവുകളെന്തെങ്കിലും പറഞ്ഞാല് നരകത്തില് അവന് അവന്റെ ഇരിപ്പിടം തയാറാക്കിക്കൊള്ളട്ടെ' (സ്വഹീഹുല് ബുഖാരി, കിതാബുല് ഇൽമ് ) എന്ന് നബിയിൽ (സ) നിന്ന് പഠിച്ചവരായിരുന്നു അവര്. അതുകൊണ്ടു തന്നെ, അവരില്പെട്ട ഒരാളും നബി(സ)യെക്കുറിച്ച് എന്തെങ്കിലുമൊരു കളവു പറയാന് യാതൊരു സാധ്യതയുമില്ലെന്ന് അവരെല്ലാവരും പരസ്പരം അംഗീകരിച്ചിരുന്നു. അതുകൊണ്ടാണല്ലോ നബി(സ) പറഞ്ഞുവെന്നോ ചെയ്തുവെന്നോ അനുവദിച്ചുവെന്നോ ഏതെങ്കിലു മൊരു സ്വഹാബി പറഞ്ഞാല് മറ്റുള്ളവര് അത് ചോദ്യംചെയ്യാതെ അംഗീകരിച്ചുവന്നത്.
ഓറിയന്റലിസ്റ്റുകളുടെയും അവരിൽ നിന്ന് മതം പഠിച്ച ഹദീഥ്നിഷേധികളുടെയും ശക്തമായ കടന്നാക്രമണ ത്തിന് വിധേയനായ സ്വഹാബി അബൂഹുറയ്റ(റ)നബി(സ) പറഞ്ഞതായി ഉദ്ധരിച്ച കാര്യങ്ങള് മറ്റു സ്വഹാബിമാര് ചോദ്യം ചെയ്യാതെ സ്വീകരിച്ചതായി വ്യക്തമാക്കുന്ന നിരവധി ഹദീഥുകളുണ്ട്. സ്ത്രീകള് പച്ചകുത്തുന്നതിനെക്കുറിച്ച് പ്രവാചകന്(സ)എന്തെങ്കിലും പറഞ്ഞി ട്ടുണ്ടോയെന്ന് ആരാഞ്ഞ ഉമറി(റ)നോട് പച്ചകുത്തുന്നത് വിരോധിച്ചുകൊണ്ടുള്ള നബികല്പനയെക്കുറിച്ച് അബൂഹുറയ്റ(റ) തെര്യപ്പെ ടുത്തുകയും അതനുസരിച്ച് ഉമര്(റ)വിധിച്ചതായും വ്യക്തമാക്കുന്ന സ്വഹീഹുല് ബുഖാരിയിലെ ഹദീഥ് ഉദാഹരണം.(കിതാബുല് ലിബാസ്.) ഒരു സ്വഹാബി യുടെ സാക്ഷ്യം മറ്റു സ്വഹാബിമാര് ചോദ്യം ചെയ്യാതെ അംഗീകരിച്ചിരുന്നുവെന്നാണല്ലോ ഇത് വ്യക്തമാക്കുന്നത്.
നബി(സ)യുടെ ജീവിതത്തെക്കുറിച്ച പരാമര്ശങ്ങളില് സ്വഹാബിമാരാരും കളവു പറയുകയില്ലെന്ന് പരസ്പരം അംഗീകരിച്ചിരുന്നുവെന്ന് പറഞ്ഞാല് ഓരോരുത്തരുടെയും സത്യസന്ധതയ്ക്ക് ഒരു ലക്ഷത്തിലധികം പേരുടെ സാക്ഷ്യമുണ്ടെന്നാണര്ഥം. നബി(സ)യുടെ അറഫാ പ്രസംഗത്തിനെത്തിയ സ്വഹാബിമാരുടെ എണ്ണം ഒരു ലക്ഷത്തിലധികമായിരുന്നുവെന്നാണ് കരുതപ്പെടുന്നത്. ഓരോരുത്തരുടെയും സത്യ സന്ധതയ്ക്ക് ഒരു ലക്ഷത്തിലധികം പേരുടെ സാക്ഷ്യം ലഭിച്ചാലും, വസ്തുനിഷ്ഠമായ തെളിവുകള് മാത്രമെ അംഗീകരിക്കൂവെന്ന് വാശിപി ടിക്കുന്ന ഓറിയന്റലിസ്റ്റുകള്ക്ക് അത് അംഗീകരിക്കുവാന് കഴിഞ്ഞുകൊള്ളണമെന്നില്ല. എന്നാല് ഹദീഥുകള് നിവേദനം ചെയ്ത ഓരോ സ്വഹാബിയുടെയും സത്യസന്ധതയ്ക്ക് നൂറുകണക്കിനാളുകളുടെ സാക്ഷ്യമുണ്ട് എന്ന വസ്തുനിഷ്ഠ യാഥാര്ഥ്യത്തിനു നേരെ കണ്ണടയ്ക്കു വാന് അവര്ക്ക് കഴിയുമോ?
ഏറെ വിമര്ശിക്കപ്പെട്ട അബൂഹുറയ്റ(റ)യുടെ കാര്യം തന്നെയെടുക്കുക. സത്യസന്ധരും വിശ്വസ്തരുമെന്ന് തെളിയിക്കപ്പെട്ട സ്വഹാബികളും താബിഉകളുമുള്പ്പെടുന്ന എണ്ണൂറോളം പേര് അബൂഹുറയ്റ(റ)യില് നിന്ന് ഹദീഥുകള് നിവേദനം ചെയ്തിട്ടുണ്ട്.(6) ഈ എണ്ണൂറോളമാളുകളും നബി(സ)യുടെ പേരില് കള്ളം പറയുന്നത് നരകപ്രവേശത്തിന് കാരണമാകുന്ന മഹാപാപ മാണെന്ന് വിശ്വസിക്കുന്നവരും ആയിരുന്നുവെന്നുറപ്പ്. അബൂഹുറയ്റ(റ)നബി(സ)യുടെ പേരില് കളവു പറയുവാന് വല്ല സാധ്യതയുമു ണ്ടെന്ന് അവര്ക്ക് തോന്നിയാല് അവരിലൊരാള്പോലും അദ്ദേഹത്തില്നിന്ന് ഹദീഥുകള് നിവേദനം ചെയ്യുമായിരുന്നില്ല. അബൂഹുറ യ്റ(റ)യെ കുറിച്ച് ഈ എണ്ണൂറോളം പേരുടെ സാക്ഷ്യം രേഖപ്പെടുത്തപ്പെട്ടതാണ്. ഇതിനെതിരെ അതിനെക്കാളധികം പേരുടെ സാക്ഷ്യമു ണ്ടെങ്കില് മാത്രമെ അദ്ദേഹത്തിന്റെ സത്യസന്ധതയെ ചോദ്യം ചെയ്യാനായി പ്രസ്തുത സാക്ഷ്യത്തെ തെളിവായി സ്വീകരിക്കാനാവൂ.
സ്വഹാബിമാരിലാരെങ്കിലും അബൂഹുറയ്റ(റ)യുടെ സത്യസന്ധതയെ സംശയിച്ചിരുന്നതായി തെളിയിക്കുന്ന യാതൊരു രേഖയും ഉദ്ധരി ക്കുവാന് അദ്ദേഹത്തിന്റെ വിമശകര്ക്ക് കഴിഞ്ഞിട്ടില്ല. അബൂഹുറയ്റ(റ)യുടെ സത്യസന്ധതയ്ക്ക് സാക്ഷ്യം വഹിക്കുന്ന എണ്ണൂറില ധികമാളുകളുടെ മൊഴിക്കെതിരായി സംസാരിക്കാനാകുന്ന സമകാലികനായ ഒരാളെപ്പോലും ഹാജരാക്കുവാന് അവര്ക്ക് സാധിച്ചിട്ടി ല്ലാത്തതിനാല് നൂറുകണക്കിന് സാക്ഷികളുടെ മൊഴി സ്വീകരിക്കുവാന് വസ്തുനിഷ്ഠതയുടെ വക്താക്കളെന്ന് അവകാശപ്പെടുന്ന ചരിത്ര വിമര്ശനരീതിക്കാര് നിര്ബന്ധിതരാണ്. അബൂഹുറയ്റ(റ)സത്യസന്ധനല്ലെന്ന് തെളിയിക്കുവാന് ചരിത്ര വിമര്ശനരീതിയുടെ വക്താക്ക ളുടെ പക്കല് കോപ്പുകളൊന്നുമില്ലെന്നര്ഥം. ഹദീഥുകള് നിവേദനം ചെയ്ത മുഴുവന് സ്വഹാബിമാരുടെയും സ്ഥിതി ഇതുതന്നെയാണ്. അവരുടെയെല്ലാം സത്യസന്ധതയ്ക്ക് നൂറുകണക്കിന് ആളുകളുടെ സാക്ഷ്യമുണ്ട്. തിരിച്ചാകട്ടെ, വിശ്വസ്തരും സമകാലികരുമായ ഒരാള്പോലും സാക്ഷ്യത്തിനില്ലതാനും!
അല്ല. നബിയെക്കുറിച്ച (സ) കൃത്യവും വസ്തുനിഷ്ഠവുമായ സംഭവവിവരണമാണ് ഹദീഥുകളിലുള്ളത്. ഹദീഥ് ശേഖരണത്തിൻെറയും ക്രോഡീകരണത്തിന്റെയും ത്യാഗപൂർണമായ ചരിത്രമറിയാത്തവരാണ് ഹദീഥുകളെക്കുറിച്ച് യാതൊരു ആദരവുമില്ലാതെ സംസാരിക്കുന്നത്. സ്വഹാബിമാരും താബിഉകളുമെല്ലാം വളരെ സൂക്ഷ്മതയോടുകൂടിയാണ് ഹദീഥുകൾ അടുത്ത തലമുറയ്ക്ക് കൈമാറിയത്. കള്ള ഹദീഥുകളിൽ നിന്ന് സമൂഹത്തെ രക്ഷിക്കുവാൻ അവർ പ്രത്യേകം ശ്രദ്ധിച്ചു. നബി (സ) യെ ക്കുറിച്ച് ഇല്ലാത്തതൊന്നും തങ്ങളുടെ നാവിലൂടെ പുറത്തുവരരുതെന്ന് അവർക്ക് നിർബന്ധമുണ്ടായിരുന്നു; അതോടൊപ്പം തന്നെ, നബിയിൽ (സ) നിന്നുള്ളതെന്ന രൂപത്തിൽ നബിയിൽ (സ) നിന്നുള്ളതല്ലാത്ത യാതൊന്നും സമൂഹത്തിൽ പ്രചരിക്കാതിരിക്കുവാനും അവർ ജാഗരൂകരായിരുന്നു. പ്രസ്തുത ജാഗ്രതയുടെ ഫലമായാണ് ഉസൂലുൽ ഹദീഥ് എന്ന ശാസ്ത്രം ഉണ്ടായിവന്നത്.
പ്രവാകാനുചരന്മാരില് നിന്ന് മതം പഠിച്ച താബിഉകള്ക്കുശേഷം, അവരില് നിന്ന് ഇസ്ലാമിക വിഷയങ്ങളെക്കുറിച്ച് മനസ്സിലാക്കിയ താബിഉത്താബിഉകളുടെ തലമുറയില് ഹദീഥ് പഠന-ശേഖരണ രംഗത്ത് വമ്പിച്ച മുന്നേറ്റം തന്നെയുണ്ടായി. അതിനായി ജീവിതം ഉഴിഞ്ഞു വെച്ച ത്യാഗികളുടെ കാലമായിരുന്നു അത്. ഇസ്ലാമിക കര്മശാസ്ത്രവിഷയങ്ങള് ക്രമരൂപത്തില് നല്കികൊണ്ട് ക്രോഡീകരിക്കപ്പെട്ട ഗ്രന്ഥങ്ങളാണ് അന്നത്തെ പണ്ഡിതന്മാര് രചിച്ചത്. ഓരോ വിഷയത്തെയും സംബന്ധിച്ച ഹദീഥുകള് ആ വിഷയത്തെക്കുറിച്ച് പറയുന്ന തിനിടക്ക് ഉദ്ധരിക്കുകയായിരുന്നു അവര് പൊതുവെ ചെയ്തിരുന്നത്. ഇത്തരം ഹദീഥ് ശേഖരങ്ങളെ മുസ്വന്നഫ് എന്നോ മുവത്വഅ് എന്നോ ആണ് വിളിക്കുന്നത്. ഇന്ന് ഉപലബ്ധമായ മുസന്നഫുകളില് ഏറ്റവും പഴക്കമുള്ളത്, ഇമാം മാലിക്കുബ്നു അനസിന്റെ (ഹിജ്റ 93-179) അല് മുവത്ത്വയാണ്. ഹിജ്റ 150ല് മരണപ്പെട്ട ഇബ്നു തുറൈബിന്റെയും 161ല് മരണപ്പെട്ട സുഫ്യാനു ഥ്ഥൗരിയുടെയും ഹിജ്റ 211ൽ മരണപ്പെട്ട അബ്ദുര്റസാഖ് അസ്വന്ആനിയുടെയും ഹിജ്റ235ല് മരണപ്പെട്ട അബൂബക്കര് ബ്ന് അബീശൈബയുടെയും മുസ്വന്നഫുകൾ പ്രസിദ്ധങ്ങളാണ്.
കുഴപ്പങ്ങളില്നിന്നും വ്യതിയാനങ്ങളില്നിന്നും മുസ്ലിം ബഹുജനങ്ങളെ സംരക്ഷിച്ച് വിശുദ്ധ ക്വുര്ആനിലൂടെയും പ്രവാചകചര്യ യിലൂ ടെയും അവരെ നയിക്കുന്നതിനു വേണ്ടി വ്യാജ ഹദീഥുകളെയും യഥാര്ഥ നബിചര്യകളെയും വേര്തിരിച്ച് മനസ്സിലാക്കുവാനും രേഖപ്പെടു ത്തുവാനുമുള്ള ത്യാഗപൂര്ണമായ പണ്ഡിത പരിശ്രമത്തോടൊപ്പം തന്നെ, വാമൊഴിയായി ലഭിച്ച ഹദീഥുകളുടെ വെളിച്ചത്തില് ദൈവിക മാര്ഗദര്ശനത്തിലൂടെ ജനങ്ങളെ നയിക്കുന്നതിനുവേണ്ടി അവര്ക്ക് മതവിധികള് പറഞ്ഞുകൊടുക്കുന്നതിന്നായുള്ളപരിശ്രമങ്ങളുമുണ്ടാ യി. ഇതിന്വേണ്ടി പരിശ്രമിച്ച പ്രധാനപ്പെട്ട പണ്ഡിതമാരുടെ പേരില് സ്ഥാപിക്കപ്പെട്ടതാണ് നാല് കര്മശാസ്ത്രധാരകളായ മദ്ഹബുകള്. തബഉത്താബിഉകളില്പെട്ട മദ്ഹബിന്റെ ഇമാമാണ് അബൂഹനീഫ നുഅ്മാനു ബ്നു ഥാബിത്. താബിഉകള്ക്കു ശേഷമുള്ള അടുത്ത തലമുറയില്-താബിഉത്താബിഉകള്ക്കിടയില്-ജീവിച്ച ഇമാം മാലിക്കുബ്നു അനസാണ് മദ്ഹബുകളുടെ ഇമാമായി ഗണിക്കപ്പെടുന്ന മറ്റൊരു മഹാപണ്ഡിതന്. മുസന്നഫുകളില് ഏറ്റവും പഴക്കമുള്ള അല്മുവത്വയുടെ കര്ത്താവായ അദ്ദേഹം ഇമാം അബൂഹനീഫയുടെ സമകാലികരിലൊരാളായിരുന്നു.മാലിക്കുബ്നു അനസ്ന്റെ ശിഷ്യനായിരുന്ന അബൂഅബ്ദില്ലാ മുഹമ്മദ്ബ്നു ഇദ്രീസുശ്ശാഫിഈയാണ് മദ്ഹബിന്റെ ഇമാമായി അറിയപ്പെടുന്ന മൂന്നാമന്: നൂറോളം ഗ്രന്ഥങ്ങളെഴുതിയിട്ടുണ്ട് ഇമാം ശാഫിഈ. ഇമാം ശാഫിഈയുടെ ശിഷ്യനായിരുന്ന അഹ്മദ്ബ്ന് മുഹമ്മദ് ബ്ന് ഹന്ബല് അബൂ അബ്ദുല്ലാ അശ്ശൈബാനിയാണ് നാലാമത്തെ മദ്ഹബിന്റെ ഇമാമായി അറിയപ്പെടുന്നത്.. മുസ്നദ് അഹ്മദ് ബിന്ഹന്ബലാണ് അദ്ദേഹത്തിന്റെ ഏറ്റവും പ്രധാനപ്പെട്ടതും പ്രസിദ്ധവുമായ ഹദീഥ്ശേഖരം.(13)
പ്രവാചക നില്നിന്ന് ഹദീഥുകള് നിവേദനം ചെയ്ത സ്വഹാബിമാരുടെ അടിസ്ഥാനത്തില് ക്രോഡീകരിക്കപ്പെട്ട ഹദീഥ് ഗ്രന്ഥങ്ങളാണ് മുസ്നദുകള് എന്നറിയപ്പെട്ടത്. ഓരോ പ്രത്യേക സ്വഹാബിയില്നിന്നും നിവേദനം ചെയ്യപ്പെട്ട ഹദീഥുകള് പ്രത്യേക അധ്യായമായാണ് മുസ്നദുകളില് ക്രോഡീകരിക്കപ്പെട്ടിരിക്കുന്നത്. ഇമാം ശാഫിഈയുടെ ശിഷ്യനും നാലാമത്തെ മദ്ഹബിന്റെ ഇമാമുമായ ഇമാം അഹ്മദ് ബ്ന് ഹന്ബ ലിന്റെ ഹദീഥ് ശേഖരമാണ് മുസ്നദുകളില് ഏറ്റവും പ്രധാനപ്പെട്ടതായി അറിയപ്പെടുന്നത്. വ്യാജ ഹദീഥുകള്ക്ക് ഹദീഥ് ഗ്രന്ഥങ്ങ ളില് സ്ഥാനം കുറയാന് മുസ്നദുകള് നിമിത്തമായി. ഒരാള് പ്രവാചകന്റെ പേരില് വല്ലതും പറയുകയും അയാള്ക്ക് നബിയില്നിന്ന് അയാള് വരെയുള്ള നിവേദകന്മാരുടെ ശൃംഖല അവതരിപ്പിക്കാന് കഴിയാതിരിക്കുകയും ചെയ്താല് അതിന്റെ സ്ഥാനം മുസ്നദുകളില്നിന്ന് സ്വാഭാവികമായും പുറത്തായിരിക്കും.
വിഷയക്രമത്തില് ഹദീഥുകളും സഹാബിമാരുടെയും താബിഉകളുടെയും അഭിപ്രായങ്ങളും ക്രോഡീകരിച്ചുകൊണ്ടുള്ള മുസന്നഫുകള് ക്കും പ്രവാചകരില് നിന്നുള്ള പൂര്ണമായ ഇസ്നാദിന്റെ അടിസ്ഥാനത്തില് ക്രോഡീകരിക്കപ്പെട്ട മുസ്നദുകളുടെയും നന്മകള് സ്വാംശീക രിച്ചുകൊണ്ട് ഹിജ്റ മൂന്നാം നൂറ്റാണ്ടിന്റെ തുടക്കത്തില് രചിക്കപ്പെട്ട ഹദീഥ് സമാഹാരങ്ങളാണ് 'സുനന്'എന്ന് അറിയപ്പെടുന്നത്. വിഷയ ക്രമത്തില് ക്രോഡീകരിക്കപ്പെട്ടതും പൂര്ണമായ ഇസ്നാദോടുകൂടി ഉദ്ധരിക്കപ്പെട്ടതുമായ ഹദീഥുകളാണ് സുനന് ഗ്രന്ഥങ്ങളില് ക്രോഡീകരി ക്കപ്പെട്ടിരിക്കുന്നത്. ഹിജ്റ 227ല് മരണപ്പെട്ട സഈദ്ബ്നു മന്സൂന് അല്ഖുറാസാനിയും 255ല് മരണപ്പെട്ട അബ്ദുല്ലാഹിബ്നു അബ്ദുറഹ്മാ ന് അദ്ദാരിമിയുമാണ് ആദ്യകാല സുനനുകളുടെ കര്ത്താക്കള്.
ഹദീഥ് പഠനരംഗത്തെ നെല്ലും പതിരും വേര്തിരിച്ച് സംസ്കരിക്കുകയും പ്രവാചകനില് നിന്നുള്ളതാണെന്ന് ഉറപ്പുള്ള ഹദീഥുകള് മാത്രം ശേഖരിച്ച് മുസ്ലിംലോകത്തിന് നല്കുകയും ചെയ്ത മഹാ പ്രതിഭാശാലിയാണ് മുഹമ്മദ് ബ്ന് ഇസ്മായീല് അല് ബുഖാരി (ഹിജ്റ 196-256). അദ്ദേഹത്തിന്റെ ത്യാഗപൂര്ണമായ പരിശ്രമങ്ങളുടെ ഫലമായാണ് സ്വഹീഹായ ഹദീഥുകളുടെ മാത്രമായുള്ള ഒന്നാമത്തെ സമാഹാരമായ സ്വഹീഹുല് ബുഖാരി മുസ്ലിംലോകത്തിന് ലഭിച്ചത്. പതിനാറ് വര്ഷങ്ങള് നീണ്ടുനിന്ന നിരന്തരമായ യാത്രകളിലൂടെ ഹദീഥുകളറിയാവുന്ന ആയിരത്തിലധികം പേരുമായി ആശയക്കൈമാറ്റം നടത്തിക്കൊണ്ട് അദ്ദേഹം ശേഖരിച്ച ഏഴു ലക്ഷത്തോളം ഹദീഥു കളില്നിന്ന് ഇസ്നാദ് പരിശോധിച്ച് ഉറപ്പു വരുത്തിയശേഷം 7397 ഹദീഥുകളെ മാത്രം ഉള്ക്കൊള്ളിച്ചുകൊണ്ടാണ് ബുഖാരി തന്റെ അല് ജാമിഉ സ്സ്വഹീഹ് രചിച്ചത്. ഇതില് തന്നെ പല ഹദീഥുകളും ഒരേ പ്രവാചകചര്യയുടെ തന്നെ വ്യത്യസ്ത രൂപത്തിലുള്ള ആവര്ത്തനങ്ങ ളാണ്. ആകെ 2602 പ്രവാചകവചനങ്ങള് വ്യത്യസ്ത നിവേദകരിലൂടെ കടന്നുവന്നവയാണ് ബുഖാരിയിലുള്ള ഹദീഥുകളെന്ന് അതിന്റെ വ്യാഖ്യാതാവായ ഇബ്നു ഹജറുല് അസ്ഖലാനി വ്യക്തമായിട്ടുണ്ട്.
അബുല് ഹുസൈന് മുസ്ലിമിബ്നുല് ഹജ്ജാജ് അല് നൈസാപൂരി (ഹി 202-261) ആണ് സ്വഹീഹായ ഹദീഥുകളെ മാത്രം ക്രോഡീകരിച്ചുകൊണ്ട് ഗ്രന്ഥരചന നടത്തിയ മറ്റൊരു മഹാവ്യക്തിത്വം. നാല്പത്തിമൂന്ന് അധ്യായങ്ങളിലായി 7563 ഹദീഥുകളാണ് അദ്ദേഹത്തിന്റെ സ്വഹീഹു മുസ്ലിമിലുള്ളത് ഇമാം ബുഖാരിയുടെയും ഇമാം മുസ്ലിംന്റെയും ശിഷ്യനായി രുന്ന അബൂബക്കര് മുഹമ്മദ്ബ്നു ഇസ്ഹാക്വ്ബ്നു ഖുസൈമയും സ്വഹീഹായ ഹദീഥുകള് മാത്രം ക്രോഡീകരിച്ചുകൊണ്ട് ഒരു ഗ്രന്ഥമെ ഴുതിയിട്ടുണ്ട്. സ്വഹീഹ് ഇബ്നു ഖുസൈമ എന്നാണ് അതിന്റെ പേര്.
ഇതിനുശേഷം പലരും ഹദീഥുകള് ക്രോഡീകരിച്ച് ഗ്രന്ഥങ്ങളെഴുതിയെങ്കിലും മുസ്ലിം ലോകത്ത് പരക്കെ അറിയപ്പെട്ടത് ഇവയിലുള്ള പ്രധാനപ്പെട്ട നാല് ഹദീഥ് സമാഹാരങ്ങളാണ്. ഹിജ്റ 275ല് അന്തരിച്ച ഇമാം അഹ്മദ് ബ്നു ഹന്ബലിന്റെ ശിഷ്യനായിരുന്ന അബൂദാവൂദ് സുലൈമാന് ഇബ്നു അശ്അഥ് അസ്സിജിസ്താനി രചിച്ച സുനനു അബീദാവൂദ്, ഹിജ്റ 279ല് അന്തരിച്ച, ഇമാം ബുഖാരിയുടെ ശിഷ്യ നായിരുന്ന അബൂഈസ മുഹമ്മദ് ബ്ന് ഈസ അത്തിര്മിദി രചിച്ച അല്ജാമിഉത്തിര്മിദി, ഹിജ്റ 303ല് അന്തരിച്ച, ഇമാം അബൂദാവൂ ദിന്റെ ശിഷ്യനായിരുന്ന അഹ്മദ്ബ്നു ശൂഐബ് അന്നസാഈ രചിച്ച സുനനുന്നസാഈ, ഹിജ്റ 273ല് അന്തരിച്ച അബൂഅബ്ദില്ലാ മുഹ മ്മദ്ബ്നു യസീദുബ്നുമാജ രചിച്ച സുനനു ഇബ്നിമാജ(24) എന്നിവയാണീ ഗ്രന്ഥങ്ങള്. സുനനു അബൂദാവൂദില് 4800 ഹദീഥുകളും ജാമി ഉത്തിര്മിദിയില് 3950 ഹദീഥുകളും സുനനുന്നസാഇയില് 5750 ഹദീഥുകളും സുനനു ഇബ്നുമാജയില് 4485 ഹദീഥുകളുമാണുള്ളത്. കൃത്യ മായി പ്രവാചകനിലല് നിന്ന് തുടങ്ങി ഗ്രന്ഥം ക്രോഡീകരിച്ചവരില് അവസാനിക്കുന്ന വിശ്വസ്തരുടെ ശൃംഖലയായ ഇസ്നാദുള്ളവയല്ല ഈ നാല് ഹദീഥ് സമാഹാരങ്ങളിലെയും ചില ഹദീഥുകളെന്ന വസ്തുത അവയുടെ സമാഹര്ത്താക്കള് തന്നെ സൂചിപ്പിച്ചിട്ടുണ്ട്. സഹീ ഹുല് ബുഖാരിയിലെയും സഹീഹു മുസ്ലിമിലെയും ഹദീഥുകള് മുസ്ലിംലോകം ചോദ്യം ചെയ്യാതെ സ്വീകരിക്കുമ്പോള് മറ്റ് നാല് ഗ്രന്ഥ ങ്ങളിലെയും ഹദീഥുകള് അവയുടെ ഇസ്നാദ് പരിശോധിച്ച ശേഷം അവ സ്വീകാര്യമാണെന്ന് ബോധ്യപ്പെട്ടതിനുശേഷം മാത്രമെ സ്വീകരിക്ക പ്പെടുകയുള്ളൂ.
മുഹമ്മദ് (സ) യുടെ ജീവിതത്തെക്കുറിച്ച് നമുക്ക് അറിവു നല്കുന്ന രണ്ടാമത്തെ സ്രോതസ്സായ ഹദീഥുകള് എത്രത്തോളം കൃത്യവും സൂക്ഷ്മവുമായാണ് രേഖപ്പെടുത്തപ്പെട്ടതെന്ന് ഹദീഥ് നിദാനശാസ്ത്രത്തിന്റെ ഗ്രന്ഥങ്ങള് പരിശോധിച്ചാല് ബോധ്യമാകും. പ്രവാചക ന്(സ)യോടൊപ്പം സഹവസിച്ചവര്, തെറ്റുകളൊന്നും വരുത്താതെ, സൂക്ഷ്മവും കൃത്യവുമായി അടുത്ത തലമുറക്ക് പറഞ്ഞുകൊടുത്തതെ ന്ന് ഉറപ്പുള്ള നിവേദനം മാത്രമെ സ്വഹീഹായ ഹദീഥായി പരിഗണിക്കപ്പെടുകയുള്ളൂ. ഇത്രയ്ക്കും കൃത്യവും സൂക്ഷ്മവുമായി രേഖപ്പെ ടുത്തപ്പെട്ട മറ്റൊരു ജീവചരിത്രവുമില്ലെന്നതാണ് വാസ്തവം. ആധുനിക കാലത്തെ ചരിത്രരചനയില് പോലും രചയിതാവിന്റെ വ്യക്തിത്വ ത്തെ വിമര്ശനവിധേയമാക്കി പറയുന്ന കാര്യങ്ങളുടെ യാഥാര്ഥ്യം മനസ്സിലാക്കുന്നതിനു വേണ്ടിയുള്ള സങ്കേതങ്ങള് വേണ്ടവിധം വികസി പ്പിച്ചെടുക്കാന് കഴിഞ്ഞിട്ടില്ല. ഒരേ വ്യക്തിയുടെ ജീവിതത്തെ രണ്ടു രൂപത്തില് നോക്കിക്കാണുന്നവര് എഴുതിയ ചരിത്ര ഗ്രന്ഥങ്ങളിലെ പരാമര്ശങ്ങളുടെ സത്യത പരിശോധിക്കുവാന് നമ്മുടെ പക്കല് കാര്യമാത്രപ്രസക്തമായ മാനദണ്ഡങ്ങളൊന്നുമില്ല.
നബി(സ) യോടൊപ്പം ജീവിക്കുകയും അദ്ദേഹത്തിന്റെ ജീവിതം നേര്ക്കുനേരെ മനസ്സിലാക്കുകയും അത് രേഖപ്പെടുത്തുകയോ മറ്റുള്ളവര്ക്ക് പറഞ്ഞുകൊടു ക്കുകയോ ചെയ്യുമ്പോള് അബദ്ധങ്ങളോ അസത്യങ്ങളോ കടന്നുകൂടാതിരിക്കുവാന് സൂക്ഷ്മത പ്രകടിപ്പിക്കുകയും ചെയ്തവരില്നിന്ന് നിവേദനം ചെയ്യപ്പെട്ട നബിചരിത്രമാണ് ഹദീഥുകളിലുള്ളത്. നബി(സ) യുടെ അകവും പുറവും മനസ്സിലാക്കിയവരുടെ നേര്ക്കുനേരെയുള്ള ചിത്രീകരണം. ആ രൂപത്തില് ഒരാളുടെയും ചരിത്രം രേഖപ്പെടുത്തപ്പെട്ടിട്ടില്ല. ആത്മകഥയ്ക്ക്പോലും ഇത്രയ്ക്ക് സൂക്ഷ്മമായ ഒരു ജീവിതാഖ്യാനം നടത്താന് കഴിയില്ല. സ്വന്തത്തിന്റെ കുറവുകള് കാണാന് ആത്മകഥാകാരന് കഴിയില്ലല്ലോ. ഒരു ലക്ഷത്തിലധികം പേരുടെ ദൃക്സാക്ഷി വിവരണത്തിന്റെ സാക്ഷ്യമാണ് സ്വഹീഹായ ഹദീഥുകള്ക്കുള്ളത്. നബി(സ) മരണപ്പെടുമ്പോള് ജീവിച്ചിരുന്ന സ്വഹാബിമാരുടെ എണ്ണം ഒരു ലക്ഷത്തിലധികമായിരുന്നല്ലോ.
ക്വുര്ആന് അവതരിപ്പിക്കപ്പെടുന്ന മുറയ്ക്ക് എഴുതി സൂക്ഷിക്കാറുണ്ടായിരുന്നതുപോലെ നബി വചനങ്ങളോ കര്മങ്ങളോ എഴുതി സൂക്ഷിക്കുന്ന പതിവ് മുഹമ്മദ് നബി(സ)യുടെ ജീവിതകാലത്ത് ഉണ്ടായിരുന്നില്ല എന്നത് ശരിയാണ്. എന്നാല് ചില സ്വഹാബികള് നബി(സ)യുടെ വചനങ്ങള് എഴുതിവെക്കുകയും സൂക്ഷിക്കുകയും ചെയ്തിരുന്നതായി രേഖകളുണ്ട്. ഖുര്ആന് വചനങ്ങളും ഹദീഥുകളും തമ്മില് കൂടിക്കലരരുതെ ന്ന് നിര്ബന്ധമുള്ളതിനാല് 'ക്വുര്ആനല്ലാത്ത മറ്റൊന്നുംതന്നെ തന്നില്നിന്ന് എഴുതി സൂക്ഷിക്കരുതെന്ന് ആദ്യകാലത്ത് നബി(സ) വിലക്കിയി രുന്നു'(അബൂസഈദുല് ഖുദ്രിയില് നിന്ന് മുസ്ലിം ഉദ്ധരിച്ചത്) വെങ്കിലും പ്രത്യേക സന്ദര്ഭങ്ങളില് അങ്ങനെ ചെയ്യാന് നിര്ദേശിച്ചിരുന്നതായും കാണാന് കഴിയും. മക്കാവിജയകാലത്ത് മക്കയുടെ പവിത്രതയെക്കുറിച്ച് നബി(സ) നടത്തിയ ഒരു പ്രഭാഷണം കഴിഞ്ഞപ്പോള് അത് തനിക്ക് എഴുതിത്തരണമെന്ന് യമന്കാരനായ അബൂശാഹ് ആവശ്യപ്പെട്ടതായും അദ്ദേഹത്തിന് അത് എഴുതിക്കൊടുക്കുവാന് പ്രവാചകന്ല നിര്ദേശിച്ചതായും ബുഖാരിയും മുസ്ലിമും രേഖപ്പെടുത്തിയിട്ടുണ്ട്. പ്രവാചകശിഷ്യനായിരുന്ന അബ്ദുല്ലാഹിബ്നു അംറുബ്നുല് ആസ്വ്(റ) , ഹദീഥുകള് എഴുതി സൂക്ഷി ച്ചിരുന്നതായി അബൂ ഹുറൈറ സാക്ഷ്യപ്പെടുത്തുന്ന ഹദീഥ് ബുഖാരിയിലുണ്ട്. തനിക്ക് ഹദീഥുകള് എഴുതി സൂക്ഷിക്കുവാന് പ്രവാച കന്(സ) അനുവാദം നല്കിയതായി അബ്ദുല്ലാഹിബ്നു അംറ് (റ) അവകാശപ്പെട്ടതായി അഹ്മദും അബൂദാവൂദും ഉദ്ധരിച്ചിട്ടുണ്ട്.
നബി ജീവിതത്തെക്കുറിച്ച് തങ്ങള്ക്കറിയാവുന്ന കാര്യങ്ങള് സ്വഹാബിമാരില് ചിലര് എഴുതി സൂക്ഷിച്ചിരുന്നുവെങ്കിലും അത് വ്യാപകമായി രുന്നില്ല. തങ്ങള് നേര്ക്കുനേരെ കണ്ട നബിജീവിതത്തിന്റെ വ്യത്യസ്ത വശങ്ങളെപ്പറ്റി അവര് മറ്റുള്ളവര്ക്ക് പറഞ്ഞുകൊടുക്കുന്ന രീതിയാ യിരുന്നു വ്യാപകമായി നിലനിന്നിരുന്നത്. വാമൊഴിയായാണ് പ്രധാനമായും നബിജീവിതത്തെ കുറിച്ച വര്ത്തമാനങ്ങള് കൈമാറ്റം ചെയ്യപ്പെട്ടതെന്ന് സാരം.
രാഷ്ട്രീയവും സൈദ്ധാന്തികവുമായ ആവശ്യങ്ങള്ക്കുവേണ്ടി വ്യാജഹദീഥുകള് നിര്മിക്കപ്പെടുന്ന അവസ്ഥയുണ്ടായപ്പോള് അതിനെതിരെ വിശ്വാസീസമൂഹം ജാഗരൂകരായി. രണ്ടാം ഖലീഫ ഉമര് (റ) തന്റെ ഭരണകാലത്ത് ഹദീഥുകള് ശേഖരിച്ച് ക്രോഡീകരിക്കുവാന് ആഗ്രഹി ച്ചെങ്കിലും ക്വുര്ആന് വചനങ്ങളും ഹദീഥുകളും തമ്മില് കൂടിക്കലര്ന്നു പോകുമോയെന്ന ഭയം കാരണം അത് ഉപേക്ഷിച്ചതായി മുഹമ്മദ് ബ്നു സഅദ് രേഖപ്പെടുത്തുന്നുണ്ട്. എന്നാല് ഈ രംഗത്ത് ക്രിയാത്മകമായ ഒരു ഇടപെടല് നടത്തിയത് രണ്ടാം ഉമര് എന്നറിയപ്പെടുന്ന ഉമറുബ്നു അബ്ദുല് അസീസ് (റ) ആണ്. താബിഉകളില്പ്പെട്ട സുപ്രസിദ്ധനായ ഭരണാധികാരിയായിരുന്ന അദ്ദേഹത്തിന്റെ കാലമായപ്പോ ഴേക്ക് വ്യാജ ഹദീഥുകളുടെ നിര്മാണം വ്യാപകമായിക്കഴിഞ്ഞിരുന്നു. മദീനയിലെ അദ്ദേഹത്തിന്റെ ന്യായാധിപനായിരുന്ന അബൂബക്കര് ബിനു ഹസമിന് അദ്ദേഹം എഴുതി: 'ദൈവദൂതരില്നിന്നുള്ള ഹദീഥുകള് താങ്കള് നോക്കുകയും എഴുതി രേഖപ്പെടുത്തുകയും ചെയ്യണം. കാരണം അറിവ് തേഞ്ഞുമാഞ്ഞു പോകുന്നതും ജ്ഞാനികള് കാലംകഴിഞ്ഞു പോകുന്നതും ഞാന് ഭയപ്പെടുന്നു. അല്ലാഹുവിന്റെ ദൂതരില് നിന്നുള്ള ഹദീഥുകളല്ലാതെ മറ്റൊന്നും സ്വീകരിക്കരുത്. അറിവ് പകര്ന്നുകൊടുക്കുകയും അറിവില്ലാത്തവരെ പഠിപ്പിക്കുകയും ചെയ്യുക; ജ്ഞാനം എല്ലാവരും രഹസ്യമാക്കുമ്പോഴല്ലാതെ നശിക്കുകയില്ല'. ഉമര് ബ്നു അബ്ദുല് അസീസ്ന്റെ നിര്ദേശപ്രകാരം മദീനയിലെ സ്വഹാ ബികളില് നിന്നും താബിഉകളില്നിന്നും അബൂബക്കര് ബ്നു ഹസം (റഹ്) ഹദീഥുകള് ശേഖരിച്ചു. അന്നു ജീവിച്ചിരുന്ന മഹാപണ്ഡിതനാ യിരുന്ന മുഹമ്മദ്ബ്നു മുസ്ലിബിനു ശിഹാബ് അസ്സുഹ്രിയും രണ്ടാം ഉമറിന്റെ ഭരണകാലത്ത് ഹദീഥുകള് ശേഖരിക്കുകയും ക്രോഡീ കരിക്കുകയും ചെയ്യുവാന് മുന്നോട്ടുവന്നു. ഇതോടൊപ്പം തന്നെ, ഇസ്ലാമികരാഷ്ട്രത്തിന്റെ വ്യത്യസ്ത കോണുകളിലേക്ക് ഹദീഥുകള് ശേഖരിക്കുവാന് ആവശ്യപ്പെട്ടുകൊണ്ട് ഉമറുബ്നു അബ്ദുല് അസീസ് കത്തുകളയിച്ചിരുന്നുവെന്ന് അബൂനുഐമിന്റെ താരിഖുല് ഇസ്ബ ഹാനില് നിന്ന് ഇബ്നുഹജറുല് അസ്ഖലാനി ഉദ്ധരിക്കുന്നുണ്ട്.(ഫത്ഹുല്ബാരി, വാല്യം 1, കിത്താബുല് ഇല്മ്) ഇങ്ങനെ ശേഖരിക്കപ്പെട്ട ഹദീഥുകള് ക്രോഡീകരിച്ചു രേഖപ്പെടുത്തിയത് ഇമാം സുഹ്രിയായിരുന്നു. അതിനുശേഷം വ്യത്യസ്ത ദേശക്കാരായ പല താബിഉകളും ഹദീഥുകള് ശേഖരിക്കുവാന് തുടങ്ങി. ഹിജ്റ 150ല് അന്തരിച്ച അബ്ദുല് മലിക്കു ബ്നു അബ്ദുല് അസീസ് ബ്നു ജുറൈജ് മക്കയിലും ഹിജ്റ 157ല് അന്തരിച്ച സഈദ്ബിനു അബിഅറൂബ മെസപ്പെട്ടോമിയയിലും ഹിജ്റ 159ല് അന്തരിച്ച അബൂഅംറില് ഔസാഈ സിറിയയിലും ഹിജ്റ 159ല് തന്നെ അന്തരിച്ച മുഹമ്മദ് ബ്നു അബ്ദിര് റഹ്മാന് മദീനയിലും ഹിജ്റ 160ല് അന്തരിച്ച സൈദ് ബ്നുക്വുദാമയും സുഫ്യാനുഥൗരിയും കൂഫയിലും ഹിജ്റ 165ല് അന്തരിച്ച ഹമ്മാദ് ബ്നു സലമ ബസറയിലും വെച്ച് ഹദീഥുകള് ശേഖരിക്കുകയും രേഖപ്പെടുത്തുകയും ചെയ്തതായി രേഖകളുണ്ട്.(ഇബ്നുല് നദീമിന്റെ അല് ഫിഹിരിസ്തില് നിന്ന് )
സ്വഹാബിമാരും താബിഉകളുമെല്ലാം ഹദീഥുകൾ രേഖപ്പെടുത്തി സൂക്ഷിച്ചിരുന്നുവെന്ന് ഇവ വ്യക്തമാക്കുന്നുണ്ട്. എന്നാൽ അവർക്കു ശേഷം മൂന്നാം തലമുറ മുതൽക്കാണ് ഹദീഥ് രേഖീകരണം വ്യാപകമായി ആരംഭിച്ചത് . ഹദീഥ് നിദാനശാസ്ത്രം വളർച്ച പ്രാപിച്ചതും അക്കാലത്ത് തന്നെയായിരുന്നു
നബിയുടെ (സ) പേരിൽ പിൽക്കാലത്ത് കള്ളങ്ങൾ പ്രചരിപ്പിക്കപ്പെട്ടിരുന്നുവെന്നത് ശരിയാണ്. അതുകൊണ്ട് തന്നെ വ്യാജങ്ങളിൽ നിന്ന് മുക്തമായ നബിചര്യ സംരക്ഷിക്കുന്നതിന് വേണ്ടിയുള്ള ശാസ്ത്രീയമായ രീതി വളർത്തിയെടുക്കുവാൻ മുസ്ലിംകൾക്ക് കഴിയുകയും ചെയ്തു. 'എന്റെ പേരില് ആരെങ്കിലും കളവുപറഞ്ഞാല് അവന് നരകത്തില് തന്റെ ഇരിപ്പിടം ഒരുക്കിക്കൊള്ളട്ടെ'യെന്ന നബി(സ)യുടെ താക്കീത് നേര്ക്കുനേരെ ശ്രവിച്ചവരായ സ്വഹാബിമാര് ഹദീഥുകള് ഉദ്ധരിക്കുകയും പഠിപ്പിക്കുകയുമെല്ലാം ചെയ്യുമ്പോള് പുലര്ത്തിയിരുന്ന സൂക്ഷ്മത അതേപോലെ പാലിക്കുവാന് അടുത്ത തലമുറയിലെ ചിലര്ക്കെങ്കിലും കഴിഞ്ഞില്ല.
മുഹമ്മദ് നബി(സ)ക്കു ശേഷം രണ്ടു ഖലീഫമാരുടെ കാലത്തുമില്ലാതിരുന്ന ചില രാഷ്ട്രീയകുഴപ്പങ്ങള് മൂന്നാമത്തെ ഖലീഫയായ ഉഥ്മാന്(റ) ന്റെ ഭരണകാലത്ത് തലപൊക്കി. അറേബ്യന് ഉപദ്വീപിന് പുറത്തുള്ള പ്രദേശങ്ങളിലേക്കടക്കം ഇസ്ലാമിക സാമ്രാജ്യം വിസ്തൃതമാവുകയും അവിടെയുള്ള നിരവധിപേര് ഇസ്ലാം സ്വീകരിക്കുകയും ചെയ്ത കാലമായിരുന്നു അത്. തങ്ങള് ജീവിക്കുന്ന സാമ്രാജ്യത്തിന്റെ മതമായ ഇസ്ലാമിന്റെ വക്താക്കളാണ് തങ്ങളുമെന്ന് വരുത്തിത്തീര്ക്കുകയും എന്നാല് ഇസ്ലാമികാദര്ശങ്ങള് സ്വന്തം മനസ്സിനകത്തേക്ക് കടക്കാതിരിക്കുകയും ചെയ്തവരും അവരിലുണ്ടായിരുന്നു. അവരില് ചിലരെങ്കിലും ഇസ്ലാമിനെയും മുസ്ലിം സമൂഹത്തെയും അകത്തുനിന്ന് നശിപ്പിക്കാമെന്ന് കരുതി ഇസ്ലാമിന്റെ കുപ്പായമണിഞ്ഞ കപടന്മാരായിരുന്നു. രണ്ടാം ഖലീഫയായിരുന്ന ഉമറുല്ഫാറൂഖ്െ(റ) ന വധിക്കുവാന് ഗൂഢാലോചന നടത്തിയ യഹൂദനായ സബഅ് ബ്നു ശാമൂനിന്റെ പുത്രനും യമനിലെ സന്ആയിലെ യഹൂദ റബ്ബിയുമായിരുന്ന അബ്ദുല്ലാഹിബ്നു സബഅ് താന് മുസ്ലിമായിയെന്ന് സ്വയം പ്രഖ്യാപിച്ചുകൊണ്ട് രംഗത്തു വരികയും മുസ്ലിംകളെ തമ്മിലടിപ്പിക്കുവാനുള്ള തന്ത്രങ്ങള് മെനയുകയും ചെയ്തത് ഉഥ്മാന്െ (റ) ന്റ ഭരണകാലത്താണ്.
അയാളും മറ്റു കപടന്മാരും തങ്ങളുടെ ആശയങ്ങള് പ്രചരിപ്പിക്കുവാനും മുസ്ലിംകളെ വഴിതെറ്റിക്കുവാനും തമ്മിലടിപ്പിക്കുവാനും വേണ്ടി നബി(സ)യുടെ പേരില് അദ്ദേഹം പറയാത്തതും ചെയ്യാത്തതും ആരോ പിച്ചുകൊണ്ട് ഹദീഥുകള് എന്ന വ്യാജേന തങ്ങളുടെ രചനകള് പ്രചരിപ്പിക്കുവാന് തുടങ്ങി. ഇത്തരം വ്യാജ ഹദീഥുകള് സ്വീകരിക്കരു തെന്ന് അന്നു ജീവിച്ചിരുന്ന പണ്ഡിതന്മാര് ജനങ്ങളോട് ആഹ്വാനം ചെയ്തു. അബ്ദുല്ലാഹിബ്നു സബഇന്റെ മസ്തിഷ്കസന്തതിയായ ശിആയിസത്തിന്റെ താത്ത്വികമായ അടിത്തറകള് സ്ഥാപിക്കപ്പെട്ടിരിക്കുന്നത് ഇത്തരം വ്യാജ ഹദീഥുകളിന്മേലാണ്. ശിആക്കളില്പെട്ട റാഫിദികളെപ്പറ്റി ഇമാം മാലിക് പറഞ്ഞത് ''അവരോട് സംസാരിക്കരുത്. അവരില് നിന്ന് ഹദീഥുകള് നിവേദനം ചെയ്യുകയും അരുത്. കാരണം അവര് കള്ളം പറയുന്നവരാണ്." എന്നായിരുന്നു.
മുഹമ്മദ് നബി(സ)യുടെ പേരില് വ്യാജ ഹദീഥുകള് നിര്മിച്ചുകൊണ്ട് ആ രംഗത്ത് കുഴപ്പങ്ങള്ക്ക് വാതില് തുറന്നുകൊടുത്തത് അബ്ദുല്ലാ ഹിബ്നു സബഇന്റെ നേതൃത്വത്തിലുള്ള ശിആക്കളായിരുന്നുവെങ്കിലും പിന്നീട് മതത്തിന്റെ അന്തരാത്മാവ് ഉള്ക്കൊള്ളാതെ ഇസ്ലാമി ലുള്ളവരാണെന്ന നാട്യവുമായി നടക്കുന്ന പലരും വ്യാജഹദീഥുകള് നിര്മിച്ചുകൊണ്ട് തങ്ങളുടെ ആശയങ്ങള് പ്രചരിപ്പിക്കുകയും സ്വാര്ഥ താല്പര്യങ്ങള് സംരക്ഷിക്കുവാന് ശ്രമിക്കുകയും ചെയ്തു. രാഷ്ട്രീയമായ ഭിന്നിപ്പുകളും അധികാരവടംവലിയും, ഇസ്ലാമിന്റെ വളര് ച്ചയോടുള്ള വിരോധവും പകയും, വര്ഗീയവും വംശീയവുമായ പക്ഷപാതങ്ങള്, ആദര്ശപരമായ ഭിന്നിപ്പുകളില് തങ്ങളുടെ കക്ഷിയാ ണ് ശരിയെന്ന് സ്ഥാപിക്കുക, അധികാരികളുടെ സാമീപ്യം സിദ്ധിക്കുക, കഥകള്ക്ക് വിശ്വാസ്യതയുണ്ടാക്കുക, നന്മയാണെന്ന് തങ്ങള് കരുതു ന്ന കാര്യങ്ങള്ക്ക് ജനപിന്തുണ നേടിയെടുക്കുക തുടങ്ങിയ ലക്ഷ്യങ്ങള്ക്കുവേണ്ടി വ്യാജ ഹദീഥുകള് നിര്മിക്കപ്പെടുകയും പ്രചരിപ്പിക്ക പ്പെടുകയും ചെയ്തു. ഇങ്ങനെ വ്യാജ ഹദീഥുകളുടെ നിര്മാണവും സംപ്രേഷണവും നടന്നുകൊണ്ടിരുന്നപ്പോള് ഹദീഥുകളുടെ സ്വീകാര്യത പരിശോധിക്കുന്നതിനുവേണ്ടി കുറ്റമറ്റ ഒരു സമ്പ്രദായം അക്കാലത്തെ പണ്ഡിതന്മാര് സ്വീകരിച്ചു. ഒരാള് ഒരു ഹദീഥ് നിവേദനം ചെയ്യുക യാണെങ്കില് നബി(സ)യില്നിന്ന് അയാള്ക്ക് അത് കിട്ടിയതെങ്ങനെയാണെന്ന് പഠിക്കുകയും നബി(സ)ക്കും അയാള്ക്കുമിടയിലുള്ള നിവേദ കന്മാരുടെ സത്യസന്ധതയും സ്വീകാര്യതയും സൂക്ഷ്മമായി പരിശോധിച്ചശേഷം അവരെല്ലാം കുറ്റമറ്റവരും സത്യസന്ധരും അബദ്ധങ്ങള് സംഭവിക്കുവാന് യാതൊരു സാധ്യതയുമില്ലാത്തവരുമാണെങ്കില് മാത്രം അത് സ്വീകരിക്കുകയും ചെയ്യുകയെന്ന നിലപാടാണ് അവര് സ്വീകരിച്ചത്. അങ്ങനെയാണ് നിവേദകപരമ്പര അഥവാ ഇസ്നാദ് പരിശോധന ഹദീഥ് പഠനത്തിന്റെ ഭാഗമായിത്തീര്ന്നത്.
കുഴപ്പങ്ങള്ക്കു ശേഷം ജീവിച്ചിരുന്ന യുവസ്വഹാബികളില് നിന്നുതന്നെ ഈ പരിശോധന ആരംഭിച്ചു കഴിഞ്ഞിരുന്നു. കളവിന്നു പ്രചാരം കൂടിത്തുടങ്ങിയപ്പോള്, താബിഉകളും ഈ നില തുടര്ന്നു. അങ്ങനെയാണ് ഉസൂലുൽ ഹദീഥ് എന്ന ശാസ്ത്രശാഖ രൂപീകരിക്കപ്പെട്ടത്.
ഇന്ന് നമുക്ക് ലഭിക്കുന്ന ഹദീഥുകള്ക്ക് രണ്ട് ഭാഗങ്ങളാണുണ്ടാവുക. സനദും മത്നും. പ്രവാചകന്(സ) പറയുകയോ ചെയ്യുകയോ അനുവദി ക്കുകയോ ചെയ്തതിനെക്കുറിച്ച് പരാമര്ശിക്കുന്ന ഹദീഥിന്റെ ആശയപ്രധാനമായ ഭാഗമാണ് മത്ന്. പ്രവാചകനില് നിന്ന് ഹദീഥ് രേഖ പ്പെടുത്തിയ വ്യക്തിയിലേക്ക് മത്ന് എങ്ങനെ എത്തിച്ചേര്ന്നുവെന്ന് വിശദീകരിക്കുന്ന ഭാഗമാണ് സനദ് അതല്ലെങ്കില് ഇസ്നാദ്. നബി(സ)യി ല്നിന്ന് സ്വഹാബിയും അദ്ദേഹത്തില്നിന്ന് അടുത്ത തലമുറയില്പെട്ട താബിഉം അദ്ദേഹത്തില്നിന്ന് അടുത്ത തലമുറയില്പെട്ടയാളും മത്ന് കേള്ക്കുകയും ഈ അവസാനത്തെ വ്യക്തിയില്നിന്ന് കേട്ടയാള് അത് രേഖപ്പെടുത്തുകയും ചെയ്യുകയാണെങ്കില് സ്വഹാബി, താബി അ്, അടുത്തതലമുറയില് പെട്ടയാള് (തബഉത്താബിഅ്) എന്നിങ്ങനെയായിരിക്കും അതിന്റെ ഇസ്നാദ്. അവതരിപ്പിക്കപ്പെട്ട ഹദീഥുകളി ലെല്ലാം മത്നിനോടൊപ്പം ഇങ്ങനെ ഇസ്നാദ് രേഖപ്പെടുത്തപ്പെട്ടിട്ടുള്ളതിനാല് ഓരോ ഹദീഥും എത്തിച്ചേര്ന്ന ശൃംഖലയിലെ ഓരോ കണ്ണി യെയും പരിശോധനാവിധേയമാക്കുവാനും അവര് സത്യസന്ധരാണോയെന്ന് മനസ്സിലാക്കുവാനും കഴിയും. ഹദീഥുകള് നിവേദനം ചെയ്ത വരെക്കുറിച്ച് പഠിക്കുകയും അവരുടെ ജീവചരിത്രങ്ങള് സൂക്ഷ്മമായി അപഗ്രഥിക്കുകയും ചെയ്ത് അവര് വിശ്വസ്തരും ഹദീഥ് നിവേദ നത്തില് തെറ്റുപറ്റാത്തവരുമാണോയെന്ന് കൃത്യമായി പരിശോധിക്കുന്ന ഒരു വൈജ്ഞാനിക ശാഖ തന്നെയുണ്ട്. 'വ്യക്തിവിജ്ഞാനീയം' (ഇല്മുര്രിജാല്) എന്നാണ് ഹദീഥ് പഠനരംഗത്ത് ഈ വിജ്ഞാന ശാഖ അറിയപ്പെടുന്നത്.
ഉസൂലുൽ ഹദീഥിന്റെ അരിപ്പയിലൂടെ വ്യാജമായ ഹദീഥുകൾക്കൊന്നും കടന്നു വരാൻ കഴിയാത്ത രൂപത്തിൽ ശാസ്ത്രീയമാണ് ഈ വിജ്ഞാനീയം. കള്ള ഹദീഥുകളുള്ളത് കൊണ്ടാണ് ഇത്രയും ശാസ്ത്രീയമായ ഒരു വൈജ്ഞാനിക ശാഖ മുസ്ലിം ലോകത്ത് വളർന്നു വന്നത്. യാഥാർഥ്യത്തെ വ്യാജനിൽ നിന്ന് തിരിച്ചറിയാൻ തക്ക ശക്തവും സൂക്ഷ്മവുമാണത്. വ്യാജഹദീഥുകളുണ്ടെന്നത് യഥാർത്ഥ നബിവചനങ്ങളെ നിഷേധിക്കുന്നതിനുള്ള കാരണമല്ല. ഒരാളെക്കുറിച്ച് ആരെങ്കിലും എന്തെങ്കിലും കളവു പറഞ്ഞുവെന്നതെങ്ങനെയാണ് അയാളെക്കുറിച്ച് സത്യസന്ധമെന്ന ഉറപ്പുള്ള കാര്യങ്ങളെ സ്വീകരിക്കാതിരിക്കുന്നതിനുള്ള കാരണമാവുന്നത്!!!
മുഹമ്മദ് നബി(സ)യിലൂടെ പൂര്ത്തീകരിച്ച മതത്തില് അദ്ദേഹത്തിന് ശേഷം യാതൊന്നും കടന്നുകൂടി മലീമസമാകാതിരിക്കുവാന് സ്വഹാ ബിമാര് ശ്രദ്ധിക്കുകയും സൂക്ഷിക്കുകയും ചെയ്തുപോന്നു. പ്രവാചകചര്യയെക്കുറിച്ച് തങ്ങള്ക്കറിയാവുന്ന കാര്യങ്ങള് മറ്റുള്ളവരി ലേക്ക് അവര് പകര്ന്നുനല്കിയത് അതീവ സൂക്ഷ്മതയോടു കൂടിയായിരുന്നു. നബി(സ) പറയാത്തതെന്തെങ്കിലും അദ്ദേഹത്തിന്റെ പേരില് അബദ്ധവശാല് തങ്ങളുടെ നാവുകളില്നിന്ന് ഉതിര്ന്നുവീഴുമോയെന്ന് ഭയപ്പെട്ട അവര് നബിചര്യയെക്കുറിച്ച് മറ്റുള്ളവരോട് പറയാന് വിസമ്മതിക്കുന്ന സ്ഥിതി വരെയുണ്ടായി. നാവില് വന്നു പോയേക്കാവുന്ന ചെറിയ പിഴവുകള് പോലും അവര് സൂക്ഷിക്കു കയും ശ്രദ്ധിക്കുകയും ചെയ്തു. ഓര്മപ്പിശകുമൂലം തെറ്റുകള് വന്നുപോകുമോയെന്ന് ഭയപ്പെട്ടവര് നിശ്ശബ്ദത പാലിച്ചു. വാര്ധക്യത്തി ലെത്തിയവര് മറവിയെ പേടിച്ച് നബിവചനങ്ങള് പറഞ്ഞുകൊടുക്കാത്ത അവസ്ഥ വരെയുണ്ടായി. ചില സംഭവങ്ങള് കാണുക.
''അബ്ദുല്ലാഹിബ്നു സുബൈര് (റ) തന്റെ പിതാവി (സുബൈറി)നോടു ചോദിച്ചു: ഇന്ന ആളും ഇന്ന ആളും ചെയ്യുന്നതു പോലെ, നിങ്ങള് റസൂൽ(സ) തിരുമേനിയില്നിന്ന് ഹദീഥ് പറയുന്നതായി കേള്ക്കുന്നില്ലല്ലോ? അപ്പോള് സുബൈര് (റ) പറഞ്ഞു: എന്നാല്, ഞാന് തിരുമേ നിയെ വേര്പിരിയാറില്ലായിരുന്നു. എങ്കിലും അവിടുന്നു ഇപ്രകാരം പറയുന്നതു ഞാന് കേട്ടിരിക്കുന്നു: ''എന്റെ പേരില് ആരെങ്കിലും കല്പിച്ചുകൂട്ടി കളവു പറഞ്ഞാല്, അവന് തന്റെ ഇരിപ്പിടം നരകത്തില് ഒരുക്കിക്കൊള്ളട്ടെ!''(സ്വഹീഹുല് ബുഖാരി, കിതാബുല് ഇല്മ്.)
സൈദുബ്നു അര്ക്വം(റ) (റ) നാടു ഞങ്ങള്ക്കു ഹദീഥ് പറഞ്ഞുതരണമെന്നു ആവശ്യപ്പെടുമ്പോള് അദ്ദേഹം ഇങ്ങനെ പറയുമായിരുന്നു? ''ഞങ്ങള്ക്കു വയസ്സു ചെല്ലുകയും മറവി ബാധിക്കുകയും ചെയ്തിരിക്കുന്നു. റസൂൽ(സ) തിരുമേനിയില്നിന്നു ഹദീഥ് പറയുന്നതാകട്ടെ, ഗൗരവപ്പെട്ട കാര്യവുമാണ്.'' സാഇബ് ബ്നു യസീദ് (റ) പറയുന്നു: മദീനയില്നിന്നു മക്ക വരെ ഞാന് സഅ്ദ്ബ്നു മാലികിന്റെ ഒന്നിച്ചു സഹവസിക്കുകയുണ്ടായി. അദ്ദേഹം നബി(സ)യെക്കുറിച്ചു ഒരു ഹദീഥും പറയുകയുണ്ടായില്ല. നബി(സ)യെക്കുറിച്ചു ഹദീഥ് പറയുമ്പോ ള് അതില് കളവു വന്നുപെട്ടേക്കുന്നതിനെ സൂക്ഷിച്ചുകൊണ്ട് ''അല്ലെങ്കില് അവിടുന്നു പറഞ്ഞപ്രകാരം'' എന്നു കൂടി അദ്ദേഹം തുടര്ന്നു പറയുമായിരുന്നു(സുനനു ഇബ്നുമാജ, കിതാബുസ്സുന്ന).
ഓര്മപ്പിശകോ അബദ്ധമോ വന്നുഭവിക്കുകയില്ലെന്ന് സ്വയംബോധ്യമുള്ള സ്വഹാബിമാര് മാത്രമാണ് ഹദീഥ് സംപ്രേഷണത്തിന് ഔല്സു ക്യം കാണിച്ചത്. തങ്ങള് പ്രവാചകനില് നിന്ന് കണ്ടതും കേട്ടതുമെല്ലാം അവര് മറ്റുള്ളവര്ക്ക് പകര്ന്നുനല്കി. വിശുദ്ധ ക്വുര്ആനിലെ നിര്ദേശങ്ങളും പ്രവാചകന്ലന്റെ ഉപദേശങ്ങളുമാണ് അവര്ക്കതിന് പ്രചോദനമായത്. ഏറ്റവുമധികം ഹദീഥുകള് നിവേദനം ചെയ്ത അബൂഹൂറൈറ (റ) പറഞ്ഞതായി ബുഖാരി ഉദ്ധരിക്കുന്നു: അബൂഹുറൈറ നബിതിരുമേനിയുടെ നടപടികള് കൂടുതലായി ഉദ്ധരിക്കുന്നു വെന്നു ജനങ്ങളതാ പറയുന്നു: അല്ലാഹുവിന്റെ കിതാബില് ഉണ്ടായിരുന്നില്ലെങ്കില് ഒരൊറ്റ വാര്ത്തയും ഞാനുദ്ധരി ക്കുകയില്ലായിരുന്നു. അതുപറഞ്ഞിട്ട്, ''വേദഗ്രന്ഥത്തില് മനുഷ്യര്ക്ക് നാം വെളിപ്പെടുത്തിക്കൊടുത്ത ശേഷം നാം അവതരിപ്പിച്ച മാര്ഗദര്ശനത്തെയും വ്യക്ത മായ ദൃഷ്ടാന്തങ്ങളേയും മറച്ച് വെക്കുന്നതാരോ അവരെ അല്ലാഹു ശപിക്കും. ശപിക്കുന്നവരെല്ലാവരും ശപിക്കും.'' (2:159) എന്ന് തുടങ്ങുന്ന രണ്ട് ക്വുര്ആന് വാക്യങ്ങള് അബൂഹുറൈറ പാരായണം ചെയ്തു കൊണ്ട് പറഞ്ഞു: മുഹാജിറുകളായ സഹോദരന്മാര് ചന്തയില് വ്യാപാരവിഷയങ്ങളില് ഏര്പ്പെട്ടിരിക്കുകയായിരുന്നു. അന്സാരി സഹോദരന്മാരോ, അവരുടെ കൃഷിയിലും. അതേയവസരത്തില് അബൂഹുറൈറ വിശപ്പടക്കിയിട്ട്, വിട്ടുപിരിയാതെ തിരുമേനിയോടൊപ്പം ഇരിക്കുകയും മറ്റുള്ളവര് ഹൃദിസ്ഥമാക്കാത്തത് ഹൃദിസ്ഥമാ ക്കുകയുമാണ് ചെയ്തിരുന്നത്.(സ്വഹീഹുല് ബുഖാരി, കിതാബുല് ഇല്മ്)
മുഹമ്മദ് നബി(സ)യോടൊപ്പം ഏറെനാള് ജീവിക്കുവാന് അവസരം ലഭിച്ച സ്വഹാബിമാരില് പലരെയും കാണുവാനോ അവരില്നിന്ന് ഹദീഥുകള് മനസ്സിലാക്കുവാനോ നബി(സ)യെ കാണുവാന് അവസരം ലഭിച്ചിട്ടില്ലാത്ത അടുത്ത തലമുറക്കു സാധിച്ചില്ല. അവര് ഇസ്ലാമി ലെത്തിയപ്പോഴേക്ക് മുതിര്ന്ന സ്വഹാബിമാരില് പലരും മരണപ്പെട്ടിരുന്നു. അതുകൊണ്ടുതന്നെ നബി(സ)യോടൊപ്പമുണ്ടായിരുന്നപ്പോള് യുവാക്കളായിരുന്ന സ്വഹാബിമാര്ക്കാണ് അടുത്ത തലമുറക്ക് ഹദീഥുകള് പറഞ്ഞുകൊടുക്കുവാന് കൂടുതല് അവസരമുണ്ടായത്. തന്റെ മുപ്പതാമത്തെ വയസ്സില് ഇസ്ലാം സ്വീകരിക്കുകയും അതിനുശേഷമുള്ള മൂന്നുവര്ഷം നബി(സ)യുടെ മരണംവരെ അദ്ദേഹത്തോടൊപ്പം വിട്ടുപിരിയാതെ ജീവിച്ച് നബിജീവിതവും മൊഴികളും നേരില് മനസ്സിലാക്കുവാന് അവസരം ലഭിക്കുകയും നബിവിയോഗത്തിനുശേഷം ഏകദേശം നാല്പത്തിയഞ്ച് വര്ഷക്കാലം സഹാബിമാരോടൊപ്പം ജീവിക്കുകയും നബിവിയോഗത്തിനുശേഷം ജനിച്ച നിരവധി പേര്ക്ക് നബിചര്യകളെക്കുറിച്ച് വിശദീകരിച്ചു കൊടുക്കാന് സാധിക്കുകയും ചെയ്ത അബൂഹുറൈറേയാണ് രേഖപ്പെടുത്തപ്പെട്ടവയില് ഏറ്റവു മധികം ഹദീഥുകള് നിവേദനം ചെയ്ത സ്വഹാബി. മറ്റൊരു പ്രധാന ഹദീഥ് നിവേദകന്, നബി(സ) മരണപ്പെടുമ്പോള് ഇരുപത്തിമൂന്ന് വയ സ്സ് പ്രായമായിരുന്ന അബ്ദുല്ലാഹിബ്നു ഉമര് (റ) ആണ്. പ്രധാനപ്പെട്ട പ്രവാചകശിഷ്യരിലൊരാളും ഉമര്(റ) ന്റ പുത്രനും അതുകൊണ്ടു തന്നെ ചെറുപ്പം മുതലേ പ്രവാചകസന്നിധിയില് ജീവിക്കുവാന് അവസരം ലഭിച്ച് നബിജീവിതത്തിന്റെ വ്യത്യസ്തങ്ങളായ വശങ്ങളെപ്പറ്റി കൃത്യമായി അറിയാന് കഴിഞ്ഞിരുന്നയാളുമായ ഇബ്നു ഉമര് (റ) മരണപ്പെടുന്നത് നബിവിയോഗത്തിന് ശേഷം ആറു പതിറ്റാണ്ടുകള് കഴിഞ്ഞാണ്. അടുത്തതലമുറയിലെ താബിഉകളില്(4) മിക്കയാളുകളെയും കാണുവാനോ അറിയുവാനോ അവസരമുണ്ടായിരുന്ന അദ്ദേഹ ത്തിന്, അതുകൊണ്ടുതന്നെ വളരെയേറെ ഹദീഥുകള് തന്റെ പിന്ഗാമികള്ക്ക് പകര്ന്നുകൊടുക്കുവാനുള്ള ഭാഗ്യമുണ്ടായി. നബിവി യോഗം നടക്കുമ്പോള് പതിനാല് വയസ്സുമാത്രം പ്രായമുള്ളയാളും അതിനുശേഷം അര നൂറ്റാണ്ടിലേറെക്കാലം ജീവിക്കുവാന് അവസരമു ണ്ടാവുകയും ചെയ്ത അബ്ദുല്ലാഹിബ്നു അബ്ബാസ് (റ) ആണ് സ്വഹാബികളില് നിന്നുള്ള മറ്റൊരു പ്രധാന ഹദീഥ് നിവേദകന്. മദീനയിലെ ത്തിയ നബി(സ)യെ സേവിക്കുവാന് സ്വന്തം മാതാവിനാല് പത്താമത്തെ വയസ്സില് നിയോഗിക്കപ്പെടുകയും അതിന് ശേഷം ഏറെക്കാലം സേവകനും സഹായിയുമായി നബി(സ)യോടൊപ്പം ജീവിക്കുകയും നബിവിയോഗത്തിന്ശേഷം എട്ടുപതിറ്റാണ്ടുകള്ക്കുശേഷം തന്റെ നൂറ്റി മൂന്നാമത്തെ വയസ്സില് മരണപ്പെടുകയും ചെയ്ത അനസ്ബ്നു മാലിക്ക് (റ) ആണ് മറ്റൊരു പ്രധാനപ്പെട്ട ഹദീഥ് നിവേദകന്. താബിഉകളില്പ്പെട്ട മധ്യവയസ്കര്ക്കും വൃദ്ധര്ക്കുമെല്ലാം ഹദീഥുകള് എത്തിക്കുവാന് തന്റെ ദീര്ഘായുസ്സ് കാരണം അദ്ദേഹത്തിന് സാധിച്ചു. തന്റെ ഒന്പതാമത്തെ വയസ്സില് പ്രവാചകപത്നിയാകുവാന് ഭാഗ്യം ലഭിക്കുകയും, എട്ടുവര്ഷത്തിലധികം അദ്ദേഹത്തോ ടൊപ്പം ദാമ്പത്യജീവിതം നയിക്കുകയും പ്രവാചകവിയോഗത്തിനുശേഷം അരനൂറ്റാണ്ടിലധികം ജീവിച്ചിരിക്കുകയും ചെയ്ത ആയിശ (റ) യാണ് ഹദീഥുകള് നിവേദനം ചെയ്ത മറ്റൊരു പ്രമുഖ വ്യക്തിത്വം. നബി(സ)യുടെ കുടുംബ-ലൈംഗിക ജീവിതത്തെക്കുറിച്ച് സമകാലിക രായ സ്വഹാബികള്ക്ക് പറഞ്ഞുകൊടുത്തതും അടുത്ത തലമുറയില്പ്പെട്ട താബിഉകളെ പഠിപ്പിച്ചതും ആയിശയായിരുന്നു.
അബൂബക്ക റിനെയും (റ) ഉമറിനെയും (റ) പോലെ നബി(സ)യോടൊപ്പം മക്കയിലും മദീനയിലുമുണ്ടായിരുന്ന സ്വഹാബിമാര് ഏതാനും ഹദീഥുകള് മാത്രം നിവേദനം ചെയ്തപ്പോള് നബിവിയോഗത്തിന്റെ സന്ദര്ഭത്തില് യുവാക്കളായിരുന്നവര്ക്ക് നൂറുക്കണക്കിന് ഹദീഥുകള് നിവേദനം ചെയ്യാന് കഴിഞ്ഞത്, അവര്ക്ക് നബി(സ)യില് നിന്ന് ഹദീഥുകള് നേര്ക്കുനേരെ കേള്ക്കാന് കഴിഞ്ഞവരുമായി സമ്പര്ക്കത്തിലേര്പ്പെടുവാന് കൂടുതല് അവസരങ്ങള് ലഭിച്ചതിനാലായിരുന്നു.
തീർച്ചയായും. ദൈവികവചനങ്ങളെ പ്രായോഗികമാക്കേണ്ടതെങ്ങനെയെന്ന് പഠിപ്പിക്കുവാൻ വേണ്ടി നിയോഗിക്കപ്പെട്ട മുഹമ്മദ്നബി(സ) കേവലമൊരു ഉപദേശിയായിരുന്നില്ല . താന് ഉപദേശിക്കുന്ന കാര്യങ്ങളെല്ലാം അദ്ദേഹത്തിന്റെ ജീവിതത്തില് പ്രാവര്ത്തികമായിക്കാണാന് അദ്ദേഹത്തിന്റെ അനുയായികൾക്ക് കഴിഞ്ഞിരുന്നു. സാധാരണക്കാരായ സ്വഹാബിമാരോടൊപ്പം കേവലമൊരു സാധാരണക്കാരനെ പ്പോലെയായിരുന്നു അദ്ദേഹത്തിന്റെ ജീവിതം. നാട്ടിലും വീട്ടിലും പള്ളിയിലും അങ്ങാടിയിലും യാത്രയിലും വിശ്രമത്തിലുമെല്ലാം അനു യായികള് അദ്ദേഹത്തോടൊപ്പമുണ്ടായിരുന്നു. അവര് അദ്ദേഹത്തിന്റെ വാക്കുകള് ശ്രവിക്കുകയും ഓര്മയില് കുറിച്ചിടുകയും ചെയ്തു; അദ്ദേഹത്തിന്റെ പ്രാര്ഥനകള് ശ്രദ്ധിക്കുകയും അവ അതേപോലെത്തന്നെ പിന്തുടരുകയും ചെയ്തു; ജീവിതവ്യവഹാരങ്ങളും നിലപാ ടുകളും സ്വഭാവങ്ങളും കൊള്ളക്കൊടുക്കലുകളും സൂക്ഷ്മമായി നിരീക്ഷിക്കുകയും അവ അനുധാവനം ചെയ്യാന് പരമാവധി പരിശ്രമി ക്കുകയും ചെയ്തു. സ്വഹാബിമാരുടെയെല്ലാം ആത്മാര്ഥമായ പരിശ്രമമായിരുന്നു അത്. നബി(സ)യെ അനുകരിക്കുവാന് അവര് ആഗ്ര ഹിച്ചു; അക്കാര്യത്തിലായിരുന്നു അനുചരന്മാരുടെ ശ്രദ്ധ. അതുകൊണ്ടുതന്നെ പരമാവധി സമയം നബി(സ)യോടൊപ്പമുണ്ടാകണമെന്ന് അവര് സ്വയം നിഷ്കര്ഷിച്ചു. തങ്ങള് നബി(സ)യോടൊപ്പമില്ലാത്തപ്പോള് അദ്ദേഹം എന്തൊക്കെയാണ് ചെയ്തതെന്നും പറഞ്ഞതെന്നും അവര് മറ്റുള്ളവരോട് അന്വേഷിച്ചു പഠിച്ചു. നബി(സ)യെ നിരീക്ഷിക്കുവാന് അവര് ഊഴം നിശ്ചയിച്ചു.
വ്യത്യസ്തങ്ങളായ ജീവിതപ്രശ്നങ്ങളില് ദൈവികവിധിയെന്താണെന്നറിയാനും, അവ പ്രയോഗവല്ക്കരിക്കുവാനും ഉത്സുകരായിരുന്നു പ്രവാചകാനുചരന്മാര്. ധര്മാധര്മങ്ങളുടെ കാര്യങ്ങളിലൊന്നും അവര് സ്വന്തമായ തീരുമാനങ്ങളെടുത്തില്ല; പ്രവാചകനായിരുന്നു എല്ലാ കാര്യങ്ങളിലുമുള്ള അവരുടെ മാര്ഗദര്ശി. അദ്ദേഹത്തോട് ചോദിച്ചറിയുകയും അദ്ദേഹം ശരിയെന്ന് വിധിച്ചത് പ്രാവര്ത്തികമാക്കു കയും ചെയ്യുകയായിരുന്നു സ്വഹാബിമാരുടെ രീതി. തങ്ങള്ക്ക് അറിയാത്ത കാര്യങ്ങളിലുള്ള പ്രവാചക നിര്ദേശം ലഭിക്കുന്നതിനായി നാഴികകള് യാത്ര ചെയ്യുവാന് അവര്ക്ക് മടിയുണ്ടായിരുന്നില്ല. ത്യാഗങ്ങള് സഹിച്ചുകൊണ്ടാണെങ്കിലും കൃത്യമായ ദൈവിക മാര്ഗനി ര്ദേശമെന്തെന്ന് മനസ്സിലാക്കിയ ശേഷം മാത്രം അത് പ്രയോഗവല്ക്കരിക്കണമെന്ന് അവര്ക്ക് നിര്ബന്ധമുണ്ടായിരുന്നു. ഉമര് (റ) പറയുന്നു: ''ഞാനും, ഉമയ്യത്തുബ്നുസൈദിന്റെ സന്തതികളില്പ്പെട്ട എന്റെ ഒരു അയല്ക്കാരനും (അയാള് മേലേ മദീനയിലായിരുന്നു) റസൂല് തിരുമേനിലയുടെ അടുക്കല് ചെല്ലുന്നതിന് ഊഴം നിശ്ചയിച്ചിരുന്നു. ഒരു ദിവസം അദ്ദേഹം ചെല്ലും, ഒരു ദിവസം ഞാന് ചെല്ലും. ഞാന് പോകുമ്പോള് അന്നത്തെ വര്ത്തമാനം ഞാന് അദ്ദേഹത്തിന്നു പറഞ്ഞുകൊടുക്കും. അദ്ദേഹം പോകുമ്പോള് അദ്ദേഹവും അങ്ങിനെ ചെയ്യും.''(സ്വഹീഹുല് ബുഖാരി, കിതാബുല് ഇല്മ്)
പ്രവാചകന്റെ (സ) നാവിൽ നിന്ന് സ്വഹാബിമാർ ദൈവവചനങ്ങള് ശ്രവിക്കുകയും, ജീവിതത്തില് നിന്ന് അവ എങ്ങനെ പ്രയോഗവല്ക്കരിക്കണമെന്ന് പഠിക്കുകയും ചെയ്തു. ധര്മാധര്മങ്ങളെ വ്യവഛേദിക്കുന്നതിനുള്ള അവരുടെ മാനദണ്ഡം നബി(സ)യുടെ വാക്കും പ്രവൃത്തിയും അനുവാദവുമായിരുന്നു. അത് അവര് പഠിക്കുകയും മറ്റുള്ളവര്ക്ക് പറഞ്ഞു കൊടുക്കുകയും ചെയ്തു. അദ്ദേഹത്തോടൊപ്പം കൂടുതല് നേരം സഹവസിച്ചവരില്നിന്ന് മറ്റുള്ളവര് നബിജീവിതത്തിന്റെ സൂക്ഷ്മാംശങ്ങള് ചോദിച്ച റിഞ്ഞു. കുടുംബ-ലൈംഗിക ജീവിതങ്ങളില് നബിമാതൃകയെപ്പറ്റി അവര് അദ്ദേഹത്തിന്റെ പത്നിമാരില്നിന്നാണ് പഠിച്ചത്. യാത്രകളില് നബി(സ)യോടൊപ്പമുണ്ടായിരുന്നവരോട് ചോദിച്ച് ഒപ്പമില്ലാത്തവര് യാത്രാമര്യാദകളെക്കുറിച്ച് മനസ്സിലാക്കി. ഈ വിവര സംപ്രേഷണ ത്തില് അവരെല്ലാം വളരെ സൂക്ഷ